Problemas Fisica Resueltos

488
F F í í s s i i c c a a U U n n i i v v e e r r s s i i t t a a r r i i a a P P r r o o b b l l e e m m a a s s d d e e F F í í s s i i c c a a Manuel R. Ortega Girón Rafael López Luque Departamento de Física Aplicada. Universidad de Córdoba.

Transcript of Problemas Fisica Resueltos

FFííssiiccaa UUnniivveerrssiittaarriiaa PPrroobblleemmaass ddee FFííssiiccaa

MMaannuueell RR.. OOrrtteeggaa GGiirróónn

RRaaffaaeell LLóóppeezz LLuuqquuee

Departamento de Física Aplicada. Universidad de Córdoba.

ii Física Universitaria

Física Universitaria Problemas de Física

Primera edición: julio 2009

© Copyright: Manuel R. Ortega Girón Rafael López Luque Editor: Manuel R. Ortega Girón CL Santa Cruz, 10 14.012 Córdoba. España. Tfnos.: +34 957 280051 (particular) +34 957 218483 (departamento) Fax: +34 957 218483 e-mail: [email protected] http://www.uco.es/users/mr.ortega Impresión: Reprografía Don Folio 14.013 Córdoba. España. I.S.B.N. Depósito legal:

© Copyright. Reservados todos los derechos.

Ninguna parte de este libro puede ser reproducida por cualquier medio, incluidas las fotocopias,

sin el permiso por escrito del autor.

Física Universitaria iii

PPrróóllooggoo

Este libro completa nuestras obras Lecciones de Física y Física Universitaria que vienen teniendo una amplia y buena acogida, durante más de dos décadas, en diversas Universidades Españolas.

Problemas de Física, así como el conjunto de la obra en la que se integra, es un libro concebido como apoyo a la enseñanza de la Física en los estudios universitarios, tanto de carácter técnico como científico, presentando un nivel apropiado para la Física que se imparte en los Primeros Ciclos de nuestras Facultades y Escuelas Técnicas.

Desde la más remota antigüedad, la enseñanza se ha enfrentado con dos problemas básicos: decidir qué conocimientos se deben transmitir (contenidos) y acertar con cómo puede hacerse esa transmisión (forma).

En el aspecto de contenidos, la mayor parte del contenido de este libro procede de nuestra experiencia personal y de los exámenes propuestos a los alumnos a quienes hemos impartido la asignatura, y corresponde a los descriptores oficiales correspondientes a los Fundamentos Físicos de la Ingeniería.

En el aspecto formal, durante la preparación de este libro hemos pretendido la consecución de dos objetivos principales que entendemos que deben orientar la docencia de las asignaturas de Física de Primer Ciclo de los estudios universitarios: familiarizar al alumno con el conjunto de los conceptos y leyes básicas que constituyen la esencia de la Física y desarrollar en el estudiante la habilidad para manejar esas ideas y para aplicarlas a situaciones concretas

En Problemas de Física hacemos un uso intensivo de figuras y esquemas para facilitar la comprensión de los problemas, su tratamiento y soluciones. Además, hay un aspecto que conviene destacar: en muchos de los problemas, las figuras representan en gran medida la solución del mismo, lo que realza la importancia de las figuras y esquemas en el planteamiento resolución de los problemas.

Córdoba, julio 2009

iv Física Universitaria

Física Universitaria v

A Estela y Olga

Desde la infancia he sido criado en el estudio de las letras y, como quiera que me aseguraban que por medio

de éstas se podía adquirir un conocimiento claro y seguro de todo aquello que es útil para la vida, yo tenía un vivísimo deseo de aprenderlas. Pero cuando acabé el

curso de los estudios, al finalizar los cuáles es costumbre ser admitido en la jerarquía de los doctos, cambié

enteramente de opinión. Por que me encontraba turbado y confuso entre tantas dudas y errores que me parecía no

haber obtenido otro provecho, al procurar instruirme, que el descubrir cada vez mejor mi ignorancia.

RENÉ DESCARTES (1596-1650)

El Discurso del Método.

vi Física Universitaria

Física Universitaria vii

FFííssiiccaa UUnniivveerrssiittaarriiaa PPrroobblleemmaass ddee FFííssiiccaa

viii Física Universitaria

MMaatteerriiaass Los códigos de materias se corresponden con el Índice de Materias de la obra Física Universitaria, del mismo autor.

M01. Álgebra vectorial. M02. Vectores deslizantes. M03. Análisis vectorial. M04. Cinemática de la partícula. M05. Cinemática del sólido rígido. M06. Principios de la Mecánica Clásica. La ley de la inercia. M07. Segunda y tercera leyes de Newton. Conservación de la cantidad de movimiento. M08. Las fuerzas de la Naturaleza. M09. Sistemas de referencia en rotación. M10. Trabajo y energía. M11. Conservación de la energía. M12. Momento angular. Fuerzas centrales. M13. Movimiento armónico simple. M15. Superposición de movimientos armónicos simples. M16. Geometría de masas. M17. Sistemas de partícilas. M18. Sistemas de masa variable. El problema de 2-cuerpos. M19. Colisiones. M20. Estática del sólido rígido. M21. Dinámica del sólido rígido. M22. Trabajo y energía en el movimiento general del sól. ríg. M24. Dinámica impulsiva del sólido rígido. M25. La ley de la Gravitación Universal. M27. Elementos de elasticidad. M29. Estática de los fluidos. M31. Cinemática de los fluidos. M32. Dinámica de los fluidos ideales. M33. Dinámica de los fluidos reales. M34. Flujo viscoso. M35. Ondas mecánicas. T00. Termodinámica. E01. Campo eléctrico. E02. Capacidad eléctrica. E03. Corriente continua. E04. Campo magnético E05. Inducción magnética. E06. Corriente alterna.

Física Universitaria: Problemas de Física Vectores. M01.1

1. Consideremos el vector A y la dirección definida por el vector B. Descompongamos el vector A en dos: uno

paralelo y otro perpendicular a la dirección del vector B. Demostrar que los vectores componentes de A son (A�B/ )eB y (B�(A�B)/ 2.

El vector A tiene como componentes los vectores A1 y A2, tal como se indica en la figura. El módulo de la componente del vector A en la dirección del vector B es la proyección de A sobre B, de modo que lo obtenemos multiplicando escalarmente A por el versor en la dirección de B; esto es,

1AB

� � � �BBA e A

De modo que el vector A1 viene expresado por

1 B� ���� � �� ��� B

BA A e

En cuanto a la componente A2, de la definición del producto vectorial se sigue la expresión del módulo de A2; esto es,

� 2 2sen senAB B A BA AB

� ��

� � � � �A B

A B

Puesto que la dirección del producto vectorial A�B es normal al plano del papel y entrante, la del producto B�(A�B) será la del vector A2, de modo que este vector vendrá dado por

� �2 2B B

� � �� � �B

A B B A BA e

A1 B

AA2

- 1 -

Física Universitaria: Problemas de Física Vectores. M01.2

2. Hallar el vector que representa la superficie del triángulo determinado por los vectores

5 8 9� � � �A i j k y 6 5� � � �B i j k concurrentes en un punto dado.

El vector que define la superficie del triángulo formado por los dos vectores viene dado por la mitad de su producto vectorial. En consecuencia, el vector S es un vector normal (perpendicular al plano) determinado por los vectores A y B, y su módulo vale 1

2 senA B � , siendo � el ángulo que forman entre sí los vectores dados, y su sentido viene determinado por la regla de la mano derecha. Analíticamente, tenemos

5 6 49 24.51 1 18 1 29 14.52 2 29 5 53 26.5

� � � � � � � �� �� � � �� � � �� � � �� � � �� � � �� � � � � � � � �� � � �� � � �� � � �� � � �� � � �� � � �� � � �� � � �� � � � S A B

Y su módulo (superficie del triángulo) es

2 2 224.5 14.5 26.5 38.9S � � � �

A

B

S

- 2 -

Física Universitaria: Problemas de Física Vectores deslizantes. M02.1

1. El módulo de la resultante de un sistema de vectores es R = 6, el invariante escalar del sistema es M�R = 30 y las ecuaciones del eje central del sistema son 2x = y = 2z. Hallar: a) el momento mínimo; b) la resultante; c) el momento respecto al origen; d) el momento con respecto al punto (2, 1, 0).

El eje central del sistema de vectores pasa por el origen de coordenadas (0,0,0) y sus ecuaciones pueden expresarse en la forma:

2 21 2 1x y zx y z� � � � � ,

por lo que su versor director es 1

1 26 1

� �� � � � �

e

a) El momento mínimo es igual a la proyección sobre el eje central del momento en cualquier punto del espacio; esto es,

mín mín mín

130 55 26 6 1

M M� �� � � � � � � � � �

M R M eR�

b) La dirección de la resultante es la del eje central: esto es, 1 1

6 2 6 26 1 1

R� � � �� � � � �� � � � � �

R e

c) Dado que el origen de coordenadas pertenece al eje central, será

0 mín

15 6 2

61

� �� � � � � �

M M

d) Aplicamos la fórmula de cambio de momentos:

P O

1 2 1 1 1 15 6 5 6 6 6 6PO 2 1 6 2 2 2 22

6 6 6 61 0 1 1 3 13

� � �� � � � � � � � � � � �� � � � � � � � � � � � �� � � � � � � � � � � � � � � � � � � �

M M R����

Eje central R

M0

- 3 -

Física Universitaria: Problemas de Física Vectores deslizantes. M02.2

2. Un sistema de vectores deslizantes es tal que en el origen el momento resultante es nulo y en los puntos A(1,0,0) y B(0,1,0) los momentos son MA = aj + k y MB = i + b j - k, respectivamente. Determinar: a) Los valores de a y b en las expresiones de los momentos. b) La resultante del sistema. c) El eje central. d) Si estuviésemos describiendo con este ejercicio el movimiento de un sólido rígido, escriba de nuevo el enunciado del problema.

a) y b) Sea R = (li + mj + nk) la resultante del sistema. Relacionamos los momentos en A y B con el momento en O:

A O�M M

B O

0 1 0AO 0

11 0

la n

a m nm

n m

�� � � � � � � ���� � � � � � � � � � �� � � � � ��� � � � � � � � �

R

M M

���� 1

1 0 1BO 1 0

1

01

11

1 0

0

l n nb m

abl

lnb

n

m

l

�������� ��� �

� � �� � � � � � � � �� ��� � � � � � � � � � � � � ��� � � � � ���� � � �

� ��� �� �� �� � �� � � � � ��

R����

de modo que

A B

0 1 11 0 1

1 1 1

�� � � � � �� � � � � � � �� � � � � � � � � � �

M M R

c) Puesto que el momento en el origen es nulo, el eje central pasa por el origen de coordenadas y tiene la dirección de la resultante R, de modo que viene dado por las ecuaciones:

1 1 1x y z x y z� � �

��

��

d) Un sólido rígido tiene un movimiento tal que, en un instante dado, las velocidades de tres de sus puntos... ... a) ... b) La rotación resultante. c) El eje instantáneo de rotación y deslizamiento.

- 4 -

Física Universitaria: Problemas de Física Vectores deslizantes. M02.3

3. Dado el sistema de vectores deslizantes de la figura, determinar: a) Los invariantes del sistema. b) El eje central. c) El momento respecto al eje Oy. d) Un sistema equivalente al anterior formado por dos vectores tales que la recta de acción de uno de ellos sea el eje Oy.

a) Los invariantes del sistema son:

1 1

2 2

=(0 1 0) P =(0,0,1)=(0 0 1) P =(1,0,0)

��� �

VV

R j k

1O

0 0 1 0 1OP OP 0 1 0 0 1

1 0 0 1 0

�� � � � � � � � � �� � � � � � � � � � � � �� � � � � � � � � � � � � � �

21 2M V V���� ����

O O

0 11 1 11 0

1�� � � �

� � � � � � � �� � � � � �

� �R MR M ��

b) Obtenemos la ecuación del eje central determinando el vector de posición de un punto E que pertenece a dicho eje:

O2

0 1 1 1/ 21 1OE 1 1 1 1/ 22 2

1 0 1 1/ 2R

�� � � � � � � �� � � � � � � � � � � � �� � � �

� � � � � � � �

R M����

y la ecuación del eje es:

E E E 1/ 2 1/ 2 1/ 20 1

1/ 211x y z

xx x y y z z x y zR R zR y� � � �� �

� � � ���

� � ��

c) El momento respecto a un eje es la proyección sobre el eje del momento respecto a un punto cualquiera de ese eje.

O

1 01 1 1

0 0yy yyM

�� � � �� � � � � � � � �� � � �

� �

� �

MM j j�

d) Sea A el vector cuya recta de acción es el eje Oy. Dado que R=A+B, el nuevo sistema será:

A

B

P (0,0,0)(0 0)(0 1- 1) P ( , , )x y z

��

���� � � � ��

AB R A

El momento en cualquier punto debe ser el mismo para los dos sistemas

AO OP�M����

B

0 (1 ) 1OP 1 1

1 (1 ) 0

x y zy xz x

��

� � �� � � � � � � �� � � � � � � � � � � � �� � � � � � � � � � � � �

A B����

(1 ) 1 11 1

(0,0

(1 )

,0)(1, 1,00 1 )

y zx x

x y

� �

� � � � �� � �� �� � � � � �� � ��� �� � � �

�� �

� �

A jB k

y1 1

1 v2

v1

x

z

yv2

v1

x

z EC

- 5 -

Física Universitaria: Problemas de Física Vectores deslizantes. M02.4

4. Sean dos sistemas de vectores deslizantes definidos por sus torsores {R;M} respectivos:

� �1 1 2 2

1 2 0 02 , 4 P 1,0,0 1 , 3 P 0,1,01 2 1 3

� � � �� � � � � � � �� � � �� � � �� � � �� � � �� � � ��� � � �� � �� � � �� �� � � �� � � �� � � �� � � �� � � �� � � �� � � �� � � �� � � �� � � �� � � �� � � � � � � �� � � �

T T

a) Reducir cada uno de los sistemas al origen de coordenadas. b) Obtener la resultante y el momento resultante del sistema total. c) Determinar el eje central del sistema total. d) Obtener el torsor resultante.

El torsor de un sistema de vectores deslizantes queda definido por su resultante R y su momento resultante M con respecto a un punto P del eje central del sistema (reducción canónica). a) La resultante Ri de cada sistema es invariante; el momento resultante Mi cambia al pasar a otro punto de reducción. Reducimos los sistemas al origen de coordenadas:

1

2

1 1,O 1,P 1 1

2 2,O 2,P 2 2

1 2 1 1 2 2 OP 4 0 2 3

1 2 0 1 4

0 0 0 1 OP 3 1

1 3 0

� � � � � � � � � �� � � � �� � � � �� � � � �� � � � �� � � � �� � � � � � � �� � � � �� � � � �� � � � �� � � � �� � � � �� � � � �� � � � �� � � � �� � � � � � � � � �� �� �� �� �� �� � � � � �� �� �� �� �� �� �� �� �� �

R M M R

R M M R

����

���� 0 11 31 3

� � � � �� � �� � �� � �� � �� � �� �� � �� � �� � �� � �� � �� � �� � �� � �� � �

b) La resultante R y el momento resultante MO del sistema total de vectores en el origen de coordenadas es la suma de las resultantes y momentos resultantes de cada uno de los sistemas:

1 2

1 0 12 1 3 141 1 2

R� � � � � �� � �� � �� � �� � �� � �� � � � � �� � �� � �� � �� � �� � �� � �� � �� � �� � �

R R R O 1,O 2,O

2 1 33 3 64 3 7

� � � � � �� � �� � �� � �� � �� � �� � � � �� � �� � �� � �� � �� � �� � �� � �� � �� � � M M M

c) Determinamos un punto P del eje central del sistema total y la ecuación de su eje central:

O2

1 3 91 1OP 3 6 114 142 7 3R

� � � � � �� � �� � �� � � �� � �� � �� � � � �� � �� � �� � �� � �� � �� � �� � �� � ��� � �

R M��� 14 9 14 1 14 3

1 3 2x y z� � �

� �

d) Proyectamos el momento MO sobre la resultante R para obtener el momento mínimo:

0mín

3 11 356 314 147 2

MR

� � � �� �� �� �� �� �� �� �� �� �� �� �� �� �� �� �

M R�� mín mín

1 135 1 53 3214 14 2 2

MR

� � � �� �� �� �� �� �� � �� �� �� �� �� �� �� �� �� �

RM

De modo que el torsor resultante es

1 15 9 1 3 3 , 3 P , ,2 14 14 142 2

� �� � � �� �� �� �� � � �� ��� � � ��� �� � �� � �� �� � �� � ��� �� � � � �� �� �� �� �� � � �� �

T

{T1}

R1

M1

P1

{T2}

R2

M2 P2

R

MO

- 6 -

Física Universitaria: Problemas de Física Análisis vectorial. M03.1

1. Sea el campo vectorial: �2 2 3 12 3 6 4x z y xy x� � � �A i j k . a) Demostrar que es conservativo. b) Calcular su circulación entre los puntos (0,0,0) y (2,-1,3).

a) Calculamos el rotacional del campo vectorial 2 2

2 2

3

012 3 06 12 12 0 0

6 6 04

� � � �� � � �� � � � �� �� �� �� �� �� �� �� �� �� ��� � � � � � � � �� �� �� �� �� �� �� �� �� �� �� �� �� �� �� �� � �� �� � � � A

x x z yy xy x xz y yx

Puesto que el campo vectorial es irrotacional, podemos asegurar que es conservativo. b) Calculamos la circulación pedida:

�(2, 1,3)

2 1 32 2 3

0 0 0(0,0,0) 0 2 2

0 0 1

2 1 3 1 32000 0 0

d 12 3 d 6 d 4 d

0 d 12 d 32 d 0 6 32 6 0296 1

��

� � �� � ��

� �

� � � � �

� � � � � � � � �

� � � �

� � �

A r�y x xz z y

x z y x xy y x z

x y y z y z

x y

z

(x,y,z)

(0,0,0)

- 7 -

Física Universitaria: Problemas de Física Análisis vectorial. M03.2

2. Consideremos el campo vectorial y z x� � �A i j k . a) ¿Es conservativo? Si lo fuese, determínese su función potencial. b) Calcular la circulación del campo vectorial entre los puntos (2,0,0) y (0,2,2�) a lo largo de la curva definida por sus ecuaciones paramétricas x = 2 cos � , y = 2 sen � , z = 4 �.

a) La condición necesaria y suficiente para que un campo vectorial sea conservativo es que sea irrotacional:

/ 1/ 1 0/ 1

x yy zz x

� � � � � �� � �� � �� � �� � �� � �� � �� �� � � � � � � �� � �� � �� � �� � �� � �� � �� � �� � �� � �� � � rot A A

de modo que es rotacional y, por ende, no es conservativo. b) El valor del campo en los puntos de la curva es 2sen 4 2cos� �� � �A i j k El vector desplazamiento infinitesimal (dr) sobre la curva dada es:

�2cos d 2sen d2sen d 2cos d d 2sen 2cos 4 d4 d 4 d

x xy yz z

� � �� � � � � �

� �

� �� � �� �� �� �� � � � � �� �� �� �� �� �� �

r i j k

De modo que: � � 2d 2sen 4 2cos 2sen 2cos 4 4sen 8 cos 8cos� � � � � � � �� � � � � � � � � � � �A r i j k i j k

Los ángulos � correspondientes a los puntos inicial y final sobre la cuva son:

1 1 1 1

2 2 2 2

P (2,0,0) : 0 4 0 0P (0, 2, 2 ) : 2 4 2 2

zz

� ��� � � � �

� � �� � �

La circulación será1:

�2

2 2 2

22

1,C 0

0 0 0

d 4sen 8 cos 8cos d

2 sen cos 8 sen cos 8 sen

4 8 8 3

� � �

� � � � �

� � � � � � �

� � �

� � � � � �

� � � � � �� � � � � � �� � � � � �� � � � � � � � �

� �A r

1 Hemos tenido en cuenta:

2 1 12 4

sen cossen d sen 2 cos d sen cos cos d sen2

� � �� � � � � � � � � � � � �

�� � � � � �� � �

- 8 -

Física Universitaria: Problemas de Física Análisis vectorial. M03.3

3. Sea el campo vectorial � � �2 2 22 2 2xy z yz x zx y� � � � � �A i j k . a) Averiguar si este campo es conservativo y, si lo fuese, determinar la función potencial correspondiente. b) En cualquier caso, calcular la circulación de este campo vectorial entre los puntos (0,0,0) y (1,2,3) a lo largo de la recta que los une.

a) La condición necesaria y suficiente para que un campo vectorial sea conservativo es que sea irrotacional; i.e. que su rotacional sea nulo:

2

2

2

2/ 2 2/ 2 2 2 0/ 2 22

� ��� � � �� � ���� �� ���� ��� ��� ���� � � � � � � � � �� ��� ��� �� �� �� �� �� �� �� �� � �� � ����

A rot Axy zx y y

y yz x z zz x xzx y

Puesto que el campo es irrotacional, es conservativo. Calculamos la función potencial asociada al campo vectorial:

� � � �

( , , )2 2 2

0 0 0(0,0,0) 0

0 0

2 2 2 2 200 0 0

, , d 2 d 2 d 2 d

0 d d 2 d

� � �� � �

� � � � � � � �

� � � � � � � �

� � � �

� � �

A r�x y z

x y z

y x x x xz z y y

x y z

x y z xy z x yz x y zx y z

x x y zx y z x y y z z x

De modo que 2 2 2

0( , , ) � � � �x y z x y y z z x� �

b) Calculamos la circulación entre los dos puntos dados como la diferencia de valores que toma la función potencial en esos dos puntos:

� � � � �

1,2,3

0,0,0d 1,2,3 0,0,0 23 0 23� � � � � �� A r � �

x

y

z

(x,y,z)

(0,0,0)

- 9 -

Física Universitaria: Problemas de Física Análisis vectorial. M03.4

4. Sea el campo vectorial A = (x + yz) i +( y + xz) j + (z + xy) k.. a) Demostrar que es un campo de potencial. b) Obtener su función potencial. c) Calcular la circulación del campo vectorial entre los puntos de coordenadas (3,0,0) y (0,3,0) a lo largo del arco de circunferencia determinado por esos dos puntos y que tiene su centro en el origen de coordenadas. d) Calcular la divergencia del campo vectorial en el origen de coordenadas. e) Utilizando el teorema de Gauss, calcular el flujo del campo vectorial a través de una esfera de radio 3 unidades centrada en (0,0,0).

a) Calculamos el rotacional del campo: // 0/

� � � � � �� � � �� � �� � �� � �� � �� � ��� � � � � � � � �� � �� � �� � �� � �� � �� � �� � �� � �� � � �� � � A

x x yz x xy y xz y yz z xy z z

por tanto, por ser irrotacional, es conservativo. b) Para obtener la función potencial calculamos la circulación del campo entre el origen de coordenadas (0,0,0) y un punto genérico (x,y,z) a lo largo de tres tramos rectilíneos en las direcciones de los ejes coordenados respectivos.

( , , )

0 0(0,0,0) 0

0

d ( ) d���������

��

� � � � �� �A rx y z

x

yz

x yz x� �

De este modo que

2 2 20

1 ( )2

� � � � �x y z xyz� �

c) Calculamos la circulación entre los dos puntos dados como la diferencia de valores que toma la función en esos dos puntos:

� � �

0,3,0

3,0,0d 0,3,0 3,0,0 0� � � �� A r � �

d) La divergencia vale

1 1 1 3�� �

�� � � � � � � �� � �

A yx zAA A

x y z

en todos los puntos del espacio. e) Teorema de Gauss:

d ( ) d 3 d 3� � � �� � �� � �A S A�S V V

V V V�

y por tratarse de una superficie esférica de 3 unidades de radio, será: 3 3 3 (unid. de flujo)

43 4 4 3 339.33

� � � � �R R� � � �

- 10 -

Física Universitaria: Problemas de Física Análisis vectorial. M03.5

5. Consideremos el campo vectorial � � �2 2 2x yz y xz z xy� � � � � �A i j k . a) ¿Es conservativo? Si lo fuese, determínese su función potencial. b) Calcular la circulación del campo vectorial entre los puntos (2,0,0) y (0,2,2�) a lo largo de la a lo largo de la recta que los une.

a) La condición necesaria y suficiente para que un campo vectorial sea conservativo es que sea irrotacional:

/ 2 0/ 2 0 0/ 2 0

x x yzy y xzz z xy

� � � � � �� � �� � �� � �� � �� � �� � �� �� � � � � � � �� � �� � �� � �� � �� � �� � �� � �� � �� � �� � � rot A A

de modo que es irrotacional y, por ende, es conservativo. Calculamos la función potencial:

� � � �

( , , )

0 0 0(0,0,0) 0

0 0

2 2 200 0 0

, , d 2 d 2 d 2 d

2 d 2 d 2 d

x y zx y z

y x x x xz z y y

x y z

x y z x yz x y xz y z xy z

x x y y z xy z x y z xyz

� � �� � �

� � � � � � � �

� � � � � � � � �

� � � �

� � �

A r�

De modo que 2 2 20( , , )x y z x y z xyz� �� � � � �

b) Calculamos la circulación entre los dos puntos dados como la diferencia de valores que toma la función en esos dos puntos:

� � � � �

0,2,22 2

2,0,0d 0,2,2 2,0,0 4 4 4 4

�� � � � �� � � � � � �� A r

Otro método Por ser conservativo, la circulación entre dos puntos es independiente del camino que sigamos. Para mayor simplicidad, calculamos la circulación entre los puntos (2,0,0) y (0,2,2�) a lo largo de la línea quebrada que se indica en la figura:

� � �(0,2,2 )

0 2 2

2 0 0(2,0,0) 0 0 0

0 0 2

0 2 2 0 2 22 2 2 2 2

2 0 02 0 0

d 2 d 2 d 2 d

2 d 2 d 2 d 4 4 4 4

y x xz z y

x yz x y xz y z xy z

x x y y z z x y z

��

� �� �

� � �� � �

� � � � � � �

� � � � � � � � � � �

� � � �

� � �

�A r

(2,0,0)

(0,2,2�)

y=0z=0

x=0z=0

x=0y=2

y

x

z

- 11 -

Física Universitaria: Problemas de Física Análisis vectorial. M03.6

6. En un sistema de coordenadas cartesianas, el campo vectorial A es perpendicular al eje z y dirigido hacia dicho eje en todo punto del espacio. El módulo del vector A en un punto cualquiera es inversamente proporcional al cubo de la distancia del punto al eje z. a) Expresar el campo vectorial A en coordenadas cartesianas. b) Demostrar que tiene función potencial. c) Determinar la función potencial. d) Describir la forma de las superficies equipotenciales.

a) Puesto que el campo tiene simetría cilíndrica, podemos “reducirlo” a un campo vectorial plano, contenido en el plano xy o plano z = 0, tal como se indica en la figura. La distancia del punto genérico P(x,y,0) al eje z y el versor re en la dirección radial vienen dados por

2 2 2 2 2

2 2OP r r

x yx y x y x yx y

� � ��

� � � � � � � ��

i ji j e e���

� � �2 2 23 4 2 2 2 2 2 2

( , ) OPrk k k kx kyx y x y

x y x y x y� �

� ��� �� �� � � � � � � � � �� �� �� �� � � ��� A e i j i j

���

b) Puesto que se trata de un campo vectorial central, podemos afirmar que es irrotacional y que, consecuentemente, tendrá función. A pesar de ello, comprobaremos que es irrotacional:

� �

/ ( , ) 0/ ( , ) 0 0/ 0

x

y

y x

x A x yy A x yz A x A y

� �� �� � �� � ��� ��� �� �� �� �� �� ��� � � � � � �� �� �� �� �� �� � �� � ���� � �� � � � �� � � � � �� ���

A

Ya que �

� �

� � �

� �

� �

� �

2 2 2 2

2 4 4 32 2 2 2 2 2 2 2

2 2 2 2

2 4 4 32 2 2 2 2 2 2 2

2 2 4 4

2 2 4 4

y

x

ky x y x kxy x yA ky kxyx x y x y x y x y

kx x y y kxy x yA kx kxyy x y x y x y x y

�� � � ���� � � � � � ��� � � � � ����� � � ���� � � � � � ��� �� � � � ���

Por consiguiente, el campo es irrotacional y existe una función potencial asociada al mismo. c) Puesto que el campo vectorial se anula en los puntos infinitamente alejados del eje, tomaremos el nivel de potencial nulo en el infinito y realizamos la integración a lo largo de una recta radial:

� 3 3 2d d d2r

k k k� � �

� � �� � �

! ! !

� � � � � �� � �A r e r� �

De modo que

�2 2( , , )

2kx y z

x y� �

d) El potencial será el mismo en todos los puntos equidistantes del eje z; por consiguiente, las superficies equipotenciales consisten en superficies cilíndricas que tienen al eje z como eje de revolución.

x

z

y

A

A P(x,y,0)

P’(x,y,z)

O

O’ er

er

x

z

yP(x,y,0)

O e

!

- 12 -

Física Universitaria: Problemas de Física Cinemática de la partícula. M04.1

1. El maquinista de un tren expreso que circula con una velocidad v1 observa a una distancia d el furgón de cola de un tren de mercancías que marcha por delante del expreso, sobre la misma vía y en el mismo sentido, con una velocidad v2, menor que la del expreso. El maquinista del expreso aplica inmediatamente los frenos, produciéndose una desaceleración constante a, mientras que el tren de mercancías continúa su marcha a velocidad constante. Determinar el menor valor de la desaceleración para que pueda evitarse la colisión.

Escribimos las ecuaciones horarias o temporales del movimiento de cada uno de los dos móviles intervinientes, conforme a la notación indicada en el esquema adjunto:

� �

� �

21 1

2 2

expreso

mercancías

12

x v t at

x d v t

� � ���� � �

En un diagrama espacio-tiempo (x,t), estas funciones están representadas por una parábola y una recta, respectivamente, como se indica en la figura. Resolviendo el sistema de las dos ecuaciones, determinamos si en algún instante coinciden en el mismo lugar la máquina del expreso con el furgón de cola del mercancías; esto es,

� �

21 2

21 2

122 2 0

v t at d v t

at v v t d

� � �

� � �

Esta ecuación de segundo grado tiene como soluciones o raíces

� � � �21 2 1 2 2v v v v ad

ta

� � � ��

Esto es, una, dos o ninguna solución. dependiendo del valor del discriminante � �2

1 2� 2v v ad� � � :

Si � >0 : � � � �22 1 2

1 2 22

v vv v ad a

d�

� � � � , dos soluciones reales distintas. Hay colisión.

Si � =0 : � � � �22 1 2

1 2 22

v vv v ad a

d�

� � � � , dos soluciones reales iguales. Hay contacto.

Si � <0 : � � � �22 1 2

1 2 22

v vv v ad a

d�

� � � � , no hay solución real. No hay colisión.

Otro método: Movimiento relativo. Describimos el movimiento del expreso en el referencial del mercancías, de modo que su velocidad es 12 1 2v v v� � . Para evitar la colisión, la velocidad relativa deberá anularse antes de que el expreso recorra la distancia d que le separa del mercancías: i.e.,

� �221 22 2 12

rel 12 2 02 2

v vvv v ad ad d

�� � � � � �

expreso mercancías

v2 v1 a

d x

x

t

desaceleracion crítica

movimiento uniforme del mercancías

movimiento del expreso

- 13 -

Física Universitaria: Problemas de Física Cinemática de la partícula. M04.2

2. Después de parar el motor de una canoa, ésta tiene una aceleración en sentido opuesto a su velocidad y directamente proporcional al cuadrado de ésta. a) Expresar la velocidad de la canoa en función del tiempo. b) Ídem la distancia recorrida al cabo de un tiempo t. c) Ídem la velocidad de la canoa después de haber recorrido una distancia x. d) Supongamos que cuando se para el motor la velocidad de la canoa era de 20 m/s y que 15 s después dicha velocidad se haya reducido a la mitad. Determinar el valor de la constante de proporcionalidad que aparece en la definición de la aceleración.

De acuerdo con el enunciado, la aceleración viene dada en función de la velocidad mediante la expresión 2a kv� � , por lo que se trata de un movimiento rectilíneo variado general; i.e., no se trata de un movimiento rectilíneo uniformemente acelerado, ya que la aceleración no es constante a) A partir de la definición de la aceleración y mediante integración obtenemos la velocidad en función del tiempo:

00

22 0

0

d d 1 1dd

1vv t

vv

v va kv k t ktt

ktv vvv

� �� � � � � � � � � � �� � ��

! !

b) Del mismo modo, a partir de la definición de la velocidad y mediante integración, obtenemos la posición o distancia recorrida en función del tiempo:

� �

00 00

0

0

0

0 0

0

1d d 1 1 dd

1d dd d

11 1 ln 1d 1 1 1 1ln ln ln

1

x t

t

u ktx t t vv kt xt x v v kt u k tv

ktvux u kt

k u k k vkv t

kkv

� � �� � � � � � �� ��

� �� �

� � � � �� �� ��

! !

!

c) De nuevo, a partir de la definición de la aceleración y mediante integración obtenemos la velocidad en función del espacio recorrido:

0

200

0

d d d d d lnd d d

v x

v

ktv v ev x v v va v kv k x ktx t x v v

�� � � �� � � � � � � �! !

d) A partir de la expresión de la velocidad en función del tiempo, obtenida en el primer apartado, despejamos la constante de proporcionalidad k y determinamos su valor:

0

-10

1 1 1 1 11 1 110 20 2015 15

0.0030

m30

v vkt k

v v t

� �� � � � � � �

x

a0 v0a v

t t0

- 14 -

Física Universitaria: Problemas de Física Cinemática de la partícula. M04.3

3. La velocidad de un vehículo quitanieves es inversamente proporcional al tiempo transcurrido desde que comenzó a nevar. Transcurrido un cierto tiempo, t0, a partir del instante en que empezó a nevar, el vehículo se pone en marcha y recorre 2 km en la primera hora y 1 km en la segunda. a) Determinar la ecuación del movimiento del vehículo, i.e., x(t). b) Calcular el valor de t0 y el de la constante de proporcionalidad. c) ¿Qué distancia recorrerá el vehículo durante la tercera hora de funcionamiento?

a) Aplicamos la definición de velocidad e integramos para obtener la distancia al origen (x) en función del tiempo:

000

d dd lnd

x t

t

x k t tv x k x kt t t t� � � � � �! !

b) Sustituimos en esta expresión los datos que nos proporciona el enunciado, expresando las distancias en kilómetros (km) y los tiempos en horas (h):

0 01 0 1

0 0

002 0 2

00

1 11 2 ln 2 ln2( )

22 3ln2 3 ln 3

t tt t x kt t

ttt t x k

tt

� � � � � ��� � " �� � � � � � ���

Desarrollamos la ecuación anterior

� � � �3 2

3 20 0 0 00 0 0

0 0 0 0

3 2 3 2 20 0 0 0 0 0 0 0

1 2 1 23ln 2ln 1 2

3 3 1 4 4 1 0

t t t tt t t

t t t t

t t t t t t t t

� � � � � � � � � �� �

� � � � � �

0

1 5 0.618 h1 1 4 1 5 22 2 1 5 0

2

t

�� ��� � � � �# � � � �

� �� ���

Ahora determinamos el valor de la constante k:

1

00

0 0

2ln 2.078 km 2.078ln1 1.618 0.618ln lnln

0.618

xt x tx k k xt ttt t

� � � � � � � �

c) Utilizamos la expresión anterior para determinar la posición de la máquina quitanieves en el instante t3 = t0+3 y el recorrido durante la tercera hora de funcionamiento:

3 3 3 23.6182.078ln 3.672 km � 0.672 km0.618

x x x x� � � � � �

2 km 1 km

t0 t0+1 t0+2

0

t =

x = 2 km 3 km

- 15 -

Física Universitaria: Problemas de Física Cinemática de la partícula. M04.4

4. Un transbordador navega en línea recta con una velocidad constante v0 = 8 m/s durante 60 s. A continuación, detiene sus motores; entonces, su velocidad viene dada en función del tiempo por la expresión

2 20 1 /v v t t� , siendo t1 = 60 s. ¿Cuál es el desplazamiento del transbordador en el intervalo 0 t� � $ ?

El espacio recorrido con velocidad constante hasta el instante t1 = 60 s es

1 0 1 8 60 480 mx v t� � � �

A partir de ese instante, la velocidad va disminuyendo, por lo que obtendremos el recorrido mediante una integración:

1 1

1

2 220 1 0 1

0 12 2 2

2 21 0 1 0 1 0 1

1

d dd d dd

1 1 1

x

x t

t

v t v tx tv x t x v ttt t t

x x v t v t v tt t

$ $

$

$

� � � � � � �

� �� � � � � �� $ �

! !

de modo que el desplazamiento total en el intervalo 0 t� � $ es

1 0 1 0 1 0 1 0 12 960 mx x v t v t v t v t$ � � � �

t60 s 120 s

v

960 m

480 m x

8 m/s

- 16 -

Física Universitaria: Problemas de Física Cinemática de la partícula. M04.5

5. El bloque de la figura está unido al extremo un hilo inextensible que pasa por una polea B. Para acercar el bloque masa hacia sí, un operario hace descender el extremo A del hilo con una velocidad constante de 1 m/s. Calcular la velocidad y la aceleración que tendrá la masa cuando pase por el punto C, indicado en la figura, situado a 8 m del operario.

Consideramos el sistema de ejes de la figura y establecemos la relación existente entre la distancia l y la distancia x:

l2 = x2 + 62 y la derivamos respecto al tiempo

d d d d2 2d d d d

l x l xl x l xt t t t� � �

Así, cuando x = 8 m y teniendo en cuenta que dl/dt es la velocidad de decrecimiento de la longitud l, que coincide con la velocidad con que desciende el extremo A del hilo

(i.e., dl/dt = - 1 m/s), se obtiene 2 2d d 8 6 51 m/s =

d d 8 4- 1.25 m/sx l l

t x t

� � � � �

Derivamos de nuevo 2 22 2

2 2

d d d dd dd d

l l x xl xt tt t

� � � � � � � � �

y teniendo en cuenta que 2

2

d 0d

lt

� y que para x = 8 m es d 5 m/sd 4

xt

� � , después de despejar se

obtiene 2 22

22 22 0.0703 m/sd 1 d d 1 25 91 m/s

d d 8 16 128dx l x

x t tt� �� � � � � �� � � � � � �� �� � � � � � � � �

��

y

x

6 m

x

l

- 17 -

Física Universitaria: Problemas de Física Cinemática de la partícula. M04.6

6. Si el cuerpo A de la figura se mueve hacia la izquierda con una celeridad de 6 m/s, determinar la celeridad del cuerpo B. Además, si la celeridad del cuerpo A disminuye a razón de 1 m/s2, determinar la aceleración del cuerpo B.

Establecemos la condición de que la longitud de la cuerda permanece constante, adoptando el convenio de signos que se indica en la figura,

A B4 2 cte.x x �

y la derivamos con respecto al tiempo

A B B A B A 12 m/s4 2 0 2 2 2 6x x x x v v � � � � � � � � � � ��� � � �

de modo que el cuerpo B se mueve hacia la izquierda (al contrario de lo indicado en la figura). Derivamos de nuevo con respecto al tiempo para obtener las aceleraciones:

A B B A B2

A4 2 0 2 2 2 ( 1 2 m/s)x x x x a a � � � � � � � � � � � ��� �� �� ��

de modo que el cuerpo B presenta una aceleración en sentido contrario a su velocidad, por lo que ésta disminuye. En la figura adjunta se indican los sentidos reales de las velocidades y aceleraciones.

A B

A B

O

xA xB

vA, aA vB, aB

- 18 -

Física Universitaria: Problemas de Física Cinemática de la partícula. M04.7

7. La deslizadera A se mueve hacia la derecha, por la guía rectilínea horizontal, con una velocidad vA constante. La deslizadera A está unida al bloque B mediante un hilo inextensible que pasa por una polea en C. Calcular velocidad y aceleración del bloque B en función de la distancia x que se indica en la figura.

Sea L la longitud del hilo. Escribimos la condición geométrica de ligadura:

2 2L x h s� y la derivamos con respecto al tiempo:

B A2 22 2

2 02

xx s xv vx h

sx h

� � � ��

� � �

Para obtener la aceleración tangencial del bloque B debemos derivar de nuevo con respecto al tiempo:

2 2 2 2A

2 2 3

2 22 2 2

t A A2 2 2 2 3 / 2 / 2

(( )) (

)xx

x hx x h x

x x x x ha s v vh v

xh x h hx �

��

� � � � �

��� ���

La aceleración normal del bloque B será: 222

Bn

A2 2

vxRx

vh

aR

� �

A

C B

O�

h

x

s

- 19 -

Física Universitaria: Problemas de Física Cinemática de la partícula. M04.8

8. Sobre un terreno horizontal, lanzamos una pelota, verticalmente hacia arriba, con una velocidad inicial de 10 m/s. El viento ejerce sobre la pelota una fuerza horizontal igual a la quinta parte de su peso. a) Calcular la altura máxima que alcanza la pelota y su velocidad (módulo y dirección) en ese instante. b) Determinar la distancia entre el impacto en el suelo y el punto de lanzamiento, así como la velocidad de la pelota (módulo y dirección) en ese instante.

Se trata de la composición de dos movimientos unifor-memente acelerados en direcciones perpendiculares entre sí, cuyas aceleraciones son

0.2x

y

a ga g

���� � ���

Mediante dos integraciones sucesivas, obtenemos

2 2

200

1 0.10.2 212

xx x

y

x a t gtv a t gtv v gt

y v t gt

� � ��� ��� ��� �� ��� � � ���

a) En el punto más alto de la trayectoria será:

0A(A) 0y

vv t

g� � �

La altura máxima alcanzada y la velocidad en ese instante serán: 2 2 2 20 0 0

A 2

00

1 10 5.10 m2 2 2 9.8

(A) (A) 0.2 0.2 0.2 10 2 m/sx

v v vy g

g ggv

v v g vg

� � � � ��

� � � � � �

b) Cuando la pelota regresa al suelo, será:

2 00 B B B

21(B) 02

vy v t gt t

g� � � � �

El alcance y las componentes de la velocidad en ese instante serán: 2 2 20 0

B 2

0B 0

00 B 0 0

4 100.1 0.4 0.4 4.08 m9.8

2(B) 0.2 0.2 0.4 0.4 10 4 m/s

2(B) 10 m/s

x

y

v vx g g

ggv

v gt g vg

vv v gt v g v

g

� � � �

� � � � � � ����� � � � � � � � ���

El módulo y dirección de dicha velocidad son: 2 2 10(B) 4 10 116 10.77 m/s arctg arg tg 2.5 68º

4v �� � � � � �

y

x1 2 3 4 5 6

1

2

3

4

5

6

g

g/5

v(A) A

� B

v(B)

v0

- 20 -

Física Universitaria: Problemas de Física Cinemática de la partícula. M04.9

9. Un muchacho que está situado a 4 m de una pared vertical lanza contra ella una pelota según indica la figura. La pelota sale de su mano a 2 m por encima del suelo con una velocidad inicial v = (10i + 10j) m/s. Cuando la pelota choca en la pared, se invierte la componente horizontal de su velocidad mientras que permanece sin variar su componente vertical. ¿A qué distancia de la pared caerá la pelota al suelo?

Tomamos un sistema coordenado de referencia con origen en el punto de lanzamiento de la pelota, como se indica en la figura. Podemos simplificar la resolución de problema observando que la pared actúa como un “espejo”, de modo que consideraremos la trayectoria “virtual” que se indica en la figura inferior. Escribimos las ecuaciones paramétricas del movimiento de la pelota y, a partir de ellas, eliminando el tiempo, obtenemos la ecuación de la trayectoria:

00

02 210 2 2

0 02

xx

yy

x x

xx v t tv

v gy v t gt y x xv v

� � � ����� � � � � ���

2 22

10 9.8 0.04910 2 10

y x x x x# � � � ��

La pelota toca el suelo cuando y = -2 m, de modo que 2 22 0.049 0.049 2 0

22.24 m1 1 8 0.049 1 1.1798(negativo)2 0.049 0.098

x x x x

x

� � � � � � � �

�� � �� � � �� �

lo que representa una distancia a la pared de 22.24 4. 18.24 m00D � � �

v0

20.049y x x� �

v0

D

y

x

Trayectoria virtual

- 21 -

Física Universitaria: Problemas de Física Cinemática de la partícula. M04.10

10. En un cierto instante la celeridad de una partícula es de 20 m/s y el módulo de su aceleración es 3 m/s2. En ese instante, los vectores velocidad y aceleración forman entre sí un ángulo de 30º. Determinar la curvatura y el radio de curvatura de la trayectoria de la partícula en ese instante.

Componentes intrínsecas de la aceleración:

tt t n n

n

cossen

a aa a

a a��

��� � � ��

a e e

De la relación existente entre la aceleración centrípeta o normal y el radio de curvatura, se sigue:

2 2 2

nn sen

v v vaa a

%% �

� � � �

Sustituyendo los valores dados en el enunciado: 2 220 267 m

sen 3sen 30ºv

a%

�� � �

La curvatura se define como la inversa del radio de curvatura: -1

2 2

1 sen 3sen 30º 0.00375 m20

av

�&%

� � � �

a

an

at

C�

v

- 22 -

Física Universitaria: Problemas de Física Cinemática de la partícula. M04.11

11. Una partícula, que se mueve con aceleración constante a = 2 i + 3 j + k (S.I.), pasa por el origen de coordenadas en el instante inicial (t = 0) con una velocidad v = - 3 i - 2 j (S.I.). a) Escribir las expresiones de la velocidad y las ecuaciones de la trayectoria en función del tiempo. b) Determinar el instante en que la velocidad es mínima y el valor de ésta. c) Dígase que tipo de trayectoria sigue la partícula (circular, rectilínea, elíptica, u otra).

Puesto que la aceleración es constante, las expresiones de la velocidad y del vector de posición serán:

0 0t� �v v a r r 210 2t t v a

a) Sustituyendo en estas expresiones las condiciones propuestas, obtenemos: 2

2 2

2

3 2 3 2 3 2 312 3 2 3 2 3 2 1.52

0 1 0 1 0.5

t tt t t t t

t t

� �� � � � � � � � � � � � � �� � � � � � � � � � � � � � � � � � � �

� � � � � � � � � � � �

v r

y las ecuaciones paramétricas de la trayectoria son 2 2 23 2 1.5 0.5x t y t z t� � � � �

b) La celeridad o módulo de la velocidad viene dado por 2 2 2 2 2( 3 2 ) ( 2 3 ) 14 24 13v t t t t t� � � � �

de modo que derivando con respecto al tiempo e igualando a cero (condición de máximo o de mínimo), tenemos:

2d( ) 2428 24 0d 28

6 s7

v t tt

� � � � � �

y la celeridad en ese instante es 2 236 6 504 1008 637 13314 24 13 2.71 (m/s)

49 7 49 491.65 m/svv �

� � � � � � �

c) Con carácter general, cualquier movimiento en el que la aceleración sea constante presenta una trayectoria parabólica. El paradigma de tales movimientos es el movimiento de un proyectil en el campo gravitatorio.

- 23 -

Física Universitaria: Problemas de Física Cinemática de la partícula. M04.12

12. Una partícula se mueve en el plano xy con aceleración constante. Para t = 0, la partícula se encuentra en la posición r0 = 4 i + 3 j m y se mueve con velocidad v0. Para t = 2 s, la partícula se ha desplazado a la posición r2 =10 i –2 j m y su velocidad ha cambiado a v2=5 i – 6 j m/s. Determinar: a) La velocidad v0. b) La aceleración de la partícula. c) La velocidad de la partícula en función del tiempo. d) La ecuación de la trayectoria. e) Las aceleraciones normal y tangencial y el radio de curvatura para t = 2 s.

Puesto que la aceleración es constante, las expresiones de la velocidad y del vector de posición serán:

210 0 0 2t t t� � v v a r r v a

Sustituyendo en estas expresiones las condiciones propuestas, obtenemos:

00

00

0 20

00

5[1] 2 5

6 2[2] 2 6

0 0 0

10 4[3] 2 2 622 3 2[4] 2 2 52

0 0 0 0

x xx x

y yy y

x xx x

y yy y

v av a

v av a

v av a

v av a

� � � � � � ���� � � � � � �� � � � ���� � � � � �

� � � � � � � � ���� � � � � � � �� � � � � ���� � � � � � � �

de modo que disponemos de cuatro ecuaciones con cuatro incógnitas. Resolviéndolas, tenemos:

00002 2

0 0

1 m/s[2] 2 61 m/s[1] 2 5[3] 2 2 6 [4] 2 2 52 m/s 3.5 m/s

yy yxx x

x x y yx x

vv avv av a v aa a

� � �� �� � �� � � �� �� � � � � � �� � �� � �� � � �

Los resultados pedidos son: 2

2 2 20

1 2 1 2 41 m/s 3.5 m/s 4.03 m/s 1 3.5 m/s 3 1.75 m0 0 0 0

t t ta t t t

� � � � � � � �� � � � � � � � � � � �� � � �

� � � � � � � �

v a v r

e) En el instante t = 2 s, serán

2t t2

n t n

2 5 5 1551 31 13.5 6 6 18661 61 61

0 0 0 0

2 155 331 13.5 186 27.561

3.97 m/s

0.61

0 0 0

av v v

a

� �� � � � � � � �� �� � � � � �� � � � � � � � � � � � � �� � � �� � � � � � � � � � � � � � � ��

�� � � � � �� � � � � � � � � � � �� � � � � � � � �

a v v a va v v

a a a

2

2

n

7

61 =0.

0 m/s

8 .6 m70

6va

% � �

- 24 -

Física Universitaria: Problemas de Física Cinemática de la partícula. M04.13

13. Las ecuaciones temporales del movimiento de una partícula son: x = 4 cos t, y = 4 sen t, z = 3t. Hállense, de forma genérica, para cualquier instante: a) La velocidad y aceleración. Determinar, también, los módulos de la velocidad y de la aceleración. b) Si la trayectoria es plana o no. c) El radio de curvatura de la trayectoria.

a) Obtenemos la velocidad y la aceleración de la partícula por derivación 4cos 4sen 4cos

d d4sen 4cos 4send d

3 3 0

t t tt t t

t tt

� �� � � � � �� � � � � � � � � � �� � � � � � � � �

r vr v a

Sus módulos son: 2 2

2 2

16sen 16cos 9 16 9 5

16cos 16sen 16 4

v t t

a t t

� � �

� � �

c) La curvatura y el radio de curvatura de la trayectoria se obtienen a partir de la velocidad y de la aceleración utilizando la expresión:

3

1v

&%

�� �

v a.

Calculamos el producto vectorial y su modulo:

2 2

2 2

4sen 4cos 12sen 12sen4cos 4sen 12cos 12cos

3 0 16sen 16cos 16

12 16 20

t t t tt t t t

t t

� �� � � � � � � �� � � � � � � � � � � �� � � � � � � � � � � �

� � �

v a

v a

El radio de curvatura es constante y vale: 3 35 125 6.25

20 20v% � � � ��v a

b) En el triedro móvil, intrínseco o de Frenet, calculamos el versor binormal:

b

12sen 0.6sen1 12cos 0.6cos20

16 0.8

t tt t

� � � �� � � � � � � �� � � � �

� �

v aev a

Este versor no es constante, ya que cambia su dirección en el transcurso del tiempo (i.e., de un punto a otro de la trayectoria), por lo que ésta no es plana, sino alabeada. En concreto, se trata de una trayectoria helicoidal uniforme, de radio R = 4, cuyo eje es el eje z, de paso constante h, tal que

32 2 6 18.851

zvh � � �

'� � � �

- 25 -

Física Universitaria: Problemas de Física Cinemática de la partícula. M04.14

14. Una partícula se mueve en el plano de tal forma que las componentes cartesianas de su velocidad vienen dadas en función del tiempo por las expresiones: 34 4xv t t� , 4yv t� (SI). En el instante inicial t0 = 0 s, el móvil se encontraba en la posición x0 = 1 m, y0 = 2 m. Calcular: a) Las componentes de la aceleración en cualquier instante. b) Las coordenadas x e y del móvil en función del tiempo.

a) Obtenemos las componentes de la aceleración derivando las de la velocidad:

� �

� �

3 23

d d 4 4 12 44 4 d dd4 d 4 4d d

xx

x

yyy

va t t tv t t t t

vv ta t

t t

� � � � �� � � ��� ��� �� � � ���

b) Obtenemos las coordenadas de posición integrando las componentes de la velocidad:

� � � �3 331 0

2 0

4 2 4 2

2 2

d 4 4 d 4 4 dd 4 4 ddd d 4 d d 4 d4d

1 2 2 12 2 2 2

x tx

y t

y

xv t t x t t tx t t tty y t t y t tv tt

x t t x t ty t y t

� �� � � � � � � � �� � �� � ���� � ��� � ���

� �� � � � ��� �� � � � �� �

! !! !

- 26 -

Física Universitaria: Problemas de Física Cinemática de la partícula. M04.15

15. El movimiento de una partícula viene dado por el vector 2 2 2 (1 ) (1 3 ) ( 2 ) t t t t t� r i j k .

a) Demostrar que dicho movimiento es plano, con el origen del vector r en dicho plano. b) Hallar un vector normal al plano del movimiento.

a) Determinamos la velocidad y la aceleración por derivación: 2

2

2

1 2 21 3 1 6 6

2 1 4 4

t tt t t

t t t

� � � � � �� � � � � � � �� � �

� � � � � �

r v a

El versor binormal a la trayectoria en un punto genérico del mismo viene dado por

b

1 12 31 1 cte.

32 3 1 1

� �� � � �� � � � � � �� � � � � � � � �

v aev a

ya que

2 2 4 24 6 24 2 1

1 6 6 2 8 8 2 2 1 2 31 4 4 12 2 12 2 1

t t tt t tt t t

� � � �� � � � � � � � � �� � � � � � � � � � � � � �� � � � � � � � � � � � � � � � � � �

v a v a

Como el versor binormal es perpendicular al plano osculador, definido por los vectores velocidad y aceleración en cada punto de la trayectoria, éste también permanecerá constante, por lo que la trayectoria descrita por el móvil es plana, por estar contenida en dicho plano. Para demostrar que el origen de coordenadas está contenido en el plano de la trayectoria (plano osculador) , es suficiente demostrar que el vector de posición r es perpendicular al versor binormal en todos los puntos de la trayectoria; i.e., que el vector de posición r está contenido en el plano osculador. En efecto,

� �2

2 2 2 2b

2

1 13 31 3 1 1 1 3 2 0

3 32 1

tt t t t t t t

t t

� � �� �� � � � � � � � �� �

� � � � �

r e� �

b) El versor binormal determinado en el apartado anterior es perpendicular al plano de la trayectoria

eb

r plano osculador

trayectoria

en

et

a

v

xy

z

- 27 -

Física Universitaria: Problemas de Física Cinemática de la partícula. M04.16

16. El vector posición de un móvil puntual viene dado en función del tiempo por la expresión 4cos10 5sen10 3cos10t t t� � �r i j k en la que todos los valores están expresados en unidades del

sistema internacional. a) Hallar la velocidad y aceleración del móvil en cualquier instante. b) Demostrar que la trayectoria es plana y determinar el versor normal a dicho plano. c) Demostrar que el origen de coordenadas está contenido en el plano de la trayectoria. d) Hallar las aceleraciones tangencial y normal y el radio de curvatura en un punto genérico de la trayectoria. e) De acuerdo con los resultados anteriores, indíquese que tipo de movimiento tiene el móvil.

a) Determinamos la velocidad y la aceleración por derivación:

2

4 cos10 40sen10 400 cos10d m d m5sen10 m 50 cos10 500sen10d s d s

3cos10 30sen10 300 cos10

t t tt t t

t tt t t

� �� � � � � �� � � � � � � � � � �� � � � � � � � � � �

r vr v a

b) El versor binormal a la trayectoria en un punto genérico del mismo viene dado por

b

15000 3 0.61 10 0 0 cte.25000 520000 4 0.8

� � �� � � � � �� � � � � � � � �� � � � � � �

v aev a

y como dicho vector es perpendicular al plano osculador, definido por los vectores velocidad y aceleración en cada punto de la trayectoria, éste también permanecerá constante, por lo que la trayectoria descrita por el móvil es plana, ya que está contenida en dicho plano. c) Para demostrar que el origen de coordenadas está contenido en el plano de la trayectoria, es suficiente demostrar que el vector de posición r es perpendicular al versor binormal en todos los puntos de la trayectoria; en efecto,

b

4cos10 0.65sen10 0 2.4cos10 2.4cos10 03cos10 0.8

tt t tt

�� � � �� � � � � �� � � � � �

r e� �

d) Las componentes intrínsecas de la aceleración y el radio de curvatura se obtienen a partir de las expresiones:

2 2

t nn

22500 500 m/s 5 m000 50 2500 50 50 500 500

va av v a

%�

� � � � � � � � � �v av a�

e) El móvil recorre una trayectoria circular de 5 m de radio con una celeridad constante de 50 m/s.

� �

2 2 2 2 2 2

2 2 2 2

2

22 2

1600sen 10 2500cos 10 900sen 10 2500 sen 10 cos 10 2500 m/s

160000cos 10 250000sen 10 90000cos 10 250000 m/s

50 m/s

500 m/s

v t t t t t

a t t t

v

a

� � � � � �

� � � �

�40sen10 400cos10

50cos10 500sen10 16000 25000 9000 sen10 cos10 030sen10 300cos10

40sen10 400cos1050cos10 500sen1030sen10 300cos10

t tt t t tt t

t tt tt t

� � � �� �� �� �� �� �� � � � � �� �� �� �� �� �� �� �� �

� � �� ��� ��� �� � � ��� ��� ��� �� �

v a

v a

� �

2 2

2 2

2 2 2 3

15000cos 10 15000sen 10 1500012000sen10 cos10 12000sen10 cos10 0

2000020000sen 10 20000cos 10

15000 20000 25000 m / s

t tt t t t

t t

� �� � �� � ���� ���� �� ��� � �� �� ��� ��� � �� �� �� � � ��

� � � �v a

eb

r plano osculador

trayectoria

en

et

a

v

xy

z

- 28 -

Física Universitaria: Problemas de Física Cinemática de la partícula. M04.17

17. Dadas las ecuaciones paramétricas (temporales) del movimiento de una partícula: x = 2t, y = t2, z = t3/3, determinar: a) Las componentes intrínsecas de su aceleración en el instante t = 1; b) el radio de curvatura de la trayectoria en dicho instante.

Calculamos la velocidad y la aceleración de la partícula en un instante genérico t:

2 42

2 23 2

2 2 04 4

2 24 4 2 1/ 3 2

tv t t

t ta t tt t t

� � � � � � � � �� � � � � � � � � �� � � � � �� � � � � � �

r v a

En el instante t=1 será: 1 2 0 31 2 3 2

2 21/ 3 1 2

vv

a

� � � � � � ���� � � � � � � � � �� � � ���� � � � � �

r v a

a) Determinamos el versor tangente a la trayectoria en ese instante: t

21 23

1v

� �� � � � � �

ve .

Calculamos el módulo de la aceleración tangencial en el instante t = 1:

� �t t

0 21 12 2 0 4 2 23 3

2 1a

� � � �� � � � � �� � � � � �

a e� �

Ahora podemos determinar las componentes intrínsecas (tangencial y normal) de la aceleración en el instante t = 1:

t t t n t

2 4 / 3 0 4 / 3 4 / 3 22 22 4 / 3 2 4 / 3 2 / 3 13 3

1 2 / 3 2 2 / 3 4 / 3 2a

� �� � � � � � � � � � � �� � � � � � � � � � � � � � � �� � � � � � � � � � � � � � � � � �

a e a a a

b) El radio de curvatura se determina a partir de los módulos de la velocidad y de la aceleración normal, ya que 2

na v %� , con

n2 4 1 4 23

a � �

De modo que 2

n

9 4.52

va

% � � �

- 29 -

Física Universitaria: Problemas de Física Cinemática de la partícula. M04.18

18. Una lancha motora, que navega río arriba, se encontró con una balsa arrastrada por la corriente. Una hora después de este encuentro, el motor de la lancha se averió. La reparación duró 30 min; durante este tiempo la lancha fue arrastrada por la corriente. Reparado el motor, la lancha navegó río abajo con la misma velocidad (respecto del río) que antes de la avería, y alcanzó a la balsa a una distancia de 7.5 km del punto de su primer encuentro. Determinar la velocidad de la corriente del río, considerándola constante.

La resolución del problema es muy simple si lo planteamos en un referencial (el del río) en el que la balsa se encuentra en reposo. En ese referencial, la lancha también está en reposo durante los 30 min que dura la reparación de la avería y su velocidad (en módulo, no en dirección) es la misma cuando navega río arriba que cuando lo hace río abajo. En consecuencia, cuando la lancha navega río abajo, después de la reparación, empleará de nuevo 1 h en alcanzar a la balsa. Así, el tiempo total que habrá transcurrido desde el primer encuentro y el reencuentro con la balsa será de 1 h + 30 min + 1 h =2.5 h. Durante ese tiempo, la balsa, arrastrada por la corriente, ha recorrido una distancia (respecto a tierra) de 7.5 km. De este modo, la velocidad de la balsa (respecto a tierra), y también la velocidad de la corriente, será:

7.5 km2.

kh5 hm3v � �

También podemos resolver el problema en el sistema de referencia de tierra. En este referencial, la balsa se desplaza con velocidad constante v0 (la misma que lleva la corriente del río). Sea v la velocidad de la lancha con respecto al río. La lancha motora lleva una velocidad �(v�v0) durante 1 h (cuando remonta el río), una velocidad v0 durante 0.5 h (durante la avería, arrastrada por la corriente) y una velocidad +(v–v0) durante un cierto tiempo t (cuado desciende por el río, hasta reencontrar la balsa). Las posiciones de la balsa y de la lancha en este referencial serán:

1 0

2 0 0 0

balsa (1.50 )lancha 1.00 ( ) 0.50 ( )

x v tx v v v v v t

� � � �

de modo que igualando esta dos expresiones (instante de reencuentro) obtenemos

0 0 0 0(1.5 ) 1.00 ( ) 0.5 ( ) 0 1 hv t v v v v v t v vt t � � � � � � � � Durante las 2.50 h, la balsa se ha desplazado 7.5 km, arrastrada por la corriente, de modo que su velocidad, que será la de la corriente, es

07.5 km2.5 h

km3h

v � �

v0 (0.5 h)

7.5 km 0 km

v-v0 (1 h)

v+v0 (1 h) v0 (2.5 h)

x lancha corriente

balsa

- 30 -

Física Universitaria: Problemas de Física Cinemática de la partícula. M04.19

19. Una pequeña embarcación es arrastrada por una corriente de 3 km/h dirigida hacia el Este. Quiere ir a un lugar situado al Nordeste de su posición actual y su velocidad de máquinas es de 15 km/h (relativa). Determinar el rumbo que debe seguir la embarcación y su velocidad efectiva (absoluta).

La velocidad efectiva o absoluta (vb) de la embarcación es igual a la suma vectorial de su velocidad relativa a la corriente de agua (vba) y a la velocidad de ésta (va); esto es,

b ba a� v v v

tal como se indica en el diagrama vectorial adjunto.

b ba a

b ba

cos 45º cos

sen 45º sen

v v vv v

� � ���

( ���

y, puesto que sen 45º =cos 45º, se sigue

� � aba ba a ba a

ba

3sen cos sen cos sen cos 0.215

vv v v v v

v� � � � � �� � � � � � � � �

Elevando al cuadrado ambos miembros de la ecuación anterior y reduciendo términos, tenemos

2 2 2 2sen cos 2sen cos 0.2 sen 2 1 0.2 0.96� � � � � � � � � � � de donde se sigue:

36.9º73.74º2

180º 73.74º 106.26º� �

�� � �� � �� 53.1º

�����

y la velocidad absoluta de la embarcación es

b basen sen 53.1º15 16.97 km/h

sen 45º sen 45ºv v �

� � � �

45º

vb

x

y

vba

va

� Este

Norte NE

- 31 -

Física Universitaria: Problemas de Física Cinemática del sólido rígido. M05.1

1. En un instante determinado, las velocidades de tres de los puntos de un sólido rígido, de coordenadas A(0,0,0), B(1,1,0) y C(0,1,1), son vA(6,-2,6) vB(4,0,5) y vC(5,-2,a). Determinar: a) El valor de a para que el movimiento sea posible. b) La velocidad angular del sólido en dicho instante. c) Las ecuaciones del eje instantáneo de rotación y deslizamiento. d) La velocidad de deslizamiento.

a) Condición cinemática de rigidez:

A C

6 0 5 0AC AC 2 1 2 1 4 2

6 1 16aa

a

� � � � � � � �� � � �� � � �� � � �� � � �� � � �� � � � � � � � � � �� � � �� � � �� � � �� � � �� � � �� � � �� � � �� � �

� �v v���� ����� � � �

b) Relacionamos las velocidades de los puntos A, B y C:

B A

C A

4 6 1 2AB 0 2 1 2

15 6 0 1

5 6AC 2

6

2l nm n

m ln l m

n� � � � � � � � � � � �� �� � � � � � �� � � � � � �� � � � � �� � � � � � �� � � � � �� � � � � � � � � � � � � � �� � � � � �� � � � � �� � � � � � � � �� � � � � �� � � � � � ��� � � � � �� � � � � �� �� � � � � � � ��� �� �� � � � �� �� �� ����

�v v �

v v �

����

���� 0 1 22 1 0

01

1

6 2 0

ml

l mm l

l

� � � � � � � � � �� �� � � � � �� � � � � �� � � � �� � � � � �� � � � �� � � � � � � � � � �� � � � �� � � � �� � � � � �� � � � �� � � � � ��� � � � �� � � � �� � � ��

de modo que 01 , con = 52

�� ��� �� �� � �� �� ����

� .

c) Determinamos las coordenadas de un punto del EIRD, sabiendo que vO = vA:

O2

0 6 101 11 2 125 52 6 6�

� � � � � �� � �� � �� � � �� � �� � �� � � �� � �� � �� � �� � �� � �� � �� � �� � ��� � �

� v

de modo que las ecuaciones del EIRD son 2

105 10 5 12 5

0 1 2 5 306x y z x

y z��� � � �� �

��

�����

d) La velocidad de deslizamiento o mínima es la proyección de la velocidad de cualquier punto del sólido sobre la dirección de '. Su módulo es

desl A

6 01 102 1 2 55 56 2

v�

� � � �� �� �� �� �� �� � � � �� �� �� �� �� �� �� �� �� �

�v � �

y su expresión vectorial:

desl desl

02 5 15 2

024

v�

� ��� �� �� � �� �� �� ����

� ��� �� �� �� �� ����

�v

- 32 -

Física Universitaria: Problemas de Física Cinemática del sólido rígido. M05.2

2. Tres puntos de un sólido rígido tienen en un instante dado las siguientes velocidades: A = (0, 0, 0) vA = a i -2 j+ a k B = (1, 1, 0) vB = b i +5 k C = (0, 1, 1) vC = 5 i +c j+ 6 k Determinar: a) Las componentes a, b, c de las tres velocidades. b) La velocidad angular del sólido. c) Ecuación del eje instantáneo de rotación y deslizamiento.

a) Relacionamos las velocidades de los puntos A, B y C:

B A

C A

1AB 0 2 1 2

5 0 5 5

6

2

5AC

b a nb a l b a nm n

a n a l m an

l m

c

� � � ��� � � � � � � � � � � �� � �� � � � � �� � � � � � �� � � � � �� � � � � � �� � � � � �� � � � � � � � � � � � � � �� � � � � �� � � � � �� � � � � � �� � � � � �� � � � � � �� � � � � �� � � � � �� �� � � � � � � � �������� � �

���

v v �

v v �

����

���� 50 5 22 1 2 2

2 1 6 6

a m na l a mm c l c l

a a l a l

� � � ��� � � � � � � � � � �� � �� � � � � �� � � � � �� � � � � �� � � � � �� � � � � �� � � � � � � � � �� � � � � �� � � � � �� � � � �� � � � � � �� � � � � �� � � � � � �� � � � � �� � � � � �� � � � � � � ����

de modo que disponemos de 6 ecuaciones con 6 incógnitas (a, b, c, l, m, n), que una vez resuelto da:

6 4 2 0 1 2a b c l m n� � � � � � � b) Las velocidades de los tres puntos y la velocidad angular del sólido en ese instante son

A B C

6 4 5 02 , 0 , 2 , 1 , con = 5

6 5 6 2�

� � � � � � � �� � � �� � � �� � � �� � � �� � � �� � � � � �� � � �� � � �� � � �� � � �� � � �� � � �� � � �� � � �� � � � v v v �

c) El ERID tiene la dirección del vector velocidad angular y pasa por un punto E tal que

O2

0 6 101 1OE 1 2 125 52 6 6�

� � � � � �� � �� � �� � � �� � �� � �� � � � �� � �� � �� � �� � �� � �� � �� � �� � ��� � �

� v����

de modo que las ecuaciones del EIRD son 5 10 5 12 5 6

0 1 22

2 6x

y zx y z ��� � � �� � �

�� ����

- 33 -

Física Universitaria: Problemas de Física Cinemática del sólido rígido. M05.3

3. Sobre un disco hay identificados tres puntos A, B y C cuyas coordenadas respecto a un sistema de referencia son A(0,0,0), B(5,3,0) y C(6,2,0) en cm. Sabiendo que se trata de un movimiento plano y que en un cierto instante es (vA)x = 4 cm/s, (vB)y = -3 cm/s y (vC)x = 16 cm/s, determinar completamente las velocidades de los tres puntos y la velocidad angular del disco en ese instante.

Puesto que el movimiento es plano y los tres puntos están contenidos inicialmente en el plano z = 0, con componentes de velocidad contenidas en dicho plano, serán nulas las componentes de velocidad de cada uno de los tres puntos en la dirección del eje z. Por el mismo motivo, el vector velocidad angular será perpendicular al plano del movimiento, por lo que tan solo tendrá componente sobre el eje z. Así, podemos escribir:

A B C

4 16 03 0

0 0 0

ba c

� � � � � � � �� � � �� � � �� � � �� � � �� � � �� � � � �� � � �� � � �� � � �� � � �� � � �� � � �� � � �� � � �� � � � v v v �

Relacionamos entre sí las velocidades de los tres puntos:

B A

C A

4 0 5 4 33 4

AB 3 0 3 3 55 3

0 0 0 0 0

4 0 6 4AC 9 0 2

0 0 0

b bb

a aa

c ca

''

''

'

'

� �� � � � � � � � � � � � ��� � � � � � � � � � � � � � � � � � � �� � � � � � � ��� � � � � �

� � � � � ��� � � � � � � �

� � � � � � � � � � � � � �� � � � � � � � � � � �

v v �

v v �

����

���� 22 4

9 66 9

0 0

ca

a

''

''

�� � � � ��� � � � � �� � � ��� �

� �

Resolviendo este sistema de cuatro ecuaciones con cuatro incógnitas resulta: a = 27cm/s, b = 22 cm/s, c = -9 cm/s y � = - 6 rad/s, de modo que tenemos:

A B C

4 22 16 027 cm/s 3 cm/s 9 cm/s 0 cm/s0 0 0 6

� � � � � � � �� � � �� � � �� � � �� � � �� � � �� � � � � �� � � �� � � �� � � �� � � �� � � �� � � �� � � �� � � ��� � � � v v v �

Determinación de la posición del C.I.R.:

A2

0 4 162 4.51 1OI AI 0 27 24 2 / 3 cm36 366 0 0 0�

� � � � � � � �� � � �� � � �� � � � �� � � �� � � �� � � � � � � �� � � �� � � �� � � �� � � �� � � �� � � �� � � �� � � ��� � � �

� v��� ���

A(0,0,0)

B(5,3,0

C(6,2,0)

vA vB

vC

I (CIR) x

y

- 34 -

Física Universitaria: Problemas de Física Cinemática del sólido rígido. M05.4

4. El rotor de un generador eléctrico está girando a 200 r.p.m. cuando el motor se apaga. Debido a efectos de fricción, la aceleración angular del rotor, en rad/s2, después de que se apaga el motor viene dada por la expresión ) = �0.01', donde ' es la velocidad angular en rad/s. ¿Cuántas revoluciones gira el rotor hasta que se detiene?

Datos: -10

200 10 10con 0.01s y r.p.s. = r.p.s. 2 =20.94 rad/s60 3 3

k k� � � �� � � � � �

A partir de la relación dada entre la aceleración y velocidad angulares, escribimos la ecuación diferencial del movimiento:

d d d d d dd d d d

k k kt t� � � �

� � � � �� �

� � � � � � �

cuya integración nos conduce a

000 0d d kk k

� �

�� � � � �� ��� � � � � � �� �

Cuando el rotor se detenga será '=0, de modo que el ángulo girado por el rotor hasta ese instante vendrá dado por

0103

0 -1

r.p.s.rev.

10000 =0.01s 3

333.33k

k �� � �� � " � ��

Otro método (más largo, pero más completo) Procedemos a una primera integración para determinar la velocidad angular en función del tiempo:

0 00

0d d d lnd

et ktk k t kt

t�

� � ��

� ���

�� � � � � � �� �

Este resultado nos indica que se necesita un tiempo infinito (?) para que se detenga el rotor. Una nueva integración nos permite obtener el ángulo girado en función del tiempo:

�00 00 0

0

0

d e d e d ed

1 et

tkt kt kt tktt k k

� ��� �

���� � � �� � � � � �� �

A partir de esta última expresión determinamos el ángulo girado cuando transcurra un tiempo suficientemente largo para poder considerar que el rotor ya se ha detenido.

1030

-1

r.p.s.rev.

1000=0.0

331

3s

333

.tk�

� ! " � ��

- 35 -

Física Universitaria: Problemas de Física Cinemática del sólido rígido. M05.5

5. Los extremos de una barra de longitud L deslizan sobre dos guías rectas perpendiculares entre sí, sin perder contacto con las mismas, de modo que el extremo B de la barra posee una velocidad vB constante, alejándose del punto de unión de las dos guías. Determinar: a) La trayectoria descrita por el punto medio de la barra. b) La velocidad y la aceleración del otro extremo de la barra en función del ángulo � indicado en la figura.

a) El movimiento del punto medio de la barra viene descrito por el vector

senOM cos OM cte.

2 20

L L��

� ��� �� �� � �� �� �� ����

���������

de modo que su trayectoria es circular, con centro en O y radio L/2. b) Se trata de un movimiento plano en el que se

determina fácilmente la posición del CIR (centro instantáneo de rotación) trazando las perpendiculares a las guías en los extremos de la barra. Tenemos

B

A B

B BA B

cosIA IB IA sen

cosBg

It

vv v L

Lv vL

vv

��

���

��� ����� � ��� � � �����

La aceleración del extremo A la calculamos derivando con respecto al tiempo la expresión de su velocidad:

�2

A B B BA B B 2 2

2

3

BA 3

d d d tg dtgd d d cos d co

cos

s cosv v v va v vt t t t L

vaL

� �� �

� �

# �

� � � � � �

A

B vB

A

B vB

I

M �

L

L sen�

L co

s�

O

vA

- 36 -

Física Universitaria: Problemas de Física Cinemática del sólido rígido. M05.6

A BI

R 0v

1v

6. Para que vire un tractor que se mueve con una velocidad v0 = 18 km/h, el tractorista frena una de las orugas de modo que el eje de la rueda motriz de ésta comienza a avanzar con velocidad v1 = 14 km/h. La distancia entre las orugas es D = 1.5 m. a) Determinar el radio de la trayectoria que describe el centro del tractor. b) ¿Cuánto tarda el tractor en dar media vuelta?

a) Designamos por R el radio que buscamos y por ' la velocidad angular asociada al movimiento circular del tractor. Puesto que conocemos las velocidades de dos puntos del tractor, A y B, y éstas son paralelas, el CIR se encuentra en un punto I de la perpendicular común a ambas velocidades en A y B. Por consiguiente, podemos escribir:

00

11

( )2 2

( ) 22

Dv R DRvDD v Rv R

��� � �� ��� $ ��� �� � �����

de modo que

0 1

0 1

618 14 1.52 18 1

m4 2

v v DRv v

� �� ��� �� � �� �� �� � ��

b) La velocidad del tractor será

0 1 18 14 16 km/h = 4.44 m/s2 2

v vv � �� � �

y describe un arco cuya longitud es 6 18.85 ms R� �� � �

por lo que empleará: 18.854.44

4.2 sstv

� � �

- 37 -

Física Universitaria: Problemas de Física Cinemática del sólido rígido. M05.7

7. El extremo superior de la varilla AB desliza a lo largo de una guía vertical (vide figura), en tanto que la varilla no pierde contacto en C con el apoyo. a) Determinar el valor del ángulo � al que corresponde una velocidad horizontal para el extremo libre, B, de la varilla. b) En ese instante, calcular las velocidades de los puntos B y C en función de la velocidad del punto A.

Método gráfico. a) Puesto que conocemos las direcciones de las velocidades de los puntos A, B y C pertenecientes a la varilla, trazando las perpendiculares a éstas en los correspondientes puntos, determinamos el CIR (Centro Instantáneo de Rotación) correspondiente al movimiento plano

de la varilla, ubicado en el punto I de intersección de las tres per-pendiculares, tal como se indica en la figura. Una vez que hemos ajustado el dibujo a las condiciones impuestas por el enunciado del problema, podemos determinar el ángulo �0 a partir de simples consideraciones geométricas. Considerando los triángulos �ADC, �ACI y �AIB, sucesivamente, encontramos la siguiente relación entre a, l y �0:

� �

20 0 0 0

2 30 0 0

ACcos AIcos cos AIcos

cos cos cos

a

l l

� � � �

� � �

� � � �

� �

De modo que 1/3

0cos al

�� ���� �� ���

b) El Teorema de las Velocidades Proyectadas (condición cinemática de rigidez) nos permite determinar fácilmente los módulos de las velocidades de los puntos B y C:

B 0 A 0 B A 0 C A 0cos sen tg senv v v v v v� � � �� � �

Método analítico (vectorial):

Determinamos analíticamente las velocidades de los puntos C y B y tenemos en cuenta que C AC 0� �v����

y que vB es horizontal.

0

C A A 0 A

0

C A 02 2 2

0 A

0 0 tgAC 0 tg

0 0 0

tg 0AC tg 0

0 0 tg

a av a a v

a aa v a

a a av

� �� �

� �� �

� � �

� � � � � � � �� � � �� � � �� � � �� � � �� � � �� � � � � � � � � �� � � �� � � �� � � �� � � �� � � �� � � �� � � �� � � �� � � �

�� � � �� �� �� �� �� �� � � � � �� �� �� �� �� �� �� �� �� � � � �

v v �

v

����

���� �2

A 0 20

0 1 tgcos

av a �� �

��� �� �� � � � ��� �� �� ���

0 0 B B 0B A A 0 0 A

A 0

0 0 cos sen senAB 0 sen cos 0 cos0 0 0 0

l l v v lv l l v v l

� � � � �� � �

� ��

� � � � � � � � � � �� � � � � �� � � � � �� � � � �� � � � � �� � � � �� � � � � � � � � � � � � � � � �� � � � �� � � � �� � � � � ��� � � � �� � � � � �� � � � �� � � � � �� � � � � v v �

����I

gualando las expresiones de vA, tenemos 30 02

0

cos coscos

a all

� ��

� �

A

a C

B

I �

D

l vC

vA

vB

- 38 -

Física Universitaria: Problemas de Física Cinemática del sólido rígido. M05.8

8. La varilla AC que se muestra en la figura tiene un movimiento plano tal que su extremo A desliza a lo largo de un eje horizontal, en tanto que la varilla pasa por un pasador fijo y orientable (B) situado a una distancia fija h del eje horizontal. Supongamos que el extremo A de la varilla se mueve con velocidad constante vA de izquierda a derecha. a) Expresar la velocidad angular (') de la varilla en función del ángulo � que se indica en la figura. b) Calcular la velocidad y aceleración del punto de la varilla que se encuentra en B en función de dicho ángulo�, expresando sus componentes en la base vectorial indicada en la figura.

a1) Método gráfico. Determinamos la posición de CIR, tal como se indica en la figura. Entonces

2

2

AB / cosIAcos cos cos

senIB AB tg tgcos cos

h h

h h

�� � �

�� �

� �

��� � � ������� � � �����

2A

B

B

A

A

IA IBo

eB sI

c s

n

vhv

v v

v � �

��

������ � ��� � ��

��

a2) Método analítico. Determinamos la velocidad de B a partir de la de A:

B A AB A

B A BB

sen 0 tg senAB cos 0 0 tg

cos tg0 0 0 0

v v h v h v v hv h h

v h

� � � � �� � �

� � ��

� � � � � � � � � �� �� � � � � �� � � � � � ��� � � � �� � � � � �� � � � �� � � � � � � � � � � � � � �� � � � �� � � � �� � � � � � �� � � � �� � � � � ��� � � � �� � � � �� � � � � v v �

����

B A 2A AB A

B

sentg cos y sen

cos tg tgv v h v h v v vv h h h

� � �� � � �

� � � � �

� � �� �� � � ��� ���

b) Determinamos la aceleración de B a partir de la de A: B A�a a AB AB( )� � � � �� � ���� ���

A A

2AAB

AB

0 20 con ( 2sen cos ) sen cos

2sen cos0 tg 10 tg 2sen

0 0 000

v vh h

hh h v

�� � � � � �

�� ��

� � � ��

� ��� �� �� � � � �� �� �� ���� � �� � � � � �� ��� � �� � � �� � � �� � � �� � � �� � � � �� � � �� � � �� � � �� � � � �� � �� � �� � � �� � � ��� � � �

���� � �����

� ��

���� �

���

2 2A

B A

AB

tg 1tg

0 0sen cos0 1 tg

( ) 0 ( ) tg 1 cos0 0 0

3sen cos

hh h

h h v

v

�� �

� ��� � � � �

��

� � � � ��� � �� �� � �� �� � �� � �� �� � �� �� � �� �� � �� � �� � � � � �� � � � � �� ��� � �� � � �� � � �� � � �� � � �� � � � � � � � �� � � �� � � �� � � �� � � � �� � �� � �� � � �� � � ��� � � �

# �

� �

a

���

32

2 2 2 2 4A

3sen cos2sen cos 2sen cos cos

0 0

vh

� � �� � � � �

� �� � ���� �� �� �� �� �� � �� �� �� � �� � � �� �� ��� �

x

y

A

B

h

O

C

x

y

A

B

h

O

C

I

vA

vB

- 39 -

Física Universitaria: Problemas de Física Cinemática del sólido rígido. M05.9

9. La varilla AC que se muestra en la figura tiene un movimiento plano tal que su extremo A desliza a lo largo de una circunferencia, en tanto que pasa por un pasador fijo y orientable (B). Supongamos que el extremo A de la varilla se mueve con celeridad constante. a) Expresar la velocidad angular (') de la varilla en función del ángulo � que se indica en la figura. b) Calcular la velocidad y aceleración del punto de la varilla que se encuentra en B en función de dicho ángulo�, expresando sus componentes en la base vectorial indicada en la figura.

Trazamos las normales a las velocidades en A y B, para determinar la posición del CIR (punto I). El triángulo ABI es rectángulo, por lo que podemos afirmar que el CIR está situado sobre la circunferencia que sirve de guía al extremo A de la varilla.

a) Una vez localizado el CIR, podemos escribir:

A BIA 2IA IBIB=2 sen

R v vr

��

�� �� � �����

de donde se sigue

AB AA

IBcte.A

enI

s2v v v vR

� �� � ��

b) Expresamos la velocidad del punto B de la varilla en función de sus componentes:

B2

B A

B cossen0

sen cossen

0v

vv

� ����� � ��� �� �� �� �� �� ��

��� �� �� �� �� ��� ����v

Para determinar la aceleración de B tendremos en cuenta que

B A AB ( AB)� � � � � �a a � � ����� ����

y teniendo en cuenta que

2A A

A

cos 2 cos 0sen 2 AB 2 cos sen 0 0

20 0 1

v vRR R

� �� � �

� � � � � �� � �� � �� � �� � �� � �� � � �� � �� � �� � �� � �� � �� � �� � �� � �� � � a � �

�����

resulta 2

2 2A A A A

B

2cos 2 0 0 cos 2cos 2 cos2sen 2 2 cos 0 0 sen 2sen 2 sen cos

2 2 2 20 1 1 0 0

v v v vRR R R R

� � � �� � � � � �

� �� �� � � � � � � � � ��� � � �� � � �� � �� � � � �� � � � �� � � � �� �� � � � � � %� � � � �� � � � �� � � � �� � � �� � � �� � � �� � � �� � � � �� � � � ��� � � � � � � �

a

que también podemos escribir en la forma

2A

B

cos 2 1 33 sen 24 0

vR

��

� �� �� �� �� %� �� �� ���� a

R

A

B �

y

x

CIR

y

x

A

B �

� 2�

' R

R

I

vB

vA

- 40 -

Física Universitaria: Problemas de Física Cinemática del sólido rígido. M05.10

10. La barra BC de la figura está articulada con la manivela AB y desliza por el interior de la guía pivotante D como se indica en la figura. En el instante representado el punto D de la barra BC se mueve a razón de 375 mm/s. Determinar la velocidad angular de la manivela en ese instante.

El punto B recorre una trayectoria circular alrededor con centro en A, por lo que su velocidad será perpendicular al vector AB; por tanto

B B B B

sen 75 / 125 0.6

cos 100 / 125 0.8

sen 0.6cos con 0.8

0 0v v

� �

� �

� � � �� �� ��� ��� �� ��� �� �� ��� �� �� ��� �� ���� �� �� � v v

La velocidad del punto D está dirigida en cada instante en la dirección de la barra BD, tal como se indica en la figura, de modo que será:

D

2 2

2 2

D D D

375 mm/s

sen 75 / 75 300 0.242

cos 300 / 75 300 0.970

cos 363 mmsen con 90 s0 0

v

v �

��

� � �

� � �

��� � � ��� �� ��� �� ��� �� �� �� ��� �� �� ��� �� ��� �� �� � ���

v v

Aplicando la condición cinemática de rigidez (teorema de las velocidades proyectadas) para los puntos B y D de la barra BC, se sigue que

� � � �

B D B

B

0.6 300 363 300DB DB 0.8 75 90 75

0 0 0 0

0.6 300 0.8 75 364 300 90 75

v

v

� � � � � � � �� � � �� � � �� � � �� � � �� � � �� � � �� �� � � �� � � �� � � �� � � �� � � �� � � �� � � �� � � �� � � � � �� � � � � � � � � � ��

v v���� ����� � � �

de donde resulta

B B

0.6 299mm mm483 483 0.8 = 386 s s0 0

v� � � �� �� �� �� �� �� �� � � �� �� �� �� �� �� �� �� �

v

125mm

100mm 300mm

375 mm/s

C

D

B

A

125mm

100mm 300mm

375 mm/s

C

D

B

A � �

x

y

- 41 -

Física Universitaria: Problemas de Física Cinemática del sólido rígido. M05.11

11. La manivela CB oscila en torno a C describiendo un arco limitado y haciendo que la manivela OA oscile en torno a O. Cuando la biela AB pasa por la posición representada, en que CB está horizontal y OA vertical, la velocidad angular de CB es 4 rad/s en sentido antihorario. Hallar las velocidades angulares de OA y AB en ese instante. Datos: a = 100 mm, b = 225 mm, c = 75 mm y d = 50 mm.

manivela BC (rotación pura en C): Determinamos la velocidad del extremo B,

B BC 4 75 300 mm/sv c�� � � �

biela AB (rototraslatorio): Determinamos el CIR gráficamente, como se indica en la figura. Entonces, escribimos

A BAB

AAB

AAB A

300100 50 22

2 rad/s

5 75

100 mm/s50

v va d b c

v

v v

� � � �

� � � �

� � ��

manivela OA (rotación pura en O): A partir de la velocidad del extremo A determinamos la velocidad angular de la manivela,

AA OA OA 1 rad/s100

100vv aa

� �� � � � �

Con notación vectorial

B C�v v CB CB CB

CB

CB0 00 0 con 4 rad/s

0 0

cc� �

� � � � � ��� � �� � �� � �� � �� � �� � � � � � �� � �� � �� � �� � �� � �� � �� � �� � �� � � �

���

B CB 75 4 300 mm/sc�# � � � � � � �v j j j

AB

A B AB CB CB AB

AB

A O

BA0 0 ( ) ( )

0 ( )0 0 0

b c a dc a d c b c

�� � �

� � � � � �� � � �� �� � �� � � ��� � �� � � ��� � � �� � � � � � � � � � � �� � � �� � � �� � � �� � � �� � �� � � � �� � �� � � � � � �

v v �

v v

���

OA

OA

OA

OA0 00 0

0 0

aa

����������� � � � � � �� � � �� � �� � � �� � �� � �� � � � � �� � �� � �� � � �� � �� � � �� � �� � �� � � �� � � ��

����

OA ABAB OA

CB ABAB CB

100 50 ( 2) 1 rad/s( ) 100( ) 0 75 4 2 rad/s

225 75

a da d a a

c b c cb c

� �� �� �

� �

� � ��� � � � � ���� � � �� �� �� �� �� � � � � �� �� � � � � ��� � ���

AB A2 rad/s 100 mm/sv� � �# �

a

b

d

c

O

A B C

a

b

d

c

O

CIR B C

A

x

y

- 42 -

Física Universitaria: Problemas de Física Cinemática del sólido rígido. M05.12

12. Una barra de 2 m de longitud está doblaba en escuadra y dotada en sus extremos de correderas articuladas. En el instante representado en la figura, la corredera A se mueve hacia la derecha con una velocidad de 5.7 m/s. Hallar la velocidad angular de la escuadra y la velocidad de la corredera B.

En primer lugar, por consideraciones puramente geométricas, determinamos los ángulos significativos, tal como se indica en la figura.

La velocidad de la corredera B se determina inmediatamente a partir de la condición cinemática de rigidez (teorema de las velocidades proyectadas) aplicada a la barra doblada. En efecto,

A B B Acos10º 0.98cos10º cos30º 5.7cos30º 0.87

6.48 m/sv v v v� � � � �

Puesto que las velocidades de los extremos de la barra tienen las direcciones de las respectivas guías, determinamos la posición del CIR (punto I) como punto intersección de las normales a dichas velocidades. En el triángulo ABI, aplicando el teorema de los senos, tenemos:

AI AB sen60º 0.87AI AB 2 1.91 msen60º sen40º sen40º 0.64

� � � �

Ahora podemos determinar la velocidad angular de la barra en el instante indicado en la figura:

AA

5.7IA1.91

2.99 raA

d/sIvv � �� � � �

1m 1m 90º

55º 40º

B

A

90º

55º 40ºB

A

1 m1 m

30º

10º

40º

1.91 m

2.17 m

vA

vB

I (CIR)

'

60º2 m 45º

- 43 -

Física Universitaria: Problemas de Física Cinemática del sólido rígido. M05.13

A

B

C

R

I

� 2R

2R

'

��

O x y

13. Una varilla AB está apoyada sobre un cilindro de radio R = 1 cm de modo que puede deslizar a lo largo de una guía tangente a dicho cilindro, como se indica en la figura. La longitud de la varilla es cuatro veces el radio del cilindro. En el instante en que el centro C de la varilla se apoya en el cilindro, la velocidad del punto A es 10 cm/s. Calcular, en dicho instante, las velocidades de los puntos B y C y la velocidad angular de la varilla.

A partir del triángulo OAC� : tg 0.5 53.13º2 2

RR

��

� ��� � � ��� ���

Aplicamos la condición cinemática de rigidez (teorema de las velocidades proyectadas) a los puntos A, B y C:

B AA B C

C

10 cm/scos cos

10cos53.13º 6 cm/sv v

v v vv

� �� � ���� � �� � ���

Método geométrico Determinamos el CIR (I) como se indica en la figura. A partir del CIR, determinamos las velocidades de los puntos de la varilla:

A B CIA IB ICv v v� � �� � �

Con IA IB AC/sen 2 / sen 53.13º 2.5 cm

IC AC / tg 4 / tg53.13º 1.5 cm

�� � � � ���� � � ���

De modo que A

A B

C

/ IA 10 / 2.5 4 rad/s10 cm/s

4 1.5 6 cm/s

vv vv

��� � � ���� � ����� � � ���

Método analítico A partir de las velocidades de A y C calculamos la velocidad angular:

C A A

C A C

C A

cos 0 2 cos 2 senAC sen 0 0 2 sen 2 cos

0 0 0 0

/ cos 2 tg 10 / cos53.13º 2 4 1 tg53

v v R v Rv R R

v v R

� � � �� � � �

� � �

� � � � � � � � � �� �� � � � �� � � � �� � � � �� � � � �� � � � �� � � � � � � � �� � � � �� � � � �� � � � �� � � � �� � � � �� � � � �� � � � �� � � � �� � � � �

� � � � � � �

v v �����

C

.13º 6 cm/s6tg tg53.13º 4 cm/s

2 2 1vR

� �

� ������ � � ��� ��

Calculamos la velocidad de B a partir de la de A:

A A

B A

B

0 4 cos 4 senAB 0 0 4 sen 4 cos

0 0 0

10 4 4 1 sen 53.13º 2.84 4 1 cos53.13º 9.6

0 0

v R v RR R

� � �� � �

� � � � � � � �� �� � � �� � � �� � � �� � � �� � � �� � � � � � � �� � � �� � � �� � � �� � � �� � � �� � � �� � � �� � � �� � � �

� � � �� � � � ��� ��� ��� � � � � � �� ��� ��� ���� �� �

v v �

v

����

2 2B

9.6tg 3.43

2.873.74º 180º 253.74º

2.8 9.6 10 cm/scm v

s � ��

� � �

� � �

�� � � �� �� �� ��� �� �� ��

- 44 -

Física Universitaria: Problemas de Física Cinemática del sólido rígido. M05.14

14. Un cilindro de radio R rueda sin deslizar sobre una superficie plana y horizontal, con una velocidad angular ' constante. Determinar: a) el eje instantáneo de rotación; b) la velocidad y la aceleración de los puntos del eje del cilindro; c) ídem de un punto cualquiera del cilindro de coordenadas (x,y,0); d) ídem de los puntos del cilindro que instantáneamente están en contacto con el plano.

Se trata de un movimiento plano, por lo que analizamos el movimiento en el plano xy indicado en la figura, con � = -�k.

a) El eje instantáneo de rotación coincide con la generatriz del cilindro que en cada instante está en contacto con la superficie sobre la que rueda sin deslizar (puntos de velocidad nula). b) Calculamos la velocidad del punto O a partiendo del punto I (CIR):

O OIOv R R� � �� � �v i

La aceleración del punto O será nula, por ser constante (en módulo y dirección) la velocidad de dicho punto; esto es,

O 0�a c) Consideremos un punto P cualquiera del cilindro, de coordenadas (x,y,0). Calculamos su velocidad y aceleración partiendo del punto O:

�P O

P O

0OP 0 0

0 0 0

R yR xy x

���

� ��� � � � � � ��� � �� � � ��� � �� � � ��� � �� � � � � � � �� � � �� � � � �� � �� � � � �� � �� � � �� � � �� � �� ��� � � �

v v �

a a

���

OP�� ����

�2

2 20

OP 00 0 0

xy xx y y

��� � �

� ��� � � � � ���� � �� � ���� � ��� � ��� � ��� � � � � � � � � �� � ��� � ��� � �� � �� �� � �� � �� �� � �� � ��� � � ���

� ����

de modo que la aceleración del punto P está dirigida hacia el eje del cilindro1. d) Las coordenadas del punto I son (0,-R,0), de modo que sustituyendo en las expresiones anteriores tenemos

�2 2

I I

00 0

0 0

R RR R

�� � �

� �� � ��� ���� ���� �� � � � � ��� �� � � �� � � �� �� ��� � �

v a j

1 En general, por ser OP&����

en el movimiento plano, el vector �OP� �� ����

tiene la misma dirección y

sentido opuesto al OP���

, ya que �OP OP� � �� � � ���� ���

� � 2 2OP OP� �� � ���� ���

vO

O

P(x,y)

I

vP

y

x

R

aI

aP

- 45 -

Física Universitaria: Problemas de Física Cinemática del sólido rígido. M05.15

15. Un disco de radio R rueda sin deslizar, sobre una superficie plana y horizontal, con una velocidad angular ', constante en dirección y sentido y aceleración ) constante. Determinar: a) la velocidad y la aceleración del dentro del disco. b) Ídem de un punto P del cilindro diametralmente opuesto al de contacto del disco con la superficie plana. c) Ídem del punto I del disco que instantáneamente está en contacto con la superficie plana.

Se trata de un movimiento rototraslatorio plano consistente en una rotación � alrededor del eje del disco y una traslación horizontal de dicho eje. Por ser un movimiento plano, basta con analizar el movimiento en el plano z = 0, con � = -�k y ) = -)k. Puesto que rueda sin deslizar, punto del disco que instantáneamente está en contacto con el plano es el centro instantáneo de rotación (CIR). a) Calculamos la velocidad del punto O a partir del punto I (CIR)

O I�v v

0O

0 0IO 0 0

0 0

d d 0 0d d 0 0

RR

R R

t t

� �

� � � � � �� � �� � �� � �� � �� � �� � � � �� � �� � �� � �� � �� � �� � �� � �� � ��� � � � � � �� �� �� �� �� �� � �� �� �� �� �� �� �� �� �� �

va

���

b) Para el punto P tenemos

P O

P O

0 0 2OP 0 0 0 0

0 0 0 0

0 0d OP OP 0 0d 0 0

R R R RR

RR

t

� � � �

� � � � � � � � � ��� � � � �� � � � �� � � � �� � � � �� � � � �� � � � � � � �� � � � �� � � � �� � � � �� � � � �� � � � �� � � � �� � � � �� � � � ��� � � � �

� � � �� �� �� �� �� �� � � � � � � � �� �� �� �� �� �� �� �� ��� �

v v �

�a a � �

���

��� ���2

200 0

0 0

RRR���

� �� � � � � �� ��� � �� � � �� � � �� � � �� � � �� � � �� � � �� � � �� � � �� � � � �� � �� � �� � � �� � � �� �� � � �

c) Aunque la velocidad del punto I del disco es instantáneamente nula, su aceleración no es nula. La calculamos a partir de la aceleración del punto O:

�I Od OI OId

0 0 0 0 0 0 00 0 0 0 0 00 0 0 0

tR

R R�

� � � �

� � � � � � �

� �� � � � � � � � � � � � � � �� � � � � � �� � � � � � � �� �� � � � � � �� � � � � � � �� � � � � � �� �� � � � � � � � �� � � � � � �� � � � � � �� � � � � � �� � � � � � �� �� � � � � � �� � � � � � �� � � � � � �� � � � � � �� � � �� � � � � � � �� ��

�a a � ���� ���

20

00 0

RR

��

� �� � �� ��� �� �� � �� �� � �� �� � �� � �� � �� � � �� �� �� � �� � �� � �

y

x

I

P

O

�,)

vP

vO

aI

aO

AP

y

x

P

O �,)

- 46 -

Física Universitaria: Problemas de Física Cinemática del sólido rígido. M05.16

16. Un disco de radio r está girando alrededor de su eje de simetría con velocidad angular � y aceleración angular �. Simultáneamente, el disco está girando, con velocidad angular constante �, alrededor de un eje fijo en el espacio que está contenido en el plano del disco y es tangente al perímetro de éste en un punto Q. a) Determinar la velocidad y aceleración del punto P del perímetro del disco diametralmente opuesto al punto Q de tangencia. b) Ídem de un punto genérico de la periferia del disco.

El disco está sometido a dos rotaciones simultáneas: una rotación intrínseca � alrededor de su eje de simetría de revolución y una rotación de precesión �. Elegido un referencial como el indicado en la figura, podemos escribir:

res

00 0 00

� �� � � � � �� � �� � �� � �� � �� � � � �� � �� � �� � �� � �� � �� � �' '� � � � � � � �

0 00 CP QP 20 0 0

r r�� � � � � �� � �� � �� � �� � �� � �� � �� � �� � �� � �� � �� � �� � �

���� ���

La velocidad del punto P del disco se obtiene como la superposición o suma de las correspondientes a cada una de las dos rotaciones; i.e.,

P

0 0 0 2CP QP 0 0 2 0

0 0 0

rr r

r

� � � � � � � � � �� '� � � � �� � � � �� � � � �� � � � �� � � � � � � � �� � � � �� � � � �� � � � �� � � � �� � � � �� � � � �'� � � � � v � �

��� ���

Determinamos la aceleración de P a partir de la aceleración del punto C (centro del disco); i.e.,

�resP C res res

d CP CPdt

� � � � � ��a a � �

��� ���

La aceleración del punto C es la aceleración centrípeta asociada a una trayectoria circular de radio r con velocidad angular � constante:

2C

0

0r

� ��� �� �� �'� �� �� �� a

Calculamos la derivada temporal de la velocidad angular resultante teniendo en cuenta que � precesa con velocidad angular �, de modo que

res res0 0 0d dd d 0 0 0 0 CP 0

d d d d0 0 0 0 0r

t t t t r

� � � �� �

� � � � � � � � � � � � � �� � � � � � �� � � � � � �� � � � � � �� � � � � � �� � � � � � � � � � ' � � ' � �� � � � � � �� � � � � � �� � � � � � �� � � � � � �� � � � � � �� � � � � � �'� � � � � � �

� �� � � � ����

Calculamos el último termino:

� 2 2res res res

00CP 0 0 0

0 0

rr r r

r

� ��

� �� � � � � � � ��' ��� � � �� � � � �� � � � �� � � � �� � � � � � � � � �'� � � � �� � � � �� � � � �� � � � �� � � �� � � � � �' '� � � � � � � �

���

Finalmente, tenemos

�2 2 2 2 2P

00 000 2

0 0r r r r

r r� �

� �

� �� � � �� � �� � �� ��� �� � ��� �� �� �� �' � � � �' � � � '��� � ��� �� �� � ��� ��� �� �� � ��� � � � a

y

z

P Q

� �, �

C

- 47 -

Física Universitaria: Problemas de Física Cinemática del sólido rígido. M05.17

Para un punto genérico El disco está sometido a dos rotaciones simultáneas: una rotación intrínseca � alrededor de su eje de simetría de revolución y una rotación de precesión �. Elegido un referencial como el indicado en la figura, podemos escribir:

res

00 0 00

� �� � � � � �� � �� � �� � �� � �� � � � �� � �� � �� � �� � �� � �� � �' '� � � � � � � �

0 00 CG cos QG (1 cos )

sen0 senr r

rr

�� �

��

� � � � � �� � �� � �� � �� � �� � � �� � �� � �� � �� � �� � �� � �� � � �

���� ����

La velocidad del punto G del disco se obtiene como la superposición o suma de las correspondientes a cada una de las dos rotaciones; i.e.,

P

0 0 0 (1 cos )CG QG 0 cos 0 (1 cos ) sen

sen0 sen cos

rr r r

rr r

� �� � � �

�� � �

� � � � � � � � � ��' �� � � � �� � � � �� � � � �� � � � �� � � � � � � � � � �� � � � �� � � � �� � � � �� � � � �� � � � ��� � � �' � � � � � v � �

���� ����

Determinamos la aceleración de P a partir de la aceleración del punto C (centro del disco); i.e.,

�resP C res res

d CP CPdt

� � � � � ��a a � �

��� ���

La aceleración del punto C es la aceleración centrípeta asociada a una trayectoria circular de radio r con velocidad angular � constante:

2C

0

0r

� ��� �� �� �'� �� �� �� a

Calculamos la derivada temporal de la velocidad angular resultante teniendo en cuenta que � precesa con velocidad angular �, de modo que

res

res

0d d d 0 0 0 0d d d 0 0 0

0 send CG cos send 0 sen cos

t t t

rr r

t r r

� � ��

� � �� � � �

� � �

� � � � � � � �� � � �� � � �� � � �� � � �� � � � � � � � � � ' � � � �� � � �� � � �� � � �� � � �� � � �'� � � � � � � � � �'� � �� � �� � �� � �� � ' � � �� � �� � �� � �� � �� � �� � �� � �

� � � � � �

� ����

Calculamos el último término:

� �2 2res res res

2

sen0 cosCG 0 cos 0 sen cos

sen cos sen

rrr r rr r r

� �� � �� � � � �� � � � �

� �� '� � � � � � � ��' ��� � � � �� � � � �� � � � �� � � � �� � � � � � � � � � ' �� � � � �� � � � �� � � � �� � � � �� � � � �� � � �' '� � � � �� �� � � �

����

Finalmente, tenemos

� �2 2 2 2 2P

22

sen0 0sensen cos sen 1 cos cos

0 cos cos sensen

rrr r r r r r

r r rr

� �� �� � � � � � � � �� � � � � �� �

� � � �� � � '� �' �� �� �� � �� ��� ��� �� ���� �' � � � � ' � � � �' � ��� �� ��� �� ��� ��� �� �� �� � ��� ��� �� �� a

y

z

G

Q

� �, �

C �

- 48 -

Física Universitaria: Problemas de Física Cinemática del sólido rígido. M05.18

17. La hélice de un avión gira a razón de 6000 r.p.m., en tanto que el avión tiene una velocidad horizontal, en línea recta, de 360 km/h. Determinar: a) El tipo de movi-miento que realiza un punto de la hélice distante 1 m del eje de la misma; b) la velocidad y aceleración de dicho punto.

a) Se trata de un movimiento helicoidal cuyo eje es el eje de rotación de la hélice (eje x, en la figura), que constituye el EIRD (Eje Instantáneo de Rotación y Deslizamiento).

b) La velocidad y la aceleración del punto P se calculan a partir de la velocidad y aceleración del punto O (en el eje de la hélice), de modo que

P O

P O

OPd OP ( OP)dt

�� � � ������ � � � � � �����

v v ��a a � �

���

��� ���

con

O

O

00 0 OP 00 0

d0 0d

v

l

t

�� � � � � �� � �� � �� � �� � �� � �� � �� � �� � �� � �� � �� � �� � �� � �� � �� � �

� �

v �

�a

���

De modo que

P

P2

0 00 0 0 00 0 0 0 0

0 0( OP) 0 0

0 0

v v vl l

l

ll

�� �

��

� � � � � � � � � � � �� � � � � �� � � � � �� � � � � �� � � � � �� � � � � �� � � � � � � �� � � � � �� � � � � �� � � � � �� � � � � �� � � � � �� � � � � �� � � � � �� � � � � �� � � � � � �� � � � �� �� �� �� �� �� � � � � � �� �� �� �� �� �� �� �� �� � ��

v

a � ����

���������� ��� ��� � �� � �� � �� � �� � ���

Y sustituyendo valores, con 26000 200 628 rad/s60�

� �� � � 1000360 100 m/s3600

v � �

resulta:

P P

2P

100628 m/s 636 m/s0

00 m/s

394384

v� � �� �� � �� � �� � ��� ��� ��� ���� � ��� � �� ��� �� �� � � �� � �� � ��� ��� ��

v

a

v x

z

y

P

vo x

z

y �

vP P

O

- 49 -

Física Universitaria: Problemas de Física Cinemática del sólido rígido. M05.19

18. Sobre un plano horizontal rueda sin deslizar un cono recto de sección circular, de generatriz l y semiángulo en el vértice �. Sea � la velocidad angular constante de rotación del cono alrededor del eje vertical indicado en la figura. Determinar: a) la velocidad angular � intrínseca de rotación del cono alrededor de su eje de simetría; b) el punto del cono cuya velocidad (con respecto al plano fijo) es máxima, así como la velocidad y aceleración de dicho punto.

Se trata de un sólido rígido sometido a dos rotaciones simultáneas: una rotación intrínseca � alrededor del eje de revolución del cono, al tiempo que este eje presenta una rotación � alrededor del eje z indicado en la figura. a) La generatriz OM del cono que en un instante dado está en contacto con el plano horizontal constituye el EIR (eje instantáneo de rotación) del cono. La velocidad del punto M perteneciente al EIR será instantáneamente nula, de modo que,

M 0sen sen

l lv l RR l

� � ��

� � � � � �

b) El punto P es el que presentará una velocidad máxima, por ser el más distante del EIR. Su velocidad y aceleración se calculan fácilmente a partir de la velocidad y aceleración del punto O (vértice del cono):

P O�v v R

P O

OP� �

a a

���

�RR R

d OP OPdt

� � � � �� � �

��� ���

con

R

R

0 0 0 0OP cos 2 cos 0 cos

sen 2 sen 0

d d d dd d d d

ll

t t t t

� � � � �� � �

� � � � � � � �� � � �� � � �� � � �� � � �� � � �� � � � � �� � � �� � � �� � � �� � � �� � � �� � � �� � � �� � � ��� � � �

� � � �

� � �

� � �� �

���

0 0 cos0 cos 0

sen 0

� �� �

� �

� � � � � �� � �� � �� � �� � �� � �� � � � �� � �� � �� � �� � �� � �� � �� � �� � ��� � � � �

de modo que

P R

RP R R

20 0 cos sen 2OP cos cos 2 0

0 sen 2 0

cos 0d OP OP 0 c

2 co

os 2d

s0

0 sen 2

0

lll

lt

l

l

� � �� � �

� ��

� � � � � �� � �� � �� � �� � �� � �� � � � � �� � �� � �� � �� � �� � �� � �� � �� � �� � �

� � ��� ��� ��� � � � � � �� ��� ��� �����

� ��� �� �� �� �� �� ���

v �

�a � �

���

��� ���

2

2 2

2

0 cos sen 2cos 0 ...0 0

0... cos sen 2

cos cos 2 cos sen

02coscot2 g

l

l

ll l

� � �� �

� � �� � � � � ��

� � � � �� � �� �� � �� �� � �� � �� �� � �� �� � �� �� � �� � �� � � � �

� ��� �� ��� � ��� �

� ��� �� ��� �� �� �� ��� � �� ���

Otro método....

xy

z

l

��R

M

P

O

C

l

R

- 50 -

Física Universitaria: Problemas de Física Cinemática del sólido rígido. M05.20

Movimiento absoluto = Movimiento relativo + movimiento de arrastre. Descomponemos todos los vectores en la base vectorial ijk asociada con el referencia absoluto o fijo xyz, con

rel arr

0 0cos 0sen� �� �

� � � �� �� �� �� �� �� � � �� �� �� �� �� �� �� �� ��� � ijk ijk

� � � �

Dado que ambas velocidades angulares son constantes en módulo y que hemos hecho coincidir los orígenes O y O’ de los referenciales absoluto (xyz) y relativo (x’y’z’), podemos utilizar las relaciones siguientes:

rel rel

arr arr

0 0 cos sen sen cos 2 senO P cos cos 2 0 ... 0

sen sen 2 0 0

0OP 0

ll ll

� � � � � �� � � �� � �

� � � � � � � �( �� � � �� � � �� � � �� � � �� � � �)� � � � � � �� � � �� � � �� � � �� � � �� � � �� � � �� � � �� � � �� � � �

� ��� �� �� � � � �� �� �����

ijk ijk ijk ijk

ijk

v �

v �

����

���

abs rel arr

2

0 cos 2cos 2 0sen 2 0

sen co 2 cos0

s 2 cos 200 0 0

0

lll

l l l l l

���

� � � ��

� � � �� �� �� �� �� �� �� �� �� �� �� �� �� �� �� �

� � � �� �� �� �� �� �� �# � � � � �� �� �� �� �� �� �� �� ��

� ��� �� �� �� �� �� ��� �

ijk ijk

ijk i k ijkj

v v v

2 2 2rel rel rel

2 2

arr arr arr

0 00 sencos 0 sensen 0 sen cos cotg

0 cos 20

ll l

l l

l

� �� � � � � � � � � �

� � � �� � � � � �� �� �� � � �� � � �� � � �� � � �� � � � � �� � � �� � � �� � � �� � � �� �� �� �� � � �� � � �� �� � � �� �

� ��� �� �� � � �� �� �� �����

ijk ijk ijk ijk

ijk

a � v

a � v 2

2Cor arr rel

abs rel ar

00 cos 20 0

00 sen 02 2 0 0 2 sen 2

0 0 0

l

ll l

� �� �

� �� � ���� �� �� �� �� �� �� �� �� � �� �� �� ��� �

� �� � � � � � ��� � �� � � �� � � �� � � �� � � �� � � � � � � �� � � �� � � �� � � �� � � � �� � �� � �� � � �� � � �� �� � � �

# � �

ijk ijk

ijk ijk ijk ijk

a � v

a a a 2 2 2 2

2

2 2r Cor

0 0cos 2 2 1 cos 2

cotgcotg

02coscotg

l l l ll

l � �� �

��

� � � � �

� � � ��� ��� �� �� ��� � ��� �� �� �� � � ���� � �

� ��� �� ��� �� �� �� ��� ijkijk ijk

a

x’ y

z

l

M

P

O

C

l

R

� z’

y’

x �

- 51 -

Física Universitaria: Problemas de Física Cinemática del sólido rígido. M05.21

19. Una moneda, de 1.5 cm de radio, rueda inclinada manteniendo un ángulo de 60º respecto al plano horizontal. En su movimiento, el punto de contacto con el plano horizontal describe sobre éste una circunferencia, de 0.75 cm de radio, cada tercio de segundo. Determinar las velocidades y aceleraciones del centro de la moneda A y del punto B de la periferia, en el instante en el que se encuentra en una posición diametralmente opuesta al punto de contacto con el plano horizontal.

La velocidad angular * vale 3 r.p.s.; o sea * = 3�2� = 6� rad/s a) Dado que r = R cos60º, el punto A permanece estacionario en la vertical del centro O de la

trayectoria circular descrita por el punto de contacto C; i.e.,

A A0 0� �v a

Además, el punto C de la moneda que en cada instante está en contacto con plano horizontal se encuentra instantáneamente en reposo, ya que nos dicen que la moneda rueda; esto es vC = 0. En consecuencia, todos los puntos del diámetro CAB se encuentran instantáneamente en reposo; i.e., dicho diámetro coincide con el eje instantáneo de rotación (EIR). En consecuencia, el punto B también se encuentra instan-táneamente en reposo; i.e.,

B 0�v

b) Determinamos la velocidad angular intrínseca de la moneda (') a partir de la condición de rodadura:

C0.750 3� rad/s1.50 2

rv r RR

� � � � � � � � �

R3 3 2 3 3 2

0 00 0sen 60º � / rad/s 0 rad/s � / rad/scos 60º 3� / 2 6� 9� / 2

� � � � �

� � � �� � � �� �� �� �� �� �� �� �� �� �� �� �� �� � � � � �� �� � � �� �� �� �� �� �� �� �� �� �� �� �� � � �� � � �� �

donde 'R es la velocidad angular total o resultante de la moneda. Determinamos la aceleración del punto B a partir de la del punto A (nula):

B A�a a RR R

d ( )d

AB ABt

� � � � �� � �

���� ����

2

2RR 0 rad/s

0

00 9 3d 0 3 3 2d 6 9 / 2

/t

��

� �

� � � �� � �� �� ��� � �� ��� � �� �� �� � � � �� �� � �� �� � � �� �� � � ��� �� ���� � � �

� � �

2B

2

2

2R

0-9 3 0d 0 3 / 4 81 / 4d 0 3 3 /

0200

4 27cm/s

13 15/ 4AB

ta

���

� � � � � �� � �� � �� � �� � �� � �� � ��� � � � � �� �� � �� � �� � �� � �� � �� �

� ��� �� �� �� �� ����� �� �� � �� �� � �

� ����

2 2B

2 231cm/s 115200 115 tg200

30º� �� � �� �a con la horizontal.

B60º

A

B

A

C O

z

y

'

*

'R

R

r

aB

60º

30º

- 52 -

Física Universitaria: Problemas de Física Cinemática del sólido rígido. M05.22

20. La cubierta del rodamiento a rodillos representado en la figura está fija, mientras que el árbol interior gira con velocidad constante '1 en sentido horario. Los rodillos ruedan sin resbalar por las pistas. Determinar: a) La velocidad angular '2 de un rodillo, indicando su sentido. b) La velocidad vB y la aceleración aB del punto B del eje de un rodillo. c) La aceleración aA del punto A de la periferia del rodillo.

Se trata de un movimiento plano en el que el CIR del rodillo se encuentra en el punto A, que se encuentra instantáneamente en reposo por no resbalar el rodillo respecto de la cubierta fija. Por la misma razón, la velocidad del punto C del rodillo tiene la misma velocidad que la del punto C perteneciente al árbol interior; esto es,

C 16v R��

a) Determinamos la velocidad angular '2 del rodillo a partir del conocimiento de la posición del CIR (punto A) y de la velocidad del punto C:

1C 1

C2 2

6AC2AC

3v RvR�

� ��� � � �

en el sentido antihorario, tal como se indica en la figura. b) La velocidad del punto B es la mitad de la del punto C, tal como se deduce de la construcción gráfica de la figura; i.e.,

B 2 13v R R� �� �

El punto B está describiendo una trayectoria circular de radio 7R con celeridad constante. En consecuencia, presenta una aceleración centrípeta (dirigida hacia el centro de dicha trayectoria) cuyo módulo es

�1B2

221B 3

7 779Rv

R RRa

��� � �

c) Determinamos la aceleración del punto A o CIR a partir de la del punto B mediante la expresión:

�A B 2 2 2 2BA BA con 0� � � � � � �a a � � � ���� ���

� �

de modo que

2 2 2 2 2A 1 1 2 1 1

2 2

9 9 9

7 7 7

0 0 000 0 00 0 9

0 00 0 0

R R R R R R� � � � �� �

� � � � � �� � � �� � �� �� � � � � �� �� � ���� � �� � �� �� � �� � �� � � �� �� � � �� � �� � � �� �� �� � � � � � � � � � � �� � � �� � � � � ��� �� � � �� � �� � �� �� � � �� � �� � �� � �� �� � � �� � � ��� � � � �� �� � �� � � �� � �� � �

a 21

72

7

0

0

R�

� ��� �� �� �� ��� �� �� ��� ���

����� ������

A

B R

6R

'1

R

A (CIR)

B

'1

R

CvC

vB '2

aB

x y

- 53 -

Física Universitaria: Problemas de Física Cinemática del sólido rígido. M05.23

21. Un disco de radio r1 = 10 cm rueda sin deslizar sobre otro disco de radio r2 = 20 cm, a velocidad angular �1 = 3 rad/s. Al conjunto de ambos discos se les dota de una nueva rotación �2 = 6 rad/s alrededor del eje del disco grande, en el mismo sentido que �1. En el instante en que los dos discos están tal como aparecen en la figura, determinar: a) Las velocidades de los puntos A y B. b) El centro instantáneo de rotación del disco pequeño. c) La velocidad del punto C.

a) En la rodadura del disco pequeño sobre el grande, al punto A le corresponde una velocidad nula (CIR). En la rotación del conjunto de los dos discos, al punto A le corresponde una velocidad

2

01.20 m/s

0

0 0.20OA 0 0

6 0A

� � � �� �� �� �� �� �� � � � �� �� ��� �� �� �� �� ����

� �� �� �� �� �� �� �� � v �

El disco pequeño está sometido a una rotación resultante

1 2 3 6 9 rad/s� � � � �� � � k k k

Calculamos la velocidad del punto B a partir de la del punto A:

B A

0 0 0.20AB 1.20 0

03.00 m/0

0s

09 0

� � � � � �� � �� � �� � �� � �� � �� � � � � � �� � �� � �� � �� � �� � �� �

� ��� �� �� �� �� ���� �� � �� � �� � � v v �

b) El CIR del disco pequeño se encuentra en el punto I, tal que

AA

1.20IA IA 0.13 m =9

13.3 cmvv ��

� " � � �� �

o sea, a una distancia de 6.6 cm del punto O. c) Calculamos la velocidad del punto C a partir de la del punto A:

C A C

0 0 0.10

1.20 0 0

0.90

2.1.10

0 9

0

0 0

m/s 2.28 mA /sC v�� � � � � �� � �� � �� � �� � �� � �� � �� � �� � � � � � � "� � �� � �� � �� � �� � ��

� ��� �� �� � �� �� �� ��� �� � �� � �� � � ��

v v �����

A

B

C

A B

C

O I x

y

�2

�1

- 54 -

Física Universitaria: Problemas de Física Cinemática del sólido rígido. M05.24

22. En el dispositivo que se muestra en la figura, el brazo tiene una longitud l y está girando alrededor de un eje fijo que pasa por O. En un instante dado, su velocidad angular es 'b y su aceleración angular )b (sentido antihorario). El otro extremo del brazo arrastra un piñón, de radio R, que rueda sin resbalar por el interior de una corona fija. a) Determinar la velocidad angular ('p) y la aceleración angular ()p) del piñón en ese instante. b) Determinar la velocidad y la aceleración de punto A del piñón en ese instante. c) Ídem del punto B.

a) Determinamos la velocidad y aceleración del punto C del brazo que serán también las del punto C del piñón:

C O�v v b b

b

C O

OC0 00 0

0 0

ll�

� � � � � �� � �� � �� � �� � �� � �� � � � �� � �� � �� � �� � �� � �� � �� � �� � �� � �

a a

���

2b

b b b b b

bb

OC OC0 0 0

( ) 0 0 00 0 0

l ll l

�� �

� �

� � � �� � � � � � �� �� �� � �� � �� �� � � � �� � �� �� � �� �� � � � � � � � � �� � �� �� � �� �� � � � �� � �� �� �� � �� � �� � �� �� � �� �� �� � � � � � � � ���� ���

La velocidad del punto B del piñón es nula (rodadura, CIR del piñón), de modo que podemos escribir:

p bC b

C p p p p b bBC (horario)BC

v l lv l lR R R R�

� � �� � � ��� � � � � � �� � �� �

b) Determinamos la velocidad y la aceleración del punto A del piñón a partir del conocimiento de la velocidad del punto B (CIR) y de la aceleración del punto C:

A B�v v p p p C

p

A C p p p

2b

b

b

BA

CA CA

0 2 0 00 0 2 2 2

0 0 0

( )

00 0

0 0lR

RR l

l Rl

� ��

��

� � � � � �� ��� � �� � ���� � ��� � ��� � ��� � �� � � � � � � ��� � ��� � ��� � ��� � �� � �� ��� � � �� � �� � �� � � �

� � � � � � �

� � � � �� ��� ��� �� �� ��� � � ��� �� �� �� � � ��� �� �� �� �

� v

a a � � �

���

��� ���

�2

b

2b

b

b

0 00 0 0

02

0

l

l lR R

R

lR l

� �

��

� �� � � �� � �� �� � �� �� �� � �� �� ��� �� � �� �� ��� � � �� �� � �� �� ��� �� � �� �� ��� �� � �� �� �� �� �� �� �� �� � � �

� �� �� �� �� �� �� �� ��� � ����

c) La velocidad del punto B del piñón es nula (CIR) y su aceleración la determinamos a partir de la del punto C (como en el apartado anterior):

B B C p p p

2b

b

b b b

CB CB0 ( )

0 0 00 0 0 0 0

0 0 0l l lR R R

l R Rl

��

� � �

� � � � � � � �

� � � �� � � � � �� � � �� � �� �� � � � �� �� � �� �� �� � �� �� �� � �� �� �� � �� �� �� � � � � � � �� � �� �� �� � �� �� �� � �� �� �� � �� �� �� � � �� � �� � �� �� �� � �� � �� � ��� �� � �� � � � � ��

v a a � � ���� ���

�22b

00

lR l�� �� �� �� �� �� �� �� �� ����

' b

C

) b

A BO

y

x

corona

piñón

brazo

- 55 -

Física Universitaria: Problemas de Física Cinemática del sólido rígido. M05.25

I1

A

B

I

R

I2

O1 O2 �1 �2

D

+

l=2R

vA

vB

+

23. Dos discos, de 15 cm de radio, ruedan sin deslizar sobre una superficie plana. Una barra de longitud igual al diámetro de los discos está unida por sus extremos a puntos de la periferia de ambos discos, mediante articulaciones. En el instante que se representa en la figura, el disco de la izquierda rueda a razón de 2 rad/s en el sentido de giro horario: a) Hállese la posición del centro instantáneo de rotación de la barra. b) Determínese el sentido de la rotación y el valor de la velocidad angular del otro disco. c) Ídem de la barra.

Datos: R =15 cm; l = 30 cm; �1 = 2 rad/s.

De la figura, + = 45º y sen 0.5 30ºRl

� �� � �

a) Se trata de un movimiento plano en el que intervienen tres sólidos rígidos. Los CIR de los discos se encuentran en los puntos I1 e I2 indicados en la figura. A partir de ellos podemos determinar las direcciones de los puntos A y B de los discos respectivos, tal como se indican en la figura que serán perpendiculares a las rectas I1A e I1B, respectivamente. Dichas velocidades serán también las que poseen los extremos A y B de la barra. Conocidas las velocidades (vA y vB) de dos puntos de la barra, el CIR de la misma se encuentra en la intersección de las perpendiculares trazadas a dichas velocidades en los puntos A y B. Por consiguiente, el CIR de la barra se encuentra en el punto I indicado en la figura y la rotación (�) de la barra tiene la dirección indicada (antihoraria). Determinamos las distancias de I a los puntos O2 y B:

�2

2 2

3O I cos tg 2 1 3 25.98 cm2

BI O I O B c3 10.98 m

l R R

R R

� �� � � � �

� � � � �

b) Calculamos la velocidad del punto A:

A 1 2 1I A 2 2 15 2 42.43 cm/sv R� �� � � � � �

El teorema de las velocidades proyectadas nos permite calcular la velocidad del punto B:

� �

A B B A

cos 90º cos 75ºcos 90º cos 42.43 12.68 cm/scos cos30º

v v v v� �

� � ��

� �� � � � � � �

Ahora, partiendo del CIR de la barra (I) , calculamos la velocidad angular de ésta:

BB

12.68IB10.98

1.15 rad/sIBvv � � � � (antihoraria)

c) Finalmente, partiendo del CIR del segundo disco (I2) , calculamos su velocidad angular:

B BB 2 2 2

2

12.68I B2

0.42 ra3

d/s0I B

v vvR

� �� � � � � (horaria)

- 56 -

Física Universitaria: Problemas de Física Mecánica Clásica. Ley de la Inercia. M06.1

1. Día solar y día sidéreo.- El día solar corresponde al intervalo de tiempo que emplea el Sol en pasar dos

veces sucesivas por un mismo meridiano terrestre. El día solar medio es de 86 400 segundos (= 24 horas). El día sidéreo corresponde al intervalo de tiempo que emplea la Tierra en completar una revolución alrededor de su eje polar. Calcular la duración del día sidéreo.

El día solar es el intervalo de tiempo transcurrido entre dos pasos del Sol por el meridiano del lugar. El día sidéreo o sideral1 corresponde al intervalo de tiempo que emplea la Tierra en completar una revolución alrededor de su eje polar. Es fácil establecer la relación que existe entre un día solar medio y un día sidéreo. Consideremos la Tierra ubicada inicialmente en T, en el instante en que culminan el Sol y una estrella mucho más lejana que pueda considerarse como una referencia fija. El meridiano local es m y el punto Sur es S. Un observador terrestre que estuviera mirando al Sur vería al Sol y a la estrella alineados y culminando (viñeta izquierda). A medida que transcurre el tiempo, la Tierra se traslada de T a T', a la vez que rota. La estrella lejana culmina de nuevo en T', mientras, que para que culmine el Sol, la Tierra deberá rotar aún el ángulo �. Se dice que el Sol "retrasa" respecto a la estrella (viñeta central). Finalmente, el Sol culmina por segunda vez en T" y se dice que ha transcurrido un día solar. El día solar medio se compone de un día sidéreo más una fracción de día correspondiente a la la rotación de ángulo � anteriormente descrita:

solar sidéreosolar solar

86400 s s1d 1d � � 236.6 3 min 56.55 s365.2422 d d

� � � � �

Este resultado también puede obtenerse teniendo presente que durante un año, que dura 365.2422 días solares, el Sol pasa por el meridiano una vez menos que el punto Aries. Así durante este intervalo ha habido un día sidéreo más que días medios, de modo que

solar sidereo

sidereo solar solar

1 año 365.2422 d 366.2422 d365.24221 d d 0.997 2696 d 86 400 86164 s366.2422

� �

� � � �

de modo que el día sidéreo es � 86400 86164 236 s 3 min 56 s� � � � más corto que el día solar.

1 En todo rigor, se llama día sidéreo al tiempo transcurrido entre dos pasos consecutivos del punto Aries por el meridiano del lugar. Excepto por la retrogradación coincide con el día sideral del que como mucho se diferencia en 0.01 s.

- 57 -

Física Universitaria: Problemas de Física Mecánica Clásica. Ley de la Inercia. M06.2

2. La cinta transportadora de viajeros de un aeropuerto tiene una longitud de 100 m y avanza con una

velocidad de 1.2 m/s. Una persona se mueve sobre la cinta con una velocidad relativa a ella de 1.5 m/s. Determinar el tiempo que estará la persona sobre la cinta: a) cuando camina en dirección del movimiento de la cinta y b) cuando camina en sentido opuesto.

a) Cuando la persona camina en la misma dirección en que se mueve la cinta, su velocidad absoluta será

P PC C 1.5 1.2 2.7 m/sv v v� � � � �

Y empleará un tiempo t en recorrer la longitud L, de modo que

P

100 37 s2.7

Ltv

� � �

b) Cuando la persona camina en la dirección contraria al del movimiento de la cinta, su velocidad absoluta será

P PC C 1.5 1.2 0.3 m/sv v v� � � � �

Y empleará un tiempo t en recorrer la longitud L, de modo que

P

100 333 s 5 min 30 s0.3

Ltv

� � � �

vC

vPC vP

L

vC

vPC

vP

L

- 58 -

Física Universitaria: Problemas de Física Mecánica Clásica. Ley de la Inercia. M06.3

3. Una persona sube por una escalera mecánica, que se encuentra parada, en 8.2 s. Cuando la escalera está en

funcionamiento, puede subir a la persona en 5.0 s. ¿Cuánto tiempo emplearía la persona en subir caminando por la escalera en movimiento?

Sea L la longitud que cubre la escalera. Cuando la escalera está parada, la persona emplea un tiempo t1 en cubrir la longitud L, modo que su velocidad, tanto absoluta (vP) como relativa a la escalera (vPE), es

PE 1 PE1

LL v t vt

� �

Cuando la escalera está en movimiento y la persona en reposo respecto de ella, se emplea un tiempo t2 en cubrir esa misma distancia, de modo que

E 2 E2

LL v t vt

� �

Cuando la escalera está en movimiento y la persona camina sobre ella, su velociadad absoluta será P PE Ev v v� � y empleará un tiempo t en cubrir la distancia L, resultando

�PE E PE ELL v v t v vt

� � � �

Así, de las tres expresiones anteriores, resulta:

1 2 1 2

1 1 1L L Lt t t t t t

� � � � (media harmónica)

Sustituyendo valores

1 2

1 2

8.2 5.0 41.0 3.1s8.2 5.0 13.2

t ttt t

�� � � �

� �

L

vE

vP

- 59 -

Física Universitaria: Problemas de Física Segunda y tercera leyes de Newton. M07.1

1. Dos cuerpos A y B, que pesan 500 N cada uno, se mantienen en equilibrio sobre sendas superficies, perfectamente lisas y perpendiculares entre sí, mediante un cable ligero y flexible que los une, según se indica en la figura. Determinar el valor del ángulo � correspondiente a la posición de equilibrio, así como las reacciones de las superficies sobre los cuerpos y la tensión del cable en dicha posición.

Cuando el sistema esté en equilibrio, la resultante de las fuerzas que actúan sobre cada cuerpo será nula. Esto es, descomponiendo en las direcciones horizontal y vertical, tenemos:

A

A

B

B

(1) cos sen30ºCuerpo A

(2) sen cos30º

(3) cos cos30ºCuerpo B

(4) sen 30º sen

T NT N P

T NN P T

��

��

��� ��

��� � �

Considerando el equilibrio del sistema en su conjunto [o sumando m.a.m. las ecuaciones (1)+(3) y (2)+(4)], se obtiene

A B

A B

sen30º cos30º A + B

cos30º sen 30º 2N NN N P

��� ��

Sistema de dos ecuaciones con dos incógnitas que nos permite calcular directamente las reacciones NA y NB:

A2 2

B

0 cos30º2 cos30º

2 sen 30ºsen 30866 N

5

º cos30ºsen 30º cos 30º 1

cos30º sen 30º sen 30º 0002 sen 30º

cos30º 2N

N PP

N PP

� �� � ��

� �, � � � �� � � ���

De las ecuaciones (2) y (1), al dividirlas m.a.m., se sigue el valor del ángulo �:

A

A

sen cos30º 500 750 250 250tg 0.577cos s

3en 30º 433 3

043

ºT P NT N

� ���

� � � � � � - ��� � � �

��.� /

y la tensión del cable será:

A433cos sen 30º 4 500 N33

cos( 30º )T N T� � � � � �

A

B

60º 30º

A

B

60º

30º

NA NB

P

T

P

60º

- 60 -

Física Universitaria: Problemas de Física Segunda y tercera leyes de Newton. M07.2

2. Una masa m colocada sobre una superficie lisa horizontal está unida a una masa M mediante una cuerda ligera que pasa por un agujero practicado en la superficie. La masa m se mueve describiendo una trayectoria circular de radio r con una celeridad v. Determinar el valor de la masa M para que ese movimiento se mantenga.

La masa m está en movimiento, describiendo una trayectoria circular con celeridad constante, bajo la acción de una fuerza centrípeta proporcionada por la tensión de la cuerda. Escribimos las ecuaciones del movimiento para cada una de los dos masas; i.e.,

2

0

vT ma mr

T Mg

� � �� �� �

� �� � ��

De modo que 2 2v vm Mg M m

r rg� �

� � � � �

Mg

T

T

M

r m

- 61 -

Física Universitaria: Problemas de Física Segunda y tercera leyes de Newton. M07.3

3. Un hilo flexible y uniforme, de longitud l, está colgado en una pared vertical pasando sobre un clavo fijo y liso. Aunque el hilo se encuentra inicialmente en equilibrio, se le separa ligeramente de dicha posición para que comience a deslizar sobre el clavo. a) Describir el movimiento del hilo, determinando su aceleración. b) Calcular la velocidad que adquiere el hilo, en el instante en que abandona al clavo.

a) Cuando desequilibramos el hilo, tirando ligeramente de uno de sus extremos, comenzará a moverse en esa dirección, aumentando continuamente su velocidad. Supongamos que, en un instante dado, una longitud x de hilo cuelga del lado derecho, como se ilustra en la figura. Los pesos de las dos porciones de hilo serán �xg y �(l-x)g respectivamente, siendo � la densidad lineal del hilo. La ecuación del movimiento del hilo será

( ) (2 )x g l x g l x lx x l g� � �� � � � � ��� ��

(2 )ga x ll

# � �

de modo que la aceleración que adquiere el hilo va aumentando a medida que aumenta x, i.e., su aceleración no es constante. b) Puesto que la aceleración no es constante, deberemos proceder a calcular la velocidad por integración.

2

2

/ 20 / 2

2 12

12

d d d d (2 )d d d d

1d (2 d2 4

2

)v l l

ll

v v x v ga v x lt x t x l

g v gv v x l x x lx gll l

v v gg ll

� � � � �

� �� � � � � ��

# � ��

! !

Otro método Puesto que no hay fricción, podemos calcular la velocidad a partir de la Conservación de la Energía:

2

2

2 12

1 2

14 2 2l lmg mg mv v gl

v gl

� � � �

#

Ep=0

L/4

L/2

v

G

G

x

�xg

�(l-x)g

l-x

- 62 -

Física Universitaria: Problemas de Física Las fuerzas de la Naturaleza. M08.1

1. El bloque A de la figura pesa 15 kg y el bloque B pesa 5 kg. El coeficiente de roza-miento entre todas las superficies en contacto vale 0.20. En cada uno de los casos que se muestran en la figura, calcular la magnitud de la fuerza F necesaria para arrastrar el bloque A hacia la derecha con velocidad constante.

En cada uno de los tres casos, la fuerza necesaria para arrastrar el bloque A, con velocidad constante, será igual a la suma de las fuerzas de rozamiento que se oponen al movimiento del bloque A y del B (en el tercer caso).

a) � �A A A B 0.20 15 5 4 kg 39.2 NF f N P P� �� � � � � � � �

b) �

�A AB A B A B B

0.20 15 5 0.20 5 4 1 5 kg 49.0 N

F f f N N P P P� � � �� � � � � � � �

� � � � � � � �

c) �

�A AB BA A B B A B B B

0.20 15 5 0.20 5 0.20 5 4 1 1 6 kg 58.8 N

F f f f N N N P P P P� � � � � �� � � � � � � � � � �

� � � � � � � � � � �

A B

F

A B

F

A B

F

A

B

F NA

NB

fA

fAB

A

B

F

fA

fAB fBA

NA

NB

A F fA

B

NA

- 63 -

Física Universitaria: Problemas de Física Las fuerzas de la Naturaleza. M08.2

2. Un bloque de masa m1 está situado sobre otro de masa m2 que a su vez se apoya sobre una superficie horizontal lisa. Siendo los coeficientes de rozamiento estático y cinético entre los dos bloques 0s y 0k, determinar: a) Fuerza máxima F que puede aplicarse al bloque de masa m2 para que el de arriba no deslice. b) Si F es la mitad de este valor máximo, determinar la aceleración de cada bloque y la fuerza de rozamiento que actúa entre ellos. c) Si F es el doble del valor máximo determinado en a), calcular la aceleración de cada bloque. Datos: m1 = 2 kg; m2 = 4 kg; �s = 0.3; �k = 0.2.

La fuerza de rozamiento estático entre los dos bloque presenta un valor máximo dado por �sm1g = 0.3�2�9.8 = 5.88 N. El rozamiento cinético presenta un valor constante igual a �km1g = 0.2�2�9.8 = 3.92 N

a) Puesto que el bloque de arriba no desliza, ambos bloques tendrán una aceleración común a’. Aplicamos las ecuaciones del movimiento al bloque superior (valor máximo del rozamiento estático) y al conjunto de los dos:

21 s 1

1 2 1 2

0.3 9.8 2.94 m/s( ) ( ) 0.3 6 9.8 17.64 N

s

s

f m a m g a gF m m a F m m g

� �

�� ) ) )�� �� � � � �� � � �� �) ) )� � � � � � � �� �� �

b) Ahora, la aceleración del bloque inferior es menor que la crítica y el bloque superior no deslizará sobre el inferior, por lo que ambos bloques tendrán la misma aceleración a”. Procedemos como antes, con F”=F’/2, comenzando por el conjunto de los dos bloques para determinar la aceleración y luego calculamos el valor del rozamiento estático:

1 21 2

1 s

2

1

1.47 m/s

2.9

17.64 / 2

4

( )( ) 62 1.47

FaF m m am m

f gN

m a mf

� ))��� )))) )) � � �� �� �� � � � �� �)) ))� *� �� ))� � � ���

c) Ahora, la aceleración del bloque inferior supera el valor crítico, por lo que el bloque superior deslizará “hacia atrás” sobre el bloque inferior, existiendo rozamiento cinético. Escribimos las ecuaciones del movimiento para cada uno de los bloques, por separado, con

2F F))) )� :

1 ´2

2

k1 1 ´k 1

k 12k 1 2 2

2

1.96 m/s

7.

0.2 9.82 17.64 0.2 2 9 8.8 /

44 m s

a gf m a m g

F m gaF m g m am

��

��

� )))� � � � ��� ))) )))� �� �� � )))� � � � � � �� � )))))) ))) � � �� �� �� ���

Fm1

m2

F m1

m2

f f +

- 64 -

Física Universitaria: Problemas de Física Las fuerzas de la Naturaleza. M08.3

3. Dos bloques, de masas m1 = 4 kg y m2 = 8 kg, están unidos mediante una varilla rígida y ligera y resbalan por un plano inclinado 30º, como se muestra en la figura. El coeficiente de rozamiento cinético entre el plano y cada uno de los bloques es �1=0.20 y �2=0.30, respectivamente. Calcular la aceleración del sistema y la tensión en la varilla, indicando si es tensora o compresora.

Comenzamos valorando la fuerza de rozamiento cinético que actúa sobre cada bloque

1 1 1 1 1

2 2 2 2 2

cos30º 0.2 4 9.8cos30º 6.79 N27.16 N

cos30º 0.3 8 9.8cos30º 20.37 Nf N m g

ff N m g� �� �

�� � � � � � �� ���� � � � � � ��

Aplicamos la 2ª ley de Newton al sistema constituido por los dos bloques: � � �1 2 1 2 1 2

2

sen 30º

12 9.8sen 30º 27.16 58.80 27.16 31.64 12 2.64 m/s

m m g f f m m a

a a

� � � � �

� � � � � � �

Aplicamos la 2ª ley de Newton al bloque pequeño:

1 1 1 1

1

1 1

sen 30º4 9.8sen 30º 6.79 4 2.64

19.60 6.79 10.56 2.25 N

m g f R m aR

R R

� � �

� � � � �

� � � �

De modo que la tensión en la varilla es tensora, como se ilustra en la figura. Aunque no es necesario, a efectos de comprobación aplicamos la 2ª ley de Newton al bloque grande:

2 2 2 2

2

2 2

sen 30º8 9.8sen 30º 20.37 8 2.64

39.20 20.37 21.12 2.29 N

m g f R m aR

R R

� � �

� � � � �

� � � �

La discrepancia de valores se debe a los redondeos; la tensión en la varilla bien puede ser igual a 2.27 N (tensora).

m1

m2

30º

30ºf1

N1

m1g

f2

N2

m2g +

30º

f1

N1

m1g

R1

R1

R2

varilla

f2

N2

m2g +

R2

+

- 65 -

Física Universitaria: Problemas de Física Las fuerzas de la Naturaleza. M08.4

4. Dos bloques, de masas m1 = 4 kg y m2 = 8 kg, descienden por un pla-no inclinado 30º, sin perder contacto entre sí, como se muestra en la figura. El coeficiente de rozamiento cinético entre el plano y cada uno de los bloques es �1=0.30 y �2=0.10, respectivamente. Calcular la aceleración del sistema y la fuerza que se ejercen los dos bloques entre si.

a) Comenzamos valorando la fuerza de rozamiento cinético que actúa sobre cada bloque

1 1 1 1 11 2

2 2 2 2 2

cos 0.3 4 9.8 cos30º 10.18 N16.97 N

cos 0.1 8 9.8 cos30º 6.79 Nf N m g

f f ff N m g� � �� � �

�� � � � � � � �� � � ���� � � � � � � ��

A continuación aplicamos la 2º ley de Newton en la dirección del movimiento a cada bloque y tenemos 2 ecuaciones con 2 incógnitas (a, R):

1 1 1

2 2 2

sen 30ºsen 30º

m g R f m am g R f m a

� � � ����� � � ���

Sumando m.a.m. obtenemos: � � �1 2 1 2 1 2

2

1 2

sen 30º16.97sen 30º 9.8sen 30º 3.49 m/s

12

m m g f f m m afa g

m m

� � � � �

� � � � ��

� �1 1sen 30º 4 3.49 4.9 10.18 4.57 NR m a g f� � � � � � � �

m1

m2

30º

N2

f2 m2gR

f1

N1

m1g

Rm1

m2

30º

+

- 66 -

Física Universitaria: Problemas de Física Las fuerzas de la Naturaleza. M08.5

5. Un bloque de masa m puede deslizar sobre una superficie plana inclinada con una pendiente del 12% (i.e., tg � = 0.12). Cuando el bloque se lanza plano arriba, recorre la mitad del espacio que cuando se lanza plano abajo, con la misma velocidad inicial en ambos casos. Calcular el valor del coeficiente de rozamiento entre el plano y el bloque.

Tanto cuando el bloque asciende como cuando desciende, serán: cos cos cosfN mg f N mg W f s mgs� � � � � �� � � � �

siendo Wf el trabajo realizado por la fuerza de rozamiento durante un recorrido s sobre el plano inclinado. Cuando el bloque sube, su energía cinética inicial se convierte en energía potencial gravitatoria y en energía disipada en el proceso de rozamiento hasta detenerse:

�20 1 1 1

1 sen cos sen cos2

mv mgs mgs mgs� � � � � �� � � �

Cuando el bloque baja, su energía cinética inicial y su energía potencial gravitatoria se convierten en energía disipada en el proceso de rozamiento hasta detenerse:

�20 2 2 2

1 sen cos sen cos2

mv mgs mgs mgs� � � � � �� � � � � �

Puesto que la velocidad inicial es la misma en ambos casos, igualando las expresiones anteriores, y operando, tenemos:

� � 21 2

1

cos sensen cos sen cos 2cos sen

smgs mgss

� � �� � � � � �

� � ��

� � � � � ��

de modo que cos sen 2 cos 2sen 3sen cos 3tg 3 0.12 0.36� � � � � � � � � � �� � � � � � � �

mg

N

f

v0

s1

mg

N

f v0

�s2

- 67 -

Física Universitaria: Problemas de Física Las fuerzas de la Naturaleza. M08.6

6. Tres cuerpos de masa m = 5 kg están unidos entre sí por dos cuerdas que pueden soportar una tensión máxima T = 20 N. Los cuerpos se encuentran sobre una superficie horizontal y los coeficientes de rozamiento son: 01 = 0.3, 02 = 0.2, 03 = 0.1. Si aplicamos al cuerpo 3 una fuerza F que aumentamos lentamente, ¿Qué cuerda se rompe y con qué fuerza mínima ocurrirá?

Cuando la magnitud de la fuerza aplicada sea tal que estemos en condiciones de movimiento inminente, las fuerzas de rozamiento tendrán el valor máximo posible, i.e.:

1 1 1

2 2 2

3 3 3

0.3 5 1.5 kg = 14.7 N0.2 5 1.0 kg = 9.8 N0.1 5 0.5 kg = 4.9 N

f m gf m gf m g

���

� � � �

� � � �

� � � �

y las tensiones de las cuerdas serán: T1 = f1 = 14.7 N T2 = f1 + f2 = 24.5 N Como T2 sería superior a la tensión de rotura (20 N), no habrá movimiento de conjunto de las tres masas, ya que la cuerda 2 se romperá antes de que eso ocurra. Así, las tres masas permanecerán en reposo y la máxima fuerza que se puede ejercer sin romper la cuerda 2 será: F = T2,máx + f3 = 20 + 4.9 = 24.9 N

1 2 3 F

T1 T2 F

f1 f2 f3

2 3 1

- 68 -

Física Universitaria: Problemas de Física Las fuerzas de la Naturaleza. M08.7

7. En el esquema de la figura calcular la fuerza F requerida para que el bloque de 24 kg empiece a subir por el plano inclinado 10º. El coeficiente de rozamiento estático de cada pareja de superficies es de 0.30.

La situación de los bloques es de movimiento inminente por lo que las tres fuerzas de rozamiento son las máximas en cada caso. La tg 10º = 0.18 es menor que 0 = 0.3, por lo que el bloque superior no tiene posibilidad de resbalar hacia abajo sobre el bloque inferior. En consecuencia, al tirar del bloque de 24 kg hacia arriba, será nula la reacción N3 indicada en la figura y los diagramas de cuerpo libre de cada uno de los bloques son los indicados.

Aplicando las ecuaciones cardinales de la estática a cada uno de los bloques, descomponiendo en las direcciones indicadas, tenemos: Bloque superior:

12 1 12

12 12 1

112

1 12

0 cos10º sen10º

0 cos10º sen10º

20 19.29 kg =cos10º sen10º cos10º 0.3 sen10º

189 N

( cos10º sen10º ) 2.35 kg = 23.03 N

x

y

F N N N

F N N P

PN

N N

��

�� � " � ��� �� � " � ����

� � �� �

� � �

++

Bloque inferior:

�' 2 12 2 2

' 2 12 2

0 cos10º 19.29 24cos10º 42.92 kg =

0 sen10º 4.17 5.79 12.88 22.83 kg =

421 N

224 Ny

x

F N N P N

F F P N N�

�� � " � � � � ����� � " � � � � � � ����

++

F20kg

24kg

10º

f12

N1

10ºP1 N3=0

N12 y x

10º x’

y’

N12 f21 F

P2 f2

N2

- 69 -

Física Universitaria: Problemas de Física Las fuerzas de la Naturaleza. M08.8

8. Determínense los valores de los ángulos ) y 1, correspondientes a la posición de equilibrio, para el sistema representado en la figura.

Las cuerdas ideales (inextensibles y de masa despreciable) transmiten íntegramente las tensiones a lo largo de ellas, por lo que tenemos el diagrama de fuerzas actuantes sobre el bloque central, correspondiente al equilibrio del sistema, que se indica en la figura. Escribiendo la condición de equilibrio, tenemos

cos 2 cos2 sen sen

2cos cos2sen sen 22

P PP P P� �� �

� �� �

� � �� �� � � �� �, � �� �� ��

��

y resolviendo este sistema de ecuaciones,... 2 2

2 2 2

2 2

2

2sen 1 cos 2 2sen 1 4cos 2

1 4cos 2(1 sen ) 1 4cos 4(1 sen )1 4(1 sen ) 4 4sen 8sen

3 4sen

� � � �

� � � �

� � �

� � � � � �

� � � � � �

� � � � �

� � 24 4sen�� �

61.04º 14.48º

78sen sen8

# �

� �

2m

) 1

2m m

2m

) 1

2m m 2mg

2mg mg

- 70 -

Física Universitaria: Problemas de Física Las fuerzas de la Naturaleza. M08.9

9. a) Determinar la aceleración máxima que puede alcanzar un automóvil de 1000 kg, con tracción en las cuatro ruedas cuando sube por una rampa de 30º respecto a la horizontal. (coeficiente de rozamiento 0 = 1). b) Calcular la inclinación máxima de la rampa por la que podrá subir este vehículo.

a) El diagrama de fuerzas del cuerpo libre es el que se indica en la primera figura, que puede simplificarse en la forma que se indica en la segunda figura. Apreciamos que la aceleración del automóvil es una consecuencia de las fuerzas de rozamiento entre las ruedas y

la calzada. Aplicamos las Ecuaciones Cardinales d la Dinámica: sen 30º sen 30ºcos30º 0 cos30º sen 30º cos30º

cos30º

f P ma f P maN P N P P ma Pf N f P

�� �

� �� � � �� �� �� �� �� � � � -� �� �� �- -� �� �� �

La aceleración máxima del automóvil será:

� � � 2cos30º sen 30º cos30º sen 30º 1 0.866 0.5 0.366 m3.59s

Pa g g gm

� �- � � � � � � � �

b) Procedemos de forma análoga, para un ángulo arbitrario:

sen sensen cos

cos 0 coscos sen

cos

f P ma f P maP ma P

N P N Pa g

f N f P

� �� � �

� �� � �

� � �

� �� � � �� �� � � � -�� �� � �� � � � � �� � � - �� � ��- -� �� �� �

Solo podrá subir si la aceleración es positiva; i.e., si el valor máximo del rozamiento es igual o mayor que la componente del peso en la dirección de la rampa:

�cos sen 0 cos sen 0 cos sen1tg 1 45º

a g� � � � � � � � �

� ��

- � . � . .

* � *

P

N f

30º

f B A P

NA

NB G

- 71 -

Física Universitaria: Problemas de Física Las fuerzas de la Naturaleza. M08.10

10. Un bloque de masa m está apoyado sobre la cara inclinada y áspera de una cuña de masa M. La cuña puede moverse libremente sobre una superficie horizontal lisa. Se aplica una fuerza F a la cuña, de modo que el bloque queda a punto de deslizar hacia arriba en el plano inclinado. Si el coeficiente de rozamiento estático entre la cuña y el bloque es0, determinar: a) la aceleración del sistema, b) la fuerza F necesaria para producir esta aceleración, c) la reacción normal de la superficie horizontal sobre la cuña.

En la figura adjunta se ha dibujado el diagrama de fuerzas correspondiente a la cuña y al bloque. a) Aplicamos las ecuaciones cardinales de la dinámica al bloque, descomponiendo las fuerzas en las direcciones x e y indicadas en la figura, teniendo en cuenta que f N�� en las condiciones de movimiento relativo inminente:

� �

2 2 2 2 202 2 2 2 2 1

2 2 2 2 2 112 2 2 2 2 1

mgN mg f N N mg N

N f ma N N N mg ma

��

�� �

��� , � � � � � ��� ����� ��� � � � � � � ��� ���

de modo que 11

a g��

��

b) Aplicamos la 2ª ley de Newton al sistema completo (M+m), teniendo en cuanta que F es la única fuerza exterior al sistema que tiene componente en la dirección del movimiento del sistema:

� �11

F M m a M m g��

�� � � �

c) Consideramos el sistema completo (M+m) y aplicamos la condición de equilibrio en la dirección vertical, considerando tan solo las fuerzas exteriores al sistema. Puesto que ni la cuña ni el bloque presentan aceleración en la dirección vertical, será:

�R Mg mg M m g� � � �

m

M

F

45º

mg M

F

45º

N

Mg

a f

x

y

RN

f

- 72 -

Física Universitaria: Problemas de Física Las fuerzas de la Naturaleza. M08.11

11. Una bloque de masa m descansa sobre una cuña y ésta, a su vez, sobre un plano horizontal sin rozamiento. Determinar la fuerza F que se debe aplicar a la cuña para que la masa m comience a ascender si entre ella y la cuña existe un rozamiento de coeficiente 0.

.

En la figura, hemos representado las fuerzas que actúan sobre cada uno de los dos cuerpo (el bloque y la cuña) que constituyen el sistema. Aplicamos las ecuaciones del movimiento al bloque, en el referencial S’ (solidario con la cuña), en el que el bloque permanecerá en reposo hasta que la aceleración que adquiera la cuña sea suficientemente grande como para que comience a ascender, por lo que la fuerza

de rozamiento que actúa sobre el bloque tendrá la dirección indicada (hacia abajo). Tenemos

00

00

sen cos 0[1] sen cos 0

cos sen 0[2] cos sen 0

con

mg f mamg N ma

N mg maN mg ma

f N

� �� � �

� �� �

�� � � �� � � � ���� �� � � � �� � � � ��� ��� ��

Aplicamos las ecuaciones del movimiento a la cuña, en el referencial fijo S, de modo que

0 0cos sen [3] cos senF f N Ma F N N Ma� � � � �� � � � � �

de modo que disponemos de un sistema de tres ecuaciones con tres incógnitas (F, N y 0). Resolvemos el sistema de ec. [1] y [2]

0

0

0

[1] cos sen[2] sen cos

sen coscos sen cos sen

N ma mgN ma mg

mgN a g

� � �

� �

� � �� � � � � �

� � � ��� �� � ����

� �� �

De la ec. [3] despejamos F:

0[3] (sen cos )F Ma N� � �� � �

de modo que sen cos ( )cos sen

F M m g� � �� � �

�� �

FM

m

F

M �

N f

N f mg

Mg

N N2 S ma0

a0

S’

- 73 -

Física Universitaria: Problemas de Física Las fuerzas de la Naturaleza. M08.12

12. Un niño coloca una báscula sobre una plataforma que puede deslizar sin fricción sobre un plano inclinado, como se indica en la figura. El niño se sube en la báscula y lee la indicación de su "peso" cuando la plataforma desciende (aceleradamente) por el plano inclinado. Si el peso del niño en condiciones normales es P = 40 kg y el ángulo de inclinación del plano es 10º, ¿cuál será la indicación de la báscula?

La aceleración con la que desciende el sistema báscula-niño a lo largo del plano inclinado es:

sena g �� y sus componentes en las direcciones horizontal y vertical (ejes xy ) son:

2

sen cos

senx

y

a ga g

� �

� �

Sobre el niño actúan las dos fuerzas que se indican en la figura: su peso (mg) y la reacción normal (N) ejercida por la plataforma de la báscula. La reacción de esta última fuerza es la que mide la báscula (peso aparente del niño). Aplicamos la ecuación del movimiento en la dirección vertical, bien sea en el referencial inercial [S] o en el no-inercial [S’] ligado al sistema acelerado:

�2

2 2

2

S sen1 sen cos

S sen 0

y

y

N mg ma mgN mg mg

N mg mg ma

�� �

� � � � � � �

) ) � � � �

Sustituyendo los valores del enunciado, obtenemos: 240 cos 10º 38.79 kgN = =�

mg

N

a0

sen cosg � �

2seng �

x

y

S S’

- 74 -

Física Universitaria: Problemas de Física Las fuerzas de la Naturaleza. M08.13

13. El coeficiente de fricción (estático y cinético) entre la caja A y la vagoneta de la figura vale 0.6. Se pide: a) Aceleración mínima de la vagoneta para que la caja no caiga. b) Tiempo que tardará en desprenderse la caja si la aceleración de la vagoneta es la mitad de la mínima.

a) En la figura mostramos el diagrama de fuerzas que actúan sobre la caja. Aplicando las ecuaciones cardinales de la dinámica de la partícula, siendo f la fuerza de rozamiento en condiciones de movimiento (de caída) inminente, tenemos:

(1)con

(2) 0N ma

f Nf mg

�� ��� ���, � ���

De modo que 29.8 16.33 m/s

0.6gN mg ma mg a� ��

� � � � �

b) Ahora, con a’ = a/2 = 8.16 m/s2, y siendo ac la aceleración de la caja será:

c

(1)con

(2)N ma

f Nf mg ma

�� ) ��� ���, � ���

2c c 0.6 8.16 9.8 4.90 m/sma ma mg a a g� �) )� � � � � � � � �

donde el signo negativo indica que está dirigida hacia abajo. Calculamos el tiempo de caída a partir de la ecuación para el movimiento rectilíneo uniformemente acelerado,

21 2 2 2 0.903 s2 4.90

ss at t ta

�� � � �

a

2 m

AB

mg

N

f a

mg

N

f a’

ac

- 75 -

Física Universitaria: Problemas de Física Las fuerzas de la Naturaleza. M08.14

14. a) )Qué fuerza horizontal constante debe aplicarse al sistema que se muestra en la figura de modo que los cuerpos de masa m1 y m2 no se muevan con respecto al M. b) Si la fuerza aplicada es la mitad de la calculada en el apartado anterior, ¿Cuáles serán las aceleraciones de los bloques m1 y m2 con respecto del bloque M?

a) Planteamos el problema en el sistema de referencia ligado al bloque de masa M, ya que en este sistema los bloques m1 y m2 se encuentran en reposo. En la figura, hemos representado el diagrama de fuerzas que actúan sobre cada uno de los tres bloques, incluidas las fuerzas de inercia (F1 y F2) asociadas a la no-inercialidad del referencial.

Escribimos las ecuaciones del movimiento para los dos bloques pequeños:

1 0 22 1 0 0

2 1

00

0T m a mm g m a a gm g T m

� � � �� � � ���/ � � ��

Y ahora, escribiendo la ecuación del movimiento para todo el sistema en su conjunto, determinamos la fuerza requerida1 para que los bloques pequeños permanezcan en reposo respecto del grande:

� �21 2 0 1 2

1

mF M m m a M m m gm

� � � � � �

b) Replanteamos el problema, de nuevo en el referencial no-inercial ligado al bloque grande, ya que nos interesa el movimiento de los bloques pequeños relativo al grande:

20 0

1

1 1' '2 2 2

mF F a a gm

� � �

Escribiendo las ecuaciones del movimiento para cada uno de los bloques pequeños, teniendo en cuenta que a1’ = a2’ = a’ (cuerda inextensible), tenemos:

�1 0 1 1 21 2 2 2

2 2 1

' ' 1'' 2 2

T m a m a m mm m a m g g m gm g T m a m

� � � �� 0 � � � ���/ � � ��

De modo que la aceleración que presentan los bloques pequeños con respecto del bloque grande2 es:

�2

1 2

'2

ma gm m

��

1 Obsérvese que, si m1 = m2 = m, será ao = g y F = (M+2m)g. 2 Obsérvese que, si m1 = m2 = m, será a’ = g/4.

N2m1g

m2g

F

M

T

T

F1

N1 S’

F2

a0

m1

m2 F M

- 76 -

Física Universitaria: Problemas de Física Las fuerzas de la Naturaleza. M08.15

15. Un ferrocarril ligero subterráneo está compuesto por tres vagones, los extremos de 100 t (toneladas) cada uno, y el central de 50 t. Al aplicar el freno se ejerce la misma fuerza de frenada en cada vagón. El coeficiente de rozamiento con los carriles vale 0.1. a) Calcular la aceleración de frenada máxima que admiten sin que deslice ningún vagón y las fuerza que ejercen los vagones en lo topes. ¿Qué vagón o vagones serían los primeros en deslizar. b) La aceleración de frenada máxima que admitirán sin que deslicen todos los vagones.

Empezamos calculando los valores máximos de las fuerzas de rozamiento: 3 3

1,m x 1

3 32,m x 2

3 33,m x 3

0.1 100 10 10 10 kg = 98 kN

0.1 50 10 5 10 kg = 49 kN

0.1 100 10 10 10 kg = 98 kN

f N

f N

f N

� � � � � �

� � � � � �

� � � � � �

á

á

á

a) Para que no deslice ningún vagón, la fuerza de frenado (interior al sistema) no puede superar ninguno de los valores máximos de las fuerzas de rozamiento. En consecuencia, las fuerzas de rozamiento serán iguales, en los tres vagones, al menor valor máximo de ellas; esto es 1 2 3 máx 2,máx =f f f f f� � � . Escribimos la ecuación del movimiento para todo el sistema en su conjunto:

3máx

máx tot máx máx 3tot

23 3 49 103250 1

00

.588 m/sff m a am

� �� � � � � �� �

y para los vagones primero y último: 3 3

1 máx 1 máx 1 máx 1 máx3 3

3 máx 3 máx 3 máx 3 máx

9.8 kN49 10 100 10 ( 0.588)

49 10 100 10 ( 0.58 9.88) kN

T f m a T f m aT f m a T f m a

� � � � � � � � � � � � � �

� � � � � � � � � � � �

de modo que el diagrama de fuerzas es el que se representa en la tercera figura. Si se superasen los límites de frenada, el primer vagón en deslizar sería el central. b) Ahora, permitimos que deslice el vagón central, pero no los de los extremos. Bastará con que la fuerza de frenado no supere el valor máximo de la mayor de las fuerzas de rozamiento. En estas condiciones, los valores de las fuerzas de rozamientos serán los máximos posibles en cada vagón. Escribimos la ecuación del movimiento para todo el sistema en su conjunto:

31,máx

1,máx 2,máx 3,máx tot máx máx 3tot

20.98 m2 /45 102 1

s50 0

ff f f m a a

m �

� � � � � � � ��

��

2m 2m m T3

T1 T3

f2

� �� T1

dirección del movimiento

f1 f3

m = 50 t +

2m 2m m T3

T1 T3

f2

��� T1

dirección del movimiento

f1 f3

+

v

froz

ffreno

- 77 -

Física Universitaria: Problemas de Física Sistemas de referencia en rotación. M09.1

P

y

z

x

b

a

' 2

' 1

1. Un punto P en el plano Oyz gira alrededor del eje Oy con velocidad angular '1, y todo el sistema de referencia gira alrededor del eje Oz con velocidad angular '2. a) Hállese la velocidad y aceleración del punto P en el instante en que está en la posición de la figura. Indíquese cual es el movimiento relativo y cual el de arrastre. b) Si el movimiento relativo se convirtiese en movimiento de arrastre y viceversa, ¿sería el mismo movimiento? ¿daría los mismos resultados de velocidad? ¿y de aceleración? Justifíquese las respuestas.

a) Velocidad angular de arrastre: arr 2�� �

Movimiento relativo:

1

rel 1 1

0 0OP 0

0 0

aba

��

� � � � � �� � �� � �� � �� � �� � �� � � � �� � �� � �� � �� � �� � �� � �� � �� � �� � � v �

���

1

rel 1 1 121

0 0( OP)= 0 0

0 0

a

a

��

� �� � � � ��� �� � �� � �� � �� � �� � � � �� � �� � �� � �� � � �� �� �� �� � � ��� � �� a � �

���

Movimiento de arrastre:

2

arr 2

2

0 0OP 0 0

0

bba

� � � � � ��� � �� � �� � �� � �� � �� � � � �� � �� � �� � �� � �� � �� � �� � �� � �� � � v �

���

22

arr 2 2 2

2

00( OP) 0 0

0 0

bb

��

� �� � � �� ��� �� � �� � �� � �� � �� � � � � � �� � �� � �� � �� � � �� �� �� � �� � ��� � � a � �

���

Aceleración de Coriolis: 1

cor 2 rel 1 2

2

0 02 2 0 0 2

0 0

aa

�� �

� � � � � �� � �� � �� � �� � �� � �� � � � �� � �� � �� � �� � �� � �� � �� � �� � �� � � a � v

Movimiento absoluto:

abs rel ar

1 2

r 00

a b� �� �� �� �� �� �� �� ���� � � �v v v abs rel arr co

21 2 2

21

r

02a b

a� � ��

� ��� �� �� � �� �� �� ���

�� � � �a a a a

b) Velocidad angular de arrastre: arr 1�� �

� Se permutan los papeles de las velocidades angulares '1 y '2 en el apartado anterior. � Se obtienen movimientos diferentes en los dos casos (trayectorias diferentes). � En el instante que se indica en la figura, la velocidad será la misma en ambos casos,

intercambiándose los papeles de las velocidades relativa y de arrastre. � Aunque también se intercambian las aceleraciones relativas y de arrastre, en el instante

que se indica en la figura, la aceleración de Coriolis es diferente en cada caso, ya que ahora es

2

cor 1 rel 1

1 2

0 02 2 0 0

0 0 2

b

b

��

� �

� � � � � ��� � �� � �� � �� � �� � �� � � � �� � �� � �� � �� � �� � �� � �� � �� � �� � � a � v

- 78 -

Física Universitaria: Problemas de Física Sistemas de referencia en rotación. M09.2

2. La barra BC de la figura gira con velocidad angular ' constante alrededor de B,

mientras que el manguito A se desliza con una velocidad v también constante respecto a la barra. Determinar velocidad y aceleración absolutas del manguito A cuando está a una distancia r del punto B.

Referencial fijo o absoluto, XYZ, con origen en el extremo B de la barra. Referencial móvil o relativo, xyz, con el mismo origen, solidario con la barra, de modo que en el instante indicado en la figura, coinciden las direcciones de los ejes correspondientes.

Este referencial está en rotación con una velocidad angular (de arrastre) que es la de la barra, de modo que:

arr arr

00 0�

� ��� �� �� � � �� �� �� ���� � � � �� �

La velocidad y la aceleración absolutas del manguito vienen dadas por las expresiones

abs rel arr rel 0

abs rel arr Cor rel 0 rel( ) 2� � � � � �

� � � � � � � � � � � �

v v v v v � ra a a a a a � r � � r � v�

con

0 0 rel rel0 0 BA 0 0 00 0

r v� � � �� �� �� �� �� �� � � � � �� �� �� �� �� �� �� �� �� � v a r v a

����

de modo que

abs rel

abs rel

00 0 00 0

0 0( ) 2

0

0 2 0 00 0

v r

v vr

vr

� �

� � � � � �� � �� � �� � �� � �� � �� � � � � � �� � �� � �� � �� � �� � �� � �� � �� � �� � �

� � � � � � � �� � � �� � � �� � � �� � � �� � � �� �

� ��� �� �� �� �� �

� � � � � � �� � � �� � �� � � �� � �� � � �� � �� � �

� � � �� � �

v v � r

a � � r � v

2

20

rv��

� �� �� �� �� �� �� �� ���

��

� ��

' B

A

Cv

' B C

A X,x

Y,y

- 79 -

Física Universitaria: Problemas de Física Sistemas de referencia en rotación. M09.3

3. Un punto P se mueve sobre la superficie terrestre de modo que su latitud �

y su longitud 2 vienen dadas por las expresiones

6 610 (S.I.) 10 (S.I.)3 6t t

� � �� �

Considerando la Tierra como una esfera de 6400 km de radio, determínense,

para y3 6

� �� � � : a) la velocidad relativa a la Tierra, b) la velocidad

absoluta y c) la aceleración de Coriolis.

a) Expresamos vectorialmente la posición y la velocidad relativa del punto P

M

sen cos cos sencos cos dcos sen sen cos cos cosdtsen cos

R Rx Ry R R Rz R R

� � � � � � � � � � �

� �� �� � � � ��� � �� � �� � �� � �� � �� � � � � �� � �� � �� � �� � �� �� � � �� �� �� � � ���

rr v

� �� �

con 6 6

6 10 106.4 10 m, m/s y m/s3 6

R � � �

� � � �� �

de modo que, para 60º y 30º3 6� �

� � � � � , serán:

1 13 6

6 1 1M 3 6

13

cos 60º cos30º 3cos 60º sen 30º 1

4sen 60º 2 3

sen 60º cos cos 60º sen 30º10 sen 60º cos cos60º cos3

1.870.0º 46

1.s 0co 60º

x Ry Rz

R�

� �� � � � ��� �� � ��� �� � ��� � �� � �� �� � � �� �� � � �� �� � � �� �� � �� � �� � �� � �� �� ��� � � ��� �� �� ��

��

r

v m/s7

� ��� �� �� �� �� ����

b) Velocidad absoluta: F M o� � � �v v v � r

61 10 3 116.362 6.4 100 1 3 3 201.52 m/s

4 4 86400 40 0 02 3

R R ��

� � � � � �� �� � � ��� � �� � �� �� �� � ��� � �� �� �� � �� � �� �� � �� � � � � � �� �� � � � �� � �� �� �� � �� � �� �� �� � �� � �� �� �� � �� � � � �� � �

� r

F

118.23201.0

1.87 116.360.46 201.52

1.076 m/s

1.070

� � � �� �� �� �� �� �� �� � � �� �� �� �� ��

� �� �� �� �� ��� �� ����� �� � � � �� v

c) Aceleración de Coriolis: Cor M2� �a v �

5Cor

21.87 0

2 0.46 01.07 2

86

6.7227.1 10 m/s

40

00�

� ��� ��� � �� � ��� � ��� � ��� � � �� � ��� � ��� ��

� ��� �� ��� �� �� ���� ��� ��� �� �� ���

a

móvil z

y

x

2 �

P r

fijo

- 80 -

Física Universitaria: Problemas de Física Sistemas de referencia en rotación. M09.4

4. En el hemisferio Norte, un automóvil, que pesa 1000 kg, circula por una autopista con una velocidad de

144 km/h. En un instante dado, el automóvil avanza en la dirección Sur-Norte en un lugar de 40� de latitud. a) Determinar la velocidad y la aceleración absoluta del automóvil en ese instante, considerando tan sólo el movimiento de la Tierra como rotación pura alrededor de su eje polar. b) Calcular el valor (módulo y dirección) de la fuerza de Coriolis en ese instante.

Consideramos los Referenciales XYZ (fijo o absoluto) y xyz (ligado a tierra, móvil o relativo), tal como se indica en la figura. Nos servimos de las expresiones correspondientes al movimiento relativo, teniendo en cuenta que oP 0� �r

���:

F M F (o)� � � �v v v � r M F

F M

(o)� �

v v

a a Fd(o)dt

� � ��a r �� � �� � r M

F M

2

(o) 2

� � �

� � �

� v

a � v

Expresamos las velocidades y aceleraciones intervinientes en la base vectorial asociada al referencial móvil o relativo (ligado a tierra) y efectuamos los cálculos correspondientes:

M F

2

F

00 (o) cos0 0

cos 0 cos 0 cos se(o) 0 0 0 cos

sen sen 0

vR

RR

R

� �

� � � � � �� �

� � � �

� � � �� � �� �� �� �� �� �� �� �� �� �� �� �� �� �� �� �� �

� � � � � � � �� � �� � � �� � � �� � � �� � � �� � � �� � � �� � � �� � � � � � � � �� � � �� � � �� � � �� � � �� � � �� � � �� � � �� � � �� �� � � �

v v

a � � R �2 2

Cor M Cor Cor

n0cos

cos 0 02 2 0 0 2 sen 2 sen

sen 0 0 0

R

vv m m v

� �

� �� � � �

� �

� ��� �� �� �� �� �� �� ����� � � � � � � � �� �� � � �� � � �� � � �� � � �� � � �� � � �� � � �� � � � � � � � � �� � � �� � � �� � � �� � � �� � � �� � � �� � � �� � � ��� � � �

a � v F a

O sea

2 2

F F2 2 2 2

0 cos sen 0 cos sen0 cos cos 0 2 sen 2 sen0 0 0 0cos cos

v v R RR R v v

R R

� � � � � �� � � � � � � �

� � � �

� � � �� � � � � � � �� � � �� �� �� � � �� � � �� �� � � �� �� � � �� �� � � �� �� � � � � � � �� � � �� �� � � �� �� � � �� �� � � �� �� �� � � �� � � �� � � �� �� � � �� �� �� � � � � �� � v a

Sustituimos los valores numéricos: 62144 km/h=40 m/s; rad/s; 6.3 10 m; 40º ; 1000 kg

86400v R m�

� �� � � � � �

2F F Cor

40 0.016 68 0 0355.8 m/s 0.003 75 m/s 3.75 N 0.385 Kg

0 0.019 88 0 0

� � � � � � � �� �� � � �� � � �� � � �� � � �� � � �� � � � �� � � �� � � �� � � �� � � �� � � �� � � �� � � �� � � ��� � � � v a F

x

y

� z vM

a(o)F

R

O

o

X

Y

Z

- 81 -

Física Universitaria: Problemas de Física Sistemas de referencia en rotación. M09.5

z, z’

x

y �

P

vC

C O

y’

5. El plano vertical representado en la figura gira alrededor del eje Oz con velocidad angular constante de 120/� r.p.m. Un disco de 2 cm de radio, contenido en dicho plano, rueda sin deslizar sobre la intersección del mismo plano con el plano xy, con una velocidad de traslación de 4 m/s. Calcular la velocidad y la aceleración del punto P del disco diametralmente opuesto al de contacto con el plano xy cuando el centro del disco está a 3m del eje z y � = �/2.

Consideremos un sistema de referencia fijo (xyz) y otro sistema de referencia móvil (x’y’z’) cuyo plano y’z’ es solidario al plano vertical representado en la figura. En el instante considerado el plano móvil coincide con el plano yz, de modo que, en ese instante, coinciden las bases vectoriales de ambos referenciales. El movimiento relativo del disco es una rodadura pura sobre el eje Oy’ y el movimiento de arrastre es una rotación pura alrededor del eje Oz con velocidad angular

arr arr

0120 2 = 4 rad/s 0 rad/s60 4

��

� ��� �� �� � " � � �� �� ���� �

La rodadura del disco en el plano móvil nos relaciona la velocidad del centro del disco, vC = 4 j m/s con la velocidad angular de rotación 'rel del mismo; i.e.,

CC rel rel rel

2004 200 rad/s 0 rad/s0.02 0

vv RR

� �� �� �� �� �� � � � � � �� �� ����

La velocidad absoluta del punto P (vP) será la suma de su velocidad relativa (vrel) y su velocidad de arrastre (varr):

rel C rel

arr arr

0 200 0 0mCP 4 0 0 8 s

0 0 0.02 0

0 0 12mOP 0 3 0 s

4 0.04 0

� � � � � � � � ��� � � � �� � � � �� � � �� � � �� � � �� � � � � � �� � � �� � � �� � � �� � � �� � � �� � � �� � � �� � � �� � � � �

� � � � � ��� � �� � �� � �� � �� � �� � � � �� � �� � �� � �� � �� � �� � �� � �� � �� � �

v v �

v �

���

���P rel arr

128 m/s0

������ � � ����

� �� �� �� �� �� ��������

���� v v v

La aceleración absoluta punto P (aP) la calculamos como la suma de la aceleración relativa (arel), la de arrastre (aarr) y la de Coriolis (aCor):

rel C�a a reldd t

�� � 2

2rel rel rel

arrarr

0mCP CP CP 0 s800

dd t

�� �� � �� �� �� ��� � � � � � � �� �� �� ��� �� �����

� �

�a

��� ��� ���

� 2arr arr

2Cor arr rel P rel ar

0 12 0mOP OP 0 0 48 s4 0 0

0 0 64m2 2 0 8 0 s4 0 0

� � � � � ��� � � � �� � �� � �� � � �� � � ��� � � � � � � �� � �� � � �� � � �� � � ��� � � �� � � �� � �� � �� � � � � � � � ��� � �� � �� � �� � �� � �� � � � � � �� � �� � �� � �� � �� � �� � �� � �� � �� � �

� �

a � v a a a

��� ���

r 2Cor

64m48 s800

� �� �� �� ��� �� �� ����� � �a

- 82 -

Física Universitaria: Problemas de Física Trabajo y energía. M10.1

1. Un proyectil de 5 g de masa lleva una velocidad de 400 m/s en el instante en que impacta en el tronco de un

gran árbol en el que penetra una distancia de 4 cm hasta detenerse. Supongamos, para simplificar, que la resistencia que presenta la madera al avance del proyectil sea constante. a) Calcular la fuerza que ejerció el proyectil sobre el árbol y la resistencia de la madera a la penetración. b) Calcular la potencia desarrollada por esa fuerza ¿Es constante? c) Estimar el tiempo empleado en el frenado del proyectil.

a) Puesto que la resistencia que opone la madera al avance del proyectil se supone constante, será constante la desaceleración que este experimenta. Podemos calcular la aceleración de frenado a partir una bien conocida fórmula del movimiento rectilíneo uniformemente acelerado:

2 22 2 6 20f 0

4002 � 0 2 10 m/s2� 2 0.04vv v a x a

x� � � � � � �

La fuerza que ejerce el proyectil sobre el árbol será igual y opuesta a la fuerza de resistencia que presenta la madera al avance del proyectil; esto es,

3 6 45 10 2 10 10 10000 NF ma �� � � � � � � b) La potencia desarrollada por esa fuerza viene dada por P Fv� . Aunque F se supone constante, la potencia va disminuyendo a medida que se ralentiza el proyectil. En el instante inicial vale

4 610 400 4 10 W 4 MWP Fv� � � � � � � �F v c) Podemos calcular el tiempo pedido a partir de

46

� � 400� 2 10 200 �s� 2 10

v va tt a

�� � � � � ��

Método de la energía: a) Durante la penetración en la madera, hasta quedar en reposo, el proyectil pierde toda su energía cinética

3 22

k 01 5 10 400 400 J2 2

E mv�� �

� � �

Como la fuerza resistente que se opone al avance del proyectil se supone constante, el teorema del trabajo y la energía cinética nos permite escribir:

kresist resist k resist

� 400� � 10 000 N� 0.04

EW F x E Fx

�� � � � � �

- 83 -

Física Universitaria: Problemas de Física Trabajo y energía. M10.2

2. Un automóvil que pesa 750 kg circula por una carretera a nivel con una velocidad 54 km/h cuando su motor

desarrolla una potencia de 10 CV. a) ¿Cuánto vale la suma de todas las resistencias (rozamiento, resistencia del aire, ...) que actúan sobre el automóvil? b) ¿Qué potencia deberá desarrollar el motor del automóvil para subir a 54 km/h una cuesta del 10% de pendiente? c) ¿Qué potencia será necesaria para que el automóvil baje a 54 km/h una pendiente del 3%? d) ¿Qué pendiente permitirá que el automóvil baje a una velocidad de 54 km/h sin que funcione el motor? (Nota: supóngase que todas las fuerzas de resistencia permanecen constantes). Datos: 1 CV = 736 W.

Datos: 54 km/h 15 m/s, 10 CV 7360 Wv P� � � � a) Puesto que la velocidad permanece constante, la potencia desarrollada por el motor se empleará en vencer todas las resistencias que se oponen al movimiento del automóvil; esto es,

7360 490.7 N 50 kg15

PP Fv f v fv

� � � � � �

b) Una pendiente del 10% representa un ángulo � tal que tg 0.1 5.7º� �� � . Ahora, la potencia desarrollada deberá vencer también la componente del peso del automóvil en la dirección del movimiento que se opone al movimiento del automóvil; esto es,

� �

sen

490.7 750 9.8sen 5.7º 15 18331 W 24.9 CV

P Fv f mg v�� � � �

� � � � � �

c) Una pendiente del 3% representa un ángulo � tal que tg 0.03 1.7º� �� � . Ahora, la componente del peso del automóvil en la dirección del movimiento favorece el movimiento del automóvil; esto es,

� �

sen

490.7 750 9.8sen1.7º 15 4055 W 5.5 CV

P Fv f mg v�� � � �

� � � � � �

d) Con el motor parado, i.e., P = 0, la resistencia estará compensada con la componente del peso del automóvil en la dirección del movimiento :

490.7 50sen sen 0.067750 9.8 750

ff mgmg

� �� � � � ��

lo que representa una pendiente de 3.83º tg 0.067 6.7%� �� � �

F f

F

f

mg

N

F

f

mg

N

- 84 -

Física Universitaria: Problemas de Física Trabajo y energía. M10.3

3. Una escalera homogénea, de masa m y longitud L, está apoyada sobre una pared

vertical lisa y sobre un suelo horizontal rugoso, formando un ángulo �0 con la horizontal (vide figura). El coeficiente de rozamiento entre el suelo y el pie de la escalera es �. Calcular el trabajo que debemos realizar para llevar la escalera a la posición vertical, empujándola horizontalmente a una distancia D de su pie.

En la figura se muestra el sistema de fuerzas que actúa sobre la escalera en un instante genérico durante el proceso reversible (sucesión de estados de equilibrio) que nos lleva desde el estado inicial (� = �0) hasta el final (� = 90º). Método Conservación de la Energía:

(movimiento inminente)

(equilibrio)

f Nf P

N P0

0��

� �� ��

De modo que la fuerza de rozamiento permanece constante durante todo el proceso y el trabajo realizado por ella es

0 0cos cosfW f L P� 0 �� � � �

Aplicamos el teorema de la energía cinética, teniendo en cuenta que �Ecinética=0, por tratarse de un proceso reversible:

cons cinética 0F fW W W W E� � , �

cons p p0 0F f F f F fW W W E W W W E W � � �, � � � , �

Variación de la energía potencial: � �1 1 1 1

p 0 02 2 2 2� sen 1 senE P L L PL� �� � � �

� � � �1 1p 0 0 0 02 21 sen cos 1 sen 2 cosF fW E W PL PL PL� 0 � � 0 �# � , � � � � �

Cálculo directo del trabajo de la fuerza F: Escribimos las ecuaciones cardinales de le Estática, tomando momentos en A:

1 12 2

cos2sen

cos 1sen cos sensen

N PF P R R F P

f R F F PDDRL PL FD R P F LL

�0 0 0�

�� � ��

� � -� � � � �. � �� / � ��

� �� � � ��

Calculamos el trabajo, teniendo en cuenta que

� �cos d sen d 1 sen dDx L D x L D LL

� � � � �� ���� � � � � � � � �� ���

De modo que

1 2 � �

0 0 0

0

90º 90º 90º12

90º1 1 1 10 0 0 02 2 2 2

cosd sen d sen cos d2sen

cos sen cos sen 1 2 cos sen

FW F x PL PL

PL PL PL

� � �

�� � � � � � �

� � � � � � � � �

� ���� � � � � � ��� ���

� � � � � � � � �

� � �

F

L D

F

L D

R

P

N

f

x

A

B

- 85 -

Física Universitaria: Problemas de Física Trabajo y energía. M10.4

4. Un bloque de masa m se deja caer sobre una cinta transportadora que se mueve con

velocidad constante v. Determinar el tiempo y el espacio recorrido por el bloque hasta que éste adquiera la velocidad de la cinta, siendo 0 el coeficiente de rozamiento entre la cinta y el bloque.

Método 1º Planteamos el problema en el referencial S. La fuerza de rozamiento que actúa sobre el bloque es la que lo acelera hasta que adquiere la misma velocidad que la cinta: esto es,

ctef N mg ma a g� � �� � � � � de modo que se trata de un movimiento rectilíneo uniformemente acelerado, en el que la velocidad inicial

del bloque es nula. Por consiguiente:

f 0v v�

2 2f 0

vvat ta

v

g

v

�� � �

�2 2

22 2vax xa

vg�

� � �

Método 2º Planteamos el problema en el referencial S’ que se mueve con velocidad constante (inercial). La fuerza de rozamiento sigue siendo la misma que antes,

ctef N mg ma a g� � �� � � � � pero v0 = -v y vf = 0

f 0

22 2 2f 0

2

0

2 0 222

vv v at v at ta

vv v ax v ax x

vg

gav

� � � � � � �

) ) )� � � � � � ��

donde el signo negativo significa que el bloque se mueve “hacia atrás” con respecto de la cinta. Método 3º El trabajo realizado por la única fuerza horizontal que actúa sobre el bloque (fuerza de rozamiento ) es igual al incremento de su energía cinética:

k

221�

2 2fW E mgx m vv xg�

� �� �

v

v

f

S S’

- 86 -

Física Universitaria: Problemas de Física Trabajo y energía. M10.5

5. Un ascensor desciende con una velocidad constante de 0.75m/s. Del techo del ascensor se desprende una de

las bombillas de 40 g, que case sobre el suelo del ascensor. La altura de la caja del ascensor es 2.2 m. Calcular el trabajo realizado por la fuerza gravitatoria sobre la bombilla y la variación de la energía cinética de la misma, desde que se desprende hasta que se estrella en el suelo del ascensor: a) en el referencial ligado a la caja del ascensor; b) en el referencial ligado al edificio. c) Explicar las diferencia existentes entre los resultados de los aparatos anteriores.

Consideramos dos sistemas de referencia inerciales: 1. Referencial inercial S, ligado al edificio. 2. Referencial inercial S’, ligado a la caja del

ascensor Durante su movimiento de caída libre, la bombilla tan solo está sometida a la fuerza gravitatoria, i.e., a su peso mg, que representa una fuerza constante. La aceleración de la bombilla será la misma (g) en ambos referenciales inerciales. El trabajo realizado por dicha fuerza será igual al producto de la misma por el desplazamiento que experimenta la bombilla, que será diferente en cada referencial. La velocidades y los cambios de energía

cinética también serán diferentes en cada referencial. El tiempo de caída es el mismo en ambos referenciales:

2caída caída

1 2 2 2.2 0.67 s2 9.8

hh gt tg

�� � � �

a) En el referencial S’:

�A B

2 2 2 2 2 2kB A B A B A

A B

1 1 1

2 2 2

0.0.040 9.8 2.2 86 J

2 0.86 J�

W W mg mgs mgh

v v gh E mv mv m v v mgh

) )

) ) ) ) ) )

) )) )� � � � � � � �

) ) ) ) ) ) )� � � � � � � �

de modo que k�W E) )� (teorema de la energía cinética).

b) En el referencial S:

0 0 caida

AB

2 2 2 2 2 2B A k B A B A

AB

1 1 1

2 2 2

2.20 0.75 0.67 2.20 0.50 2.70 m

0.040 9.8 2.7

2 �

1.06 J

1.06 J

s s v t h v t

W W mg mgs

v v gs E mv mv m v v mgs

)� � � � � � � � � �

� � � � � � �

� � � � � � � �

de modo que k�W E� (teorema de la energía cinética).

c) El trabajo realizado es mayor en el referencia S que en el referencial S’; lo que está de acuerdo con las correspondientes variaciones de la energía cinética. La explicación radica en que tanto la energía cinética como sus cambios, depende del referencial en el que se mida. Podemos asegurar que el trabajo suplementario que se mide en el referencial S coincide con la variación suplementaria del energía cinética que se mide en ese mismo referencial.

A

B

s

s’

A’

B’

A’

B’

h v0

v0

v0t

S S’

S’

- 87 -

Física Universitaria: Problemas de Física Conservación de la energía. M11.1

1. Desde lo alto de una torre de 30 m de altura se lanza un cuerpo con una velocidad de 20 m/s y una

inclinación de 45º (en elevación) con respecto a la horizontal. a) ¿Con qué velocidad (módulo) llegará al suelo? b) ¿Se conseguirá que llegue al suelo con más velocidad si lanzamos el objeto con otro ángulo de inclinación? ¿Con cuál?

a) Conservación de la energía: 2 2 2 20 f f 0

1 1 2 20 31.43 m/s2 9.8 302 2

mgH mv mv v v gH� � � � � � � � �

b) En el planteamiento y resultado del apartado anterior no interviene el ángulo de lanzamiento. Por consiguiente, no se conseguirá ni más ni menos velocidad para otros ángulos.

H=30 m

v0=20 m/s

Vf=31 m/s

�0=45º

- 88 -

Física Universitaria: Problemas de Física Conservación de la energía. M11.2

2. Una masa puntual m está unida a un hilo inextensible de masa despreciable y

longitud l. La masa se deja caer desde la posición horizontal con velocidad inicial nula, como se indica en la figura. Determinar, en función del ángulo �, la velocidad de la masa, la aceleración tangencial, la aceleración normal y la tensión de la cuerda. Particularizar los valores de las magnitudes anteriores para � = 0º y � = 90º.

Conservación de la energía 210 2 2 sen

2mgh mv v gh gl �� � � � �

La aceleración centrípeta o normal se determina fácilmente por tratarse de una trayectoria circular:

2

n 2 senva gl

�� �

La aceleración tangencial se obtiene a partir de la ecuación del movimiento tangencial:

t tcos cosmg ma a g� �� �

También podemos obtenerla como la derivada de la celeridad (módulo de la velocidad):

t2 send d d d d cos2 sen 2 cos

d d d d d 2 senglv v v va gl gl g

t t l l�� �

� � �� � � �

� � � � � �

El módulo de la aceleración es 2 2 2 2 2 2 2t n cos 4 sen 1 3sena a a g g g� � �� � � � � � .

Para determinar la tensión de la cuerda, escribimos la componente radial de la ec. del movimiento:

�n nsen senN mg ma N m g a� �� � � �

en la que sustituimos el valor de la aceleración normal para obtener la tensión en función del ángulo �:

( sen 2 sen ) 3 senN m g g mg� � �� � �

ángulo velocidad ac. tangencial ac. normal mód. aceleración tensión

0º 0 g 0 g 0

90º 2gl 0 2g 2g 3mg

l � m

l

m

h

mg

N

- 89 -

Física Universitaria: Problemas de Física Conservación de la energía. M11.3

A

B

h60º

� m

3. Una masa m está suspendida de un hilo de longitud � atado en A. Si se libera desde la posición indicada, a que distancia mínima h habrá que situar un obstáculo fijo B, para que el hilo se enrolle en el obstáculo.

Conservación de la energía entre 1 y 2: 22

22

1cos 60º2

2 (1 cos 60º ) [1]

mgl mgl mv

v gl gl

� � � �

� � �

El radio de la trayectoria circular centrada en B es R = l – h. Conservación de la energía entre 2 y 3:

2 22 3

2 23 2

1 1( ) ( )2 2

4 ( ) 4 ( ) (4 3 ) [2]

mg l h mv mg l h mv

v v g l h gl g l h h l g

� � � � � �

� � � � � � � �

Aplicamos la 2ª ley de Newton en el punto más alto de la trayectoria circular, teniendo en cuenta que, en las circunstancias críticas de mínima velocidad, la tensión de la cuerda sería nula:

3mg T�2

23cp 3 ( ) [3]

( )vma m v g l h

l h� � � �

Combinamos las expresiones [2] y [3]:

(4 3 ) ( ) 5 4 45

h l g l h g h l h l� � � � �

A

2

1

3

B

60º

h

l

l-h

mgT

v2

v3

- 90 -

Física Universitaria: Problemas de Física Conservación de la energía. M11.4

4. Un pequeño bloque de masa m desliza sin rozamiento por una guía en

forma de lazo como la indicada en la figura. El bloque parte del reposo desde el punto P.

a) Determinar la altura mínima, h, desde la que debe partir el bloque para alcanzar la parte superior del lazo sin separarse de la guía. Ahora, supongamos que la altura h sea igual a 5R. b) Determinar la reacción de la guía sobre el bloque en el instante en el que éste alcanza la parte superior del lazo. c) Calcular las componentes normal y tangencial de la aceleración del bloque en dicho instante.

a) Conservación de la energía: 2 21(2 ) 2 4

2mgh mg R mv v gh gR� � � � [1]

Ecuación del movimiento en el punto más alto (Q):

mg N�2

2vm v gRR

� � [2]

con N = 0 en las condiciones críticas de llegar hasta Q. Combinando las expresiones [1] y [2], se sigue

2 g 4h g� R g�52

R h R �

b) Rescribimos las expresiones [1] y [2] con h’=2h: 2 2

2 2

1 5(2 ) (2 ) 4 4 4 4 62 2

6 5

mg h mg R mv v gh gR g R gR gR

v vmg N m N m mg mg m ggR R

m

) )� � � � � � �

) )� � � � � � �

c) Las componentes intrínsecas de la aceleración, en el punto Q, son: 3 t 0a � , ya que no hay componente de fuerza en la dirección tangencial

3 2

n cp 6va aR

g)

� � � , es la componente normal o centrípeta de la aceleración.

m

P

h R

mg N

v Q

- 91 -

Física Universitaria: Problemas de Física Conservación de la energía. M11.5

5. Una masa m, de pequeñas dimensiones, está sujeta mediante un hilo ligero

y longitud l a un punto fijo O. La masa se abandona desde el punto A, como se indica en la figura. Consideremos el instante en que el hilo forma un ángulo � con la horizontal. a) Determinar la velocidad de la masa. b) Calcular las componentes intrínsecas de la aceleración de la masa. c) Hallar la tensión del hilo. d) Particularizar los resultados anteriores para � = 0º; � = 30º; � = 90º; recogiendo los valores en una tabla y dibujando los vectores en un esquema para cada caso.

a) Como el sistema es conservativo, aplicamos el Principio de Conservación de la Energía para determina la velocidad en función del ángulo:

210 22

mgh mv v gh� � � �

con h =l sen�, de modo que

2 senv gl ��

b) La aceleración centrípeta o normal asociado al movimiento circular es:

2

n 2 sena v gl

�� �

c) Para determinar la aceleración tangencial y la tensión del hilo escribimos las ecuaciones del movimiento:

n

tt

sen sen 2 sencos

3 sencos

T mga

T mg ma mg mgmg ma g

� � ��

��

� �� � � � �� �� � ��

�� �� �� ��

d) Casos particulares:

� 0º 30º 90º

2 senv gl �� 0 gl 2v gl�

3 senT mg �� 0 1.5 mg 3mg

n 2 sena g �� 0 g 2g

t cosa g �� g 0.87g 0

O

A

m

l � l

O A h

l

mg

T an

at �

- 92 -

Física Universitaria: Problemas de Física Conservación de la energía. M11.6

6. Una cadena de acero de 3 m de longitud y 20 N/m de peso está estirada sobre una mesa horizontal de forma

que 2 m de la misma permanecen sobre la mesa y 1 m cuelga verticalmente desde el borde de la misma. En estas condiciones, el peso del segmento colgante es justamente suficiente para arrastrar toda la cadena fuera del borde. a) ¿Cuál es el coeficiente de rozamiento estático entre la cadena y la mesa? Supongamos que, una vez que la cadena está en movimiento, el coeficiente de rozamiento cinético sea tan pequeño que pueda despreciarse. b) Calcular el trabajo realizado sobre la cadena por la fuerza de la gravedad desde que empieza a deslizar (2 m de cadena sobre la mesa) hasta que toda la cadena abandona la mesa. a) Determinar la velocidad de la cadena en el instante en que ésta abandona la mesa.

El peso de la cadena es mg = 3 m � 20 N/m = 60 N. a) En las condiciones del enunciado, el peso de la porción que cuelga (

0xm = 1/3) está compensado justamente por la fuerza de rozamiento sobre la otra porción que permanece sobre la mesa:

� 0

0 0

0

1 / 3 0.52 / 3

xx x

x

mm g f m m g

m m� �� � � � � �

b) El trabajo realizado sobre la cadena por la fuerza de la gravedad es igual a la disminución de la energía potencial gravitatoria de la cadena:

�p,in p,fin8 8� 0.5 1.5 60 80 J

3 6 6pmW E E E g mg mg

� ���� � � � � � � � � � � � ��� ���

c) La disminución de la energía potencial de la cadena es igual, en todo instante, al incremento de su energía cinética. Considerando los instantes indicados en el enunciado, será:

2k p

1 8 8� � 5.11 m/s2 6 3

gE E mv mg v� � � � �

x

l-x m-mx

mx f

mxg

- 93 -

Física Universitaria: Problemas de Física Conservación de la energía. M11.7

7. Una partícula de masa m está situada en la cima de una semiesfera lisa, de radio R, que está apoyada por su

base sobre un plano horizontal. Cuando desplazamos ligeramente la partícula de su posición de equilibrio comienza a deslizar sobre la superficie de la esfera. La posición de la partícula queda determinada en cada instante por el ángulo � que forma el radio-vector correspondiente con la vertical. Determinar el valor del ángulo para el cuál la partícula se despega de la semiesfera.

Conservación de la energía: 2 21cos 2 (1 cos )

2mgR mgR mv v gR� �� � � �

Componente radial de la ecuación del movimiento: 2 2

cos cosv vmg N m N mg mR R

� �� � � �

Sustituyendo en esta ecuación el valor de la velocidad, tenemos

cos 2 (1 cos ) (3cos 2)N mg mg mg� � �� � � � � En el instante en que la partícula se despega de la semiesfera, la reacción normal será nula (“se rompe la ligadura”), de modo que

max max max 48.23cos 2 cos3

2º� � �� � �

mg

N

R

- 94 -

Física Universitaria: Problemas de Física Conservación de la energía. M11.8

8. Un carrito pesa 8 kg y se mueve sin fricción, con una velocidad de 1 m/s,

sobre unos rieles rectilíneos y horizontales. Dejamos caer verticalmente un pequeño objeto de 2 kg de masa sobre el extremo delantero del carrito. Inicialmente, la velocidad del objeto es nula; pero, como consecuencia de su fricción con el carrito (coeficiente 0 = 0.1), termina quedando en reposo sobre el carrito, con tal que la longitud l de éste sea suficientemente grande. a) Calcular la velocidad final del sistema y el tiempo empleado en alcanzarla. b) Determinar el valor mínimo de la longitud del carrito que permita que el objeto se detenga sobre él.

a) Sea vf la velocidad final del sistema cuando el objeto, después de deslizar sobre el carrito, queda en reposo sobre el mismo. Puesto que no existen fuerzas externas al sistema carrito objeto que tengan componentes en la dirección horizontal, se conserva la cantidad de movimiento del sistema en esa dirección. Esto es,

0 f f 08( ) 1 0.8 m/s

10MMv M m v v v

M m� � � � � �

La única fuerza que actúa sobre el objeto (en la dirección de su movimiento) es la de fricción, de modo que la ec. de su movimiento es:

2obj obj obj 0.1 9.8 0.98 m/sf ma mg ma a g� �� � � � � �

de modo que el objeto posee un movimiento uniformemente acelerado, partiendo del reposo, hasta que adquiere la velocidad final vf. Podemos escribir:

ff obj f f

obj

0.8 0.82 s0.98

vv at v a t ta

� � � � �

b) El trabajo realizado por la fuerza de rozamiento durante el recorrido x que realiza el objeto sobre el carrito es igual a la pérdida de energía cinética que experimenta el sistema durante ese proceso. Esto es,

2 20 f

1 1 ( )2 2

Mv M m v mgx�� � �

Despejamos el recorrido x d esta expresión y obtenemos 2 2

2 20 f

1 1 8 1 10 0.8( ) 4 3.22 2 2 2 0.41 m 41cm0.1 2 9.8 1.96

Mv M m vx

mgx�

� �� � � �

� � � � �� �

de modo que la longitud del carrito deberá ser mayor de 41 cm a fin de que el objeto no se salga por la parte trasera del carrito.

(sigue…)

l

M m v0

M

m

v

f f

l

- 95 -

Física Universitaria: Problemas de Física Conservación de la energía. M11.9

Otro método b) La única fuerza que actúa sobre el carrito en la dirección de su movimiento es la fuerza de rozamiento. Calculamos la aceleración del carrito:

2car car car

20.1 9.8 0.25 m/s8

mf Ma mg Ma a gM

� �� � � � � � � � � � �

En el referencial del carrito, el objeto se mueve inicialmente con una velocidad –v0 (esto es, hacia la parte trasera del carrito) y va disminuyendo su velocidad (relativa) hasta que finalmente queda en reposo en ese referencial. La aceleración del objeto con respecto al carrito (aceleración relativa) es:

� � 2rel obj car 0.98 25 1.23 m/sa a a� � � � � �

de modo que el objeto presenta un movimiento uniformemente retardado relativo al carrito. Utilizando la bien conocida fórmula de la cinemática 2 2

0 2v v ax� � , durante el proceso de frenado, obtenemos el espacio que recorre el objeto sobre el carrito:

2 22 00 rel

rel

10 2 0.41 m 41cm2 2 1.23vv a x xa

� � � � � ��

de modo que la longitud del carrito deberá ser mayor de 41 cm a fin de que el objeto no se salga por la parte trasera.

- 96 -

Física Universitaria: Problemas de Física Fuerzas centrales. M12.1

1. La velocidad de escape es aquella velocidad inicial que necesita cualquier proyectil para escapar de la atracción gravitatoria de un planeta u cualquier otro cuerpo de gran masa ¿Calcular la velocidad de escape desde la superficie terrestre?

Datos: Radio ecuatorial de la Tierra: 6371 km; Masa de la Tierra: 5.979�1024 kg; G = 6.672�10-11 (SI)

Conservación de la energía para el lanzamiento del proyectil con una velocidad inicial vescape para conseguir que llegue a una hipotética distancia infinita con velocidad nula:

2escape

1 0 02

Mmmv GR

� � �

De modo que

2escape escape

1 22

Mm GMmv G vR R

� �

Podemos expresar este resultado en función de g (intensidad del campo gravitatorio en la superficie del planeta), ya que

2 2

mG mg g GR R� �

� �

de modo que

escape2 2GMv gR

R� �

Sustituyendo valores para la Tierra, tenemos 11 24

3escape 6

2 2 6.672 10 5.979 10 11.2 10 m/s 11.2 km/s6.371 10

GMvR

�� � � �� � � � �

La velocidad de escape en la superficie de la Tierra es de 40 320 km/h. A velocidades inferiores, el proyectil se convertiría en un satélite artificial en órbita elíptica alrededor de la Tierra. Esta velocidad inicial es aplicable tan solo a proyectiles; esto es, objetos que carecen de un medio de propulsión propio de modo que dependen de su impulso inicial para vencer la atracción gravitatoria.

r v

R vesc

- 97 -

Física Universitaria: Problemas de Física Fuerzas centrales. M12.2

2. Órbita geoestacionaria. Supóngase que se desea establecer en el espacio una base interplanetaria que se mueva en una órbita circular en el plano ecuatorial de la Tierra y a una altura tal que permanezca siempre sobre el mismo punto. a) ¿Cuál deberá ser el radio de esa órbita? b) ¿Qué velocidad tendrá en satélite en dicha órbita?

Para que la órbita sea geoestacionaria, el satélite deberá girar en su órbita con la misma velocidad angular con que rota la Tierra alrededor de su eje:

52 2 7.29 10 rad/s86164T

� �� �� � � �

siendo el periodo el correspondiente a un día sideral ( = 86164 s). a) Aplicamos la Ley de la Gravitación Universal al satélite en órbita, teniendo en cuenta que la fuerza de atracción gravitatoria le imprime una aceleración centrípeta, de módulo 2R� , de modo que

2 2 3 3cp2 2

Mm GMF G ma m R GM R RR

� ��

� � � � �

Para determinar la masa de la Tierra, consideraremos un cuerpo de masa m en caída libre cerca de su superficie, siendo RT el radio de la Tierra:

2T

2T

gRMmF G mg MR G

� � �

que sustituimos en la expresión anterior para obtener 1/32 2

3 T T2 2

gR gRR R� �

� ��� �� � � �� ���

De modo que

� �

1/3266

T25

9.8 6.37 1042.14 10 m 6.62

7.29 10R R

� ��� �� �� �� � � �� �� �� �� ���

b) La velocidad orbital se puede calcular multiplicando su velocidad angular por el radio orbital:

5 67.29 10 42.14 10 3072 m/s 11060 km/hv R� �� � � � � � �

R

F M

m

- 98 -

Física Universitaria: Problemas de Física Fuerzas centrales. M12.3

3. Un satélite de comunicaciones describe una órbita circular ecuatorial en el mismo sentido de rotación de la Tierra, a una altura de 800 km sobre su superficie. ¿Por dónde sale el satélite, por el Este o por el Oeste? ¿Durante cuánto tiempo permanecerá visible (sobre el horizonte) desde un lugar llano y despejado situado en el Ecuador?

Datos: Radio ecuatorial de la Tierra: 6371 km; Masa de la Tierra: 5.979�1024 kg; G = 6.672�10-11 (SI)

Radio de la órbita del satélite

s 6371 800 7171 kmR � � �

Ángulo central correspondiente al orto (salida) y ocaso (ocultación) del satélite en el horizonte:

T

s

6371cos 0.8882 717154.64º 0.9537 rd

RR

� � �

� �

La velocidad angular asociada con el movimiento del satélite en su órbita alrededor de la Tierra se deduce a partir de la Ley de la Gravitación Universal:

2 3T s Ts cp s s s s2 3

s s

1.04 10 rd/sM m GMG m a m RR R

� � �� � � � �

La velocidad de angular de la Tierra es 5T

2 7.29 10 rad/s86400�

� �� � � .

Puesto que estamos observando el satélite desde la Tierra, su velocidad angular relativa a la Tierra será:

4rel s T 9.67 10 rad/s� � � �� � � � �

donde el signo positivo indica que la salida (orto) se produce por el OESTE. Dicho de otro modo, la velocidad angular del satélite es superior a la terrestre, por lo que el satélite “se adelanta” a ésta. El satélite estará visible mientras recorre un arco de circunferencia de ángulo central � con una velocidad angular relativa a la Tierra �rel; esto es,

4rel

0.9537 986 s9

16 m.67 10

in 26 st �� �� � � �

RT

Rs A

B

�T

�s

�rel

Hemisferio Norte

- 99 -

Física Universitaria: Problemas de Física Fuerzas centrales. M12.4

4. Masa del Sol. Conocidos los semiejes mayores de las órbitas de la Tierra y de la Luna, 149.6×106 km y 384.0×103 km, respectivamente y los correspondientes periodos de revolución, 1 año y 27.32 días, calcular la masa del Sol en unidades de la masa de la Tierra.

A partir de la Ley de la Gravitación Universal deducimos la expresión de la Tercera Ley de Kepler para órbitas circulares:

2 22 2 3 3 2 3

2 2

4 4MmF G m R GM R R T RR T GM

� �� �� � � � �

Aplicamos la Tercera Ley de Kepler a los sistemas Sol-Tierra y Tierra-Luna y dividimos las expresiones miembro a miembro:

22 3

2 3T T 2 3S ST T T L T

2 32L S L T T L2 3

L LT

4

4

T RGM MT M R T R

T M R M T RT R

GM

���� �� � � � �� � �� � �� � � � � �� � �� �� �� �� � � �� �����

Finalmente, sustituimos los valores dados en el enunciado del problema:

2 33 6 3S

T

27.34 149600 5.60 10 59.129 10 331 10365.24 384

MM

� � � �� �� �� � � � � � �� �� �� �� �� �

Conocida la masa de la Tierra, podemos calcular la del Sol: Masa de la Tierra: 5.9736 × 1024 kg Masa del Sol: 1.9891 × 1030 kg

RT

RL

LS T

- 100 -

Física Universitaria: Problemas de Física Fuerzas centrales. M12.5

5. Lunas de Marte. Los semiejes mayores de las dos Lunas del planeta Marte, Phobos y Deimos, miden 9.408×103 km y 23.457×103 km, respectivamente. El periodo de revolución orbital de Phobos es de 4.65 horas. Con esos datos se deben calcular la masa del planeta Marte y el periodo de revolución de Deimos.

A partir de la Ley de la Gravitación Universal deducimos la expresión de la Tercera Ley de Kepler para órbitas circulares:

2 22 2 3 3 2 3

2 2

4 4MmF G m R GM R R T RR T GM

� �� �� � � � �

Aplicamos la Tercera Ley de Kepler al sistema Marte-Phobos:

� �

3632 224P

M 22 –11P

9.408 104 4 1.758 10 kg6.693 x 10 4.65 3600

RMG T� � �

� � � ��

Aplicamos la Tercera Ley de Kepler a los sistemas Marte-Phobos y Marte-Deimos y dividimos las dos expresiones miembro a miembro:

22 3

3/2P P 3/22 3M P P D

D P2 32D D P2 3

D DM

423.4574.65 18.31 h9.4084

T RGM T R RT T

T R RT R

GM

���� �� � � � �� �� � ��� � � � �� �� �� �� ���� � � �� �����

M

P

D

RD

RP

- 101 -

Física Universitaria: Problemas de Física Movimiento armónico simple. M13.1

1. Una partícula realiza un movimiento armónico simple con una frecuencia angular de 10 rad/s. En el instante inicial, la partícula presenta una elongación positiva de 8 cm y se mueve con una velocidad de 60 cm/s acercándose a la posición de equilibrio. a) Determinar la ecuación del movimiento (elongación en función del tiempo). b) Determinar el primer instante en el que la partícula pasa por la posición de equilibrio. c) Ídem en el que la partícula se encuentra instantáneamente en reposo.

a) Comenzamos escribiendo las ecuaciones del m.a.s. e imponiendo las condiciones iniciales:

� �

2 2 20

0 22 20

0

sensen sen

0coscos cos

x Ax A t x A

t vv Av A t A

�� � �

� �� � � ��

�� ��� �� �� � ��� � � � � � �� ��� � ���� � ���� �� �� ���� ��

de modo que 2 2

2 2 200

0

0

608 64 36 100 10 cm 0.10 m10

10 8tag 1.33 126.87º 0.705 rad 2.215 rad60

vA x A =

xv

��

� � �

� � � ��� �� �� � � � � � � �� �� �� ��� � �

�� � � � � � �

y la ecuación pedida es � �0.10sen 10 0.7 cos 10 0.705 (S.I05 .)vx tt � � �� ��

b) Deberá transcurrir menos de medio periodo o ciclo de oscilación; esto es, para una fase de � rad, como se ilustra en el diagrama fasorial:

� �

1

1

1

0 0.10sen 10 0.705

10 00.093 s

.7050.0295

t

tt

� �

� �

� �

� �

c) Análogamente, el instante corresponde a una fase de 3�/2 rad, como se ilustra en el diagrama fasorial:

2

2

2

0 cos 10 0.705310 0.7052

00.079 . s5 250

t

t

t

��

� �

� �

� �

x0

x

x

+

vv0

Diagrama fasorial

c)

a)

b)

- 102 -

Física Universitaria: Problemas de Física Movimiento armónico simple. M13.2

2. Una partícula de 4 kg de masa se mueve a lo largo del eje x bajo la acción de la fuerza2

16F x�� ��� �� �� �� ���

, donde

x se expresa en metros y F en newtons. Cuando t = 2 s, la partícula pasa por el origen, y cuando t = 4 s, su velocidad es de 4 ms-1. Determinar la frecuencia, la amplitud del movimiento y el ángulo de fase inicial.

Puesto que la fuerza es de la forma 2

, con 16

F kx k �� � � , se trata de un movimiento

armónico simple, sobre el eje x, alrededor de la posición de equilibrio x = 0, de modo que x representa la elongación. La frecuencia angular de tal movimiento es

2 rad 1 Hz 16 s16 4 8 s 2 16

k Tm

� � �� �

�� � � � � �

La expresión de la elongación y de la velocidad en función del tiempo es � �0 0sen cosx A t v A t� � � � �� � � �

Con las condiciones impuestas en el enunciado, será

� �

0 0

0

0 0

0

2 0 2 rad42 s sen 2 0

2 2

32 32 32 2 m14.4s2cos

4 24 s cos 4 432 32 32 2

5 2co

3 rad4

m1s

2

4.4s

4

t A

A

t AA

�� � � �

� �

� � � � � �

� �� �

� � �

� �� �

��� � � � � � ����� � � ���� � � � � ������� � � � ��� � �� ��� ��� ��� ��� � � ���� � � � � �� � �� ���� ��� ���

� ��

- 103 -

Física Universitaria: Problemas de Física Movimiento armónico simple. M13.3

3. Un objeto de 12 kg cuelga del extremo de un muelle de constante elástica 300 N/m. Iniciamos las oscilaciones libres del sistema comunicando al objeto una velocidad inicial hacia abajo de 50 cm/s. a) Calcular la frecuencia y el periodo de las oscilaciones. b) Determinar la amplitud de las oscilaciones. c) Expresar la elongación en función del tiempo. d) Si inicialmente hubiéramos comunicado al objeto una velocidad mayor, indíquense cuales de las magnitudes anteriores se modificarán y de que forma, justificando las respuestas.

a) La frecuencia de las oscilaciones del sistema masa-muelle está definida por la magnitud de la masa y el valor de la constante elástica del muelle:

300 2512

5 1=2 2

5 rad/s

0.80 Hz 1.26 s

km

T

��

� � �

� � � �

� � � �

b) De conformidad con el enunciado de este problema, al instante inicial (t = 0) le corresponde una elongación nula, por lo que la fase inicial también será nula. De la expresión de la elongación se obtiene por derivación la de la velocidad

sen cosx A t v A t� � �� � En el instante inicial (t = 0) serán:

0 00 0t x v A�� � �

de modo que

0 505

10 cmvA�

� � �

c) Elongación en función del tiempo: 0.10sen 5 (SI)x t�

d) La frecuencia de las oscilaciones es una propiedad intrínseca del sistema masa-muelle, por lo que no depende de las condiciones iniciales, de modo que no se modificaría en el supuesto del enunciado. La amplitud de las oscilaciones depende de las condiciones iniciales, de tal modo que, si la velocidad inicial impuesta aumenta, también lo hace la amplitud. Aunque la fase inicial depende de las condiciones iniciales, en este caso continuará siendo nula, ya que tan solo cambia el módulo de la velocidad inicial, pero no su dirección.

m

k

v0

x

x=0

- 104 -

Física Universitaria: Problemas de Física Movimiento armónico simple. M13.4

4. Disponemos de tres muelles idénticos. a) Los unimos en serie, uno a continuación de otro, y fijamos uno de los extremos libres al techo, en tanto que del otro extremo suspendemos un bloque de masa m. Cuando duplicamos la masa suspendida, el extremo inferior del conjunto serie desciende una distancia adicional h. ¿Cuánto vale la constante elástica de cada muelle? b) Con los tres muelles disponemos ahora un montaje paralelo (cada muelle tiene un extremo unido al techo) y suspendemos una masa 3m. ¿Cuál será la frecuencia de las oscilaciones de este sistema?

Sea k la constante elástica de cada uno de los muelles.

a) Asociación de muelles en serie: serie

1 1ik k

Los tres muelles idénticos equivalen a un muelle único cuya constante elástica sería

serie serieserie

1 3 33kk k k

k k� � �

La tensión F (carga, en este caso) que soporta un muelle es directamente proporcional a la deformación x mismo (ley de Hooke):

�� ��

FF kx F k x kx

� � �

Así, para el muelle “equivalente serie” tenemos: �

serie serie

23 3mg mg mgk k k

hmghh

�� � � �

b) Asociación de muelles en paralelo: par ik k�

Los tres muelles idénticos equivalen a un muelle único de constante elástica

par93 mgk k

h� �

La frecuencia de las oscilaciones de una masa sujeta a un muelle viene dada por

sistkm

��

que en nuestro caso nos conduce a

par 9 /3 3

3 1 322

gk mg hm

gh hm

�� �

� �� � � � �

k

k

k

m h

- 105 -

Física Universitaria: Problemas de Física Movimiento armónico simple. M13.5

5. Un arquero tensa la cuerda de su arco desplazando su punto medio 60 cm hacia atrás; para ello, debe ejercer una fuerza sobre la cuerda que se incrementa uniformemente desde cero hasta 120 N. a) Calcular la constante elástica del arco. b) ¿Qué trabajo se ha realizado para tensar el arco? En esas condiciones, se dispara horizontalmente una flecha de 20 g hacia un blanco situado a 30 m. c) Determinar la velocidad con la que sale impulsada la flecha. d) ¿Cuánto se desviará verticalmente del blanco?

a) En el supuesto de que la fuerza ejercida sea directamente proporcional a la elongación que experimenta el punto medio de la cuerda del arco, i.e., que se cumpla la ley de Hooke, será

120 200 N/m0.60

FF kx kx

� � � �

b) El trabajo realizado será igual a la energía elástica almacenada en el sistema 2 2

elast1 1 200 0.60 36 J2 2

W E kx W� � � � � �

c) La energía elástica se convierte en energía cinética de la flecha, que saldrá impulsada con una velocidad v0:

2 elastelast k 0 0

21 2 36 60 m/s2 0.020

EE E mv vm

�� � � � �

d) Bajo la acción gravitatoria, la flecha describe una trayectoria parabólica.

02

220

1 22

x v tgy xvy gt

� ���� � ��� � ����

de modo que, para x = 30 m, será 2

2

9.8 30 1.225 m2 60

h � � � ��

h

xy

P

v

v0

- 106 -

Física Universitaria: Problemas de Física Movimiento armónico simple. M13.6

6. Un bloque de masa m se deja caer desde una altura h sobre un muelle de constante elástica k. Determínese el acortamiento del muelle justamente antes de iniciarse la subida de la masa.

Dado que el sistema es conservativo, aplicamos el Principio de Conservación de la Energía. Tanto en el instante inicial como en el de máximo acortamiento del muelle son nulas la energía cinética, ya que el bloque se encuentra instantáneamente en reposo, de modo que tiene lugar una conversión de energía potencial gravitatoria en elástica; esto es,

2 21 2 2 02

mgh mgx kx kx mgx mgh� � � � � �

Resolvemos la ec. cuadrática, 2 2 2mg m g kmgh

xk

3 ��

de modo que tenemos dos soluciones 2

1

2

2

2 0

2 0

mg mg mgx hk k k

mg mg mgx hk k k

� ���� � � .�� ���

� ���� � � *�� ���

una positiva y otra negativa, que corresponden a los instantes en los que el bloque se encuentra instantáneamente en reposo, tal como se indica en la figura. Puesto que nos piden el máximo acortamiento del muelle, éste será

2

1 2 0mg mg mghk k k

x� ���� ��� ���

� .

Obsérvese que el acortamiento del muelle correspondiente a la posición de equilibrio, con el bloque encima de

él, vale 0mgxk

� , y que la amplitud de las oscilaciones del sistema, en el caso de que el bloque quede unido al

muelle, vale 2

2mg mgA hk k

� ���� ��� ���

Caso particular: Si fuese h = 0, sería: 1 2 02 0mg mg mgx x x A

k k k� � � �

k

m

k

h

x1

Ep,grav=0 x2

+

En re

poso

En reposo

m

m

- 107 -

Física Universitaria: Problemas de Física Movimiento armónico simple. M13.7

7. En el interior de un tubo vertical, cerrado por su base inferior, hay un muelle de 100 espiras y constante elástica k. Sobre dicho muelle se apoya, (por simple contacto, sin ningún tipo de unión), un bloque de masa m. Determinar: a) La amplitud máxima con que puede oscilar el bloque sin despegarse del muelle y la frecuencia angular ' de las oscilaciones. b) Si decidiésemos recortar el muelle para que la misma masa oscilase con frecuencia angular 2' ¿cuántas espiras deberíamos dejar?

a) Puesto que el muelle y el bloque tan solo están en contacto (sin unión fija), ambos permanecerán en contacto en tanto que el muelle esté comprimido. Esto es, el muelle nunca podrá alargarse más allá de su longitud natural, ya que no puede ser “traccionado” por el bloque. Durante las oscilaciones del sistema, la energía permanece constante (sistema conservativo). Tomamos como nivel de referencia para la energía gravitatoria la correspondiente a la posición más baja del bloque y sabemos que la distancia entre las dos posiciones extremas es el doble de la amplitud de las oscilaciones (i.e., 2A) y que en ellas el bloque se encuentra instantáneamente en reposo.

Conservación de la energía En la elongación máxima positiva (longitud natural del muelle):

cin grav el0 0E E mgA E E mgA� � � �

En la elongación máxima negativa (muelle comprimido):

�2 2cin grav el

10 2 22

E E mgA E k A E mgA kA� � � � � � �

De donde: 2 22 2 2E mgA mgA kA mgA

kmgA kA� � � � ��

b) La frecuencia de las oscilaciones del sistema masa-muelle es /k m�� , por lo que, para una masa dada, será

2 4k k kk

��

))� � �

Por otra parte, es bien sabido que dos muelles idénticos conectados uno a continuación de otro equivalen a un muelle de constante elástica dada por

eq eqeq

1 1 1 2 22kk k k

k k k k� � � � �

A la inversa, si tenemos un muelle de constante k y lo cortamos en dos partes iguales, dispondremos de dos muelles de constante k’ = 2k ; si volvemos a cortar en dos parte iguales cada una de estas mitades, tendremos cuatro muelles de constante k’’ = 2k’ = 4k, que es lo que pretendemos. Por consiguiente, puesto que el muelle original tiene 100 espiras, deberemos quedarnos con un muelle de tan solo 25 espiras.

m

k

-A

+A

0

x

- 108 -

Física Universitaria: Problemas de Física Movimiento armónico simple. M13.8

8. El movimiento del pistón de un automóvil de 500 g de masa podemos considerarlo vibratorio armónico simple. Si la carrera del pistón (doble de la amplitud) es 10 cm y la velocidad angular del cigüeñal es de 3600 r.p.m., calcular: a) Aceleración del pistón en el extremo de la carrera. b) Fuerza resultante que se ejerce sobre él en el extremo de la carrera. c) Velocidad máxima del pistón.

Comenzamos escribiendo las ecuaciones del m.a.s.: 2 2sen( ) cos( ) sen( )x A t v A t a A t x� � � � � � � � �� � � � � � � � �

a) La aceleración del pistón en el extremo de su recorrido es la que corresponde a la elongación máxima x=A (amplitud), de modo que

2a A��

con A = 0.05 m y 2 rad3600 376.9960 s�

�� � � , resulta

2 320.05 376.9 m7.

s9 11 10a �� � �

b) Aplicando la segunda ley de Newton, tenemos 3 30.5 7.11 1 3. 5 100 5 NF ma� � � � ��

c) La velocidad máxima en el m.a.s. se alcanza en la posición de elongación nula y viene dada por

max 0.05 376.99 m18.85s

v A�� � � �

- 109 -

Física Universitaria: Problemas de Física Movimiento armónico simple. M13.9

9. Un reloj de péndulo que ha sido cuidadosamente ajustado para marcar el tiempo correcto en un lugar donde g = 9.823 m/s2 retrasa 40 s por día cuando se lleva a otro lugar geográfico. ¿Cuánto vale g en ese lugar?

El periodo de las oscilaciones del péndulo (simple) viene dado por la expresión:

2 lTg

�� ,

de modo que en un lugar donde el valor de la gravedad sea menor, el periodo será mayor (el reloj atrasa).

Para expresar cuantitativamente esta circunstancia, relacionaremos el cambio �T en el periodo con el �g en la intensidad gravitatoria mediante el método de la derivada logarítmica:

� 1 �2

T lT l

�1 � 1 � � �22 2

g g g Tg g g T

� � � � �

lo que pone de manifiesto que un aumento del periodo (el reloj retrasa) está asociado con una disminución del valor de la intensidad de la gravedad. Sustituyendo los valores dados en el enunciado:

2� 40� 2 2 9.823 0.009 m/s86400

Tg gT

� � � � � �

2� � 9.823 0.009 9.814 m/sg g g g g g) )� � � � � � �

Otro método Relacionamos los periodos y las intensidades gravitatorias en los dos lugares dividiendo miembro a miembro las expresiones del periodo del péndulo:

0 20 0 0

00

2

2

lTg T Tg g g

T g TlTg

��� ��� � ��� �� � � �� � ��� � �� �����

A partir del dato del retraso por día, podemos obtener la relación (cociente) entre los periodos en los dos lugares:

0

0 0 0 0

� 40 86440186400 86400

T TT T TT T T T

�� � � � �

Combinando estos dos resultados, calculamos la intensidad gravitatoria pedida: 2 2

200

864009.823 9.814 m/s86440

Tg gT

� � � �� �� �� � �� �� �� ��� � �

- 110 -

Física Universitaria: Problemas de Física Movimiento armónico simple. M13.10

10. Introducimos agua en un tubo en U de sección constante, colocado en posición vertical, y desnivelamos momentáneamente las dos ramas de la U. a) Calcular la frecuencia y el periodo de las oscilaciones que se producen; b) ¿Variarán los resultados del apartado anterior si en lugar de agua se colocara mercurio? Datos: densidad del mercurio, 13.6 g/cm3.

En la situación de equilibrio, el líquido alcanza el mismo nivel en las dos ramas. Supongamos que, por algún procedimiento, se desnivela el líquido en las dos ramas. Consideremos un instante genérico, ilustrado en la figura, cuando el nivel del líquido en cada una de las ramas presenta una elongación x (desplazamiento respecto del nivel de equilibrio). Método de las fuerzas: Sobre la totalidad de líquido actúa una fuerza F, no compensada, igual el peso de la columna de líquido de sección S y altura 2x; i.e,

�2F m g S x g�)� � � �

de sentido contrario a la elongación x. La masa de todo el líquido es �SL. La ecuación del movimiento se escribe

� 22 0gF mx S x g SLx x xL

� �� � � � ��� �� ��

que es la ecuación diferencial correspondiente a un m.a.s. con

2 2 22

g lTl g

�� �

�� � �

Método de la energía:. Expresamos la energía del sistema correspondiente a una elongación genérica, teniendo en cuenta que la porción líquida contenida en el codo no cambia su energía potencial. Los centros de masas de las ramas verticales de líquido se han marcado mediante un punto, cuya altura es la mitad de la longitud de dicha columna. Tenemos,

� �

� �

2k p

2 22 2 2 2

12 2 2

1 12 2 2

m h x m h xE E E mx h x g h x gl l

m mmx g h x h x mx g h xl l

� � � �� �� �� �� � � � � � � �� �� �� �� �� �

� � � �� � � � � � � �� �� � � �

� �

Puesto que el sistema es conservativo, su energía permanece constante, de modo que

d 22 0dE m gmxx gxx mx x xt l l

� ���� � � � ��� ��� ��� � � ��

y como esta relación debe satisfacerse en cualquier instante, cualquiera que sea la velocidad del sistema, podemos escribir

2 0gx xL

� ���

que es la ecuación diferencial correspondiente a un m.a.s. (la misma obtenida anteriormente). b) La frecuencia tan solo depende de la longitud de la columna líquida. Si el tubo contiene un mismo volumen de agua, de mercurio,... la frecuencia de las oscilaciones serán las mismas, con independencia de la densidad del líquido.

x

F

2x

x

h

x

Ep=0

- 111 -

Física Universitaria: Problemas de Física Movimiento armónico simple. M13.11

11. Una balsa de madera de 3 m de largo, 2 m de ancho y 0.5 m de grosor tiene una densidad de 0.6 g/cm3. Esta balsa está flotando en agua de densidad 1 g/cm3, en ella se sube una persona que pesa 75 kg; en el momento de subirse se producen unas oscilaciones verticales. a) Calcular la frecuencia de dichas oscilaciones. b) Ídem para cuando la persona se baja de la balsa.

Calculamos la superficie, el volumen y la masa de la balsa: 2

3

3m

3 2 6 m3 2 0.5 3 m

0.6 10 3 1800 kg

SVm V�

� � �

� � � �

� � � � �

Cuando se sube la persona, se produce un hundimiento adicional A tal que

pm p

m

75 0.021 m 21 mm600 6

mSAg m g A

S�

�� � � � �

hasta que se alcanza una nueva posición de equilibrio. Supongamos que la balsa se hunde una distancia adicional x (elongación) a partir de la situación de equilibrio de flotación; aparecerá una fuerza restauradora igual al peso del volumen de agua adicional desplazado

rec aF Sxg�� �

Entonces, aplicando la 2ª ley de Newton, podemos escribir �rec a sist sist a 0F Sxg m x m x Sg x� �� � � � ��� ��

que es la ecuación diferencial de un movimiento armónico simple cuya frecuencia angular viene dada por

a a

sist sist

12

Sg Sgm m� �

� ��

� �

a) Cuando la persona se encuentra sobre la balsa, será

1 1000 6 9.8 0.89 Hz 1.12 s2 1800 75

T��

� �� � �

b) Cuando la persona se baja

1 1000 6 9.8 0.91 Hz 1.10 s2 1800

T��

� �� � �

x

Frec

- 112 -

Física Universitaria: Problemas de Física Movimiento armónico simple. M13.12

12. Una moneda permanece en reposo sobre una plataforma horizontal que realiza un movimiento armónico simple de frecuencia v. a) Si la plataforma oscila verticalmente, ¿cuál será el valor máximo de la amplitud A de las oscilaciones que permita a la moneda permanecer en contacto permanente con la plataforma? b) Supongamos ahora que la plataforma oscila horizontalmente y que sea μ el coeficiente de rozamiento estático entre la moneda y la plataforma. ¿Cuál será, entonces, el valor máximo de A que permita a la moneda permanecer en reposo respecto a la plataforma, sin deslizar?

a) Consideramos una elongación genérica x y escribimos la ec. del movimiento de la moneda en el supuesto de que permanezca sobre la plataforma:

2N mg mx N mg mx mg m x�� � � � � ��� �� donde hemos tenido en cuenta la relación existente entre la aceleración y la elongación en el m.a.s.; esto es, 2x x�� ��� , que presenta su valor máximo para x = A. La moneda permanecerá sobre la plataforma en tanto que N >0, de modo que

22 2 20

4g gN mg m A A�� � �

� � . * � .

b) Ahora, interviene una fuerza de rozamiento estático de valor f N mg� �- �

De nuevo consideramos una elongación genérica x y escribimos la ec. del movimiento de la moneda en el supuesto de que permanezca en reposo respecto de la plataforma,

2f mx m x�� � ��� de modo que la fuerza de rozamiento tiene en cada instante dirección opuesta a la elongación y presenta su valor máximo para x = A, tal que 2

máxf m A��

Combinando estas dos ecuaciones tenemos:

22 2 24g gm A mg A � �

� �� � �

* * �

mg

N x

mg

N f x

- 113 -

Física Universitaria: Problemas de Física Movimiento armónico simple. M13.13

13. Un deportista que pesa 60 kg se lanza desde un puente sujeto a una cuerda elástica de 30 m de longitud natural (practica “puenting”) llegando justamente a tocar la superficie del agua situada a 40 m por debajo en la vertical de donde inició el salto. a) Calcular la constante elástica de la cuerda. b) Determinar la aceleración máxima a la que estará sometido el deportista y en qué punto la adquiere. c) Una vez que se haya amortiguado la caída, de modo que la cuerda permanezca siempre tensa, determinar la frecuencia de las oscilaciones verticales que experimentará el deportista.

a) Conservación de la energía entre A y C:

� �

20 2

0

1 202

mglmgl k l l kl l

� � � � ��

de modo que

2

2 60 9.8 40 N470.410 m

k � � �� �

b) En un instante genérico, la ecuación del movimiento del deportista se escribe en la forma

F mg ma� � siendo F la tensión de la cuerda. La aceleración máxima ocurre en el instante en el que la cuerda elástica presenta su máximo alargamiento (l = 40 m) y, por ende, su máxima tensión;

� �máx máx máx máxmáx máx� �kF mg ma k l mg ma a l g

m� � � � � �

o sea 2

máx470.4 10 9.8 78.4 9.8 68.6 m/s 7

60a g�

� � � � � �

c) Una vez que se haya amortiguado la caída, de modo que la cuerda permanezca siempre tensa, el deportista experimentará un m.a.s. vertical cuya frecuencia es

470.4 12.8 rad/s 0.446 Hz 2.24 s60 2

k Tm

�� �

� �� � � � � � �

La posición de equilibrio corresponde a un alargamiento x0 de la cuerda tal que su tensión equilibre el peso del deportista; esto es

0 060 9.80 1.25m470.4

mgF mg kx mg xk

�� � � � � �

l 0 =

30 m

l = 4

0 m

F

mg

av0

A

B

C

- 114 -

Física Universitaria: Problemas de Física Movimiento armónico simple. M13.14

14. Un muelle de 20 cm de longitud y 10 N/m de constante elástica está unido por uno de sus extremos a un eje vertical y en el otro extremo a una pesa de 100 g situada un plano horizontal. Cuando el eje rota a razón de 1 r.p.s., la pesa describe una trayectoria circular sobre el plano horizontal (sin fricción) al tiempo que oscila radialmente. Determinar el alargamiento del muelle y la frecuencia de las oscilaciones.

Describimos la situación en un sistema de referencia en rotación en el que la pesa se encuentra en reposo. En este sistema de referencia, la fuerza que proporciona el muelle extendido será igualada por la “fuerza centrífuga”; esto es, muelle cfF F� . Siendo �� la velocidad angular de rotación del eje y �l el alargamiento que experimenta el muelle, será:

� �2 2

2 2 22

2� � � � ml lk l m l l k m l ml l kk m

m

� �� � �

� �� � � � � �

� �

� �� � �� �

Y sustituyendo los valores dados en el enunciado

22

22

0.20 2 7.896� 0.130 m 13 cm10 60.5220.1

ll km

��

� �

�� � � � �

� �

La frecuencia angular de las oscilaciones vendrá dada por la bien conocida expresión general:

10 1010 rad/s 1.59 Hz0.1 2 2

km

�� �

� �� � � � � �

- 115 -

Física Universitaria: Problemas de Física Superposición de m.a.s. M15. 1

1. Expresar la elongación en función del tiempo para los m.a.s. siguientes: a) Movimiento con periodo de 1 s tal que para t = 0 son v = 3 cm/s y a = 0. b) Movimiento con frecuencia 0.25 Hz tal que para t = 0 son v = 0 y a = �16 cm/s2. c) Es la superposición de los m.a.s. cuyas ecuaciones, en el S.I. de unidades, son:

1 240.06sen 2 0.08sen(2 )x t x t�

� �� ���� � ��� ����

Ecuaciones del m.a.s.:

0

02 2 2 2

0 0

sensen( )cos( ) 0 cos

sen( ) sen

x Ax A tv A t t v Aa A t x a A x

�� �� � � � �

� � � � � � �

�� � �� � � �� ��� �� � � �� �� �� �� �� � � � � � � � ��� ��

a) 21s 2 rad/sTT�

� �� � �

Como a0 = 0, será x0 = 0 y sen+ = 0, de modo que + = 0º o 180º y cos+ = ±1. Como v0 = +3 cm/s (positiva), será cos+ >0, de modo que + = 0º, de donde resulta

00

3 0.48 cm2

vv A A�� �

� � � �

0.0048 sen(2 ) (S.I.)x t�# �

b) 0.25 Hz 2 2 0.25 rad/s2�4 ' �4 �� � � � � �

Como v0 = 0, será cos+ = 0, de modo que + = ±90º y sen+ = ±1. Como a0 = �16 cm/s2 (negativa), deberá ser sen+ >0, de modo que + = +90º, resultando

00 2 2 2

16 16 4 6.48 cm( / 2)

ax A' � �

� �� � � � � � �

0.0648sen( ) 0.0648 cos( ) (S.I.)2 2 2

x t t� � �� �#

c) Se trata de superponer (sumar) dos m.a.s. de la misma dirección y de la misma frecuencia. Recurrimos a la representación fasorial (vide figura):

2 2 2 2 21 2 1 2 1 22 cos 0.06 0.08 2 0.06 0.08 cos 45º 0.01679A A A A A � �� � � � � � � � � � �

1 1 2 2

1 1 2 2

sen sen 0.06 sen 45ºtg 0.3465cos cos 0.06 cos 45º 0.08

A AA A

� ��

� �� �

� � �� � �

0.13 m 19.11º 0.33 radA �� � �#

0.13sen(2 0.33) (S.I.)x t� �# �

v0

x0

a0

A1

A2

A A

� � 45º

v0

x0

a0

+

- 116 -

Física Universitaria: Problemas de Física Superposición de m.a.s. M15. 2

2. Calcular la ecuación del movimiento armónico resultante de la composición de:

1 23sen 4sen6 4

x t x t� �' '� � � �� � � � � �

Ambos m.a.s. tienen la misma frecuencia, lo que nos permite utilizar de modo inmediato la representación de Fresnel, compleja o fasorial de tales movimientos. Esto es,

� �

1 30º1

2 45º2

3 2.60 1.503sen 30º4 2.83 2.834sen 45º

x t

x t

� � � � �� �� �� � � �� � � � �� �� ���

��

jj

1 2 38.6º 0.67 rad5.43 4.33 6.94 6.94� � � � � �� � � j

y la ecuación del movimiento resultante es � �6.94sen 0.67x t'�

Otro método A partir de la representación geométrica de Fresnel (fasorial) del m.a.s., resulta inmediato la determinación del módulo de m.a.s. resultante:

� �2 2 2 21 2 1 2 1 22 cos 3 4 2 3 4 cos15º 6.94A A A A A + +� � � � � � �

así como la fase inicial del mismo:

1 1 2 2

1 1 2 2

sen sen 3 sen 30º 4 sen 45ºtg 0.80 38.6º 0.67 radcos cos 3 cos30º 4 cos 45º

A AA A

+ ++ ++ +

� �� � � � � �

� �

y la ecuación del movimiento resultante es � �6.94sen 0.67x t'�

A1

Re

Im

A2

A

+1 +2

+

- 117 -

Física Universitaria: Problemas de Física Superposición de m.a.s. M15. 3

3. Una partícula se mueve en el plano xy de modo que las componentes cartesianas de su aceleración vienen dadas por ax = -9x y ay = -9y (S. I.) a) ¿Qué tipo de movimiento se produce sobre cada eje? b) Determinar los vectores de posición y de velocidad de la partícula sabiendo que en el instante inicial el punto pasa por el origen de coordenadas con una velocidad: v0 = i + 2j (S. I.) c) Determinar la trayectoria del punto y el máximo alejamiento del origen de coordenadas.

a) Las ecuaciones que describen el movimiento de la partícula sobre los ejes x e y son:

sen(39 9 09

)sen(39 )0

x A tx x x xy ty y By y

��

� � � � � ���� �� �� �� � � ��

�� ���� ��

ya que, por ser de la forma, 2 0x x�� ��� , representan sendos m.a.s. simples en direcciones perpendiculares, con la misma frecuencia angular 3 rad/s�� . b) Imponemos las condiciones iniciales a las soluciones de

sen( ) 0 0 sen 30

sen( ) 0 0 sen 3x A x A t

ty B y B t

� �� �

� �� � � �� �� �� "� �� �� � � �� �� �

Derivando con respecto al tiempo, obtenemos las componentes de la velocidad, a las que imponemos las condiciones iniciales:

3 cos3 3 1 1/ 3 m0

3 cos3 3 2 2 / 3 mx A t x A A

ty B t y B B

� �� � � �� �� �" �� �� �� � � �� �� �

� �� �

de modo que podemos escribir 1

32

3

sen 3 cos3 3sen 32cos3 6sen 3sen 3

x t x t x ty t y ty t

��� �� � �� � �� ��� � �� � �� � �� � �� �� �� �����

� ��� ��

c) Se trata de dos m.a.s. de la misma frecuencia, en fase y en direcciones perpendiculares, de modo que la trayectoria de la partícula es rectilínea, como resulta fácil comprobar eliminando el tiempo entre la ecuaciones paramétricas de la misma. Esto es,

1

32

3

sen 31/ 3 12 / 3 2se 3

2n

x t xyy t

y x

��� ���� � � ������

��

El máximo alejamiento del origen será: 2 2 1 49

539

D A B� � � � �

A

B

x

y

v0

- 118 -

Física Universitaria: Problemas de Física Geometría de masas. M16.1

1. Una pieza de maquinaria, de masa m, está constituida por un disco de material homogéneo, de radio R, al que le falta una porción circular de radio R/2, tal como se indica en la figura. a) Determinar la posición del centro de masa de la pieza. b) Calcular los momentos de inercia de la pieza con respecto a los ejes coordenados (x,y,z) indicados en la figura. Expresar los resultados en función de R y de la masa m de la pieza.

Comenzamos determinando las masas del disco completo (sin merma alguna) y de la porción suprimida (negativa) en función de la masa m de la pieza:

211 1

2 212 4

223 21 2 4

4 43 3( / 2)

1133

mm R m mmm R Rm m mm m m R m

��

�� ��

��

� �� � �� � � �� �� � �� � ��� � �� � � � � � �� � �� � � � �� �� � �� � �� � ��� �� �� a) Centro de masa. Aplicamos el teorema correspondiente a la determinación del centro de mas de un cuerpo compuesto por otros cuerpos:

13

1 1 2 2cm

1 2

02

6

Rmm x m x Rxm m m

��� � � �

� b) Momentos de inercia. Con carácter general, determinamos los momentos de inercia de un disco de masa m y radio R con respecto a los ejes que se indican:

2 21 12 4zz xx yy zz yyI mR I I I I mR� � � �

y aplicando el Teorema de Steiner

2 2 2 2 2 2' ' ' '

1 3 1 52 2 4 4z z y yI mR mR mR I mR mR mR� � � � � �

Ahora, aplicamos estos resultados para el cálculo de los momentos de inercia pedidos en el enunciado del problema:

22 2 2 2 2

1 1 2 2

22 2 2 2 2

1 1 2 2

22 2 2 2 2

1 1 2 2

1 1 1 4 1 1 1 1 154 4 4 3 4 3 4 3 48 481 5 1 4 5 1 1 5 114 4 4 3 4 3 4 3 48 481 3 1 4 3 1 2 1 262 2 2 3 2 3 4 3 8 48

xx

yy

zz

RI m R m R mR m mR mR

RI m R m R mR m mR mR

RI m R m R mR m mR mR

� ���� � � � � � ��� ����

� ���� � � � � � ��� ����

� ���� � � � � � ��� ���� de modo que

2 2 25 11 1316 48 24xx yy zzI mR I mR I mR� � �

x

y

R

x

y

R

y’

- 119 -

Física Universitaria: Problemas de Física Geometría de masas. M16.2

2. Una semiesfera y un cono, ambos macizos y homogéneos, construidos con el mismo material y del mismo radio, están soldados por sus bases. Calcular el valor máximo de la altura del cono que permita el conjunto comportarse como un tentetieso (i.e., que no vuelque) al apoyarlo sobre una superficie horizontal.

Centro de masa del cono: d

d

z Vz

V�

��

Descomponemos el cono el rodajas (discos) perpendiculares al eje de revolución, de modo que

2d d r z RV r z r zR H H

�� � �

2 2 32 2 2

2 20

1d d d3 3

HR R HV V r z z z R HH H

� � � �� � � � �� � �

2 2 43 2 2

2 20

1zd d4 4

HR R HV z z R HH H� � �� � �� �

2 2

2

1d 34

1 4d3

R Hz Vz H

V R H

�# � � �

��

(medido desde el vértice)

Centro de la hemisfera.

2 2 3 3

sen cos d sen dd d sen ( sen )d sen dr R z R z RV r z R R R

� � � �

� � � � � � � �

� � � �

� � � � �

03 3 3

/ 2(como es sabido)

2sen d 3

V R R�

� � � �� � �� 04 40

4 3 4

/ 2/ 2

send sen cos d4 4

Rz V R R�

� �� � � � �

� �� �� � � � �� ��

� �

4

3

d 342 8d3

Rz V

z RV R

�# � � �

��

Centro de masa del cuerpo compuesto Deberá estar situado por debajo del punto O para que el cuerpo se comporte como un tentetieso (equilibrio estable). En las condiciones críticas será:

cono cono hemisf hemisfcm cono cono hemisf hemisf

cono hemisf

2 3 2 2

0 0

1 3 2 3 33 4 3 8

3

V z V zz V z V zV V

R H H R HR R H R� �

�� � � �

� � �

z

R

r

z

z R�

z

r

R

H

O

z

- 120 -

Física Universitaria: Problemas de Física Sistemas de partículas. M17.1

1. Una varilla ligera de longitud l puede girar sin rozamiento alrededor de su centro. Se colocan en sus extremos sendas masas 2 m y m y se abandona el sistema desde la posición de la varilla horizontal. En el instante en que la varilla alcanza la posición vertical, determinar: a) la celeridad de las masas; b) la vector cantidad de movimiento del sistema; c) la velocidad del centro de masa del sistema.

Posición del c.m. del sistema constituido por las dos masas:

�cm

2 2 22

2 3 6

l l lm mly

m m

� ���� � � �� ��� � � � �

a) Puesto que el sistema es conservativo, aplicamos el Principio de Conservación de la Ener-gía, tomando como nivel de referencia la posición inicial:

� 210 2 32 2 2 3l l glmg mg m v v� � � � �

b) La cantidad de movimiento del sistema es la suma de las cantidades de movimiento de las dos masas:

1 2 23glmv mv mv m� � � � � �p p p i i i i

c) La cantidad de movimiento del sistema es igual al producto de su masa por la velocidad de su centro de masa; esto es,

� cm cm1 1

3 3 3 3 3imv glm v

m m � � � � �

pp v v i i i

m2 m

m

2 mv

Ep=0

v

vmv

x

y

cm

l/2

l/2

- 121 -

Física Universitaria: Problemas de Física Sistemas de partículas. M17.2

2. Un muchacho está situado en el centro de una plataforma circular de radio R = 3 m. La plataforma, inicialmente en reposo, puede girar sin fricción alrededor de su eje. El muchacho hace girar alrededor de su cabeza una masa de 2 kg sujeta con una cuerda de longitud R/2, con una velocidad tal que la masa da una vuelta cada 3 segundos. a) ¿Debido a la conservación de qué magnitud física se pone en movimiento la plataforma? Razonar la respuesta. b) Determinar la velocidad y sentido del movimiento de la plataforma y la energía cinética del sistema en movimiento, sabiendo que el momento de inercia del conjunto plataforma y muchacho, respecto al eje de la plataforma, es 600 kg�m2.

Datos: 2

2 2m m

2 2.094 rad/s 2 1.5 4.5 kg.m3 2

RI m��

� ���� � � � � ��� ���

Por tratarse de un sistema aislado, el momento angular del sistema completo con respecto al eje de rotación permanecerá constante, de modo que la plataforma adquiere una rotación en sentido opuesto a la de la masa de 2 kg. Conservación del momento angular del sistema aislado:

mm m p p p m

p

4.5 2. 0.0157 rad/094 s600

II II

� � � �� � � � �

Energía cinética: 2 2 2 2

k m m p p1 1 1 14.5 2.094 600 0.0157 9.870 0.074 9.994 J2 2 2 2

E I I� �� � � � � � � � �

m

- 122 -

Física Universitaria: Problemas de Física Sistemas de partículas. M17.3

3. Dos prismas triangulares, de masas M y m, y anchuras a y b, están en reposo, tal como se indica en la figura adjunta, sobre un tablero hori-zontal liso. Las superficies de contacto entre los dos prismas son, tam-bién, perfectamente lisas. Determinar el retroceso del prisma inferior hasta el instante en que la cara vertical del prisma superior alcanza el tablero horizontal. Aplicación numérica: M = 10 kg, m = 2 kg, a = 40 cm y b = 10 cm.

Método 1º. Centro de masas. Todas las fuerzas externas que actúan sobre el sistema constituido por los dos prismas (pesos y reacción normal en la base del prisma inferior) tienen dirección vertical. Puesto que no hay fuerza externa alguna que de componente en la dirección horizontal, se conserva la cantidad de movimiento del sistema en esa dirección (aunque no en la dirección vertical). En consecuencia la componente horizontal de la velocidad del centro del sistema permanece constante en el transcurso del movimiento. Puesto que partimos del reposo, la posición horizontal del c.m. del sistema permanece invariable. Expresamos esta circunstancia en la forma siguiente. En la figura adjunta hemos representado las

posiciones de los c.m. de cada uno de los prismas mediante las distancias x1 y x2 que se indican. En cada una de las dos situaciones, inicial y final, se determina la posición del c.m. del sistema mediante el Teorema de Varignon:

� �1 21 2

cm

M x X m x a b XMx mxxM m M m

� � � � ��� �

� � [1]

de modo que será � �1 2 1 2Mx mx M x X m x a b X� � � � � � �

De donde se sigue

1Mx 2mx� 1Mx� 2MX mx� � � � �

m a b mX

M m X m a b

mX a bM m

� � �

� � �

� ��

Y sustituyendo los valores dados en el enunciado

�2 40 10 5 cm10 2

X � � ��

(sigue…)

b

M

m

a

X

x1

x2

x2

a-b

x1

b

M

m

a

- 123 -

Física Universitaria: Problemas de Física Sistemas de partículas. M17.4

Método 2º. Conservación de la cantidad de movimiento. Todas las fuerzas externas que actúan sobre el sistema constituido por los dos prismas (pesos y reacción normal en la base del prisma inferior) tienen dirección vertical. Puesto que no hay fuerza externa alguna que de componente en la dirección horizontal, se conserva la cantidad de movimiento del sistema en esa dirección (aunque no en la dirección vertical). Siendo ddXt

la velocidad del prisma inferior,

ddxt

la componente horizontal de la velocidad relativa del prisma superior respecto del

inferior, d dd dx Xt t

� ��� � �� ��� la componente horizontal de la velocidad absoluta del prisma superior,

la conservación de la componente horizontal de la cantidad de movimiento del sistema se expresa en la forma:

� d d d0 0d d dxX x XMV m v V M mt t t

� ���� � � � � ��� ��� [2]

De modo que � �

0 0

d d d d d

d d d dX a b

M X m x X M m X m xm mX x X x

M m M mmX a b

M m

� � � � � �

� � � � � �

� � ��

� �

Y sustituyendo los valores dados en el enunciado

�2 40 10 5 cm10 2

X � � � � ��

b

Mm

a

X a-b

v vx

V

x

- 124 -

Física Universitaria: Problemas de Física Sistemas de partículas. M17.5

4. Una cuña de masa M se encuentra en reposo sobre un tablero horizontal, como se muestra en la figura. En la parte más alta de la cuña reposa un pequeño bloque de masa m, a una altura h sobre el tablero horizontal. Todas las superficies son perfectamente lisas. Abandonamos el sistema, de modo que el bloque desciende y la cuña retrocede. Encontrar la velocidad de retroceso de la cuña en el instante en que el bloque toca el tablero horizontal.

Todas las fuerzas externas que actúan sobre el sistema formado por la cuña y el bloque (pesos y reacciones normales en la base de la cuña) tienen dirección vertical, por lo que se conserva la componente de la cantidad de movimiento en la dirección horizontal. Esta circunstancia se expresa mediante la ecuación h 0mv MV� � [1]

Además, al no existir rozamientos, el sistema es conservativo y podemos escribir la ecuación que expresa la conservación de la energía,

�2 2 2h v

1 12 2

mgh MV m v v� � � [2]

donde V es la velocidad final de la cuña y vh y vv son las componentes horizontal y vertical, respectivamente, de la velocidad del bloque en el momento en que éste abandona a la cuña. Disponemos de dos ecuaciones y tres incógnitas (V, vh y vv). Necesitamos una tercera ecuación que obtenemos de imponer que, en el sistema de referencia de la cuña, la velocidad del bloque �h v,v V v� es siempre tangente al plano inclinado, es decir

�vv h

h

tg tgv v v Vv V

� �� � ��

[3]

Despejando vh en la ecuación [1] y sustituyéndola en la ecuación [3] tenemos

h v tgM M mv V v Vm m

�� �� ��� � � � �� ���

Sustituyendo estas dos expresiones en la ecuación [2] nos queda una ecuación con V como incógnita única, de la que despejamos V para obtener:

� � � �2 2

22 2 2

2 2tg 1 tg tg

m gh m ghVM m M M m M m M m� � �

� �� � � �� � � � �� � � �� �

Multiplicando numerador y denominador por cos2� y extrayendo la raíz cuadrada de la expresión anterior, obtenemos finalmente la velocidad pedida:

2 2

2

2 cossen

m ghVM m M m

��

�� �� �� ��

Vv

vh

vv

M

m

h

- 125 -

Física Universitaria: Problemas de Física Sistemas de masa variable… M18.1

1. Una balanza de resorte está ajustada para leer el cero. Desde una altura de 5 m sobre el platillo de la balanza, dejamos caer un chorro de perdigones, a razón de 20 perdigones por segundo, que chocan contra el platillo, rebotan hacia arriba con la misma velocidad y salen abandonando definitivamente el platillo. Si cada perdigón pesa 200 mg, )cuál será la lectura de la balanza?

La velocidad que tiene cada uno de los perdigones cuando chocan contra el platillo se calcula a partir del principio de conservación de la energía:

21 2 2 9.8 5 9.9 m/s2

mgh mv v gh� � � � � �

Cuando un perdigón colisiona elásticamente con el platillo, experimenta un cambio en su cantidad de movimiento expresado por:

final inicial� ( ) 2m m m� � � � � �p p p v v v

Este cambio está dirigido hacia arriba, de modo que el platillo tendrá que proporcionar impulso en esa dirección; i.e., ejercer una fuerza hacia arriba sobre cada perdigón. Como al platillo llegan n = 20 perdigones por segundo, el cambio de la cantidad de movimiento por unidad de tiempo, esto es, la fuerza, será

d � 2d

n nmt

� ���� � ��� ��� pF p v

Y sustituyendo los valores tenemos

�6 32 2 20 200 10 9.9 79.2 10 N 8.1gF nmv � �� � � � � � � � �

Esta será la indicación de la balanza, ya que los perdigones ejercen sobre el platillo una fuerza igual y opuesta a la que el platillo ejerce sobre ellos (Tercera Ley de Newton).

v

v

+ F

- 126 -

Física Universitaria: Problemas de Física M.19 Colisiones

1. Un automóvil de 1200 kg, que inicialmente viaja con una velocidad de 27 m/s, choca contra la parte posterior de un camión que pesa 9000 kg y se desplaza en la misma dirección y sentido a 22 m/s. La velocidad del camión en el instante inmediato posterior al choque es de 23 m/s. a) Determinar el coeficiente de restitución. b) Calcular la energía mecánica que se ha perdido en el choque. ¿Cómo se explica esta pérdida de energía?

a) Conservación de la cantidad de movimiento durante el choque:

A A B B A A B B

BA A B B

A

9000( ) 27 (22 23) 19.5 m/sm 1200

m v m v m v m vmv v v v

) )� � �

) )� � � � � � �

Aplicamos la definición del coeficiente de restitución:

A B

A B

19.5 0.2327

7022

v vev v

) )� �� � � � �

� �

de modo que se trata de una colisión parcialmente elástica. b) Durante el choque disminuye la energía mecánica (cinética) del sistema.

2 2 2 2A B

1 1 1 1

2 2 2 21200 27 9000 22 2 615 400 JkE mv mv� � � � � � �

2 2 2 2A B

1 1 1 1

2 2 2 21200 19.5 9000 23 2 608 650 JkE mv mv) ) )� � � � � � �

6750 Jk k kE E E �)4 � � �

La energía mecánica (cinética) que ha desaparecido se ha convertido en otras formas de energía, asociadas a la deformación de los vehículos colisionantes.

A B

vB vA

A B

v’B v’A

- 127 -

Física Universitaria: Problemas de Física M.19 Colisiones

2. Un coche de 1500 kg que viaja hacia el este con una velocidad de 25 m/s choca en un cruce con una furgoneta de 2500 kg que viaja al norte con una velocidad de 20 m/s. Hallar la dirección y el módulo de la velocidad de los vehículos después de la colisión, suponiendo que los vehículos sufren una colisión perfectamente inelástica (es decir, quedan unidos).

Puesto que la colisión es perfectamente elástica, los dos cuerpos permanecen unidos después del choque y tan solo se conserva la cantidad de movimiento del sistema coche-furgoneta. Cantidad de movimiento del coche:

1 1500 25 37500 kg.m/sp � � �

Cantidad de movimiento de la furgoneta:

2 2500 20 50000 kg.m/sp � � �

Cantidad de movimiento de sistema coche-furgoneta:

2 21 2 37500 50000 62500 kg.m/sp p p� � � � �

Velocidad común después del choque es

1 2

62500 15.625 m/s4000

pvm m

� � ��

En una dirección que forma un ángulo �, como se indica en la figura, tal que

2

1

50arctg arctg 53º37

pp

�� � �

p1

p2 p

- 128 -

Física Universitaria: Problemas de Física M.19 Colisiones

3. Consideremos dos partículas, de masas respectivas m1 y m2, que efectúan una colisión perfectamente elástica frontal, de modo que sus velocidades antes de la colisión sean v1 y v2 = v1, con � 0. Supongamos que fuesen iguales las energías cinéticas iniciales de las partículas. Calcular el valor (o valores) que deberá tener el parámetro para que la partícula "1" quede en reposo después de la colisión y la relación entre las masas de ambas partículas para que sea posible esa situación.

Ambas partículas tienen inicialmente la misma energía cinética, con v2 = v1; por consiguiente

22 2 2 2 21 2

1 1 2 2 1 1 2 22 1

1 12 2

m vm v m v m v m vm v

�� ��� �� � � �� �� ���

Conservación de la cantidad de movimiento, con v2 = v1:

1 1 2 2 1 1m v m v m v)� � �212 2 1 2 2 1 1 1 2

2

1mm v v v v v v v vm

� � � �) ) )� � � � � � �

Regla de Huygens-Newton, con v2 = v1:

2 1v v) )� � � �2 1 2 2 1 11v v v v v v�)� � � � � � � �

Igualando las expresiones de v’2 en las dos últimas ecuaciones:

� �2 2 21 11 1 2 1 0v v� � � � � � � �� � � � � � � � �

Resolvemos la ecuación de segundo grado:

1

2

1

2

10.4142 (alcance) 0.1725.831 1 1 1 2

1 2.4142 (frontal) 5.83

mmmm

��� � � ���� 3 � �� � � 3 � ��� � �����

v1 v2

v’2

reposo

� > 0

v1 v2

v’2

reposo

� < 0

- 129 -

Física Universitaria: Problemas de Física M.19 Colisiones

4. Una pelota de ping-pong rebota escaleras abajo, escalón por escalón, de tal modo que todos los rebotes son idénticos. El coeficiente de restitución o percusión entre la pelota y las baldosas vale 0.9 y cada escalón tiene una altura de 19 cm. Determinar la altura de rebote de la pelota sobre cada escalón.

Consideremos un rebote aislado; por definición de coeficiente de restitución (e), será

2 21 1 22 2

2 1 221 12 2

2con

2

v gh v hv ev ev hv gh

�� ��� � ��� ���

La pelota de ping-pong realiza el bote desde una altura h1, respecto al escalón; después de botar alcanza una altura h2. Como todos los rebotes son idénticos, después del bote estará a la misma altura respecto al escalón siguiente, o sea

1 2h h a� �

de modo que 2

222 2

2 1h ee h a

h a e� �

� �

de donde 2

2 2

1

0.9 0.810.19 0.19 0.81 81cm =1 0.9 0.1981

m

100 cm19

h

h

� � � � ��

� � �

h1

h2

h1

v2

v1

h1 h2

- 130 -

Física Universitaria: Problemas de Física M.19 Colisiones

5. Dejamos caer una pelota de ping-pong desde una altura h0 sobre un suelo duro, liso y horizontal. Observamos que después del quinto rebote la pelota sólo asciende hasta una altura h0/2. a) Determinar el coeficiente de restitución de los rebotes. ¿Es el mismo en todos ellos? b) Calcular la fracción de energía que se disipa en los rebotes. ¿Es la misma en todos ellos? ¿Por qué? c) ¿Cuántos rebotes deberán transcurrir para que la altura de rebote se reduzca a la centésima parte de h0?

a) En la figura representamos los rebotes sucesivos. Obviamente, la relación existente entre las velocidades indicadas y las respectivas alturas es:

2 para 0,1,2,...n nv gh n� �

donde el subíndice n se refiere a la velocidad y a la altura alcanzadas tras el n-ésimo rebote. Designamos por e el coeficiente de restitución y aplicamos la regla de Huygens-Newton a cada uno de los rebotes sucesivos:

1 02

2 1 01 03

3 2 0

...

nn n

v ev

v ev e vv ev e v

v ev e v �

� ����� � ��� � ��� � �������

El valor del coeficiente de restitución, que es el mismo en todos los rebotes, lo calculamos a partir de los datos para el quinto rebote:

5 5 1/105 55 0

0 0

1 (0.5 0.93)2

3v hv e v e ev h

� � � � � �

b) La pérdida de energía en el n-ésimo rebote será: 2 2 2 2 2 2 21 1 1 1

1 12 2 2 2

2 2

� ( ) ( 1)�� (1 ) (1 ) cte

n n n n n n n

nn

E E E mv mv m e v v mv eEE e E e

E

� �� � � � � � � �

# � � � � � � �

La fracción de energía perdida en cada rebote es la misma en todos ellos y viene dada por 2� (1 ) 0.13 13%

n

E eE

� � �� �

c) A partir de la expresión de la velocidad tras el n-ésimo rebote, 0n

nv e v� , tenemos:

2 0

0 0 0 0

log( / )2 log log2 log

log 0.012 log 0

33.2 rebotes.933

n nn n n n nv h h h h he e n e nv h h h e

n

� � � � �

# � ��

h0 h1

h2

v0

v2 v1

v1

- 131 -

Física Universitaria: Problemas de Física M.19 Colisiones

6. Sea un sistema formado por una masa de 300 g y un muelle de constante elástica igual a 120 N/m, situados en reposo sobre un plano horizontal liso, como se indica en la figura. Sobre esta masa, inicialmente en reposo, choca elásticamente otra masa idéntica a la anterior con una velocidad de 5 m/s. Determinar: a) La velocidad de cada una de las masas inmediatamente después del choque. b) La amplitud del movimiento armónico subsiguiente. c) El tiempo que tardan en volver a chocar las dos masas. d) ¿Variará el resultado del apartado anterior si la velocidad con la que choca la masa incidente fuese el doble? Justifíquese la respuesta.

a) Aplicamos el Principio de Conservación de la Cantidad de Movimiento y la Regla de Huygens-Newton (con e = 1) :

1 2 1 2 1

1 2 1 2 2

1

2

2 0 05 m/2 s2

mv mv mv v v v vv v v v v v v v

vv v

� � � �) ) ) ) )� � � � �� � � �� � � � � � � �� � � �) ) ) ) )� � � � � �

) �) ��� � �� � � � ��

por lo que el primer bloque queda en reposo y el otro adquiere la velocidad del primero. b) Después de la colisión, la energía cinética del bloque unido al muelle se convierte íntegramente en energía potencial elástica cuando este bloque alcanza su elongación máxima (amplitud, A); esto es,

2 21 1 0.35 0.25 m2 2 120

25 cmmmv kA A vk

� � � � � �

c) Calculamos la frecuencia de las oscilaciones y, a partir de ella, el periodo:

2 220 rad/s 0.31 s20

k Tm

� ��

�5(6

� � � � � �678

Obviamente, el tiempo que tardan en reencontrarse los dos bloques es el correspondiente a media oscilación, i.e., un semiperiodo, de modo que

0.162

sTt4 � �

d) No variará el resultado, puesto que el tiempo que tardarán en reencontrase los dos bloques será un semiperiodo, en cualquier caso (en tanto que sea m1=m2), siendo T independiente de la amplitud de las oscilaciones.

v k m m 1 2

(reposo) v1 2

A

1 2

- 132 -

Física Universitaria: Problemas de Física M.19 Colisiones

m1m2

v1k

m1+ m2

v’ k

m2

v’2

m1

v’1

7. Un bloque de masa m1 = 1 kg desliza sobre una superficie horizontal lisa con una velocidad de 6 m/s. El bloque choca con otro de masa m2 = 2 kg que está en reposo unido a un resorte horizontal de constante k = 1200 N/m. Determinar la frecuencia con la que oscilará el sistema tras el choque y la amplitud del movimiento en los siguientes casos: a) El choque es totalmente inelástico y las dos masas quedan adheridas después del choque. b) El choque es perfectamente elástico.

a) Choque totalmente inelástico: La cantidad de movimiento se conserva:

� 11 1 1 2 1

1 2

1' 6 2 m/s3

mm v m m v v vm m

� � " � � ��

1 2

1200 20 rad/s3

km m

�� � ��

10 Hz 3.18 Hz2�

� �� �

� � " �

Conservación de la energía después del choque:

12

� 21 2

1'2

m m v� � 2 1 2 0.1 m220

m m vkA A v Ak �

)�)" � � � " �

b) Choque perfectamente elástico: El coeficiente de restitución es e=1. �1 2 1 2 1 1' ' ' 'v v v v v v� � � " � �

La cantidad de movimiento se conserva: � � �1 1 1 1 2 2 1 1 2 1 1 1 2 1 1 2 1m v m v m v m v m v v m m v m m v) ) ) ) )� � � � � " � � �

1 21 1

1 2

1 6 2 m/s3

m mv vm m

� �) � � � ��

2' 6 2 4 m/sv � � �

por lo que la masa 1 vuelve hacia la izquierda.

2

1200 24.5 rad/s2

. H2

3 9 zkm

�� � �

�� � � " � " �

Conservación de la energía después del choque:

12

22 2

12

m v) � 2 22

241200

0.16 mmkA A v Ak

)" � � " �

Nota: Las masas no vuelven a chocar ya que T = 0.256 s y el tiempo que tarda m1 en recorrer la amplitud es:

1

0.16 0.08 s2

Atv

� � �)

- 133 -

Física Universitaria: Problemas de Física M.19 Colisiones

8. Una partícula de masa m se desplaza a velocidad v cuando choca elásticamente con otra de igual masa que se encontraba en reposo. Después de la colisión, la trayectoria de la primera partícula forma un ángulo de 30º con respecto a su dirección original. a) Determinar la velocidad de cada una de las partículas tras el impacto. b) Calcular la percusión que recibe cada una de las partículas.

a) Conservación de la cantidad de movimiento:

1 2 1 2 [1]m m m� � � �v v v v v v

Conservación de la energía cinética: 2 2 2 2 2 21 1 1

1 1 1 22 2 2 [2]mv mv mv v v v� � � �

Las ecuaciones [1] y [2] corresponden a una suma vectorial en cuadratura, por lo que las direcciones de las partículas después del choque determinan un ángulo recto, de modo que

1 2 290º 60º 0 º90 3 º� � �� � � � �

y las velocidades de las partículas después del choque son

1 20.87 0.cos30 50º cos 60ºv v v vv v� � � �

b) La percusión que experimenta cada una de las partículas es igual a la variación de su cantidad de movimiento durante el choque; i.e., � � �� �p vm , de modo que

��2 2 22� � �� � � � vvm mv mv m 1 2 2m� � � � v� � 11 2 2 mv� �� �

en las direcciones en que cambia la velocidad de cada una de las partículas.

Otro método Escribimos de nuevo las ecuaciones [1] y [2] que expresan, respectivamente, la conservación de la cantidad de movimiento y de la energía cinética durante el choque, descomponiendo la primera de ellas en sus componentes longitudinal y transversal:

3 31 2 2 2 2 12 2

1 2 2 221 11

1 2 2 1 2 2 2 22 22 22 2 2

1 2 2 2 2 2 2 21 2 2 1

cos coscos30º cos

0 sen 30º sen sen sen sen2 4

v v v v v vv v v

v vv v v vv v

v v vv v v v v v

� ��� � � �

�� � � � ��� �� � � �� ��� �� � � � �� �� �� �� �� �� �� � � � � ���

que constituyen un sistema de tres ecuaciones con tres incógnitas 1 2 2( , , )v v �) ) . Resolviendo dicho sistema de ecuaciones se obtienen de nuevo los resultados que ya conocemos.

2 2 2 232 2 1 14

2 2 22 2 2 2 2 21 2 1 1

2 2 2 2 2 142 22 2

2 2 22 1

cos 3

4cos 1 sen 1 cos4 4

v v v vv

v v v v v vv v

v v v

� � �

�� � � ����� �� � � � � � � � ������ � ���

De modo que 2v 2 231 14 3v vv v� � � 2 2 2 31

1 1 1 1 14 22 3v v v vv v v� � � �

30ºv

v

v1

v2 60º

30º

v

v1

�2 60º

30º

�1 �v1

- 134 -

Física Universitaria: Problemas de Física M.19 Colisiones

9. Una partícula de masa m se desplaza con una velocidad v cuando choca elásticamente con otra de igual masa que se encontraba en reposo. Después de la colisión, la trayectoria de la primera partícula forma un ángulo de 60º con respecto a su dirección original. a) Determinar la velocidad de cada una de las partículas tras el impacto. b) Calcular la percusión que recibe cada una de las partículas.

a) Conservación de la cantidad de movimiento:

1 2 1 2 [1]m m m� � � �v v v v v v

Conservación de la energía cinética: 2 2 2 2 2 21 1 1

1 1 1 22 2 2 [2]mv mv mv v v v� � � �

Las ecuaciones [1] y [2] corresponden a una suma vectorial en cuadratura, por lo que las direcciones de las partículas

después del choque determinan un ángulo recto, de modo que

1 2 290º 30º 0 º90 6 º� � �� � � � �

y las velocidades de las partículas después del choque son

1 20.50 0.cos 60 87º cos30ºv v v vv v� � � �

b) La percusión que experimenta cada una de las partículas es igual a la variación de su cantidad de movimiento durante el choque; i.e., � �m� �p v� , de modo que

�2 2 22� mm mv mv� �� � � �v v� 1 2 2m� � � � v� � 1 2 0.87mv� �� �

en las direcciones en que las que cambia la velocidad de cada una de las partículas.

Otro método (nada recomendable) Escribimos de nuevo las ecuaciones [1] y [2] que expresan, respectivamente, la conservación de la cantidad de movimiento y de la energía cinética durante el choque, descomponiendo la primera de ellas en sus componentes longitudinal y transversal:

1 11 2 2 2 2 12 2

1 2 2 223 1 1

1 2 2 1 2 2 2 22 22 22 2 2

1 2 2 2 2 2 2 21 2 2 1

cos coscos 60º cos

3 30 sen 60º sen sen sen sen2 4

v v v v v vv v v

v vv v v vv v

v v vv v v v v v

� ��� � � �

�� � � � ��� �� � � �� �� �� � � � � �� �� �� �� �� �� �� � � � � ���

que constituyen un sistema de tres ecuaciones con tres incógnitas 1 2 2( , , )v v �) ) . Resolviendo dicho sistema de ecuaciones se obtienen de nuevo los resultados que ya conocemos.

2 2 2 212 2 1 14

2 2 22 2 2 2 2 21 2 1

2 2 2 2 2 12 22 2

2 2 22 1

cos

3 4 3 3cos 1 sen 1 cos4 4 4

v v v vv

v v v v v vv v

v v v

� � �

�� � � ����� �� � � � � � � ������� � ���

De modo que 2v 2 211 14 v vv v� � � 2 2 23

1 1 1 1 14 2 0.5v v v vv v v� � � �

60º v

v

v1

v2

60º

30º

v

v1

�2

60º

30º

�1 �v1

- 135 -

Física Universitaria: Problemas de Física M.19 Colisiones

10. En una mesa de billar, la bola 1 se mueve con una velocidad de 5 m/s y choca con la bola 2 de modo que ésta se introduce en la tronera de la esquina, como se indica en la figura. Determinar la velocidad y la dirección de cada bola después del choque, considerándolo: a) completamente elástico; b) con un coeficiente de restitución de 0.95.

a) Es bien sabido que cuando una partícula incidente colisiona elásticamente contra otra partícula de la misma masa que se encuentra en reposo, las partículas se mueven después de la colisión en direcciones que son perpendiculares entre sí. Así, el esquema de colisión es el que se ilustra en la figura, con �1 = 30º y �2 = 60º. Puesto que en esta colisión se conservan tanto la cantidad de movimiento como la energía cinética, podemos escribir:

1 2 1 1 2 1

1 2 1 22 2 2 2 2 21 1 1

1 2 1 1 2 12 2 2

cos30º cos 60º sen 60º cos 60ºsen 30º sen 60º 0 cos60º sen 60º 0

mv mv mv v v vmv mv v v

mv mv mv v v v

� �) ) ) )� � � � ��� �� �� �) ) ) )� � � �� �� �� �� �) ) ) )� � � �� ���

de modo que disponemos de un sistema de tres ecuaciones con dos incógnitas (v’1 y v’2). A partir de las dos primeras se obtiene fácilmente

12

1 2 1 1 12

2 11 2 2

sen 60º sen 60º cos 60º sen 60º sen 60ºcos 60ºcos 60º sen 6

4.33m/s2.50m/s0º cos 60º 0

v v v v vv

vvv v v

� � �) ) )� � � �� �� � � � � �� � �)) ) �� �� � �� )� ��

) �

b) Para resolver este problema de choque oblicuo analizamos por separado sus componentes frontal y transversal (tangencial). Para ello, tomamos una base vectorial apropiada definida por los ejes x e y: el eje x sobre la recta que une los centros de las esferas en el instante de la colisión (componente frontal) ; el eje y es normal al anterior (componente transversal), como se muestra en la figura. Componente frontal: Aplicamos el Principio de Conservación de la Cantidad de Movimiento y la Regla de Huygens-Newton,...

1 1 21 2 1 2 1 2 1 2

1 2 1 2 1 2 1 22 1 2

1 coscos cos 21cos cos

2

xx x x x

x x x x x xx

ev vmv mv mv v v vev v e v v v v e v v v

�� �

� �

� ��� ) ��� �) ) ) )� �� � �� �� � � � � �� � �) ) ) ) �� � � � � � �� � ��� ) �����

Componente transversal: Suponiendo que las bolas sean lisas, las fuerzas impulsivas que actúan durante la colisión son normales a las superficies en el punto de contacto de modo que no tiene componentes sobre el eje y. En consecuencia, las componentes de las veloci-dades normales a la línea de los centros no se alteran durante la colisión; i.e.,

1 1 2 2sen 0y yv v v�) )� �

Para calcular el ángulo �1 bajo el que sale dispersada la esfera incidente, calcularemos primero �1+�2; esto es,

v’1

v1

v’2

�2 �1

v1

1

2 30º

v1 v’1

v’2

x (frontal) y (transversal) �2

�1

bola 1 (incidente bola 2

(blanco)

- 136 -

Física Universitaria: Problemas de Física M.19 Colisiones

1 1 21 2 2

11 2

sen 2tg( ) tg1 1cos2

y

x

v vev ev

�� � �

)� � � �

�) �

Sustituimos los valores dados en el enunciado del problema para obtener los resultados numéricos correspondientes a cada caso: e = 1 (colisión elástica)

21

1 2

1 2

1

2 1

2

2 1

1 1

25cos 60º 2.5 m/s

4.33 m/s 2.5 m

05sen 6

/s

30

0º 4.33 m/s 0

2tg( ) tg 901

ºº1

xx

y y

v v

vvv v

�� � � � �

���� ) � ��� )� � �� �� �� �) )� � �� �� �� �� �� �� ����

� � � ! � � �

) )� �

e = 0.95 (colisión parcialmente elástica)

1 2

1 2

2 2

2

1

1 11 2

2

1 0.95 1 0.95

2 25cos 60º 0.0625 m/s 5cos 60º 2.44 m/s

5sen 60º 4.33 m/s 0

0.0625 4.33

2tg( ) tg 60º 69.

2.44 m/s4.33 m/

28 8

s

299.2º1 0.95

.2

x x

y y

vv

v v

v v

� � � � �

� �

)

� �� �� �) )� � � �� �� �� �� �� �) )� � �� �� �� �� �� �� �� �� �� �� �

� �

�) �

�� � � �

º

- 137 -

Física Universitaria: Problemas de Física Estática del sólido rígido. M20.1

1. Una escalera AB, de 10 kg de masa, 2.5 m de longitud y centro de masa situado a mitad de la misma, se encuentra apoyada sobre una pared lisa y un suelo rugoso, cuando su extremo inferior se encuentra a una distancia de 1.5 m de la pared. a) Calcular las reacciones en los apoyos A y B. b) Determinar el valor mínimo del coeficiente de rozamiento entre el suelo y la escalera para que ésta pueda permanecer en equilibrio en la posición indicada en la figura.

Determinamos el lado vertical del triángulo: 2 2 2 22.5 1.5 2 mb l a� � � � �

a) Aplicamos las Ecuaciones Cardinales de la Estática, descompo-niendo las fuerzas en las direcciones horizontal y vertical y tomando momentos en el punto A.

B B

A A

B B

0 3.75 kg =0 1

36.8 N98.0 N

36.

0 kg =1.5 10 3.75 kg =

2 2 2 28 N

f N f NN mg N mg

a amg N b N mgb

� � � � ����� � � � ������ � � � � �� ���

b) El valor mínimo del coeficiente de rozamiento es

AA

2 1.52 4

0.375

a

bmgf af N

N mg b� �- 9 � � � �

A

B

1.5m

A

B

G

a

b

l

mg

NB

NA

f

- 138 -

Física Universitaria: Problemas de Física Estática del sólido rígido. M20.2

2. Una barra uniforme pesa P y se mantiene en equilibrio, apoyada en una pared vertical y en un suelo horizontal, ambos lisos, en la posición indicada en la figura, gracias a la cuerda horizontal CD. Determinar las reacciones en los apoyos de la barra y la tensión en la cuerda en función del ángulo �.

En la figura mostramos el diagrama de fuerzas que actúan sobre la varilla. Aplicamos las ecuaciones cardinales de la estática tomando momentos en el punto C:

(1)

(2) 100 kgA 2(3) cos CG sen CBsen

3

R T

N P

lR P N� � �

���� �� ���, � ������ � �����

Con 2 1CG )3 2 6

ll� ���� � ��� ���

de modo que la tercera ecuación se escribe en la forma 2(3) cos sen sen3 6 3l l lR P N� � �� �

Y, teniendo en cuenta las dos primeras ecuaciones, resulta: 4 tg 2 tg

34 3 tg tg4

R P N

R P R T P

� �

� �

� �

� � �

D

A

B

G

P N

l/3

�2l/3

R

TC

D

A

B

l/3

� 2l/3

C

G

- 139 -

Física Universitaria: Problemas de Física Estática del sólido rígido. M20.3

3. La varilla homogénea AB, que tiene una longitud de 1.6 m y pesa 2 kg, se apoya por su extremo inferior en un muro vertical y, por un punto intermedio C, en otra varilla fija, horizontal y paralela al muro, a una distancia de 10 cm de éste. Suponiendo que no existan rozamientos entre la varilla AB y los demás elementos en contacto con ella, determínense en la posición de equilibrio el ángulo formado por la varilla y el muro y las reacciones en los apoyos de la varilla.

a) En la figura mostramos el diagrama de fuerzas que actúan sobre la varilla. Aplicamos las ecuaciones cardinales de la estática tomando momentos en el punto A:

A

(1) costg

(2) sen

(3) sen2 sen

N R PP R N

l aP R

��

��

� ��� ��� � ��� ��� ��, ���� �����

Que constituye un sistema de tres ecuaciones con tres incógnita (N, R y �). Sustituyendo la (2) en la (3):

2

3

sen sen sen2 sen 2 sen

2 2 0.1 1sen1.6 8

1sen 30º2

l a l aR R

al

� � �� �

� �

� �

� � � �

� �

De la ecuación (2):

sen 2 4 kgsen

PP R R P��

� � � �

Y de la (1), se sigue:

3cos 4 2 3 3.46 kg2

N R �� � � �

Las direcciones y sentidos de estas reacciones son las indicadas en la figura.

Otro método Puesto que sobre la varilla solo actúan tres fuerzas (P, N y R), éstas deben ser concurrentes en un punto tal como el D. En consecuencia, el problema se reduce a una simple condición geométrica de que el centro de gravedad (G) de la varilla se encuentre en la vertical del punto D. Resolvemos considerando los triángulos ACE, ACD y ADG:

� 2 3 2AC AD sen AGsen sen sen sensen 2

a l al

� � � � ��

� � � � �

Que es el mismo resultado obtenido anteriormente.

C

A

B

C

A

B

G

P

N

R l/2

a

D

E

- 140 -

Física Universitaria: Problemas de Física Estática del sólido rígido. M20.4

4. La barra AB de la figura tiene sección recta uniforme, de masa m y longitud 1. Determinar el ángulo � correspondiente al equilibrio.

De la figura, se siguen fácilmente las siguientes relaciones entre ángulos: 90º 90º ( )90º 90º ( )

� � � � � �

� � � � � �

) )� � � � � �) )� � � � � �

Aplicando las ecuaciones cardinales de la estática, tomando momentos en G :

A B

A B

G A B

0 [1] sen sen

0 [2] cos cos

0 [3] sen sen2 2

x

y

F N N

F N N Pl lM N N

� �

� �

� �

���� � ����� � � ������ ) )� �����

++

+

Puesto que tan sólo estamos interesados en determinar el valor del ángulo � correspondiente a la posición de equilibrio, podemos eliminar fácilmente las reacciones en los apoyos de entre las ecuaciones [1] y [3]:

A B

A B

sen sen sen sencos( ) cos( ) cos( ) cos( )

N NN N

� � � �� � � � � � � �

�� �� $ ���� � � � ���

de modo que, desarrollando las expresiones trigonométricas, tenemos sen sen

cos cos sen sen cos cos sen sensen cos cos sen sen sen cos sen cos sen sen sen

2sen sen sen (cos se

20.1

n sen cos )cossen( ) sen15ºtg 1.3660

2sen sen 2sen 30º s n 4 ºº

e 5

� �� � � � � � � �

� � � � � � � � � � � �� � � � � � � �� �

��

��

� � �

� � � � �

�� � ��

A

B )

1

P

NBNA 1

)

y

x

� )’

1’ G

A

B

45º

30º

- 141 -

Física Universitaria: Problemas de Física Estática del sólido rígido. M20.5

5. Una barra de longitud l se apoya sin rozamiento sobre una pared de perfil circular tal como se indica en la figura. a) Determinar la fuerza horizontal que se debe aplicar en B para mantener la barra en equilibrio. b) Si en lugar de aplicar una fuerza en B se aplicase un par en B, ¿qué valor debería tomar dicho par?

a) Las fuerzas que actúan sobre la barra son las que se representan en la figura. Tomamos momentos en O:

3sen 60º2

32 4

lFl P Pl PF� � �

b) En este caso las fuerzas que actúan sobre la barra serán las que se representan en la figura. De nuevo tomamos momentos en O, para obtener:

3sen60º2 2

34

lM P P M Pl� � �

Lo que resulta obvio, ya que el momento del par (invariante) debe ser igual al de la fuerza F inicialmente aplicada con respecto al punto O.

Cálculo de reacciones Ecuaciones cardinales de la estática:

AA

B A

B A

3

43

2

1cos30º cos30º 2sen 30º

1 1 3sen 30º2 2 4

PFN PF NN N P

N P N P P P

������ � � ���: �� �� � � �� �� �� �� ��� � � � � �����

NB

P B

NA

60º

F

O

A

l

l

B

NB

PB

NA

60º

M

O

A

- 142 -

Física Universitaria: Problemas de Física Estática del sólido rígido. M20.6

6. Un bloque prismático de sección cuadrada y masa m se encuentra apoyado sobre un plano liso horizontal y otro inclinado rugoso. Sabiendo que la situación representada corresponde a la de límite de equilibrio: a) Dibujar el diagrama de fuerzas. b) Determinar el coeficiente de rozamiento.

a) En la figura adjunta hemos representado el diagrama de fuerzas. b) Escribimos las ecuaciones cardinales de la estática, descomponiendo en las direcciones horizontal y vertical y tomando momentos en B:

2 2

1 2

1

2

B

[1] cos30º sen 30º tg 30º[2] sen 30º cos30º

[3] sen 30º 2 cos15ºcon [4]

N f N fN N f P

N l Plf N�

�� : � ���� � � ����� ����

Puesto que tan solo estamos interesados en el valor del coeficiente de rozamiento, es suficiente combinar las ecuaciones [1] y [4] para determinarlo:

2 21 1tg30º

tg30º 0.5771.73

3N N� �� � � �

60º 30º

60º 30º

15º 60º

N1

N2

P

l

B

A

f

l

- 143 -

Física Universitaria: Problemas de Física Estática del sólido rígido. M20.7

a

b

T

P

N

f

G

A

Psen�

Pcos�

7. Un bloque rectangular de dimensiones a = 1 m y b = 3 m se sitúa sobre un plano inclinado tal como se indica la figura. Una cuerda sujeta la parte superior del bloque para evitar que caiga por el plano, ¿cuál será el ángulo � máximo para que el bloque no deslice por el plano?

Cuando el ángulo tiene el valor crítico, el bloque está a punto para volcar deslizando sobre la arista A de su base y las fuerzas que actúan sobre el mismo son las indicadas en la figura. Aplicando las ecuaciones cardinales de la estática en las condiciones críticas, tomando momentos en A, tenemos:

A

sencos cos sen

1 sen cossen cos2 22 2

con

T f PN P T P P

ab a T P PTb P P bf N

�� � � �

� �� �

� � ����� � � � ��� �� �� �� "� �� � � �� �� �� ������ ����

1 sen cos cos sen2 2

sen cos 2 cos 2sen

se tn s g 22 co

aP P P Pb

aba a

bb

� � � � �

� � � � �

� � � ��

� � �

� � �

� ��� � � �� � �����

Sustituyendo los valores dados en el enunciado, resulta 1tg 2 0.1 0.53 28.1º3

� �� � � � ��

Otro método.- Tengamos en cuenta que: 1º) La reacción resultante R (suma de la reacción normal N y de la fuerza de rozamiento f), en las condiciones de movimiento inminente, forma un ángulo � con la normal al plano inclinado, tal que

tgff NN

� � � � �

2º) Puesto que sobre el bloque solo actúan tres fuerzas (P, T y R), éstas deben ser concurrentes en un punto tal como el D. De la figura, correspondiente a la posición crítica, se sigue

BC+CD / 2 tg2G

2tg/B

ab

a bb

� �

�� � ��

a

0 = 0.1b

T

P

R

f

G

A

BC

D

� �

N

- 144 -

Física Universitaria: Problemas de Física Estática del sólido rígido. M20.8

8. Una caja de embalaje contiene un frigorífico, pesa 300 kg y tiene forma de paralelepípedo rectangular de 2 m de alto por 80 cm � 80 cm de base. El coeficiente de rozamiento entre la caja y el suelo vale 0.30. Si deseamos arrastrarla sobre el suelo mediante la aplicación de una fuerza horizontal: a) ¿Cuál debe ser la magnitud de esa fuerza? b) ¿A qué altura sobre el suelo podemos aplicar esa fuerza sin riesgo de vuelco?

En la figura hemos representado el diagrama de fuerzas que actúan sobre la caja. Obsérvese que actúan dos pares de fuerzas: el par [F,f] provoca el vuelco sobre E y el par [P,N] se opone a dicho vuelco. Escribimos las Ecuaciones Cardinales de la Estática, tomando momentos en B (punto de aplicación de la reacción normal N):

B

N PF f

Fh Px

, � �

��

De las dos primeras ecuaciones se sigue:

F f N P F P� � �� 9 � 9

Esto es 0.3 300 90 kgF 9 � �

La tercera ecuación nos permite relacionar la distancia h (punto de aplicación de la fuerza externa) con la distancia x

(“punto de aplicación” de la reacción normal N): P P xh x xF P� �

� - �

de modo que el valor máximo de h que no produzca el vuelco será el que corresponda al valor máximo posible de la distancia x: esto es,

máxmáx

/ 2 40 133 cm 1.33 m0.3

x lh� �

- � � � �

P F

f

N

x

E B

O H

h

l

- 145 -

Física Universitaria: Problemas de Física Estática del sólido rígido. M20.9

9. Deseamos transportar en una carretilla un bloque homogéneo, de masa m, cuyas di-mensiones se especifican en la figura. Sea � el coeficiente de rozamiento entre la base del bloque y la plataforma de la carretilla. Determinar los valores máximos de la aceleración de la carretilla (acelerando y frenando) para que no haya movimiento relativo entre el bloque y la carretilla.

a) Cuando la carretilla está acelerando, si la aceleración es excesiva, el bloque puede volcar rotando alrededor del eje A. En esas circunstancias, el diagrama de fuerzas en el referencial de la carretilla, que es no inercial, es el que se india en la figura. Tomando momentos en A, tenemos

� �0 0A

1 2 2lmg ma l a g� ��

b) Cuando la carretilla está frenando, si la aceleración es excesiva, el bloque puede volcar rotando alrededor del eje B o resbalar hacia delante (respecto de la carretilla). En esas circunstancias, el diagrama de fuerzas en el referencial de la carretilla, que es no inercial, es el que se indica en la figura. b.1) Vuelco. Tomando momentos en B, tenemos

� �0 0B

1 22 4lmg ma l a g) )� ��

b.2) Resbalamiento.

¨0

¨0¨0

ma f f mgN mg a gfa gf N m

��

��

� )) �� � ��� �� �� ))� �� �� � )) � �� ��� ����

Por consiguiente, � Si � < 0.25, tiene preferencia el resbalamiento. � Si � > 0.25, tiene preferencia el vuelco

4l

l

l

4l

l

l

A

B mg

ma0

a0

4l

l

l

A

B

mg ma0

a0 N f

- 146 -

Física Universitaria: Problemas de Física Estática del sólido rígido. M20.10

10. Una placa rectangular y homogénea, de dimensiones 30 cm x 20 cm, pesa 2 kg y está unida a un eje vertical de modo que en A está articulada con el eje y en B tan solo se apoya en él, como se indica en la figura. a) Determinar las reacciones en A y en B cuando el sistema está en rotación con una velocidad angular de 30 r.p.m. b) ¿A partir de que valor de la velocidad angular no se apoyará en B?

Reconducimos el problema a un problema de estática (equilibrio estático) analizándolo en un referencial en rotación en el que la placa se encuentra en reposo. En estas condiciones, el diagrama de fuerzas es el que se muestra en la figura, incluida la fuerza centrífuga. Cálculo de la fuerza centrífuga: Sea un elemento de masa d dm b x�� , de modo que

2 2cf

2 2 2 2 2cf 0

d d d

1 1 1d2 2 2

a

F x m bx x

F b x x ba ba a ma

� � �

� � � � � � �

� �

� � � ��a) Escribimos las ecuaciones cardinales de la estática, tomando momentos en A:

Acf B

B cf A

[1] [3]

[2] 2 2R P b aF bN PN F N

� ��� � ���, � ����

de modo que disponemos de tres ecuaciones con tres incógnitas (NA, NB y RA) De la primera, se sigue que AR P�

De la tercera, obtenemos

�2

2B cf

1 1 12 2 2 2 2

ma bN aP bF mga m ab gb b b

��

� �� � ���� �� � � � � �� �� ��� � ��� �

y sustituyendo en la segunda ecuación 2

2 2 2A B cf

1 1 12 4 2 2 4 2 2

mga mga ma bN N F ma ma ma gb b b

�� � �

� ��� �� � � � � � � � �� �� ���

b) Para una cierta velocidad angular crítica, �crít, desaparece la ligadura en B (i.e., NB=0): 2 2crít crít

B crít20 0

2 2 2b bma gN g g

b b� �

�� ��� �� � � � �� �� ���

Sustituyendo los valores dados,

A

2

A

2

B

crít

2 0.30 0.209.80.30 m2 0.20 2

0.20 m2 0.30 0.202 kg 9.82 0.20 230 r.p.m. rad/s

2 9.8 9.9 rad/s0.

2 kg 19.6 N

16.2 N 1.65 kg

13.2 N 1.35 kg

94.5 r.p.m2

.

R

Nabm N

� �

� �� � �� �� � � �� �� ��� ��� � �� �� � �� � � ��� � �� � ��� �� ��� � � � � ����

� � �

�������������������������

A

B

a=30 cm

b=20 cm

RA

NA

NB

P

Fcf

dx

x

G C

- 147 -

Física Universitaria: Problemas de Física Estática del sólido rígido. M20.11

11. Una puerta de garaje pesa 60 kg y está montada como se muestra en la figura. Las ruedas están enmohecidas de modo que no ruedan sino que deslizan en la guía, siendo 0.4 el coeficiente cinético de rozamiento. La distancia entre las ruedas es de 2 m y cada una de ellas dista 50 cm de los bordes verticales de la puerta. Se empuja la puerta mediante una fuerza horizontal constante de modo que se mueva uniformemente. a) Si la línea de acción de dicha fuerza dista 1 m de la guía, ¿cuál es la fuerza ejercida por cada una de las ruedas sobre el carril? b) Encontrar la máxima distancia h a la que se puede aplicar la fuerza horizontal F sin que ninguna rueda se separe del carril.

a) Aplicamos las ecuaciones cardinales de la estática, tomando momentos en el punto A:

A B A B

A B A B A B1

B 2B B

( )

A

2 2

hb

F f f N N F P F PP N N P N N N P N

b b N P FFh N b P Fh N b P

� � �� �� �� �� � �� � � � � ��� �: �� � �� � �� � " � � " � �� � �� � �� � �� � � � ���� �� � � �� �� �� �

��

de modo que resulta

A A1

B B2

24 kg42 kg 16.8

0.kg

18 kg 7.2 kg

4 600.4 42

30 24 0.4 18

FN fN f

� � �� � � �� � � � � �

b) En las condiciones del enunciado, será NB = 0; i.e., 2 2 22

2 2 02.5 m

.4h h bP F P h b h

b b� �

�� � " � " � � �

h

F

F

h fA

NB NA

fB

P

b

A B

- 148 -

Física Universitaria: Problemas de Física Estática del sólido rígido. M20.12

12. Deseamos apilar un cierto número de ladrillos uno sobre otro, como se muestra en la figura, de modo que obtengamos el máximo saliente. Deseamos apilar un cierto número de ladrillos uno sobre otro, como se muestra en la figura, de modo que obtengamos el máximo saliente. a) Obtener el criterio que debemos seguir para conseguir nuestro objeti-vo. b) Demostrar que se puede conseguir un saliente tan grande como queramos sin más que apilar un número suficientemente grande de ladrillos.

a) En la figura, hemos representado mediante un circulito la posición del centro de gravedad de cada uno de los ladrillos. Mediante un triangulito, hemos representado la posición del centro de gravedad del ladrillo que sirve de base y de todos los que tiene encima. El criterio que debemos seguir para apilar los ladrillos es que el centro de gravedad de los

ladrillos que se encuentre encima de uno dado no sobresalga sobre el borde de este último, tal como se ilustra en la figura para una condiciones críticas de equilibrio. La mayor longitud de la parte saliente del ladrillo superior (1) es igual a l/2. Los ladrillos sucesivos sobresalen sobre los que les sirven de base una distancia dada por

(12)

(123)

(1234)

0 ( / 2)ladrillo 2 G =2 4

2 0 ( / 2)ladrillo 3 G =3 6

3 0 ( / 2)ladrillo 4 G =4 8

m m l lm

m m l lm

m m l lm

� � � � �

� � � � �

� � � � �

deduciéndose de modo obvio la regla a seguir en el caso de que hubieran más ladrillos. b) La distancia máxima en que la parte derecha del ladrillo superior (1) sobresale sobre el ladrillo inferior (n-ésimo) que sirve de base, se expresa en la forma:

1 1 1 1 1 1 1 12 4 6 8 2 1 2 3 4

ll� � � �� �� �� � � � � � � � �� �� �� �� �� �

� �

que es la bien conocida serie armónica, que es divergente. Así, para un número ilimitado de ladrillos, esta suma tiende hacia infinito, con lo que queda demostrado el aserto propuesto.

l/2l/4

l/6

(1)(2)

(3) (4)

G123 G12 G1

- 149 -

Física Universitaria: Problemas de Física Estática del sólido rígido. M20.13

A

B

C

L

PL

30º

F L

R

T

N

13. La barra homogénea AB, de longitud L y peso P, mantiene en equilibrio gracias a una articulación en el punto A y a una cuerda, de longitud también L, que actúa unida a B y C. a) ¿Qué tensión tendrá la cuerda BC? b) ¿Qué reacción habrá en A?

Comenzamos determinando el ángulo � que forma la cuerda con la horizontal:

cos30º cos cos 1 cos30º 0.1340 82.3ºL L L � � �� � � � � � Escribimos las Ecuaciones Cardinales de la Estática, tomando momentos en A:

cos82.3º 0

sen82.3º 0

A cos30º sen(82.3º 30º ) 02

T RT N P

LP TL

���� � ���� , � � ������ � � �����

y resolviendo este sistema de tres ecuaciones con tres incógnitas (T, N y R) resulta:

0.1340 0 0.1340 0.07330.9910 0 0.9910 0.45770.4330 0.7912 0 0.5473

T R R T PT N P N P T PP T T P

� � � � ����� � � � � � ���� � � ����

de modo que la tensión de la cuerda es 0.5473T P�

y la reacción en A (que actúa sobra la barra) tiene de componentes

0.4577 0.0733N P R P� � lo que representa una resultante

2 20.4577 0.0733 0.46350.4577tg 6.24 80.9º0.0733

F P P

� � �

� � �

A

B

C

L

L

L 30º

- 150 -

Física Universitaria: Problemas de Física Estática del sólido rígido. M20.14

14. El extremo superior de una varilla, de masa m y longitud l, está articulado a una deslizadera que desliza a lo largo de una guía vertical lisa (vide figura), en tanto que la varilla no pierde contacto en B con el apoyo liso. Determinar el valor del ángulo � correspondiente al equilibrio y las reacciones en los apoyos de la varilla. Aplicación numérica: m = 10 kg, l = 100 cm y a = 5 cm.

Escribimos las Ecuaciones Cardinales de la Estática del Sólido Rígido, descomponiendo en las direcciones horizontal y vertical y tomando momentos en A:

A B

B

B

cossen

A = sensen 2

N NP N

a lN P

��

: �

� � � �� �� �� �� �� �� �� � �

De modo que disponemos de tres ecuaciones con tres incógnitas. Combinando la tercera con la primera:

2 3 3B B

2sen sen sen2 2l l aN P Na a l

� � �� � �

1/3 1/32 10sen 0.4642 27.7º100

al

� �� � � �� �� �� � � �� �� �� �� �� �

A

B

10 19.04 kg 187 Ntag tag 27.7º

10 21.54 kg 211 Nsen sen 27.7º

PN

PN

� � � �

� � � �

Método de la energía Expresamos la energía potencial de la varilla (nivel de referencia en B) en función del ángulo que forma con la vertical (1 grado de libertad):

pcos coscos cos

2 sen sen 2l lE mg a mg a� �

� �� �

� � � �� �� �� � � � �� �� �� �� �� �

En la posición de equilibrio estable, la energía de potenciar debe tener un valor mínimo. Así, determinamos el valor del ángulo correspondiente a dicho valor mínimo:

2 2p

2 2

d sen cos sen sen 0d sen 2 2 senE l l amg a mg� �

� �� � �

� � � �� � �� ���� � � � ��� �� �� ��� � �

De modo que 3

2

2sen sen2 senl a a

l� �

�� �

A

Ba �

l

a

B

l/2

A

C

P

NB

D NA

- 151 -

Física Universitaria: Problemas de Física Estática del sólido rígido. M20.15

15. Una viga AB, de 5 m de longitud y 60 kg de masa (distribuida uniformemente), está apoyada en el suelo. Se levanta el extremo B, a una altura de 3 m, mediante una fuerza aplicada en B, siempre perpendicular a la viga. Hállense: la fuerza aplicada en B, la reacción del suelo en A y el coeficiente de rozamiento mínimo necesario para que A no deslice.

Datos: m= 60 kg, l = 5 m, h = 3 m 3sen 0.6 36.87º5

hl

� �� � � �

Escribimos las ecuaciones cardinales de la estática, tomando momentos en el punto A

A

[3]

[2]

[1]

2 3 6sen 14.4 kg5 5 25sen

8 17cos cos 40.8 kg =25 25

1 2cos cos 24 kg =2

141.1 N

399.8 N

235.2

25

N

f F P Pf F

N F P N P F P P PlFl P F P mg

��

� �

� �

��� �� � � � � ��� �� � �� �� ��� �� � " � � � � � �� �� �� �� �� ��� � � � �� ��� ���

:

��

Para que no exista deslizamiento en esta posición deberá ser 6

170.353f

N�. � �

NP

F

f A

B

G h

l

- 152 -

Física Universitaria: Problemas de Física Estática del sólido rígido. M20.16

16. La viga AB, de masa 10 kg y 1 m de longitud, está cargada y apoyada como se indica en la figura. Determinar la reacción en el apoyo A y la tensión del hilo BC cuando m1 = 2 kg y m2 = 7 kg.

En la figura presentamos el diagrama de fuerzas del “cuerpo libre” correspondiente a la viga, con

7 2 5 kg 0.4 m10 kg 0.5 m

F bP a

� �� � � �� �� �� �� �� �� �� �

Escribimos las Ecuaciones Cardinales de la Está-tica, tomando momentos en A:

cos30ºsen 30º

sen 30º

x

y

T RT R F Pl T bF aP

�� ���� � � ����� � ���

de modo que disponemos de 3 ecuaciones con 3 incógnitas (T, Rx, Ry). Sustituyendo valores y resolviendo el sistema de ecuaciones, tenemos:

cos30º cos30º 14cos30º 12.1 kg 119 Nsen 30º 5 10 15 15 sen 30º 15 7 8 kg 78 N

7sen 30º 0.4 5 0.5 10 7 14 kg 137 Nsen 30º

x x

y y

T R R T =T R R T

T T

� � � � ��� �� � � � � � � � � � ������� � � � � � � � �����

El módulo y la dirección de la reacción en el apoyo A son: 2 2 2 212.1 8 14.5 kg 145 N

8=arctg arctg 33.4º12.1

x y

y

x

R R R

RR

� � � � � �

� �

B

m1 m2

A

C

0.4 m

30º

A

a

30º

Ry

Rx

P F

b

- 153 -

Física Universitaria: Problemas de Física Estática del sólido rígido. M20.17

17. Sobre un canalón cilíndrico AB de 10 kg se enrolla un hilo que se engancha en el extremo B, tal como se representa en la figura. a) Determinar la magnitud mínima de la fuerza F con que debemos tirar del otro extremo del hilo para levantar el canalón. b) Calcular el valor mínimo del coeficiente de rozamiento que debe tener el canalón con el suelo para que el punto A no deslice.

Cuando la fuerza F es la mínima necesaria para levantar el canalón, la reacción en B es nula y el diagrama de fuerzas del cuerpo libre para el canalón es el que se indica en la figura. Aplicamos las ecuaciones cardinales de la estática, tomando momentos en el punto O:

A

A

O A A

0 (1) cos 0

0 (2) sen 0

0 (3) 0

x

y

F F N

F N F mg

M N R FR N F

� �

�� � " � ����� � " � � ����� � " � � " ����

++

+

de modo que disponemos de un sistema de tres ecuaciones con tres incógnitas (F, NA y 0). De la primera ecuación, teniendo en cuenta el resultado de la tercera, se sigue:

cos 0 cos cos 0 7º .30 8F F� � � �� � � � � y, análogamente, de la segunda ecuación obtenemos:

10 9.8sen 01 sen 1 sen 30º

196 NmgF F mg F��

�� � � � � �

� �

F �

NA

B 0 NA

mg

O

x y

G R

F 30º

B A

- 154 -

Física Universitaria: Problemas de Física Estática del sólido rígido. M20.18

18. Un cilindro homogéneo de 1.2 m de diámetro pesa 1 t y descansa sobre la plataforma de un camión según se indica en la figura. Los bloques representados se utilizan para impedir que ruede el cilindro cuando acelere el camión. Determinar la aceleración de éste que haría que el cilindro rodara sobre el bloque.

Supongamos que el camión frena con una aceleración constante -a0. Sobre el cilindro “aparece” una fuerza de inercia dirigida hacia delante (en el sentido de la marcha) dada por -m(-a0) = ma0. En la figura hemos representa el diagrama del cuerpo libre o diagrama de fuerzas que actúan sobre el cilindro, en un referencial solidario con el camión en el que, en las condiciones críticas, el cilindro aun permanece en equilibrio, aunque manifiesta una tendencia a rodar sobre el borde A del bloque indicado. Aplicamos tan sólo la segunda ecuación del equilibrio, tomando momentos en A:

0 0( ) bma R h mgb a gR h

� � ��

A partir de la figura, por aplicación del teorema de Pitágoras, tenemos: 2 2 2 2( ) 2 (2 )

10(120 10) 33.17 cm

b R R h Rh h h R h

b

� � � � � � �

# � � �

De este modo, la aceleración pedida es 2

033.17 0.66

60 106.5 m/sa g g� � �

Obviamente, las mismas consideraciones nos llevarán a los mismos resultados en el caso de que el camión acelere, solo que entonces la fuerza de inercia tendrá sentido opuesto al indicado en la figura y la rodadura se presentará sobre el borde B del bloque trasero.

Otro método: Aplicamos tan sólo la primera ecuación del equilibrio, en las direcciones horizontal y vertical; i.e.,

00

0cos

cotgsen

cotga gma N amg N g

���

�: ��� $ � �� ����

con 50 5sen 56.4º cotg 0.6660 6

R hR

� � ��

� � � � �

de modo que 2

0 cotg 0.66 6.5 m/sa g g�� � �

10cm60cm

mg ma0

h

O

b A B

Fuerza de inercia

- 155 -

Física Universitaria: Problemas de Física Estática del sólido rígido. M20.19

60º

N

P

F

f

60º A

O

19. Calcular la fuerza horizontal tangencial F mínima necesaria para que el disco de la figura, de masa 25 kg, ruede por el plano inclinado a 60º. Hállese también la fuerza que imprime el plano inclinado al disco, y el coeficiente de rozamiento mínimo para que en esas condiciones no deslice.

Aplicamos las ecuaciones cardinales de la estática, tomando momentos con respecto al eje que pasa por el punto A; i.e.,

cos sen[1][2] cos sen

A [3] sen ( cos ) (1 cos )

F f NN f PPR F R R FR

� �� �� � �

� � ��: ��� � ���� � � � ������

Disponemos de tres ecuaciones con tres incógnitas (F, N , f).

sen 3 14.43 kg = 14[3]1 cos 3

1.5 NF P F P F��

" � � � "�

(+ m.a.m.)( sen )

( cos )

sen cos[1] sen cos[2] cos senN f F

N F PN f P

� �� �

� ��

� � ��� ;; ;; � ��� � ���

De modo que 2sen co 25 kg = 245.0 Ns

1 cosN P P P P NN�

��

� � � �" � "�

cos 1 cos 3

sen sen 314.43 kg = 1 1 N2 .[ 4 5] P N

f P P F f F f� �

� �

� �� � � � " � � "

A partir de la definición del coeficiente de rozamiento, tenemos

( 3 /3

. 83 53) 0f PN P

�� � � �

La ecuación [3] también puede escribirse tomando momentos con respecto al eje que pasa por el punto O; de modo que las ecuaciones cardinales de la estática quedan en la forma:

cos sen[1][2] cos sen

O [3]

F f NN f P

fR FR f F

� �� �

�� � �: ���� � ����� � " ������

La resolución de este sistema de tres ecuaciones con tres incógnitas (F, N, f) nos lleva a los mismos resultados anteriormente obtenidos.

F

60º

- 156 -

Física Universitaria: Problemas de Física Estática del sólido rígido. M20.20

20. Una esfera uniforme de radio R y masa M, se mantiene en reposo sobre un plano inclinado de ángulo � mediante una cuerda horizontal como se muestra en la figura, siendo el coeficiente de rozamiento entre las superficies 0 . Determinar: a) La tensión de la cuerda. b) La fuerza normal ejercida sobre la esfera por el plano inclinado. c) La fuerza de rozamiento que actúa sobre la esfera. Aplicación numérica: M = 3 kg, � = 30º y 0 = 0.5.

Aplicamos las ecuaciones cardinales de la estática, tomando momentos con respecto a un eje en O:

O

0 [1] cos sen 0 [2] cos sen

0 [3]

x

y

F T f MgF N Mg TM TR f R T f

� �� �

� " � �

� " �

� " � �

de modo que disponemos de 3 ecuaciones con 3 incógnitas (T, N y f ). Rescribimos la ec. [1] con T = f :

sen1 co

(1 cos ) sens

T Mg T Mg� ���

��

� �

y de la ec. [2] se sigue: 2 2 2sen cos cos sen 1 cos

cos1 cos 1 cos 1 cos

cos senN Mg T Mg Mg Mg Mg� � � � ��

� � �� �

� � ��

� � �

� ���� � � � � ��� ����

de modo que sen sen

1 cos 1 cosT Mg N Mg f Mg� �

� �� � �

� �

y sustituyendo valores

0.8s 0en 4 k30º 31

gc

3 kg 0.804 kgos30º

T N f� � � � ��

Como la magnitud del la fuerza de rozamiento es tal que

max 0.5 3 1.5 kgf f N�- � � � �

efectivamente hay equilibrio.

y x T

Mg

f N

A

� O

- 157 -

Física Universitaria: Problemas de Física Estática del sólido rígido. M20.21

21. Dos tableros pesan 10 y 20 kg y miden 2 y 4 m de longitud, respectivamente. Los tableros están articulados en sus extremos, entre ellos y en el techo y soportan un cilindro de 1 m de diámetro y 30 kg de peso. No hay rozamiento en ningún contacto. a) ¿Qué reacciones soportarán las articulaciones A y C? (exprésense en forma vectorial) b) ¿Qué fuerza se ejercen los dos tableros en la articulación B?

Geometría del problema 2sen 0.5 30º4

AC BCcos30º 4 3 / 2 2 3 3.46 m

BD BE ODcotg30º 1/ 2 3 3 / 2 0.87 m

� �� � �

� � � � �

� � � � � �Aplicamos las ecuaciones cardinales de la estática al cilindro y a cada uno de los tableros, por separado: Cilindro

1 2 1

2 2

3

3

3 / 2

sen 30º tg30º 30 10 3 kg

30cos30º 20 3 kgcos30º

N N N P

PN P N

���: � � � ������� � � � ������

Tabla vertical

� �

A B 1 A B B

A B 1 A B

AA 1B

10 3 10 3 7.5 9.82 kg10

2 3 / 4 10 3 7.5 kg2 3 / 2

X X N X X XY Y P Y Y

xx N

�� ��� �: � � � � � � �� �� ��� �� � � �� �� �� �� � � ��� �� �����

Tabla oblicua

C 2 B

C 2 2 B

3C 2 2 C2

C B 2

3B C 2 2 2

3 31 1C C 2 23 2 4 3

B

7.5 kg

sen 30ºcos30º

2 3 2 3

sen 30º 9.82 10 3

cos30º 14 35.67 kg.33 20 2

14.33

0 3

7.5 1 5 3 k0 g

X N XY P N Y

X P N Y

X X N

Y Y P N

Y X P N

�� : � ���� � � � ���� � � ����� � � � � ����� � � � � � � ����� � � � � � � � ���

A B C(7.5, 45.7) kg (9.82, 35.7) kg ( 7.5, 14.3) kg� � � � �# R R R

o bien, en newtons:

A B C(73.5, 448) N (96.2, 350) N ( 73.5, 140) N� � � � �# R R R

A

90º

B

C

4m 2m

A

90º

B

C

l2

30º30º P2

P

N2

N1

P1 30º

YA YC

XC

XA

YB

XB

�=30º

l1 OD

E

- 158 -

Física Universitaria: Problemas de Física Estática del sólido rígido. M20.22

22. Una barra homogénea AB de 5 N de peso y 4 m de longitud está articulada en A una pared vertical y mantenida en su extremo superior mediante un hilo horizontal BD, formando un ángulo de 45º con la vertical. La barra soporta un disco, de 3 N de peso y 1 m de diámetro, que se encuentra también en contacto con la pared. Considerando despreciables los rozamientos, determinar la tensión del hilo y la reacción en la articulación A.

En primer lugar determinamos las distancias a y b del punto de contacto C a los extremos de la barra:

0.5tg 22.5º 1.21 m 0.4142

r a a� � �

En la figura adjunta hemos representado los diagramas de fuerzas que actúan sobre cada uno de los dos cuerpos (el disco y la barra). Aplicamos las ecuaciones cardinales de la estática al disco,

1 disco1 2

2 disco discodisco 2

2 3 N2 2 2 3 2 4.24 N2 22

N PN N

N P PP N

��� � � �� �� �� �� � � �� � � � � �� �� �/ � �����

y a la barra, tomando momentos en A,

2 2

barra 2

barra2 barra 2

2 2 4.31 32 2

2 5 32

2 2 2 1.21 2 3 2 2.52 2 2 2 4

1.31 N

8 N

4.31 N

X X

Y

N N T N T N

N P N

Pl aN a P Tl T Nl

��� � � � � � � ��������/ � � � � �������� � � � � � � ������

B

45º

C

Pdisco

N2

G

N2

NX

T

a

b

Pbarra A

D

NY

r

N1

- 159 -

Física Universitaria: Problemas de Física Estática del sólido rígido. M20.23

23. Un semicilindro homogéneo, de peso P1 y radio R, se apoya en su base sobre un plano horizontal rugoso. Una varilla homogénea AB, de longitud l y peso P2, está articulada a la pared por su extremo A y se apoya en la superficie lisa del semicilindro formando un ángulo de 60º con la vertical. a) Determinar el valor mínimo del coeficiente de rozamiento del semicilindro con el plano horizontal para que la posición indicada sea de equilibrio. b) En dicha posición, calcular la reacción en la articulación A.

De la geometría de la figura se sigue:

tg 60º tg 60º 3h h R RR

� � �

a) En la figura se muestra el diagrama de fuerzas que actúan sobre cada uno de los dos cuerpos. Aislamos la varilla y tomamos momentos en A:

21 2 21 2 2 2sen 60º 3sen 60º

2 2 44 3l l l lhN P N P P P

h RR� ���� � � ��� ���

Aislamos el semicilindro y aplicamos la primera condición de la estática:

12 2

2

1 1 21 1 12 1 2

cos 60º8

8 33sen 60º8

lf N PR lPf

N RP lPlN P N P PR

��� � ����� 9 ��� ��� , � � � �����

b) Aislamos la varilla y aplicamos la primera condición de la estática:

h 21 2

v 21 2 v 2 21 2

cos 60º8

3sen 60º sen 60º 18

lT N PR

lT N P T P N PR

��� � ������� � �� �� � �� , � � � � � �� �� � ���� � ���

60º

A

B

60º

A

B

P1

P2

N21

N12 N1

Th

Tv

60º C

O

h

h

30º

60º Rf

- 160 -

Física Universitaria: Problemas de Física Estática del sólido rígido. M20.24

24. Una escalera se apoya contra un cilindro liso de radio R, fijo sobre una superficie horizontal. La escalera forma un ángulo de 60º con la superficie horizontal y su longitud es 5 R/2. Determinar: a) La fuerza que el cilindro ejerce sobre la escalera. b) La fuerza de rozamiento que evita que la escalera deslice. c) La fuerza normal que la superficie horizontal ejerce sobre la escalera. d) El valor mínimo del coeficiente de rozamiento entre el suelo y la escalera para mantener el equilibrio.

Geometría del problema: 5AB 2.525AG 1.25

2 4AC cotg 30º 3 1.73

R R

R R

R R R

��� � � ������� � � ������ � � ������

Ecuaciones cardinales de la estática:

C

C A

A C

0 cos30º

0 sen 30º

0 AC AG sen 30º

[1]

[2]

[3]

x

y

F N f

F N N mg

M N mg

�� � ����� � � ����� � ����

+++

Resolvemos el sistema de tres ecuaciones con tres incógnitas (NA, NC y f):

C 0.AG sen 30º (5 / 4) (1/ 2) 5 324

[ 63AC 3

3] RN mg mg mg mgR

� � � �

C5 3 3 5cos30º24 2 16

[1 0.31] mgf N mg mg� � � �

C5 3sen 30º 148

0.82] 2[ A mgN mg N mg� ��� �� � � � �� �� ����

En las condiciones de deslizamiento inminente es:

min min0.310.8

0. 82

3A

fN

� �� � " �

G

NC

C

ANA

mg O

30º

30º

30º f

B

x

y

R

- 161 -

Física Universitaria: Problemas de Física Estática del sólido rígido. M20.25

25. Un tablero rectangular uniforme, de longitud 25 cm, se apoya sobre un cilindro de 5 cm de radio y sobre el suelo, como se indica en la figura. Tanto el tablero como el cilindro pesan 5 kg. ¿Cuánto deben valer, como mínimo, los coeficientes estáticos de rozamiento entre cilindro y tablero, entre cilindro y suelo y entre tablero y suelo para que el sistema permanezca en equilibrio?

Determinamos la posición del punto D de contacto del tablero con el cilindro: 5AD 18.66 cm

tg15º tg15ºR

� � �

Para que el sistema esté en equilibrio, deberán estarlo el tablero y el cilindro por separado. Aplicamos las ecuaciones cardinales de la estática al tablero, tomando momentos en A:

� �

3 1 3

1 3 3

A 3

1 sen 30º cos30

2 cos30º sen 30º

3 AD cos30º2

N f f

N N f PLN P

� �

, � � �

��

y al cilindro (momentos en C): � � �

2 3 3

2 3 3

C 3 2

4 cos30º sen 30º

5 cos30º sen 30º

6

f f N

N P N f

f R f R

� �

, � � �

��

de modo que disponemos de 6 ecuaciones con 6 incógnitas (N1, N2, N3, f1, f2 y f3) que resolvemos para obtener:

� �

� � � �

3

3 2

2 3 2 3 3

2 3 3

1 3 3

cos30º 5 25cos30º3 2.90 kg2 18.662AD

6sen 30º sen 30º4 1 cos30º sen 30º 2.90 0.78 kg

1 cos30º 1 cos30º5 cos30º sen 30º 5 2.90cos30º 0.78sen 30º 7.90 kg

2 cos30º sen 30º 5 2.90

PLN

f f

f N f f N

N P N f

N P N f

�� � �

��

� � � � � � �� �

� � � � � � �

� � � � � � � �1 3 2 3 3

cos30º 0.78sen 30º 2.10 kg

1 sen 30º 1 cos30º cos30º 0.78 kgf N f f f

� �

� � � � � �

1 2 3 1 2 30.78 kg 2.10 kg 7.90 kg 2.90 kgf f f N N N# � � � � � �

y los coeficientes de rozamiento pedidos serán:

31 21 2 3

1 2 3

0.78 0.78 0.780.37 0.10 0.272.10 7.90 2.90

ff fN N N

� � �9 � � 9 � � 9 � �

(sigue…)

30º 5 cm

25 cm

15º

25 cm

5 cm 15º

P

f2

f3

f1

N1

N3

N2

P

N3

f3

AB

DE

C

- 162 -

Física Universitaria: Problemas de Física Estática del sólido rígido. M20.26

Aplicando las ec. cardinales de la estática al sistema completo (tablero+cilindro), tomando momentos en A:

� �

2 1

1 2

A 2

i

ii 2

iii AB AB cos30º2

f f

N N PLN P P

, � �

� ��

De modo que

2 1

1 2

2

2 10 7.90 2.10 kg/ 2 12.51 cos30º 1 cos30º 5 7.90 kg

18.66AB

f fN P N

LN P

� � � � �

� � � �� �� �� � � � � �� �� �� �� �� �

Y estas tres ecuaciones, junto con el sistema de ecuaciones (1)-(2)-(3) o el (4)-(5)-(6) nos conduce a los mismo resultados que antes.

- 163 -

Física Universitaria: Problemas de Física Estática del sólido rígido. M20.27

26. Un cilindro de radio r, y peso P, se apoya sobre un suelo y una pared rugosas (coeficiente de rozamiento, �). Determinar el momento mínimo que hay que aplicar al eje del cilindro para que dicho cilindro deslice.

Puesto que estamos interesados el “momento mínimo”, se entiende que el cilindro permanece en reposo, tanto de traslación (lo impide la pared), como de rotación, en tanto que no se supere dicho valor mínimo.

En la figura se representan las fuerzas que actúan sobre el cilindro (peso, rozamientos y reacciones normales en los apoyos), con

1 1 2 2f N f N� �� �

Las Ecuaciones Cardinales de la Estática, descompo-niendo en las direcciones horizontal y vertical y tomando momentos con respecto al eje del cilindro, nos permiten escribir:

1 2 1 2

1 2 1 2

par 1 2 1 2O

0

( )

N f N Nf N N N P

M f r f r N N r

��

�� � � � ���� , � � � ����� � � � ��� �

De las dos primeras se sigue:

1 2 22

2 2 2

1 011 11 11 1 11 0

1 1

N PP

N PP

��� ��� �

��� �

� ��� � ��� � �� �� � � ��� � ��� � ���

y sustituyendo en la tercera:

par 2 2 2

111(1 )

1PrM P P r�

��

����� ���� � � ��

���� �� � ��

m

M r

Mpar

f1

f2

N2

N1

P

r

O A

B

- 164 -

Física Universitaria: Problemas de Física Estática del sólido rígido. M20.28

27. Una semiesfera hueca de 10 kg descansa sobre un plano horizontal. Sobre un punto de su borde se coloca una masa m, inclinándose la semiesfera un ángulo de 45º. Calcúlese el valor de la masa m.

Determinación del centro de masa de una capa hemiesférica: 2

cm

2 212

d (2 ) d 2 sen dd sen

cos(4 ) 2

S r R Rz S r R

zz RSS R R

� � � � ���

� �

�� � ���� ���� " �� ���� � ����

/22

cm0

/23 2/2

2 00

1 ( cos )2 sen d

2 sensen cos d2 2 2

z R RS

R R RR

��

� � � �

� �� � �

� �

� � �

Condición de equilibrio: Tomamos momentos en O.

2

O

1

0 cos

tg

sen2RmgR MG

m M

� �

��

� " �+ M

con + = 45º, será 12m M�

con M =10 kg, será 5 kgm �

r

R 5�

5Rd�

z

R

mg

Mg

N G

O 45º+

m M

45º

- 165 -

Física Universitaria: Problemas de Física Estática del sólido rígido. M20.29

28. Dos placas rectangulares, de espesores despreciables, se apoyan por sus aristas de igual longitud sobre sendos realces de un plano horizontal separados una distancia a. Una placa tiene la otra arista de longitud a y la otra placa de longitud 2a y masa doble que la anterior. Despreciando los rozamientos, hállense el ángulo de equilibrio que forma la placa mayor con el suelo horizontal y el módulo y dirección de la reacción que se ejercen ambas placas.

Consideraciones geométricas de interés:

1 21 2

1 2

1

1 2 2

sen sen 22 180º

cos cos 2

senACTeorema senos: ACsen sen sen

a a

� �� �

� �

�� � �

� ���� � �� � ���

� �

En la figura mostramos el diagrama de fuerzas actuantes sobre cada una de las placas. Aplicaremos la 2ª condición del equilibrio ( 0�M ) a cada una de las placas, tomando momentos en los puntos A y B respectivamente. Placa grande (momentos en A)

12 2

2

2 2 2

1 2

senAC 2 cos 2 cossen

2sen cos sen 2sen sen 2

R Pa R a Pa

R P P P R P

�� �

�� � �� �

� �

� � � �

Placa pequeña (momentos en B)

� �1 2 1 2 1

1 1 2 1 2

cos cos cos sen sen2

cos 2cos cos 2sen sen 0

aP R a R a� � � � �

� � � � �

� �

� � �

Ahora, tenemos que resolver esta ecuación trigonométrica teniendo en cuenta las relaciones geométricas establecidas inicialmente para eliminar las referencias al ángulo �1:

2 2 2 2 22 2 3 2 2

2 2 2 2 2 2 22 2 3 2

2 2 2 2 2

2 32 2

cos 2 2cos 2 cos 2sen 2 sen 0

cos sen 2cos 2sen cos 4sen cos 0

cos cos 1 2cos 2sen cos 0

2cos 1 2cos

� � � � �

� � � � � � �

� � � � �

� �

� � �

� � � � �

� � � � �

� � 32 22cos 2cos� �� �

22 2

2 2 1

0

2cos 2cos 1 0

cos 0.3660 68.53º 42.94º AC 0.73a

� �

� � �

� � �

� � � �

A B

C

a

2a

a

2a

A B

C

a

a

�2

G2

G1

2P P

R

R

F2,x F1,x

F1,yF2,y

�1

H

�2

�2

- 166 -

Física Universitaria: Problemas de Física Estática del sólido rígido. M20.30

29. Dos bolas lisas de igual tamaño y de peso P están contenidas en el interior de una cavidad cilíndrica, tal como se indica en la figura. Determinar las reacciones en todos los contactos.

El diagrama de fuerzas del cuerpo libre para cada una de las dos bolas es el que se indica en la figura. Aplicamos las ecuaciones cardinales de la estática al sistema completo (las dos bolas), tomando momentos con respecto al punto O2. Tenemos

1 2

31

2 1 1 2

2O 2

N NN PPa N a N P

, �

� ��

de modo que

1 2 30.5 2N N P N P� � �

Para determinar las reacciones N12, es suficiente con expresar la condición de que la resultante de las tres fuerzas (concurrentes) que actúan sobre la bola superior sea nula:

12 1 2 212 1

12

sen 5cos 2

N NN N P P

N P��

� ��� � � ��� , ���

a

2a

a

2a N12

N2

N3

P

O2

N1

N12

� O1

� P

- 167 -

Física Universitaria: Problemas de Física Estática del sólido rígido. M20.31

30. Dos cilindros idénticos se apoyan en una pared vertical y un suelo horizontal, tal como se indica en la figura. Qué inclinación mínima � debe tener la cuña para que haya equilibrio, no existiendo rozamiento en ninguno de sus contactos.

En la figura presentamos el esquema de las fuerzas que actúan sobre cada cilindro en posición genérica, con � > � min. Entonces aplicando las ecuaciones cardinales de la estática al sistema �+� y tomando momentos en O2, obtenemos

1 2

3 2

1 1

12 2

32 3

11 2

sen2 cos2 2

sen[ ] tg

4 2cos 2

N NP N NaN aP P N

N P PP NN P N

N P

��

��

� ����� � � ���� � ���� �� � ��� $ �� �� � �� �� ����� ���

En las condiciones críticas de equilibrio, el cilindro inferior se encontrará a punto de rodar alrededor de C y perderá contacto con la superficie horizontal, por lo que será N3 = 0. El correspondiente valor del ángulo �, esto es � min, se obtiene a partir del resultado anterior

min3

tg4 2

PP N

� �� min

14

14º�� �

a

2a

P

P N3

N2

N1 O1

O2

A

B

C

� �

N12

N21

- 168 -

Física Universitaria: Problemas de Física Estática del sólido rígido. M20.32

31. Dos bolas idénticas, de masa m y radio r, están colocadas en el interior de un tubo cilíndrico (abierto en sus bases) de diámetro 3r. El conjunto descansa sobre un plano hori-zontal, como se muestra en la figura. Determinar la masa mínima que deberá tener el tubo cilíndrico para que el sistema no vuelque.

1cos 60º tg 60º 3 sen 60º 3 / 22 2rr

� �� � � � �

Aplicamos la primera ecuación cardinal de la estática a cada una de las bolas:

12 1

2 12

3 2 3

4 24

3cos 3tgsen 2 3

3

3cos3sen

2

N mgN N mgN mg N

N mg

N N N mgN N mg

N mg

��

��

��� ��� ���� � � � �� ��� �� � �������� �� ��� � � �� �� �� �� ����

L

Las fuerzas que actúan sobre el tubo son N1, N3, N5 y Mg. Las dos primeras (N1,N3) constituyen un par de fuerzas que tienden a volcar el tubo, cuyo momento es:

�par 132 sen 3

3M r N r mg rmg�� � �

En las condiciones críticas de vuelco, el tubo estará sometidos a dos pares de fuerzas de sentidos opuestos: el par de vuelco (N1,N3) y el par recuperador (N5,Mg). Aplicamos la segunda ecuación cardinal de la estática al tubo, tomando momentos en A:

3 22 3rrmg Mg M m

� ���- 9�� ���

3r

m,r

m,r

3r

mg

mg

N1

N3

N2

N2

N4

Mgr

2r

N3

N1

N5 �

A

- 169 -

Física Universitaria: Problemas de Física Estática del sólido rígido. M20.33

32. Un tractor de 2 t de peso, cuyas medidas más significativas se indican en el esquema, arrastra una carga de la que tiene que tirar con una fuerza horizontal de 1 t (a 1.5 m de altura): a) En tales condiciones, determinar las componentes vertical y horizontal de las fuerzas que actúan en el contacto de cada una de las ruedas con el terreno. b) ¿Qué arrastre máximo horizontal puede realizar el tractor sin que se levanten sus ruedas delanteras? c) Determinar el ángulo máximo de elevación que puede tener el terreno para que el tractor, sin arrastre, lo suba sin que se levanten sus ruedas delanteras.

a) En los tractores, tan solo las ruedas traseras son “tractoras” por lo que se “agarran al terreno”; por el contrario, las ruedas delanteras, en tanto que se mantenga constante la velocidad del tractor, mantendrán constante su velocidad angular, por lo que no necesitan “agarrarse al terreno”. En definitiva, el diagrama de fuerzas del cuerpo libre, aplicadas al tractor, es el que se indica en la figura. Aplicando las ecuaciones cardinales de la

estática, en las direcciones vertical y horizontal y tomando momentos en B, tenemos

A B A B

B

B

B

AA A

2 (por eje)(por eje)

2 1.5 2 1.5 2 (por

1.75 t1

0. jt e e)25

N N P N Nf F

N

N F P Nf

N

� �� � � � � �� �� �� �� � �� �� �� � � � � � �� �

��

b) Rescribimos las ecuaciones anteriores para la condición crítica (NA = 0):

AN B

B máx

A2

N P

f F

N

� �

�B

máx máxmáx

B

B

2 (por eje)

(por eje)

1.5 2

2 t

1.3 t

11 .3 t.5

N

f

N

F FF P

� �� � �� �� ��� � � �� �� �� � � � �� ����

c) Para la pendiente crítica será NA = 0 y, al no arrastrar carga alguna, será F = 0, por lo que tan solo quedan las tres fuerzas indicadas en la figura, que deberán ser concurrentes en B (para que el momento sea nulo). En consecuencia, el problema se reduce a una simple condición geométrica de que el centro de gravedad (G) se encuentre en la vertical del punto B:

1tg 0. 33.7º61.5

� �� � ��

1 m 2 m

FG

1.5 m

1 m

2 m

FG

1.5 mNA fB B A

NB

P

1 m

2 m

G 1.5 mfB

B A

NB

vertical

horizontal

terreno�

- 170 -

Física Universitaria: Problemas de Física Estática del sólido rígido. M20.34

400

300

200

400 kg

<100

A

B

F

37º

250

33. En el mecanismo que se esquematiza en la figura, todas las cotas están expresadas en milímetros y se supone despreciable el peso del propio mecanismo. a) Determinar el valor de la fuerza F que permite el equilibrio. b) Calcular las reacciones en los cojinetes A y B.

a) Cálculo de la fuerza aplicada. La condición de equilibrio de la polea móvil (inferior) implica que:

2 200 kgP T T� " � Tomando momentos con respecto al eje x, obtenemos

50 40200250 50250

kgF T F �� � � " � �

Las componentes de la fuerza F son cos37º 32 kg sen 37º 24 kgy zF F F F� � � � � �

b) Calculo de las reacciones en los apoyos del eje. � Tomamos momentos con respecto al eje z

(perpendicular al papel) en A y en B (Fig. inferior izquierda):

B B

A A

53 k(A) 300 32 500 0

(B) 300 32 200

g

21 kg

R R

R R

� � � � " �

� � � " �

� Tomamos momentos con respecto al eje y (perpendicular al papel) en A y en B (Fig. inferior derecha):

B B

A A

493 kg(A) 400 400 300 24 500

(B) 400 700 300 24 920 17 kg0

N N

N N

� � � � � �" �

� � � � � " �

A

RA

32 kg

x

y

RB

400 300 200

B

NA

P = 400 kg

x

z

NB

24 kg

400 300 200

A B

z

P

T T

T T

P = 400 kg

y

37º

F

- 171 -

Física Universitaria: Problemas de Física Estática del sólido rígido. M20.35

34. En el mecanismo que se representa en la figura se aplica un par mediante dos fuerzas de 100 N aplicadas en los puntos D y E de la aleta. Todas las cotas están expresadas en milímetros (mm). Determinar la fuerza F necesaria para establecer el equilibrio y las reacciones en los apoyos fijos B y C. (Se desprecia el peso de la aleta).

Las fuerzas que actúan sobre la estructura son: B (0, 0, 0)C (0, 0, 0.2)F (0, 0.04, 0.04)

x y

x y

B BC CF

� � �� � � �� � �

B i jC i jF i

y el par de fuerzas aplicadas en D y E, cuyo momento es

� � �par 0.120 100 cos30º sen 30º 12 cos30º sen 30º 6 3 6 m N� � � � � � � �M i k i k i k

Aplicamos las ecuaciones cardinales de la estática, tomando momentos en B:

B par0 BA BC 0� � � � � � � � � �+ +F F B C M M F C���� ����

Esto es:

0 00

00x x x

y yy

F F B CB CF

�� �� � � ���� �� � �� � � ��� ����

++ +F

B

6 3 0.26 3 0 00 0 0.04 0 0 0.04 0.2 0

6 0.04 0 0.2 6 0.040

yx

y x

CF CC F C

F

� �� � � � �� � � � � � ���� �� �� � �� � � �� �� �� � � �� � �� ��� �� � � ��� � � � � � � � �� � � �� �� � � � �� � � � � � �� � �� � � �� � � � �� � �� � � � �� � � � �� � �� � �� � � � � � ��� � +M

Y resolviendo el sistema de cinco ecuaciones con cinco incógnitas, tenemos:

6 36 3 0.2 0 30 3 52 N0.2

0.040.04 0.2 0 0.2 30 N0.2

66 0.04 0 150 N0.04

y y

x x

C C

FF C C F

F F

��� � � � � � � � �������� � � � � � ������� � � � � � ������

150 30 180 N00 30 3 52 N

x xx x

y y y y

B F CF B CB C B C

� � � � � � �� �� � �� �� � �� �� � � � � �� �� �

En definitiva: � �150 N 180 52 N 30 52 N� � � � � � �F i B i j C i j

120

30º

200

40

z

A B

C

E

D

F

x

y

40

By

Cy

Cx

Bx

F

C

B

A

E

D Mpar

30º

30º

z

x

- 172 -

Física Universitaria: Problemas de Física Estática del sólido rígido. M20.36

Otro método (mucho más cómodo) Una vez calculado el momento del par de fuerzas, como ya hemos hecho en el método anterior, imponemos la 2ª condición de la Estática anulando los momentos con respecto a los tres ejes coordenados que se indican en la figura, teniendo en cuenta que el momento de una fuerza con respecto a un eje es nulo si la línea de acción de la fuerza es coplanaria con el eje (i.e., lo corta o es paralela al eje):

parpar

parpar

cos30º 6 3 0 cos30º BC 0 52 N0.2BC

BA 150 0.04 0 BA BC 0 30 N0.2BC

sen 30º 6 0 sen 30º AF 0 150 N0.04AF

x y y

y x x

z

MM M C C

FM F C C

MM M F F

� � � � � � � � �

� �� � � � � � � �

� � � � � � � � �

Ahora, imponemos la 1ª condición de la Estática para determinar las reacciones en el punto B: 150 30 180 N0

00 30 3 52 N

x xx x

y y y y

B F CF B CB C B C

� � � � � � �� �� � �� ��� � � � � �� �� � � � � �� �� �+F F B C

En definitiva: � �150 N 180 52 N 30 52 N� � � � � � �F i B i j C i j

120

30º

200

40

z

A B

C

E

D

F

x

y

40

By

Cy

Cx

Bx

F

C

B

A

E

DMpar

30º

30º

z

x

- 173 -

Física Universitaria: Problemas de Física Dinámica del sólido rígido. M21.1

1. Dos masas m1 y m2 (m2 > m1), unidas mediante una cuerda inextensible y ligera,

penden de una polea de masa M, radio R y momento de inercia I respecto a su eje de rotación. Sobre el eje de la polea se ejerce una fuerza F vertical y hacia arriba, superior al peso del sistema, de tal magnitud que no produce aceleración sobre la masa mayor m2. Considérese que la cuerda no resbale sobre la polea. Determinar las aceleraciones de la masa pequeña m1 y del eje de la polea, la aceleración angular de ésta y la magnitud de la fuerza F.

Aplicamos las ecuaciones cardinales de la dinámica a cada una de las masas y a la polea, con el convenio de signos indicado en la figura y tomando momentos en el eje de la polea:

1 1 1 1

2 2

1 2

2 1

(1)(2) 0(3)(4)

T m g m aT m gF Mg T T MaT R T R I�

� �

� �

� � � �

� �

Disponemos de cuatro ecuaciones con seis incógnitas (T1, a1, T2, F, a, ). Obtenemos una nueva ecuación a partir de la condición de inextensibilidad de la cuerda (condición de ligadura)

� �1 2 1 22x x x x R l x x x l R� �� � � � � � � � �

que la derivamos dos veces con respecto del tiempo para obtener una relación entre las aceleraciones:

1 2 1 22 0 2x x x a a a� � � � ��� �� �� 1(5) 2a a �

La condición de que la cuerda no resbala en la polea nos proporciona otra ecuación:

1(6) aa a R a RR

� � �� � � �

Resolvemos el sistema de ecuaciones:

�1 1 1 2 12 1 1 2

2 21 2

(1) 2(4) 2

(2) 2

T m a m g m mIm g m a m g a a gIT m g R mR

� � �� � � �

� �

� �1 1 2 1 2 1(3) 2 2F Ma Mg m a m g m g M m m g M m a� � � � � � � � � �

� �1 2 11 2

1 2

2

2

M m m mF M m m gIm

R

� ��� �� � � �� ��� � � � �� �� �� �� ���

F

m2m1

F

m1g m2g

T2

T2 T1

T1

Mg

B A O +

+

x x2 x1

- 174 -

Física Universitaria: Problemas de Física Dinámica del sólido rígido. M21.2

2. La barra uniforme de masa m y longitud L de la figura está articulada en su

extremo superior A. Inicialmente se halla en reposo en la posición vertical. Se tira de la cuerda con una fuerza T. En el instante inicial, determinar la aceleración angular de la barra y la reacción en el pasador en A.

Las fuerzas que actúan sobre la barra se indican en rojo en la figura adjunta. El movimiento de la barra consiste en una rotación pura alrededor del eje que pasa por A. Aplicando la ecuación fundamental de la dinámica de la rotación, tomando momentos con respecto al eje A, se sigue:

2A A

3 1 9 2cos 45º4 3 8

TM I LT mLmL

� �� � " �+ �

El centro de masa de la barra describe una trayectoria circular. En el instante indicado, el c.m. tiene una aceleración centrípeta nula (por se nula su velocidad) y una aceleración tangencial (horizontal) dada por

9 22 16xl Ta

m�� � �

Aplicando la ecuación fundamental de la dinámica de traslación al centro de masa de la barra, se obtienen las dos ecuaciones siguientes:

xcm

cos 45ºsen 45º 0

x

y

T R mam

P T R� � ���� " ��, � � � ���

+ F a

de donde

9 2 2cos 45º12

216 6x xR a T T Tm

� ��� �� � � � �� �� ���� 2

2y mg TR ��

Las reacciones en el pasador A son iguales y opuestas a las calculadas y se indican en azul en la figura.

Ry

Rx A

P T

45º

G

3L/4

A

T3L/4

- 175 -

Física Universitaria: Problemas de Física Dinámica del sólido rígido. M21.3

3. En el esquema que se representa en la figura, el plano carece de

rozamiento. La cuerda es de masa despreciable, pasa a través del centro de masas de cada bloque y no desliza en la polea. La polea tiene forma de cilindro de radio R y masa m2. Siendo m1 = m; m2 = 2m; m3 = 4m y � = 30º. Determinar: a) El momento de inercia de la polea respecto a su eje. b) El momento resultante de las fuerzas que actúan sobre el sistema (las dos masas, la cuerda y la polea) respecto al centro de la polea. c) El momento cinético (o momento angular) del sistema respecto al centro de la polea cuando las masas se mueven con velocidad v. d) La aceleración de las masas. e) Las tensiones en la cuerda.

a) El momento de inercia viene dado por

222 21 1 2

2 2I m R mR mR� � �

b) Las fuerzas que intervienen en el movimiento del sistema son las que se representan en la figura. El momento neto con respecto al eje que pasa por O es

1O 3 1 2( sen ) 4M m g R m gR mg R m RR mgg�� � � � �

c) El momento angular respecto a O vendrá dado por

O 1 2 4 6L m vR I m vR mvR mvR m mvRvR�� � � � � � �

ya que v R�� . d) A partir de los resultados anteriores, podemos determinar directamente las aceleraciones. En efecto, teniendo en cuenta

que OO

ddL Mt

� , resulta

2Od d d 1(6 ) 6 6d d d

1.636

m/sL vmrv mr mra mgr a gt t t

� � � � � �

e) Para calcular las tensiones en las cuerdas debemos escribiremos las ecuaciones del movimiento para cada uno de los bloques:

761 1 1 1 1

1 13 3 3 3 3 2 6

43

7( )6

sen ( sen ) (4 ) ( )

T m g m a T m a g m g

m g T m a T m g a m

mg

mgg� �

� � � � � �

� � � � � � �

vv

m3 m2

m1

� m3g

m1g

T1

T1

T3

T3 N3

O

vv

'

- 176 -

Física Universitaria: Problemas de Física Dinámica del sólido rígido. M21.4

4. Una caja de 15 kg está sujeta al extremo de una cuerda inextensible

arrollada sobre un tambor uniforme de 40 kg y 600 mm de diámetro, según se indica en la figura. En el instante representado, la caja está cayendo a 9 m/s. Determinar el par constante de frenado que hay que aplicar al tambor para que la caja quede en reposo tras descender 3 m.

Método dinámico: El movimiento del bloque es un movimiento uniformemente acelerado, por lo que aceleración (de frenado) que detendrá al bloque se calcula a partir de la expresión:

22 2

0 2 vv v ax a� � �2 2 20 0

2

9 m13.52 2 2 3 s

v vx x

�� � � � � �

Llamando a la aceleración angular del tambor, la condición de ligadura que relaciona el movimiento de éste y con el del bloque se expresa por

a R�� Escribimos las ecuaciones del movimiento: Bloque (movimiento de traslación):

( )mg T ma T m g a� � � � Tambor (movimiento de rotación):

2 2par par

1 1 12 2 2

TR M I MR M TR MR TR MRa� � �� � � � � � �

Sustituyendo los valores dados en las expresiones anteriores, calculamos la tensión T de la cuerda y el momento del par pedido:

( ) 15 (9.8 13.5) 349.5 NT m g a� � � � � �

par40 13. 185.5( ) 349.5 0.3

29 N m

2.MaM T R

� �� ��� � � � � ��� ���

Método de la energía: Mientras que el bloque desciende una distancia h, hasta detenerse, el tambor gira un ángulo �; además, en cada instante, la velocidad del bloque está relacionada con la velocidad angular del tambor:

00

3000 910 rad 30 rad/s300 0.3

vhh R v RR R

� � � �� � � � � � � �

El trabajo realizado por el par de frenado, par parW M �� , es igual a la disminución de la energía del sistema, que viene dada por:

2 2 21 1 10 02 2 2

2 2 2 21 1 1 10 02 4 2 4

( )

15 9.8 3 15 9 40 9 1858.5 J

E mgh mv MR

mgh mv Mv

� �� � � � �

� � � � � � � � � � �

Y el momento de frenado será:

parpar par par 181858.5

105.9 N.m

W EW M M ��

� ��

� � � � �

' Mpar

300 mm

40kg

15kg 9m/s

m

mg

Mpar

T

M,R T

+

+

- 177 -

Física Universitaria: Problemas de Física Dinámica del sólido rígido. M21.5

5. Una varilla de longitud L y masa m puede girar sin rozamiento alrededor de un eje

horizontal O que pasa por su punto medio. En uno de los extremos de la varilla hay adherida una masa puntual m. Se abandona el sistema en posición horizontal. Determine: a) La posición del centro de masa. b) El momento de inercia del sistema respecto al eje O. c) La velocidad angular cuando la varilla alcance la posición vertical. d) La aceleración angular cuando la varilla forma un ángulo � con la vertical. e) La fuerza que ejerce el eje sobre la varilla cuando ésta alcanza la posición vertical.

a) Tomamos el origen en O: � � 1

cm 4

/ 2 0OG

2 4m L m Lx h L

m� �

� � � � �

b) El momento de inercia del sistema es la suma de los momentos de inercia de la masa y de la varilla:

22 2 2

O1 1 1 1

2 12 4 12 3LI m mL mL mL

� � � �� �� �� � � � �� �� �� �� �� �

c) El movimiento del sistema consiste en una rotación pura alrededor del eje O. Puesto que el sistema es conservativo, la conservación de la energía se expresa en la forma:

2 2 2O

1 1 302 2 3L gmg I mgL mL

L� � �� � � � �

d) Aplicamos la Ecuación Fundamental de la Dinámica de la Rotación, tomando momentos en el eje de rotación:

O2O

2 sen2 sen 342 sen sen1 23

Lmgmgh gmgh II LmL

��� � � �� � � �

e) Aplicamos la Ecuación Fundamental del Movimiento del c.m. del sistema:

� �cm cm2 2 2R mg m a R m g a� � � �

y teniendo en cuenta que las componentes tangencial y normal del c.m., para la posición vertical ( � = 0 ), son:

2t n

3 3 3sen 04 8 4 4L g g L ga h a h

L� � � �� � � � � � �

resulta que acm = an , de modo que

�cm3 72 2 14 2

R m g a mg mg� ���� � � � ��� ���

L

mm O

L

mm O G

h

O

m

G

R

2mg

R

2mg

G acm

O

m

- 178 -

Física Universitaria: Problemas de Física Dinámica del sólido rígido. M21.6

6. El árbol vertical de la figura gira con una velocidad inicial de 20 rad/s cuando la masa

puntual A de 2.5 N de peso comienza a deslizarse lentamente hacia fuera a lo largo de miembro horizontal. Determinar la disminución de la velocidad angular del árbol cuando la masa A se desliza desde 75 mm hasta 600 mm a partir del eje del árbol.

Puesto que no existe momento dinámico externo con respecto al eje de rotación (vertical), se conserva el momento angular o cinético con respecto a dicho eje. Esto es:

11 1 2 2 2 1

2

II II

� � � �� " �

Despreciando la masa de la varilla y el momento de inercia del dispositivo, serán: 22

1 1 1 12

2 22 2

I m III m

� ���

� � �� � �� � �" � �� �� ��� � ���

de modo que 2 2

12 1

2

75 20 0.31 rad/s600

�� �

� � � ��� ���� � � �� ��� �� ��� � �

Por consiguiente, será

2 1 19.69 rad/s� �� �� � ��

A

'

525 mm 75 mm

A

'

525 mm 75 mm

- 179 -

Física Universitaria: Problemas de Física Dinámica del sólido rígido. M21.7

7. Considere una barra delgada, con masa m = 4 kg y longitud l = 1.2 m, que oscila sin rozamiento en un plano

vertical alrededor de un eje horizontal que pasa por un punto de la barra situado a 1/4 de uno de los extremos de la misma. a) Expresar la aceleración angular de la barra en función del ángulo que forma con la vertical. b) Calcular el periodo de las pequeñas oscilaciones de rotación de la barra.

a) La única fuerza que produce momento respecto al eje de rotación es el peso de la barra que está aplicado en el centro de gravedad de la misma. Planteamos la ecuación para la dinámica de la rotación de la barra para una posición genérica en la que ésta forma un ángulo � con la vertical: i.e.,

O O Osen4lM I mg I� � �� � ��� ��

donde IO es el momento de inercia de la barra respecto al eje de de rotación y ��� la aceleración angular de la barra. Calculamos I aplicando el teorema de Steiner:

22 2 2

O cm1 1 7

4 12 16 48lI I m ml ml ml

� ���� � � � ��� ���

Por lo que la ecuación para la aceleración angular resulta ser 27 12sen sen

4 48 7l gmg ml

l� � � �� � ��� ��

b) La ecuación anterior resulta ser la de un péndulo compuesto. Haciendo la aproximación de pequeñas oscilaciones, i.e., sen � �= , tenemos la ecuación del movimiento de rotación, que podemos escribir en la forma

12 07

gl

� �� ���

Que corresponde a oscilaciones armónicas simples cuya frecuencia angular y periodo vienen dados por

12 12 9.8 2 23.74 rad/s 1.68 s7 7 1.2 3.74

g Tl

� ��

��

� � � � � ��

G

mg

M

l

l/4

O

- 180 -

Física Universitaria: Problemas de Física Dinámica del sólido rígido. M21.8

8. a) Calcular la frecuencia de las pequeñas oscilaciones de un aro de radio R colgado de la pared mediante un

clavo horizontal. ¿Cuál es la longitud reducida de este péndulo físico? b) Repetir el apartado anterior si se suprime la mitad inferior del aro.

a) El Teorema de Huygens para los puntos conjugados en un péndulo compuesto nos permite escribir

2cm

G' ' I mRmhh I h Rmh mR

� � � �

de modo que el conjugado de O es O’ (obvio) y la distancia entre ellos es la longitud reducida del péndulo físico constituido por el aro en sus pequeñas oscilaciones; eso, es

' 2h h R R R�� � � � � La frecuencia de las pequeñas oscilaciones será:

1 1 22 2

Rg g�

�� �

� �

b) Aplicamos el Primer Teorema de Pappus-Guldin para determinar la posición del c.m. (G) de medio aro, siendo CG=:

2 2 24 2 GOS sL R R R h R R�� � �� � �

� ��

� � � � � � � �����

El Teorema de Steiner nos permite determinar el momento de inercia de medio aro con respecto a su c.m.:

�22 2 2 2 2

C G G C 2 2

44I I m I I m mR m R mR�

� �� �

�� � � � � � �

De nuevo, el Teorema de Huygens nos permite determinar el conjugado O’ de O:

� �

�2cm

G 2

4 2' ' GO'

2Imhh I h R Rmh

� � �� � �

� �� � � � �

����

�22' 2h h R R R��

�� �

��� � � � �

de modo que, de nuevo el conjugado de O es el punto O’ diametralmente opuesto. La frecuencia de las pequeñas oscilaciones será:

1 1 22 2

Rg g�

�� �

� �

O

O’

G

h

h’

O

O’

C

h

h’

G

- 181 -

Física Universitaria: Problemas de Física Dinámica del sólido rígido. M21.9

9. Un disco uniforme, de radio R y masa m, posee una pequeña perforación practicada en el

punto A, a una distancia h del centro del disco. a) Determinar la frecuencia de las pequeñas oscilaciones que se producen cuando el disco está suspendido en el punto A. b) ¿A qué distancia h hay que hacer el agujero para que la frecuencia sea máxima?

a) Escribimos la ecuación fundamental del movimiento de rotación alrededor de un eje fijo, tomando momentos respecto al eje de suspensión que pasa por el punto A

AA

sen sen 0mghmgh II

� � � �� � � ��� ��

y, como para valores pequeños de � podemos establecer la aproximación sen � = � , resulta

A

0mghI

� �� ���

que es la ecuación del movimiento, que corresponde a un m.a.s.de rotación (por ser de la forma la forma 2 0� � �� ��� ) en el que la frecuencia angular es,

22 2 2 2

2 2

2 221 2 22

A

mgh mgh gh ghI R h R hmR mh

� � ��� � � � �� ��

b) La frecuencia angular será máxima cuando lo sea ' 2; esto es,

� � � �

� �

2 2 2 2 2 2

2 22 2 2 2

d d 2 4 20 2 2 0d d 2 2

R h h h R hg gh h R h R h

� � � � �� � � �

% % � �

lo que se consigue cuando el numerador es nulo

2 22 0 222

RR h h R� � � �

h A

M,R

A

G

mg

+ h

- 182 -

Física Universitaria: Problemas de Física Dinámica del sólido rígido. M21.10

10. Consideremos un disco, de masa m y radio R, que oscila sin rozamiento en un plano vertical alrededor de un

eje horizontal y perpendicular al disco y que pasa por un punto del mismo situado a R/2 del centro del disco. a) Expresar la aceleración angular del disco en función del ángulo que forma con la vertical. b) Calcular el periodo de las pequeñas oscilaciones de rotación del disco.

a) La única fuerza que produce momento respecto al eje de rotación es el peso del disco que está aplicado en el centro de gravedad del mismo. Planteamos la ecuación para la dinámica de la rotación del disco para una posición genérica en la que ésta forma un ángulo � con la vertical: i.e.,

O O Osen2RM I mg I� � �� � ��� ��

donde I0 es el momento de inercia del disco respecto al eje de de rotación y ��� la aceleración angular del disco. Calculamos I0 aplicando el teorema de Steiner:

22 2 2

O cm1 1 3

2 2 4 4RI I m mR mR mR

� ���� � � � ��� ���

Por lo que la ecuación para la aceleración angular resulta ser 23 2sen sen

2 4 3R gmg mR

R� � � �� � ��� ��

b) La ecuación anterior resulta ser la de un péndulo compuesto. Haciendo la aproximación de pequeñas oscilaciones, i.e., sen � � �, tenemos la ecuación del movimiento de rotación, que podemos escribir en la forma

2 03

gR

� �� ���

Que corresponde a oscilaciones armónicas simples cuya frecuencia angular y periodo vienen dados por

2 323 2

g RTR g

� �� �

G

mg

m

R

R/2 O

- 183 -

Física Universitaria: Problemas de Física Dinámica del sólido rígido. M21.11

11. Un cilindro, de masa m y radio R, desciende por un plano inclinado un ángulo � respecto de la horizontal.

Determinar el valor máximo de dicho ángulo para que el cilindro ruede sin deslizar sobre el plano, siendo � el coeficiente de rozamiento entre el plano y el cilindro.

Escribimos la ecuación fundamental del movimiento de rotación, tomando momentos con respecto a la generatriz del cilindro en contacto con el plano inclinado:

2 2 2O

1 3 2 sensen2 2 3

gmgR I mR mR mRR�

� � � � �� ���� � � � ��� ���

Rescribimos la ec. fundamental de la rotación tomando momentos con respecto al eje de simetría del cilindro (que pasa por el c.m.):

2cm

1 12 2

R f I mR f mR� � �� ���� � ��� ���

Sustituyendo en esta ecuación el resultado obtenido en la primera, tenemos

1 sen3

f mgR ��

de modo el que rozamiento estático entre el cilindro y el plano inclinado tendrá que ser tanto mayor cuanto mayor sea el

ángulo de inclinación de éste, a fin de mantener la rodadura (sin deslizamiento) del cilindro. Puesto que el rozamiento estático no puede superar un cierto valor máximo, será

�1 sen cos tg 3 arctg 33

f N mg mg� � � � � � � �- - - -

y la rodadura sin deslizamiento tan solo será posible si se cumple esta última condición. Por consiguiente, el ángulo pedido es

�max = arctg 3� �

m, R

f

mg

N

cm

O

- 184 -

Física Universitaria: Problemas de Física Dinámica del sólido rígido. M21.12

12. Un cilindro macizo baja rodando sin resbalar por un plano inclinado. a) Calcular la aceleración del centro de

masa del cilindro. b) Determinar el valor mínimo de la fuerza de rozamiento (estático) entre el plano y el cilindro a fin de que éste ruede sin resbalar. c) Calcular el valor mínimo del coeficiente de rozamiento para que el cilindro no resbale. d) Estudiar el movimiento del cilindro en función de diversos valores del coefi-ciente de rozamiento. e) ¿Se conserva la energía total del cilindro cuando éste rueda sin resbalar?

abc) Aplicamos las ecuaciones cardinales de la dinámica de traslación y rotación al cilindro, tomando momentos en el c.m.:

2

[1] cos 0[2] sen

1[3]2

N mgmg f ma

fR mR

��

� �� �

� ���� �� ��� �

que, junto con la condición de rodadura, [4] a R��

constituyen un sistema de cuatro ecua-ciones con cuatro incógnitas (N, f, a, ).

Sustituyendo la [4] en la [3] tenemos 1 12 2

f mR ma�� ���� ��� ���

De la [2] se sigue 3 2sen sen2 3

mg ma f ma a g� �� � � � que es la aceleración

pedida. El valor mínimo de la fuerza de rozamiento necesaria para la rodadura es

1 1 2 1sen sen2 2 3 3

f ma m g mg� �� � �

De la definición del coeficiente de rozamiento estático, se sigue 13

mínsen 1 1tg tg

cos 3 3mgff N

N mg�

� � � � ��

- 9 � � �

d) Tipo de movimiento del cilindro: 3 Si � = 0, el cilindro no puede rodar, simplemente desliza (traslación pura). 3 Si 0 < � < �mín , el cilindro rueda y resbala. 3 Si � � �mín , el cilindro rueda sin resbalar (rodadura).

e) La energía se conservará en tanto que no haya resbalamiento (rozamiento cinético) entre el cilindro y el plano; esto es, que sea � � �mín .

mg

f

N

- 185 -

Física Universitaria: Problemas de Física Dinámica del sólido rígido. M21.13

13. a) Hallar la aceleración del centro de masa de un cilindro macizo de 20 cm de radio y 10 kg de masa, que

rueda sin deslizar sobre un plano inclinado 30º. b) Si el cilindro parte del reposo, que velocidad alcanzará al descender 5 m de altura.

a) Escribimos las Ecuaciones Cardinales de la dinámica de traslación (c.m) y de rotación alrededor de un eje que pasa por el c.m., así como la condición de rodadura:

cm

2

cm

sencos 0 1con

2

mg f maN mg

I mRRf Ia R

��

��

� � ����� � ��� ��� ���� ����

que constituyen un sistema de cuatro ecuaciones con cuatro incógnitas (acm, , f, N). Operando con las dos últimas ecuaciones, tenemos

22 cm

cm

cm

11 1

22 2

Rf mR aRf mR f maRa R

���� � � ����� ��

y sustituyendo este valor en la primera ecuación 2

cmcm cm1 2 1sen sen 3.27 m

3/

2 3smg m aa ma g g� � ���� �

b) Se trata de un movimiento de traslación con aceleración constante a lo largo del plano. Descender una altura de h = 5 m implica un desplazamiento x a lo largo del plano

5 10 msen sen 30º

hx�

� � �

y la velocidad alcanzada, partiendo del reposo, será

cm2 2 3.27 10 8.08 m/sv a x� � � � �

mg

N

f c.m.

5 m

- 186 -

Física Universitaria: Problemas de Física Dinámica del sólido rígido. M21.14

14. Una bola esférica, maciza y homogénea, de radio r y masa m, rueda sin resbalar por un plano inclinado un

ángulo � con la horizontal. a) Determinar la fuerza de rozamiento actúa sobre la bola, indicando gráficamente su dirección y sentido. b) Calcular coeficiente de rozamiento mínimo que se requiere entre el plano y la bola pera evitar que resbale.

a) Aplicamos las ecuaciones cardinales del movimiento de la bola, tomando momentos en C (c.m. de la bola), de modo que:

2

[1] cos 0 cos[2] sen

2 2[3] 5 5

[4]

N mg N mgmg f ma

fr I mr f mr

a r

� ��

� � �

� � � ����� � ������ � � ������ ���

que junto con la condición de rodadura [4] constituyen un sistema de cuatro ecuaciones con cuatro incógnitas (N, f, a, ).

Sustituyendo la ec. [4] en la ec. [3] y resolviendo el sistema de ec. [2] y [3], tenemos [1] cos[2] sen

2 7 5( ) sen sen2 2 5 5 7[3] 5 5

N mgmg f ma

mg ma ma ma a gf mr ma

�� �

� ����� �� � �� �� �� � � � � ��� �� � � �� �� ��

de modo que 2 sen7

25

mgf ma �� �

b) El coeficiente de rozamiento mínimo que se requiere será 2 sen

ctg

os27

7mgff N

N mg

�� �

��- 9 � �

f

mg

N

C

A

+ +

+

- 187 -

Física Universitaria: Problemas de Física Dinámica del sólido rígido. M21.15

15. Un carrete está constituido por dos discos de radio R unidos por un eje de radio r,

siendo m la masa e I el momento de inercia del conjunto con respecto a su eje de simetría de rotación. Alrededor del eje está enrollada una cuerda o cinta ligera, sujeta por su otro extremo a un punto fijo. Calcular la tensión de la cuerda y la aceleración de carrete cuando desciende por un plano inclinado perfectamente liso, tal como se indica en la figura.

En la figura mostramos el esquema de fuerza que actúan sobre el carrete. Aplicando las ecuaciones cardinales de la Dinámica del Movimiento Plano del Sólido Rígido, tomando momentos con respecto al eje del carrete y con el convenio de signos indicado en la figura, tenemos:

O

sencos 0

mg T mamN mg

I Tr I

��

��� � ���� � �� �� � �� �� ��� ��� � ����

++

F a

M ��

Por otra parte, como el punto de aplicación de la fuerza T se encuentra instantáneamente en reposo, dicho punto constituye el CIR del movimiento en ese instante, por lo que entre la aceleración del c.m. y la aceleración angular establecemos la condición de rodadura:

a r�� En definitiva, tenemos

2

2

sen sen

cos

Img T ma m a mgr

N mgI ITr I a T ar r

� �

� � �� �� �� � � ��� � ��� � ��� ������ � � �����

de modo que

� �1 /sen sen

/ 1 / 1

2

2 2

I mra = g T mgI mr I mr

� ��� �

Rr

I

R r

I

T

N

mg

+ +

- 188 -

Física Universitaria: Problemas de Física Dinámica del sólido rígido. M21.16

16. Un disco delgado de 600 mm de diámetro y masa 60 kg se mantiene sobre un plano

inclinado gracias a un bloque y a un cable arrollado en su superficie según se indica en la figura. Determinar la tensión del cable y la aceleración del centro de masa del disco una vez suprimido el bloque, con lo que el disco podrá deslizarse libremente por el plano inclinado. El coeficiente de rozamiento entre disco y plano vale 0.20.

Al suprimir el bloque, el disco inicia un movimiento acelerado partiendo del reposo. El movimiento del disco es un movimiento plano en el que el centro instantáneo de rotación (CIR), i.e., el punto de velocidad instantánea nula, se encuentra en la posición que se indica en la figura. En estas condiciones, el punto A de contacto del disco con el plano inclinado desliza sobre éste, por lo que, al tratarse de un contacto rugoso, aparece una fuerza de

rozamiento (f). Ecuaciones del movimiento:

1 2

Geje

eje

rotación e

[1]

n

[2]

[ G3]

sencos 0

x

y

mg T f maN mg

T R

� � ������ � ������� f R� 212 mR� ��

�������������

Ecuaciones complementarias:

G

rozamiento en A

rodadura en el CIR

[4][5]

f Na R��

�� ���� �� �� � �� �� �

Disponemos de cinco ecuaciones con cinco incógnitas (T, f, aG, N, ). Resolvemos sustituyendo [4] y [2] en [1] y [5] en [3]; obtenemos dos ecuaciones con dos incógnitas (T, aG):

G

1G2

sen coscos

mg T mg maT mg ma

� � �

� �

� � � ����� � ���

Sumando miembro a miembro estas dos ecuaciones obtenemos: 3 2

G G2 32

G 3

(sen 2 cos ) (sen 2 cos )sen5 3.33 m/0º 2 0.2 cos50º ) 0.34 s

mg ma a ga g g� � � � � �� � � �

# � � � � � �

1 21 1G2 3

13

1cos (sen 2 cos ) cos (sen cos )3

(sen 50 175.º 0.2 cos50º ) 60 17. 3 N89 kg =

T ma mg mg mg

T

� � � � � � � � � �� � � � � � �

# � � � � �

50º

50º mg

T

f N G

A

CIR

x

y

aG

- 189 -

Física Universitaria: Problemas de Física Dinámica del sólido rígido. M21.17

17. Deseamos acelerar una esfera homogénea de 2 kg de masa y 5 cm de radio,

situada sobre un plano horizontal rugoso, ejerciendo sobre ella una fuerza horizontal constante cuya línea de acción pasa por el centro de la esfera. El coeficiente de rozamiento estático entre la esfera y el plano vale 0.3. a) ¿Cuál será la máxima aceleración que podemos comunicar a la esfera con tal que ruede sin resbalar sobre el plano? b) ¿Cuál será la magnitud de la fuerza que produzca esa aceleración máxima?

Aplicamos las ecuaciones fundamentales de la dinámica del sólido rígido, tomando momentos en el centro de masa de la esfera, escribimos la ecuación correspondiente a la rodadura y resolvemos:

cmcm22

5 2cm cm5

cm (condición de rodadura)

F f maF f ma

f R mRf ma

a R

��� � �� �� � ��� �� � �� � �� ���� ���

De modo que 7

cm52

cm5

F maf ma

� ����� ���

En tanto que la esfera no resbale sobre el plano, tenemos rozamiento estático entre la esfera y el plano, de modo que

5cm 2

72cm5 2

a gf N mg ma mg F mg

f mg

00 0 0 0

0

6��6 � � 6 � 6�� 6�

Sustituyendo los valores dados en el enunciado: 25

cm 272

0.3 9.8 7.35 m/s0.3 2 2.1 kg 20.6 N

0.3 2 0.6 kg 5.9 N

aFf

�� - � ���� - � � ����� - � � ���

F

f N

P

F

- 190 -

Física Universitaria: Problemas de Física Dinámica del sólido rígido. M21.18

18. Una esfera maciza y homogénea, de masa m y radio r resbala sin rodar sobre una

superficie horizontal rugosa bajo la acción de una fuerza F dirigida horizontalmente y aplicada a una altura h < r, como se indica en la figura. Determinar la aceleración de la esfera y el coeficiente de rozamiento entre ésta y el plano.

Puesto que en el enunciado del problema nos aseguran que “resbala sin rodar”, el movimiento de la esfera consiste en una traslación pura (no hay rotación). Interviene una fuerza de rozamiento cinético en el punto de contacto de la esfera con el plano sobre el que desliza sin rodar, tal que

k kf mg��

Escribimos las Ecuaciones Cardinales de la Dinámica del movimiento plano del Sólido rígido, tomando momentos en el centro de masa de la esfera:

�k cm

k

F f maF r h f r

� � ����� � ���

De la segunda ecuación se sigue:

k k k1 1h h Ff F mgr r mg

� �� � � �� �� �� � � � �� �� �� �� �� �

De la primera ecuación obtenemos la aceleración de la esfera:

kcm 1 1F f F h hFa

m m r mr� �� �� ��� �� � � � ��� ��� �� �

F m,r

h

F

fk

C

h

m,r

- 191 -

Física Universitaria: Problemas de Física Dinámica del sólido rígido. M21.19

19. Una esfera maciza y homogénea, de masa m y radio R rueda (sin resbalar) so-

bre una superficie horizontal rugosa bajo la acción de una fuerza F dirigida horizontalmente y aplicada a una altura h < R, como se indica en la figura. De-terminar la aceleración de la esfera y el coeficiente de rozamiento mínimo entre ésta y el plano que impida el resbalamiento.

Aplicamos las ecuaciones fundamentales de la dinámica del sólido rígido, tomando momentos en el centro de masa de la esfera (C), y tenemos en cuenta la condición de rodadura:

� 2

[1]2[2]5

[3]

F f ma

fR F R h mR

a R

� � ����� � �� ��� � � �� � ��� � ��� ���

de modo que disponemos de tres ecuaciones con tres incógnitas (f, a, ). Sustituyendo las ecuaciones [3] y [1] en la [2], tenemos:

� �2 2 5[2] 15 5 7

hfR F R h Rma R F f f FR

� ���� � � � � � � �� ���

Sustituyendo este resultado en la [1], obtenemos

2

5 5[1]7 7

5[3]7

h h Fma F f F aR R m

a h FR R m

� � � �

� �

en las direcciones indicadas en la figura. En las condiciones de rodadura, existirá rozamiento estático entre la esfera y el plano. Aplicando la definición del coeficiente de rozamiento estático, será

ff N mgmg

� � �- � 9

De modo que deberá ser

mín517

h FR mg

�� ���� � �� ���

Otro método: Como antes, pero tomamos momentos en el CIR (situado en I).

2

[1] 517 57[2] 15 77 5

[3] 5 7

F f ma hf F ma Ff h FRFh mR

mg R mgR hF ma ma Fa R h R

� �

� � � �� � ��� �� �� � � �� �� � �� �� � � �� � � �� �� �� 9 � �� �� �� �� �� �� �� � � �� � � �� �� �� ��

F

m,R

h

Fm,R h f

C + +

I

- 192 -

Física Universitaria: Problemas de Física Dinámica del sólido rígido. M21.20

20. Un disco, de 300 g de masa, 10 cm de radio y de pequeño espesor, pivota (gira)

alrededor de un diámetro vertical sobre el centro de una plataforma circular horizontal de mayor tamaño, 450 g de masa y 20 cm de radio, que puede girar alrededor de su eje de simetría vertical, común con el de rotación del disco pequeño. Cuando el disco pequeño se encuentra girando a 130 r.p.m., se libera el disco grande para que pueda empezar a girar. Una vez que, por efecto del rozamiento entre ambos, se igualen sus velocidades de rotación, ¿cuál será esa velocidad angular de rotación común? Determínense, también, los momentos de inercia necesarios para la resolución de este ejercicio.

Para determinar el momento de inercia de un disco circular con respecto a uno de sus diámetros, aplicamos el teorema de los ejes perpendiculares:

2diám diám

1 122 4zz xx yy zzI I I I I I mR� � � � �

de modo que los momentos de inercia de los discos que intervienen en este problema, con respecto a sus respectivos ejes de rotación, son

22 21 1 1

2 2 22 2

2 1 1 12 2 2

12 2 450 204 12

1 300 102

I m R I m RI m RI m R

���� � � ��� � � ��� ��� ����

La rotación por pivotamiento del disco pequeño sobre el grande transmite un par a este último y lo acelera; la reacción de ese par retarda la rotación del disco pequeño. Finalmente, se igualan las velocidades angulares a un valor común �’. Puesto que no intervienen momentos exteriores al sistema, se conservará el momento angular del mismo; esto es,

� 11 1 2

21 21

130 10 r.p.m.131

II I I II II

�� � � �) )� � � � � �

� �

'

(1)

(2)

x y

z

- 193 -

Física Universitaria: Problemas de Física Dinámica del sólido rígido. M21.21

21. En la figura se muestra esquemáticamente el tren de aterrizaje de un avión visto desde

atrás. El radio de la rueda es de 40 cm y su momento de inercia es de 2.5 kg·m2. El avión despega a una velocidad de 180 km/h. Después del despegue, se recoge el tren de ate-rrizaje girándolo lateralmente a razón de 45� por segundo. Determinar la magnitud del par ejercido sobre la rueda por su soporte e indicar las direcciones de las magnitudes vectoriales implicadas.

Datos

Durante la recogida del tren de aterrizaje, el modulo del momento angular de la rueda permanece constante, pero su dirección cambia, ya que gira con una velocidad angular *. El par ejercido sobre la rueda es igual a la variación de su momento angular por unidad de tiempo; esto es,

245.4 N md 312.5d 4

M Lt

�� � � " � � � � �

LM � L

y la dirección de M es la que se indica en la figura.

Momento de inercia: I = 2.5 kg·m2

Radio de la rueda: R = 0.40 m

Velocidad: v = 180 km/h = 180/3.6 m/s = 50 m/s

Velocidad angular rueda: ' = v/R = 50/0.40 = 125 rad/s

Momento angular rueda: L = I' = 2.5 � 125 = 312.5 kg·m2/s

Velocidad angular tren: * = 45º /s = �/4 rad/s = 0.7854 rad/s

45º/s

L

M

*

- 194 -

Física Universitaria: Problemas de Física Sólido Rígido: Trabajo y energía. M22.1

1. La barra homogénea de la figura puede girar sin rozamiento alrededor de un

eje horizontal que pasa por uno de sus extremos. Se coloca en posición horizontal y se abandona. Determinar: a) La aceleración angular de la barra en el instante en que se deja en libertad. b) La fuerza ejercida por el eje sobre la barra en ese instante. c) La velocidad del centro de masa de la barra cuando ésta alcanza la posición vertical.

a) Aplicamos la ecuación fundamental de la dinámica de rotación, tomando momentos con respecto al eje de rotación:

2 32

12 3Lmg I m g

LL� � ��� �

b) El c.m. de la barra describe una trayectoria circular, de radio L/2, de modo que las componentes intrínsecas de su aceleración, en el momento inicial, son

2

t n3= 0

2 4 / 2L va g a

L�� � �

Aplicando las ecuaciones del movimiento del c.m, tenemos:

n

t

034

014

x

y

x

y

NN ma

mg N ma m N gg m

� �� �� �� �� � � �� �� � �

����� ��

c) Puesto que el sistema es conservativo, y solo estamos interesados en la velocidad final, nos serviremos del Principio de Conservación de la Energía para determinar la velocidad angular de la barra en el instante pedido:

2 2

21

3

1 302 2L mgL mgL gmg I

I LmL� �� � � � � �

y la velocidad de su centro de masa, en ese instante, será:

cm3

2 21 32

L L gvL

gL�� � �

m, L

m, L

Ny

mg

Nx

++

+

Ep=0

L/2 vcm

- 195 -

Física Universitaria: Problemas de Física Sólido Rígido: Trabajo y energía. M22.2

2. Una barra de longitud L y masa m puede girar alrededor de un eje fijo que

pasa por uno de sus extremos O. Dejamos caer la barra desde la posición horizontal (� = 0), partiendo del reposo. Determinar: a) La aceleración y velocidad angulares de la barra en función del ángulo � . b) La reacción en O cuando � = �/2.

Momento de inercia de la barra respecto de O: 2O

13

I ml�

a) Conservación de la energía:

2 2 21 1 3 se0 ( sen )2 2

n3

lmg m gll

� � � �� ���� � � �� ���

�� �

A partir de la velocidad angular, por derivación determi-namos la aceleración angular:

2 3 3 3sen 2 cos cos2

g g gl l l

� � �� � � � �� � �� ��� � ��

b) Para 2 3 02

gl

�� � �� � �� ��

Escribimos la ecuación del movimiento del centro de masa:

cm2

02con

3 32 2 2

xx x

y yy

laR mam

R mg ma l g la gl

��� � ��� �� �� �� � �� �, � �� �� � � �����

F a��

de modo que

32

052

x

yy mg ma mg mg

R

R mg

������ � � � � ��

��

O

mg

l/2

Rx

Ry

v

m, L

m, L O

mg

l/2

l/2

Ep=0

- 196 -

Física Universitaria: Problemas de Física Sólido Rígido: Trabajo y energía. M22.3

3. Una varilla homogénea está apoyada en posición vertical sobre uno de sus extremos en

contacto con un plano horizontal, de modo que inicialmente se encuentra en equilibrio inestable. La desplazamos ligeramente de esa posición para que comience a caer. Calcular su velocidad angular cuando alcanza la posición horizontal sobre el plano horizontal: a) Suponiendo que la varilla no resbale sobre el plano. b) Suponiendo que el plano sea perfectamente liso.

a) La varilla presenta una rotación pura alrededor de su extremo inferior en E. Aplicamos el principio de Conservación de la Energía a los instantes inicial y final:

2 2 2E

1 1 1 [1]2 2 2 3lmg I ml� �

� ���� � �� ���

De modo que

3gl

��

b) Como no actúan fuerzas que den componente en la dirección horizontal, el centro de masas de la varilla no puede desplazarse en esa dirección, por lo que tan solo desciende verticalmente. El movimiento de la varilla consiste en una traslación (vertical) del c.m. simultánea con una rotación alrededor de un eje horizontal que pasa por el c.m. (G). Aplicamos de nuevo el principio de Conservación de la Energía a los

instantes inicial y final: 2 2G G

1 1 [2]2 2 2lmg mv I �� �

Puesto que el punto E tan solo se desplaza horizontalmente, podemos determinar la velocidad del c.m. (G) localizando previamente el C.I.R. (I) correspondiente a la posición final de la varilla, tal como se indica en la figura, de modo que coincide con el extremo inferior (E) de la varilla, de modo que �G / 2v l�� . Sustituimos este valor, así como el valor del momento de inercia, en la expresión de la conservación de la energía, para obtener

2 2 2 21 1 1 1 [3]2 2 4 2 12lmg m l ml� �

� � � �� �� �� �� �� �� �� �� �

De donde resulta:

3gl

��

Obsérvese que el resultado es el mismo que en los dos casos. Se debe a que, en el segundo caso, en el instante final el movimiento consiste en una rotación pura en el CIR=E, de modo que hubiéramos podido escribir directamente la misma ecuación [1] en este segundo apartado.

m,l G

E

E G

� vG

m,l G

I

I

G

G

E

�I

m,l

- 197 -

Física Universitaria: Problemas de Física Sólido Rígido: Trabajo y energía. M22.4

4. Un cubo homogéneo está apoyado sobre una de sus aristas en contacto con un plano

horizontal, de modo que inicialmente se encuentra en equilibrio inestable. Lo desplazamos ligeramente de esa posición para que comience a caer. Calcular su velocidad angular cuando una de sus caras choca con el plano horizontal: a) Suponiendo que la arista no resbale sobre el plano. b) Suponiendo que el plano sea perfectamente liso.

Determinamos la distancia 2EG2

l�

a) El cubo presenta una rotación pura alrededor de su arista en E. Aplicamos el principio de Conservación de la Energía.

2E

2 12 2 2

lmgl mg I �� �

Con 2

2 2 2 2 2E G EG

1 2 1 1 2212 2 6 2 3

lI I m ml m ml ml ml� �� � ���� �� � � � � � ���� ���� ��� ��

De modo que

�2 2 23 2 12 1 2 0.62

2 2 2 3 2

gl gmgl mg mll l

� ��

� � � �

b) Como no actúan fuerzas que den componente en la dirección horizontal, el centro de masas del cubo no puede desplazarse en esa

dirección, por lo que tan solo desciende verticalmente. El movimiento del cubo consiste en una traslación (vertical) del c.m. y una rotación alrededor de un eje horizontal que pasa por el

c.m. (G). Aplicamos de nuevo el principio de conservación de la energía:

2 2G G

2 1 12 2 2 2

lmgl mg mv I �� � �

Puesto que el punto E tan solo se desplaza horizontalmente, podemos determinar la velocidad del c.m. (G) localizando previamente el C.I.R. correspondiente a la posición final del cubo, tal como se indica en la figura, de modo que G / 2v l�� , que sustituimos en la expresión de la conservación de la energía para obtener

22 2 22 1 1 1

2 2 2 4 2 6l lmgl mg m ml� �

� � � ��� ���� � � �� �� �� ��� � �

De modo que

�212 2 1

0.995

g gl l

��

� �

m l

m

l

G

E

E G

l

m

l

G

E

E G

l

CIR

- 198 -

Física Universitaria: Problemas de Física Sólido Rígido: Trabajo y energía. M22.5

5. Dadas dos esferas, una maciza y la otra hueca, describir detalladamente un experimento que, sin dañar las

esferas, nos permita averiguar cual es la maciza y cual la hueca. Hacer los cálculos necesarios para justificar los resultados del experimento.

Si dejamos rodar las dos esferas por un plano inclinado, abandonándolas simultáneamente partiendo del reposo, la que llegue antes al pie del plano será la esfera maciza, ya que presenta menos “inercia a la rotación” que la esfera hueca. Método de la energía. El Principio de Conservación de la Energía nos permite determinar las velocidades que alcanzan las esferas cuando llegan al pie del plano inclinado, en el supuesto de que haya rozamiento suficiente para que se produzca la rodadura (sin resbalar):

22 2 2 2 2

2 22 2

1 1 1 1 1 2 22 2 2 2 2 1

v I mgh ghmgh mv I mv I m v v I IR R m R mR�

� ���� � � � � � � ��� ��� � �

donde hemos tenido en cuenta la condición de rodadura, v = �R. Puesto que la expresión del momento de inercia es de la forma kmR2, resulta que la velocidad es

2 21

ghvk

��

de modo que es independiente de la masa y del radio, pero depende del momento de inercia, siendo tanto mayor cuanto menor sea en momento de inercia. Por tanto, la esfera que llega antes al pie del plano es la esfera maciza.

22esf. maciza

esf. maciza

2esf. maciza5

2 2esf. hueca esf. hueca 2

3

esf. hueca

2 1021 7

1.43

1.20

52 2 63 1 5

ghv ghI mR

g

ghv

hI mv

ghR v gh

���� � � � �� � �� � �� �� " "� �� �� �� � � �� ���

.

� �� ��

Método dinámico. Podemos demostrar que la aceleración que adquiere cada una de las esferas, en el supuesto de que haya rozamiento suficiente para que se produzca la rodadura (sin resbalar), tan solo depende del momento de inercia de las misma, i.e., de la distribución de la masa en las mismas.

2

22(condición de rodadura)

sensen

sen

1

mg f maImg m afR I If a R

a

g

RI

Ra

mR

��

���

� �� �� �� � � �� �� � ��� � � � � �� � �� ��� � � ��� ���� ��� ��

��

Resulta que la aceleración es independiente de la masa y del radio, dependiendo tan solo del ángulo de pendiente y del momento de inercia. Así, conocidos los momentos de inercia implicados, tenemos las aceleraciones respectivas:

esf. maciza 25

esf. hueca 23

0.sen 5 sen1

71 sen

0.60

7se senn 3 sen

1 5

ga

gga

gg

g

��

��

� � ��

� � ��

de modo que la esfera maciza se acelera más que la hueca.

N f

mg � h

- 199 -

Física Universitaria: Problemas de Física Sólido Rígido: Trabajo y energía. M22.6

6. Un rodillo macizo, de sección circular, de radio r y masa m, descansa

sobre un borde horizontal de un escalón y empieza a rodar hacia fuera, sin resbalar, con velocidad inicial despreciable. Calcular el ángulo que girará el rodillo antes de que pierda contacto con el borde del escalón, así como su velocidad angular en ese instante.

En tanto que no pierda contacto con el borde del escalón, el movimiento del rodillo es una rotación pura alrededor de un eje que coincide con dicho borde.

Conservación de la energía: (nivel de referencia a la altura del escalón)

�2 2312 2

23

3

4

2 4

cos

(1 co

(1 co

)

s )

s

mgr mgr mr

mg mr

r g

� �

� �

� �� �

� � "

� � "

Movimiento radial del centro de masas: 2cos nN mg ma mr� �� � � �

Combinamos las dos ecuaciones anteriores para obtener: � �74 4 4 4

3 3 3 3 3cos (1 cos ) cos cos cosN mg mg mg mg� � � � �� � � � � � � �

de modo que 13 (7cos 4)N mg �� �

El rodillo pierde contacto con el borde del escalón cuando N=0, ya que entonces desaparece la ligadura, lo que equivale a

7cos 4 0 cos 4 7 55.2º� � �� � " � " � La velocidad angular del rodillo en ese instante será:

2 4 4 4 4(1 cos ) (1 )3 7 73

47

gg gr r r

gr

�� � �� � � � � "

m,r �

m,r

mg

N f

- 200 -

Física Universitaria: Problemas de Física Sólido Rígido: Trabajo y energía. M22.7

7. En el interior de un semicilindro hueco de radio R, una esfera de radio r y masa m,

parte del reposo desde la posición A. Calcular la velocidad y aceleración de la esfera cuando pasa por el punto más bajo, suponiendo que en todo instante rueda sin deslizar.

a) Para determinar la velocidad del c.m. de la esfera, aplicamos el Principio de conservación de la energía,

� 2 2cm

1 102 2

mg R r mv I �� � � � �

Con la condición de rodadura v r�� obtenemos

�2

2 2 21 1 2 72 2 5 10

vmg R r mv mr mvr

� �� �� �� �� � � �� �� �� �� �� �

de modo que

�107

v R r g� �

b) Cuando la esfera pasa por la posición más baja, que corresponde a la posición de equilibrio (energía potencial mínima), su velocidad es máxima y la aceleración tangencial de su c.m. es nula. El c.m. de la esfera tan solo presenta aceleración normal o centrípeta, dirigida hacia el centro de la trayectoria que describe, de radio (R – r) y cuyo módulo es

2

n107

va gR r

� ��

Y la reacción normal (N) será: 10 177 7

N mg mg N mg� � �

A

m, rR

m, r

R

I

v�

Ep=0

R

I mg

N

an

- 201 -

Física Universitaria: Problemas de Física Sólido Rígido: Trabajo y energía. M22.8

8. Una bola hueca, de espesor despreciable, masa m y radio r, se lanza rodando

por un plano horizontal con velocidad de su centro v0 y recorre un bucle situado en un plano vertical, siempre rodando, de radio R, del cual se despega cuando forma un ángulo � con la horizontal, tal como se indica en la figura. Hállese: a) El momento de inercia de la bola con respecto a un eje diametral. b) La velocidad v0 que debe de tener para que se despegue en �.

a) Puesto que toda la masa de la bola se encuentra situada a la misma distancia de su centro, el momento de inercia con respecto a este es 2

OI mr� . Entonces, aplicando el teorema que nos dice que la suma de los momentos con respecto a tres ejes perpendiculares que se interceptan en un punto es igual al doble del momento de inercia con respecto a dicho punto, tenemos

O D D2

O23

2 3 2xx yy zzI I I I I II rI m�� � � � �

b) Puesto que la bola rueda sin deslizar en todo momento, no hay disipación de energía por rozamiento, de modo que la energía total de la bola permanece constante. Podemos escribir

�2 2 2 20 0

1 1 1 11 sen2 2 2 2

mv I mgR mv I� � �� � � � �

con 0 0v r�� y v r�� , de modo que nos queda

� �

2 2 2 2 2 20 0

20

2 20

2

1 1 2 1 1 21 sen2 2 3 2 2 35 51 sen6

6 1 se6

n5

mv mr mgR mv mr

mv mgR mv v v gR

� �

� �� � � � �

� � � � � �

Escribimos la componente radial de la ec. del movimiento del centro de masa de la bola:

�220

cp6cos 1 sen5

vvN mg ma m m mgR R

� �� � � � � �

En el momento crítico en el que la bola se despega de la pista, será N = 0, de modo que

� �0 20

2 6cos 1 s 6cos 1 e5

en5

s nv gvg RgR

� � � ��� ���� � � �� ���

�R

m,r v0

� N

mg

- 202 -

Física Universitaria: Problemas de Física Sólido Rígido: Trabajo y energía. M22.9

9. Los ejes de dos cilindros de masas m1 y m2, y radios r1 y r2

respectivamente, están unidos por una cinta inextensible que pasa a través de una polea de masa despreciable. Los cilindros ruedan sin deslizar sobre sendos planos, inclinados un ángulo �. a) Calcular las aceleraciones de ambos cilindros. b) Si el sistema parte del reposo, determinar la energía cinética de ambos cilindros tras recorrer 1 m sobre los planos.

Datos: m1 = 1 kg, m2 = 1.5 kg, r1 = 15 cm y r2 = 20 cm, � = 30º.

a) Escribimos las ecuaciones del movimiento de rotación para cada uno de los cilindros, tomando momentos con respecto a las generatrices de contacto con los planos (E.I.R.) para cada uno de los cilindros:

21 1 1 1 1 1 1 1

22 2 2 2 2 2 2 2

3 3( sen ) sen2 23 3( sen ) sen2 2

Tr m g r m r T m g m a

m g r Tr m r m g T m a

� � �

� � �

� �� �� �� � � �� �� �� � � �� �� �� � � �� �� �� �� �

donde hemos tenido en cuenta que, por ser la cinta inextensible, las velocidades y aceleraciones de ambos cilindros son idénticas en cada instante. También hemos considerado la condición de rodadura para cada uno de los cilindros. Sumando m.a.m. estas dos últimas expresiones, se tiene

22 1

1

12

21

3( ) sen ( ) 2( ) sen3( )2

m m g m ma gm m

m m a ��� � � �

��

y sustituyendo los datos 2(1.5 1.0) 9.8sen 30º3(1.0 1.5

0.653 m/)

sa �� �

y las aceleraciones angulares de los cilindros son 2 2

1 21 2

0.653 0.6534.35 rad/s 3.27 rad/s0.15 0.20

a ar r

� �� � � � � �

b) Puesto que la aceleración es constante, calculamos las velocidades de cada cilindro a partir de la expresión 2 2

0v v� 2as� :

1 2 2 2 0.653 1 1.14 m/sv v v as� � � � � � �

La energía cinética de cada cilindro viene dada por la expresión:

2 2 2 2 2 2k

1 1 1 1 1 32 2 2 2 4 4

E mv mr mv mv mv�� ���� � � � ��� ���

y la energía cinética que adquiere cada uno de los cilindros es: 2 2

k,1 k,20.3 31.0 1.14 1.5 1.144

98 J 1.47 J4

E E� � � � � �

m1, r1 m2, r2

� �

m2g

� �

N1

f1

T

m1g

N2

f2

T

+ +

- 203 -

Física Universitaria: Problemas de Física Sólido Rígido: Trabajo y energía. M22.10

10. La rueda representada en la figura consiste en un semicírculo de madera que pesa

100 N contenido en un aro circular de acero de 45cm de diámetro y peso y grosor despreciables. Si rueda sin deslizamiento por un piso horizontal y tiene una velocidad angular de 15 rad/s en sentido horario cuando su centro de masa G se halla directamente debajo del centro C de la rueda. a) Determinar la velocidad angular de la rueda cuando G se halle directamente a la izquierda de C. b) Calcular las componentes normal y de rozamiento de la fuerza que el suelo ejerce sobre la rueda cuando G se halla directamente a la izquierda de C.

Determinación del c.m. de un semicírculo: 2º teorema de Pappus: V=2�ycmS

343

cm 212

1 1 42 2 3

RV RyS R

�� � � �

� � �

4 4 0.225 0.0955 m3 3

R�

� ��

� � ��

Cálculo del momento de inercia: IC=½mR2=0.258 kg.m2

IG=IC-m72= 0.165 kg.m2

IO=IG-m(R-7) 2=0.336 kg.m2

IP=IG+m(R2+572)=0.774 kg.m2 a) Conservación de la energía:

22 2 2 O O1 1

O O P P P2 2P

2I mgmg I II

� �� � � �

�� � � �

22 2p

P

0.336 15 2 100 0.0955 73.0035 (rad/s)0.774

8.54 rad/s

� � � �� �

b) Ecuaciones del movimiento:

G

x

y

f maN mg mafR N I� �

��� ���� � ����� � ���

con 2

x

y

a Ra

� ����

�� � ���� ���

ya que

2

22G C

00 0 0 00 0 0 0

CG CGR R� � � � � �

� ���

�� � �

� � � ��

� �� � � � � � � � ��� � � �� � � � �� � � � �� � � � �� � � � �� � � � � � � � � �� � � � �� � � � �� � � � � �� � � �� � � �� � � � � �� � � � � a a �

���� ����

De modo que 2

G

( )f m RN mg mfR N I

� � ���

� �

��� � ���� � � "��� � ����2

6.08 N72.4 N

= - 33.6 rad/s

fN�

��

'

G

C

G

C 7

O

P

C

O

GC

P

G

C

P

G

N

f

mg

- 204 -

Física Universitaria: Problemas de Física Sólido Rígido: Trabajo y energía. M22.11

11. En el sistema representado en la figura, el rodillo rueda sin resbalar

sobre el suelo horizontal rugoso y los dos muelles tienen la misma constante elástica k. a) Determinar la frecuencia de las oscilaciones del sistema. b) Si separamos el rodillo una distancia A de su posición de equilibrio y lo abandonamos partiendo del reposo, calcular la velocidad máxima que adquiere.

a) Método de la energía Consideremos un desplazamiento genérico del sistema a partir de su posición de equilibrio (elongación, x):

2 2 2 2 2 2 2 2 2cm cm cm

1 1 1 1 1 1 322 2 2 2 2 4 4

E mv mR kx mv mv kx mx kx�� � � �� �� �� � � � � � � �� �� �� �� �� �

ya que cm cm(rodadura), con v R v x�� � � .

Como el sistema es conservativo, la energía total permanece constante, de modo que, derivando con respecto del tiempo, tenemos:

d 3 32 2 0d 2

02

43

E mxx kxx x mx kxt

kx xm

� ���� � � � � �� ��� � ������ � � ��

que es la ecuación diferencial de un m.a.s. cuya frecuencia angular es

frec4 1 43 2 3

k km m

� ��

� �

b) La velocidad máxima se presenta cuando el rodillo pasa por su posición de equilibrio y podemos calcularla a partir de la frecuencia angular y de la amplitud:

max frec43

kv A Am

�� �

Al mismo resultado llegaremos a partir de la conservación de la energía:

2 2 2 2max max

2 2 2 2max max ma

2 2maxx

1 1 1 122 2 2 2

1 1 34

44 32

kA mv mR

kA mv mv kv Am

mv

�� � � �� �� �� � � �� �� �� �� �

� �� �

k

m,R x k

- 205 -

Física Universitaria: Problemas de Física Sólido Rígido: Trabajo y energía. M22.12

12. En el dispositivo que se ilustra en la figura, constituido por un muelle ideal de

constante k, una polea de radio R y momento de inercia I y una pesa de masa m, determinar la frecuencia de las oscilaciones de la pesa y escribir la expresión de su elongación en función del tiempo.

Método de la energía: Puesto que el sistema es conservativo, la energía total del mismo permanece constante, de modo que, para una elongación x genérica, podemos escribir:

�22 20

1 1 1 cte.2 2 2

E mx I k x x mgx�� � � � � ���

En el supuesto de que la cuerda no resbale sobre la polea, podemos establecer la siguiente condición de ligadura:

x R�� �� que, sustituida en la expresión de la energía, nos permite expresar la energía en función de la elongación x y de sus derivada temporal:

�2 2 2 20 02

1 1 1 1 cte.2 2 2 2

IE mx x kx kx kx mg xR

� � � � � � �� �

Derivamos esta expresión con respecto al tiempo:

�0 02 2

02

d 0 0dE I Imxx xx kxx kx mg x mx x kx kx mgt R R

Im x kx mg kxR

� � � � � � � � � � �

� ��� � � � ��� ���

��� ��� � � �� ��

��

que es la ecuación diferencial correspondiente a un movimiento armónico simple con

0

2

k mgxI kmR

�� ��

La expresión de la elongación en función del tiempo es �0 senx x A t� �� � �

donde la amplitud de las oscilaciones (A) y la fase inicial (+) se determinan a partir de las condiciones iniciales. Método de las fuerzas: Para una elongación genérica, escribimos las ecuaciones del movimiento para cada uno de los tres elementos implicados (bloque, polea y muelle):

� �

11

1 2 21 2 0 2

2

2 0(condición rodadura)

/

mg T mxmg T mx

T R T R I IT T I R x mg kx m x kxT kx R

T k x xx R

� � �� �� � � �� �� �� � � �� �� � �� � � � � � ��� � � ��� � � �� �� � � �� ���� ���

����

���� ��

����

que es la misma ec. dif. de un m.a.s. obtenida por el método de la energía.

I, R

k m

x

I, R

k m x

mg

T2

T2 T1

T1

T2

- 206 -

Física Universitaria: Problemas de Física Sólido Rígido: Trabajo y energía. M22.13

13. El cilindro macizo y homogéneo que se muestra en la figura, de masa m y radio R, está

suspendido del techo mediante una cuerda ligera. Uno de los extremos de la cuerda está unido directamente al techo; el otro lo está a un muelle de constante elástica k. De-terminar la frecuencia de las oscilaciones del sistema si la cuerda no resbala sobre el cilindro.

Método de la energía: Puesto que el sistema es conservativo, la energía total del mismo permanece constante, de modo que, para una elongación x genérica, podemos escribir:

�22 2 21 1 1 1 2 cte.2 2 2 2

E mx mR k x mgx�� ���� � � � ��� ���

��

En el supuesto de que la cuerda no resbale sobre la polea, podemos establecer la siguiente condición de ligadura:

x R�� �� que, sustituida en la expresión de la energía, nos permite expresar la energía en función de la elongación x y de sus derivada temporal:

2 2 2 2 21 1 32 2 cte.2 4 4

E mx mx kx mgx mx kx mgx� � � � � � � �� � �

Derivamos esta expresión con respecto al tiempo: d 3 34 0 4 0d 2 2

3 8 242 3 3

E mxx kxx mgx x mx kx mgt

kmx kx mg x x gm

� ���� � � � � � ��� ���

� � � �

��� � � � ��

�� ��

que es la ecuación diferencial correspondiente a un movimiento armónico simple con

83

hm

�� y 02 / 3

8 / 3 4g mgx

k m k� �

Método de las fuerzas: Para una elongación genérica, escribimos las ecuaciones del movimiento para cada uno de los tres elementos implicados (polea y muelle):

1 2

2 21 2

21

(condición rodadura)

1 22 122 2 2

3 8 242 3 3

mg F F mx

mg kx F mxF R F R mR

kx F mxF k x kx

x Rkmg kx mx x x gm

��

� � � ����� � � � ���� � � �� �� � �� �� � �� �� �� ����� ���

� � � �

��

����

��

����

�� ��

que es la misma ec. dif. de un m.a.s. obtenida por el método de la energía.

k

m,R

x

F2

mg

F1

k

m,R

x

- 207 -

Física Universitaria: Problemas de Física Sólido Rígido: Trabajo y energía. M22.14

14. En el dispositivo que se muestra en la figura, el collarín ligero por el que pasa la

varilla y al que están unidos dos muelles idénticos, permite que éstos permanezcan horizontales. Los muelles trabajan a extensión-compresión y poseen su longitud natural cuando la varilla está en posición vertical. Determinar la frecuencia de las pequeñas oscilaciones de la varilla.

Método dinámico Cuando desplazamos la barra de su posición de equilibrio, aparecen las fuerzas recuperadoras en las direcciones que se indican en la figura. Tomamos momentos en A y obtenemos:

21A 2 sen con tg2 3lkxh mg ml x h� � �

� ���� � � ��� ��� � ��

Para oscilaciones pequeñas es sen tg� � �= = de modo que

2 21 122 3

kh mg l ml� � �� � % % � ��

2

2

6 3 02

kh gml l

� �� ��� �� � �� �� ���

��

y la frecuencia angular de las pequeñas oscilaciones es 2

2

6 32

kh gml l

� ��

Método de conservación de la energía Expresamos la energía total del sistema para una elongación arbitraria:

�2 2 21 13 2

2 2 2 2

1 cos 2 con tg2 21 1 cos tg cte. 6 2

lE ml mg kx x h

E ml mgl kh

� � �

� � �

� � � �

� � � �

y derivamos con respecto al tiempo: 2

22

d 1 1 2 tgsen 0d 3 2 cosE khml mglt

��� � � �

�� � � ���� � �

Para pequeñas oscilaciones es 2sen tg y cos 1� � � �= = = , de modo que

2 2 2 21 1 1 12 0 2 03 2 3 2

ml mgl kh ml mgl kh� � � � �� ���� � � � � ��� ���

�� ��

que es la ecuación diferencial de un m.a.s. con 2

22

22

1 2 3 621 23

mgl kh g khl mlml

��

� � �

k k

h m,l

x kx

A

mg

kx

- 208 -

Física Universitaria: Problemas de Física Sólido Rígido: Trabajo y energía. M22.15

15. Un péndulo compuesto está formado por una varilla de masa despreciable y longitud

4R, que está unida a una esfera de masa m y radio R. En el punto medio de la varilla se conecta un muelle ideal, con constante elástica k, en posición horizontal, como se muestra en la figura, que está relajado cuando la varilla está en posición vertical. Determinar la frecuencia de las pequeñas oscilaciones del péndulo en el plano de la figura.

Mediante el teorema de Steiner, calculamos el momento de inercia de la esfera con respecto al eje de rotación en O:

2 2 2O

2 127(5 )5 5

I mR m R mR� � �

Método de Newton. Consideramos una posición genérica, caracterizada por un desplazamiento angular � del péndulo, y planteamos la ecuación fundamental de la rotación

tomando momentos con respecto al eje de rotación:

O

2O

(5 sen ) (2 sen )(2 cos )

5 sen 4 sen cos

M mg R k R R

mgR kR I

� � �

� � � �

� � � �

� � � � ��

que, en la aproximación de pequeñas oscilaciones, con sen y cos 1� � �= � , se reduce a

2 2127 (5 4 ) 05

25 200

127

mR mgR kR

mg kRmR

� �

� �

� � �

�� �

��

��

que es la ecuación diferencial de un m.a.s. en el que la frecuencia angular vale

�25 20127mg kR

mR�

��

Método de la energía. Expresamos la energía total del péndulo (cinética + potencial) en función del ángulo �:

2 2 2 2 2 2O

1 1 127(5 cos ) (2 sen ) 5 cos 2 sen cte2 2 10

E I mg R k R mR mgR kR� � � � � �� � � � � � �� �

como la energía permanece constante, su derivada temporal será nula, lo que nos conduce a 2 2

2 2

d 127 5 sen 2 sen cosdt 5

127 5 sen 2 sen cos 05

E mR mgR kR

mR mgR kR

�� � � � � �

� � � � �

� � � �

� ���� � � ��� ���

��� � �

� ��

que, en la aproximación de pequeñas oscilaciones, con sen y cos 1� � �= � , se reduce a

�2 2 25 20127 (5 4 ) 0 05 127

mg kRmR mgR kR

mR� � � �

�� � � � ��� ��

que es la ecuación diferencial de un m.a.s. cuya frecuencia angular vale

�25 20127mg kR

mR�

��

2R

2R

k

M

2R

2R F

2R

mg

O

+

5Rcos�

Ep=0

2Rsen�

- 209 -

Física Universitaria: Problemas de Física Sólido Rígido: Trabajo y energía. M22.16

16. En un tubo cilíndrico horizontal hay dos discos, de masa m, unidos por un hilo

que los mantiene a una distancia l uno del otro, y a distancia l/2 del centro del tubo O. Este dispositivo está girando libremente alrededor de un eje perpen-dicular al tubo que pasa por O, con una velocidad angular '. En un cierto instante se rompe el hilo que une ambos discos (dejándolos libres). El rozamiento entre los discos y el tubo se considera despreciable, el momento de inercia del tubo respecto al eje de giro es It = ma2 (donde a es un parámetro con dimensiones de longitud) y el momento de inercia de cualquiera de los discos respecto a un eje paralelo al de giro que pasa por el centro de masa del disco es Id = (ma2)/2. a) Hállese la velocidad angular de giro del dispositivo descrito cuando ambas masas llegan a sus sendos extremos del tubo. b) ¿Con qué velocidad se aleja del eje en el momento que abandonan el tubo?

22 2 2 2 2 2

t d t d1 1 1 12 22 2 2 4 2

lI ma I ma m ma ml I I I ma ml� ���� � � � � � � � ��� ���

a) Por no existir momento externo, el momento angular del sistema se conserva:

i i f f

2 22 2

if i 2 2

2 2f

cte.12 421 422

L I I

ma mlI a lI a Lma mL

� �

� � � �

� �

� �# � � �

��

b) Por no existir fuerzas disipativas (rozamiento) se conserva la energía mecánica; concretamente, se conserva la energía cinética del sistema:

2 2 2 2 2 21 1 1i i f f r r i i f f2 2 2

22 2 2 2 2 2i i ir i i f i i i r i i2

f f f

2 2

12 1 12

I I mv mv I I

I I Imv I I I v II I m I

� � � �

� � � �

� � � �

� � � �� �� �� �� � � � � �� �� �� �� �� �� �

de modo que sustituyendo los valores de Ii e If y operando se obtiene

2 2 2 22 2 2 2 2ir i i i2 2

f

2 22 2 2

i2 2

1 1 4 4 11 22 2 4 2

1 44 4

I a L a lv I ma mlm I m a L

L l a la L

� �

� � � �� �� � �� �� � ��� �� � � � �� �� �� � �� � ��� �� � �� � � �� �� �� �� �� �� ��

De modo que

�2 2

2 2 2 2r 2 2

1 44 4

a lv L la L

��

� ��

l

L

O

'

- 210 -

Física Universitaria: Problemas de Física Sólido Rígido: Trabajo y energía. M22.17

17. Determinar la aceleración angular que adquieren los tres cilindros idénticos de

la figura cuando sobre uno de ellos se ejerce un par M. Los mencionados cilindros pueden girar alrededor de sus respectivos ejes, sin que exista deslizamiento en los contactos entre diferentes cilindros.

En la figura hemos representado las fuerzas de rozamiento entre lo cilindros. Por se idénticos y no existir deslizamiento entre ellos, las aceleraciones angulares de los tres cilindros tendrán

el mismo módulo y las direcciones indicadas en la figura. Aplicamos la ecución fundamental de la dinámica de la rotación alrededor de un eje fijo a cada uno de los cilindros:

12

12 23

23

Cilindro 1: Cilindro 2: Cilindro 3:

M r f Ir f r f Ir f I

��

� �

� �

���

Sumando las tres ecuaciones resulta

1 222

33 3

23

M MM II m

Mmr r

� �� � � �

siendo m la masa y r el radio de cada cilindro.

M

)

f12

M

f21 f23

f32

) )

� � �

- 211 -

Física Universitaria: Problemas de Física Sólido Rígido: Trabajo y energía. M22.18

1

2

O G

O

G '2

E

E

x

y

N mg

7

18. Un cilindro de masa total m que tiene una mitad de densidad doble que la otra se abandona con velocidad nula sobre un plano sin rozamiento en la posición indicada en la figura. Determinar la velocidad y aceleración angulares en el instante inicial y en el instante en que el diámetro AA’ sea horizontal.

Cálculo del centro de masa Semicilindro macizo: Aplicamos el 2º Teorema de Pappus.

�2

3esf 1 semicirculo 1 1

4 42 23 2 3

R RV S R ��� � �� �

�� " � " �

Cilindro completo:

1 1 2 2 2 1 2 1 1

1 2 2 2

2 42 3 9

m m m m Rm m m m� � � � �

� ��

� �� � � " �

� �

Cálculo de momentos de inercia Momentos de inercia con respecto a los ejes longitudinales en los puntos O y G (centro de masa)

2O

12

I mR� 2 2G

12

I mR m�� �

Estudio dinámico Puesto que no existe rozamiento, se produce resbalamiento en los puntos de contacto del cilindro con el plano. Siendo a la aceleración del centro de masa y aplicando la 2ª Ley de Newton

0 0x x

y

ma am

N mg ma

�� � " ��� �� � ���+ F a

Por ser siempre nula la componente horizontal de la aceleración, el centro de masa no adquiere velocidad en la dirección horizontal; i.e., tan solo “cae verticalmente”. Posición 1: Obviamente es '1 =0 (condición inicial). Tomamos momentos en el punto O (ya que su aceleración es horizontal y, en consecuencia, está dirigida hacia el c.m.)

O 1 1O

89

mg gIR

mI

g ��

�� � � �� "

Posición 2: Principio de conservación de la energía: 2 2G2 G 2

1 102 2

mg mv I� �� � � �

pero vG2=0 (su movimiento es vertical y ha alcanzado el punto más bajo de su trayectoria). Por consiguiente

2 222

GG

212

mgI

mg I ���

�� " �

Tomando momentos en el centro de masa (punto G): G 2 2 00 I � �� " �

A’

A

- 212 -

Física Universitaria: Problemas de Física Dinámica impulsiva del sólido rígido. M24.1

1. Un sólido rígido, de masa m, está suspendido de un eje horizontal situado a una distancia h de su centro de masa, como se indica en la figura. Sabemos que cuando aplicamos una percusión , horizontal y perpendicular al eje, cuya línea de acción pasa a una distancia 2h de eje de suspensión, dicho eje absorbe la mitad de dicha percusión. Determinar el momento de inercia del sólido con respecto al eje de suspensión.

Escribimos la Ecuaciones Cardinales de la Dinámica Impulsiva (movimiento plano) del Sólido Rígido, tomando momentos en el eje de suspensión:

1G2G

2 22mvmv

h Ih I�� � �

�� �� �

� �) �� �� �� �) � � �� � �� �� ��

Dividiendo miembro a miembro estas dos ecuaciones, tenemos

G

G

44 hmvIh Imv�

�� �

La velocidad del punto G, en la rotación pura � alrededor del eje de suspensión, viene dada por

Gv h��

De modo que sustituyendo esta expresión en la del momento de inercia, nos queda

�224 4 2hm hI mh m h��

� � �

Si en el enunciado se omitiera la palabra “absorbe” en “dicho eje absorbe la mitad de dicha percusión”, el planteamiento y el resultado sería distinto. Escribimos la Ecuaciones Cardinales de la Dinámica Impulsiva (movimiento plano) del Sólido Rígido, tomando momentos en el eje de suspensión:

3GG 2

2 22mvmv

h Ih I�� � �

�� �� �

� �) �� �� �� �) � � �� � �� �� ��

Dividiendo miembro a miembro estas dos ecuaciones, tenemos

G

G

443 3

hmvIh Imv�

�� �

La velocidad del punto G, en la rotación pura � alrededor del eje de suspensión, viene dada por

Gv h��

De modo que sustituyendo esta expresión en la del momento de inercia, nos queda

�22 24 4 1 423 3 3 3

hm hI mh m h mh��

� � � �

O

G

Ph

h

O

G

Ph

h

O

G

Ph

h

- 213 -

Física Universitaria: Problemas de Física Dinámica impulsiva del sólido rígido. M24.2

2. Una varilla uniforme de longitud l y masa m cuelga verticalmente y está sujeta por una articulación en su extremo superior. Golpeamos la varilla en su extremo inferior aplicando una fuerza horizontal F que dura un tiempo muy pequeño �t. a) Determinar momento angular que adquiere la varilla respecto al extremo superior. b) ¿Qué percusión habría que dar en la forma indicada para que la varilla llegará a alcanzar una posición vertical hacia arriba? c) Siendo l = 1 m; m = 2.5 kg; F = 100 N y �t = 1/50 s, averiguar si la varilla alcanzará la posición vertical hacia arriba.

a) La percusión que recibe la varilla viene dada por �= F t�

De las leyes de la dinámica impulsiva se sigue: � �L l l tl L L F� �� � ��

b) Expresamos la energía cinética que adquiere la varilla inmediatamente después de la percusión en función de la magnitud de ésta:

2 2 2 22

k 2

1 3 32 2 2 2

L lE II ml m

� ��� � � �

ya que entre el momento angular y la velocidad angular existe la relación:

21con3

LL I I mlI

� �� � �

La energía (cinética + potencial gravitatoria) se conserva en el movimiento de la varilla posterior a la percusión:

2 2 2

3

3 32 2 2 2l lmg mg mgl

m mm gl� �

�� � � � ��

c) Con los datos del problema, serán: 21 kg m100 2 N s 1 2 2

50 sL�

�� � � � � � �

El valor mínimo de la percusión, calculado en el apartado b) sería

min2 9.8 12.5 6.39 N s

3�

� �� � �

de modo que no alcanza la posición vertical

m,l

O

�=F�t

N.R.

- 214 -

Física Universitaria: Problemas de Física Dinámica impulsiva del sólido rígido. M24.3

3. Sobre una varilla homogénea, de masa M y longitud l, que puede girar alrededor de un eje fijo horizontal A, impacta elásticamente una partícula de masa m con velocidad v0 perpendicular a la varilla. a) Calcular la velocidad de la partícula y la velocidad angular de la varilla justamente después de la colisión. b) Determinar la percusión en el eje A.

a) La cantidad de movimiento del sistema no se conserva, por tener el sistema un eje impuesto, en el que, en principio, aparecerán percusiones reaccionales.. Conservación del momento angular con respecto al eje de suspensión que pasa por A:

20 0

2 2 1 2 ( )3 3 3l lmv mv Ml m v v Ml� �

� ���� � � ��� ��� [1]

Conservación de la energía cinética:

2 2 2 2 2 2 2 20 0

1 1 1 1 3 ( )2 2 2 3

mv mv Ml m v v Ml� �� ���� � � ��� ���

[2]

Dividimos m.a.m. las dos ecuaciones anteriores:

2 2 2 20 0

0

3 ( ) 3( )2 ( ) 2m v v v vMl lm v v Ml

��

�� �

� ��

[3]

que sustituida en la ecuación [1] nos conduce a

0 003( ) 4 3

4 32 ( )

2m Mvv vm

m v MM

vv ��

��

��

y sustituyendo este resultado de nuevo en la ecuación [1] obtenemos:

004 32 (1 )

4 312

4 3m MM vm

ml v

m M lm

M� �

�� �

��

b) La resultante de las percusiones que actúan sobre el sistema es igual al cambio que experimenta la cantidad de movimiento el sistema completo; esto es,

eje eje cm 0� mv Mv mv�� � � � �+ p

con cm 06

2 4 3l mv v

m M�� �

�, de modo que

2

eje cm 0 04 3 6 0

4 3 4 3m mM mMmv Mv mv m v

m M m M�

� �� �� �� � � � � � �� �� �� � ��

como era de esperar ya que el impacto tiene lugar en el centro de percusión, por lo que la varilla gira libremente alrededor del eje A.

Otro método Podemos llegar directamente a la expresión [3] a partir de la regla de Huygens-Newton con un coeficiente de restitución e = 1 (elástico):

BB 0 0 0

0

21 3( ) 20 3

v ve v v v l v v v lv

� ��

� � � � � � � � ��

[3]

cpmv0

l/3

l

A

cm

- 215 -

Física Universitaria: Problemas de Física Dinámica impulsiva del sólido rígido. M24.4

4. Una viga uniforme, de longitud 2l y masa m, está sostenida hori-zontalmente por dos apoyos, A y B, a una distancia x del centro G de la viga. a) Determinar la distancia x para que, al suprimirse súbitamente uno de los apoyos, no varíe en ese instante la reac-ción en el otro. b) Justamente después de suprimir el apoyo B, calcular la aceleración del punto G de la viga.

a) Los puntos A y B deben ser conjugados; de modo que sus distancias al punto G tienen cumplir la condición de Huygens para el centro de percusión, cm'mhh I� , de modo que será:

�22 21 1 32 0.5812 3 33

lmxx mx m l ml x l l� � � � � �

b) Antes de retirar el apoyo B, serán 1

A B 2F F mg� �

Justamente después de retirar el apoyo B, en las condiciones del apartado anterior, la viga estará sometida a las fuerzas

1A 2F mg P mg� �

Aplicamos la primera ecuación cardinal de la dinámica del sólido rígido para calcular la aceleración del centro de masas de la viga:

1A cm cm2

1cm 2

P F ma mg mg ma

a g

� � � �

Otro método Antes de retirar el apoyo B, serán

1A B 2F F mg� �

Justamente después de retirar el apoyo B, en las condiciones que impone el enunciado del problema, la viga estará sometida a las fuerzas

1A 2F mg P mg� �

Aplicamos las ecuaciones cardinales de la dinámica del sólido rígido, tomando momentos en G, y tenemos en cuenta que tendrá lugar una rotación instantánea alrededor de A:

11cm 2cm2

2 231 12 3 2

2 2 2 2 331 1cm 2 2 3 3

/

/

a gmg mg ma

mgx ml gx l

a x g gx l x l x l

� �

�� �� � � �� �� �� �� � �� �� �� �� ��� � � � �� ��

x x A B

G

x x A B

G

P

FBFA

xA

G

P

FA

- 216 -

Física Universitaria: Problemas de Física Dinámica impulsiva del sólido rígido. M24.5

5. Una varilla rectilínea y uniforme, de masa m y longitud l, que dispone de un gancho en uno de sus extremos, cae libremente en posición horizontal. En el instante en que su velocidad es v0, la varilla queda enganchada en un eje horizontal fijo. a) Determinar el estado de movimiento de la varilla inmediatamente después de quedar enganchada en el eje. b) Calcular la reacción percusional en el eje. c) Determinar el cambio que experimenta la energía cinética de la varilla.

a) Durante el proceso impulsivo, la varilla está sometida a una percusión, por lo que no se conservan ni su cantidad de movimiento ni su energía cinética. En cambio, se conserva el momento angular de la varilla con respeto al eje, por ser nulo el momento percusional con respecto al mismo; esto es,

2 00

12

323

lmv ml

l v� �

� ���� �� �� ��

y la velocidad del c.m. de la varilla será

cm 03

2 4lv v�� �

b) La percusión que recibe la varilla es igual al cambio que experimenta su cantidad de movimiento; esto es,

0 0cm 0 014

3( )4

m v v m v v mv�� ���� � � � �� ��� ��

(hacia arriba)

c) Cambio de la energía cinética:

20 0

2 02 2 2

20

200

12

�1 1 1 1 3 32 3 2 3 8

82

1E mv

E E EvE ml ml mv

m

l

v�

���� ��� � � ���� �� � � � ��� ��� �� � � ��� ��� � �� � �� � ��

� � ��

de modo que hay una pérdida de energía cinética igual al 0

� 1/ 8 0.25 25%1/ 2

EE

� � � � .

Otro método a) y b) Aplicamos las ec. cardinales de la dinámica impulsiva, tomando momentos en el c.m. de la varilla:

cm 02

cm 02cm 0

1 6 61 2 1202 12

m v vl l v l vl v vml

��

�� �

�� � � ���� $ � � � ��� �� � ����

y la condición de ligadura (rotación alrededor del eje): cm 2lv ��

Resolviendo este sistema de 2 ec. con dos incógnitas, tenemos

0cm 0 0

3 3 12 4 4

v v v mvl

� �� � � �

gancho

eje v0

m, l

v0

m, l

vcm

m, l

�+

+

- 217 -

Física Universitaria: Problemas de Física Dinámica impulsiva del sólido rígido. M24.6

6. Una varilla rectilínea y uniforme, de masa m y longitud l cae libremente en posición horizontal. En el instante en que su velocidad es v0, la varilla golpea elásticamente el borde de una cuchilla rígida y fija. a) Determinar el estado de movimiento de la varilla inmediatamente después del golpe. b) Calcular la percusión sobre la varilla. c) Determinar el cambio que experimenta la energía cinética de la varilla.

a) Durante el proceso impulsivo, la varilla está sometida a una percusión externa en su extremo izquierdo, por lo que no se conservan ni su cantidad de movimiento, ni su energía cinética. Aplicamos las ecuaciones cardinales de la dinámica impulsiva, tomando momentos en el c.m. de la varilla, y las dividimos m.a.m. para eliminar �:

cm 02

cm 02 m.a.m.cm 0

1 6 61 2 1202 12

m v vl l v l vl v vml

��

�� � $

�� � ����� � � � ��� �� �����

(1)

Aplicamos la Regla de Huygens-Newton con e =1 (colisión perfectamente elástica), siendo vA la velocidad del punto A de la varilla inmediatamente después de la colisión y obtenemos dicha velocidad a partir de la del centro de masa C de la varilla:

A 0

cm 0 cm 0A cm

2 22

2

v vlv v v l vlv v� �

� � ���� � � � � � ��� � ����

(2)

Resolvemos el sistema de ecuaciones (1) y (2):

cm 0cm 0

cm 0 0m.a.m.cm 0

0.56 68 4 32 2

v vv l vv v vv l v

l

� ��

� ��� �� �� �� � � �� �� � � �� �� ��

Método 2º. Conservación del momento angular en A:

20 cm 0 cm

1 6 62 2 12l lmv mv ml v v l� �

� ���� � � ��� ��� (1)

Método 3º. Aplicamos la Regla de Huygens-Newton en A. Además, sabemos que el conjugado del punto de percusión A se encuentra en A’, a una distancia 2l/3 del punto A y que su velocidad después de la percusión no se altera (v0), por lo que obtenemos la velocidad de A a partir de ella:

�A 0 0

A 0

32 / 3

v v vv v l l

��

� � ��� ��� � ���

A partir de la velocidad de A determinamos la del centro de masa

cuchilla

v0

m, l

v0

m, l

vcm

m, l �

++�

AC

A

v0

m, l

vcm

m, l �

++

A

C A

A’

v0

v0

- 218 -

Física Universitaria: Problemas de Física Dinámica impulsiva del sólido rígido. M24.7

0cm A 0 0

3 0.52 2

vl lv v v vl

�� � � � � �

b) La percusión que recibe la varilla es igual al cambio que experimenta su cantidad de movimiento. Calculamos la percusión a partir de la primera ec. de (1) en la que sustituimos los resultados anteriores:

� �0cm 0 0

10.5 12

m v v m v mv� � � � � � � � (hacia arriba)

c) Variación de la energía cinética:

2 20 0 0 0

222 2 2 2 20 0cm 0

0

1 1�2 4

�31 1 1 1 1 1 1 0.5 50%2 2 12 2 4 2 12 4

E mv E E E mv

Ev vE mv ml m ml mv El�

� �� �� �� � � � �� �� �� � � �� �� �� � � �� ��� ��� � � � � �� � � � �� ��� ��� � �� �� �� � �� � � �� �� de modo que la pérdida de energía cinética representa el 50% de la inicial.

- 219 -

Física Universitaria: Problemas de Física Dinámica impulsiva del sólido rígido. M24.8

7. Una barra uniforme de longitud l1 y masa M puede girar libremente en el plano vertical alrededor de una bisagra O. De la bisagra pende una partícula de masa m colgada de un hilo de longitud l2. La barra se abandona a partir del reposo en la posición indicada y al chocar con la partícula, ésta queda adherida a la barra. Determinar la velocidad angular que adquiere el conjunto inmediatamente después del choque.

1º.- Movimiento inicial de la barra. Aplicamos el Principio de conservación de la energía (nivel de referencia en O):

�2 2 21 1 112 3

1

302l gMg Ml

l� �� � � " �

2º.- Colisión inelástica manteniendo un eje fijo. Se produce una percusión en el eje, por lo que no se conserva la cantidad de movimiento del sistema. En cambio, el momento angular respecto del eje (O) permanece constante.

�2 21 1

1 1 23 3 2 2 2 2 21 1 11 1 2 1 23 3 3

2 (condicion de ligadura)

Ml Ml mv lMl Ml ml Ml ml

v l� �

� � � ��

� ) )� � �� ) ) )" � � � ��� ) )���

o sea 21

132 21

1 23

21

2 21 2 1

33

MlMl ml

Ml gMl ml l

� ��

) � ��

l2

l1

O M

m

- 220 -

Física Universitaria: Problemas de Física Dinámica impulsiva del sólido rígido. M24.9

8. Una placa rectangular, de lado l y masa m uniformemente distribuida, puede girar alrededor de un eje fijo horizontal que coincide con uno de sus bordes, como se indica en la figura. Separamos la placa hasta la posición horizontal y la abandonamos partiendo del reposo. Cuando alcanza la posi-ción vertical, la placa colisiona (coeficiente de restitución, 0 < e < 1) contra el borde de otra placa idéntica que se encontraba en reposo sobre un plano horizontal. a) Determinar las velocidades de cada placa justamente después de la colisión. b) Calcular la reacción percusional suministrada por el eje.

Primera fase: Durante la rotación de la primera placa se conserva la energía de ésta, de modo que

2 2 20 0 0

1 1 3 302 2 3l g gmg ml

l l� � �

� ���� � � � ��� ���

Segunda fase: Durante la colisión entre las dos placas, 3 No se conserva la cantidad de movimiento del sistema (por

existir una reacción externa en el eje), 3 Se conserva el momento angular con respecto al eje:

2 20 0

1 1 33 3

ml ml mvl l v l� � � �� � � �� �� �� � � �� �� �� �� �� �

3 No se conserva la energía cinética del sistema (por tratarse de una colisión parcialmente elástica). Sin embargo, puesto que se supone conocido el coeficiente de restitución, podemos aplicar la Regla de Huygens-Newton en el punto A de contacto durante la colisión (frontal); esto es,

�0 00l v e l l v el� � � �� � � � � � �

Disponemos de dos ecuaciones con dos incógnitas (�, v):

0 00

00 0

1 3 13 4 2

11 1

4 2

el v l

el v el ev l v l

� � � �� �

� �� �

� �� ��� �� � �� �� �� � �� � �� � � �� � �� � �� � � �� � �� � �� �� �� �� �� �

b) La percusión que suministra el eje es igual al cambio que experimenta la cantidad de movimiento del sistema.

Antes de la colisión: 0 cm 0 2lp mv m�� �

Después: 1 cm 0 0

1 2 0

2 0

1 3 1 332 4 2 8

1 84

l e l ep mv m m ml ep p p mlep mv ml

� � ��

� � ��� � � � �� ��� � � ��� �� � �����

De modo que

eje 0 0 0 03 1 1 1 3

8 2 8 8e e ep p ml ml ml m gl� � � �

� � �� � � � � � � �

m, l

m, l

m, l

+ +

N.R.

l/2

m, l

m, l

+ +

v

�lA

E �E

-�

- 221 -

Física Universitaria: Problemas de Física Dinámica impulsiva del sólido rígido. M24.10

9. Un tren de engranajes de radios r1 y r2 y masas m1 y m2 se acoplan del modo en que se indica en la figura, permaneciendo los ejes sobre los puntos fijos O1, y O2. Supongamos que los dientes sean lo suficientemente pequeños como para poder considerar cada engranaje como un cilindro homogéneo. a) Determinar la velocidad angular con que cada engranaje inicia el movimiento al incidir una percusión tangencialmente en el punto A. b) Calcular las reacciones percusionales en los ejes.

a) Durante la percusión � en el punto A, aparecen las percusiones, �1 , �2, �12 y �21 que actúan tal como se indica en la figura, con �12 = �21 (acción-reacción). Para cada cilindro, el momento de percusional con respecto del eje de rotación es igual al incremento del momento angular; i.e.,

211 12 1 1 1 1 1 12

2121 2 2 2 2 2 22

r I m r

r I m r

� � � �

� � �

� � �

� �

r

y como la condición de rodadura da lugar a que

1 1 2 2r r� ��

de modo que tenemos tres ecuaciones con tres incógnitas (�, �1 y �2). Resolviendo este sistema de ecuaciones tenemos:

1112 1 1 12 1 2 11

1 2 1 121 121 2 2 2 2 1 12 2

21 2 2

2( )

( )2

( )

m r m m rm m r

m r m rm m r

��

� � �� �

�� � � �

��� ��� �� � �� �� � � � �� �� �� �� ��� ���

b) Podemos calcular �12

2121 12 2 2 22

1 2

mm rm m�

� � �� � ��

Cálculo de las reacciones percusionales en los ejes: Para cada cilindro, la resultante de todas las percusiones es igual al incremento de su cantidad de; i.e.,

cm� 0m v � � �

puesto que el centro de masa de cada cilindro permanece en reposo. Así, escribimos las ecuaciones

1 12 1 12

12 2 2 12

00

� � � � � �� � � �

� � � � �

� � �

de modo que

1 2 21 2

1 2 1 2

2m m mm m m m

� � � ��

� �� �

en las direcciones que se indican en la figura.

A

�1

�2

�21

�12

O1

O2

�1�2

AO1

O2

�1 �2

- 222 -

Física Universitaria: Problemas de Física Elementos de elasticidad. M27.1

1. Una barra de sección rectangular 100 � 50 mm y 2 m de longitud, sometida a una tracción de 50 t

experimenta un alargamiento de 1 mm y una contracción lateral de 0.007 mm en la arista de 50 mm. Calcular: a) El módulo de Young de la barra. b) El valor del coeficiente de Poisson. c) La contracción que experimenta la arista de 100 mm de la sección recta. d) Dimensiones de la sección recta si se somete a la barra a una tracción de 40 t.

a) A partir de la definición del módulo de Young, se sigue:

0

0

//

l

l

lF S FEl l l S

�� � �

� � �

311

2

2000 50 10 9.81 0.0

N1.965 0

10 m10

0.

E �� �

� � ��

b) A partir de la definición del coeficiente de Poisson, tenemos:

0 0

� 0.� � / 0.007 / 50� / 1

28/ 2000

b l b bb l l l

� ��

� � � � � � �

c) La deformación transversal en la arista de 100 mm será: 4

04

� � 10.28 1.4 102000

0.014m� 1.4 10 1 0 m0

a

a

a la l

a a

� �

� � � � � � � �

� � � � � � �

d) Teniendo en cuenta la proporcionalidad directa entre fuerzas y deformaciones,

99.9888 mm

40 ( 0.014) 0.0112 mm50

100 0.011240 ( 0.007) 0.0056 mm50

50 0.00 49.995 44 m6 m

a F Fa aa F F

ab F Fb bb F F

b

8 8 8, 8� � , � , � � � � �,

� � �8 8 8, 8� � , � , � � � � �

,� � �

F

- 223 -

Física Universitaria: Problemas de Física Elementos de elasticidad. M27.2

2. Una barra de sección rectangular a � b = 100 mm � 50 mm y 2 m de longitud experimenta un alargamiento

de 1 mm y una contracción lateral ,b = � 0.007 mm cuando se la somete a una tracción de 50 t. a) Calcular el módulo de Young y el coeficiente de Poisson de la barra. b) Calcular la variación de la sección recta de la barra.

a) A partir de la definición del módulo de Young, se sigue:

� �

0

112

350 10 9.8 / 0.100 0.050/� / 1/ 2000

N1.96 10m

l

l

F SEl l

��

� � �� � � � �

A partir de la definición del coeficiente de Poisson, D

l

��

�� � , tenemos:

0

� / 0.007 / 50� / 1/ 2000

0.28b bl l

��

� � � � �

b) La deformación transversal en la arista de 100 mm será: 4

04

� � 10.28 1.4 102000

0.014m� 1.4 10 1 0 m0

a

a

a la l

a a

� �

� � � � � � � �

� � � � � � �

El cambio unitario que experimenta la sección recta de la varilla es: 4 4 4� � � 1.4 10 1.4 10 2.8 10S a b

S a b� � �� � � � � �� � � � �

Y el cambio que experimenta la sección transversal es 4 2� 2.8 10 100 50 1.4 mmS � �� � � � � �

F

ab

l

F

- 224 -

Física Universitaria: Problemas de Física Elementos de elasticidad. M27.3

3. Un cable de acero, de 1 mm2 de sección, se sujeta horizontalmente, sin

tensión, a dos soportes distantes 300 cm. Se suspende una carga P en el punto medio del cable produciéndose en el mismo un esfuerzo de 1000 kg/cm2. a) Calcular el alargamiento que experimenta el cable. b) De-terminar el peso de la carga. Datos: E = 2�106 kg/cm2

a) A partir de la definición del módulo de Young (ley de Hooke) tenemos: / �

� /F S l FE l

l l E S��

� � �

De modo que

26 2

300 cm� 1000 kg/cm 0.15 cm2 10 kg/cm

l � � ��

b) Determinaremos el ángulo � a partir de las longitudes inicial y final del cable:

�o/ 2 300cos = 0.9995 1.81

� / 2 � 300.15l l

l l l l� �� � � " �

� �

Tras la deformación, el sistema constituido por el alambre y la carga suspendida queda en equilibrio, por lo que será 9F = 0; esto es,

2 senP F �� Y sustituyendo los datos

2 º2

kg2 1000 0.01 cm sen (1.81 ) 0.63 kgcm

P� ���� � ��� ���

300 cm

P

l/2

P(l+�l)/2

�F F

- 225 -

Física Universitaria: Problemas de Física Elementos de elasticidad. M27.4

4. Un peso cuelga de un cable de acero de 2 m de longitud 1 mm2 de sección. Se desea sustituir dicho cable

por otro de cobre de 3 m de longitud. a) ¿Qué sección deberá tener el cable de cobre para que se alargue lo mismo que el de acero? b) ¿Ídem para que se rompan a la misma tensión?

Datos: 11 2 8 2

rup

11 2 8 2rup

acero: 2.0 10 N/m 4 10 N/m

cobre: 1.1 10 N/m 2 10 N/m

E

E

� � � �

� � � �

Definición del módulo de Young: / �� /F S FlE l

l l ES��

� � �

a) Imponemos la condición de que ambos cables experimenten el mismo alargamiento:

ac Cu ac Cuac Cu Cu ac

ac ac Cu Cu Cu ac

� � Fl Fl E ll l S SE S E S E l

� � �

y sustituyendo valores

Cu2.0 3 1 2.73m1.1 2

mS (� � � �

b) Imponemos la condición de que sean iguales las tensiones de ruptura en los dos cables:

ruprup rup rup rup ac

ac Cu ac ac Cu Cu Cu acrupCu

F F S S S S�� �

�� � �

y sustituyendo valores 2

Cu4 12

2 mmS � � �

F

l

- 226 -

Física Universitaria: Problemas de Física Elementos de elasticidad. M27.5

5. De dos hilos de longitud l, sección S y módulos de Young E y 2E se cuelga una masa m

inicialmente en reposo, como se indica en la figura. Determinar la frecuencia angular y la amplitud de las oscilaciones verticales del sistema.

En cada instante los dos cables tienen igual deformación x, de modo que �

1 2

1 2 eq

2

3

E SESF x F xl l

ESF F F x k xl

� �

� � � �

de modo que la constante elástica de los dos hilos es

eq3ESk

l�

El sistema equivale a una masa suspendida de un muelle de constante elástica keq, por lo que la frecuencia de sus oscilaciones es

eq 3km

ESml

�� �

Dado que la energía mecánica se conserva, igualamos la energía del sistema en la posición inicial (1) (cuando se cuelga la masa) y la de máxima deformación (2)

1 1p kE E�1elE�

2pE�2kE�

2elE�

�2eq

eq

12 22

3

mg A k A

mgA mglESk

� �

E 2E

m

posición inicial

N.R. 2A

1 2 posición de equilibrio

elongación máxima

E 2E

m

l

m A

x

F1 F2

posición de equilibrio

- 227 -

Física Universitaria: Problemas de Física Elementos de elasticidad. M27.6

6. Una barra homogénea de 1000 N de peso se cuelga en posición horizontal de tres

hilos de igual sección y longitud inicial como se indica en la figura. Determinar en que relación se alargan los hilos (unos respecto a otros) y que fuerza soporta cada uno de ellos en los siguientes casos: a) El hilo central es de acero y los de los extremos de cobre. b) El hilo de acero es el de uno de los extremos y los otros dos de cobre. Datos: El módulo de Young del acero es doble que el del cobre.

a) En virtud de la simetría que presenta el problema, los tres hilos experimenta el mismo alargamiento, por lo que de la definición del módulo de Young y de las ecuaciones de la estática se obtiene:

cobre acero acero acero aceroacero cobre

cobre acero cobre cobre cobre

2 2E F F FE E E F� � �

��

� � � � � �

cobre acero

acero cobre

cobre

acero

252 1000 0 N500 N2

F FF F

FF

� �� �� �� � ��

��� ��� �� �

b) Cuando colocamos el hilo de acero en un extremo, la barra se inclina, ya que, a igualdad de esfuerzos, el acero se alarga menos que el cobre. Las ecuaciones de la estática, tomando momentos en B, se escriben en la forma:

1 2 3 1 2

1 31 3

2F F F P F F PF FFb F b

� �� � � � �� �� � � �� � �� �� ��

ya que F1 y F3 presentan el mismo brazo con respecto a B. A partir de este resultado, teniendo en cuenta la definición del módulo de Young, se sigue:

1 acero1 1 13 1

3 3 3 cobre 3

2 1 2EFF E

�� �� �

� � �� � � � �

Así que el hilo 3 experimenta una deformación doble que el 1. La deformación del hilo 2 será 2 11.5� �� , de modo que las deformaciones (alargamientos) se encuentran en la relación 2 : 3 : 4 Al comparar las deformaciones de los hilos de cobre (2 y 3) se obtiene:

2 2 22 3 1

3 3 3

1.5 0.75 0.75 0.753

F F F FF

� �� �

� � � � � �

De modo que

13

12

2

2

1 363.6 N272.7 N36

2 10000. 5

3.7

6 N

F FF

FF

F

F

���� � �� �� � � �

�� ��� �� ���

a

a/2 a/2

A B C

P

FCu FCu

Fac

cobre

cobre

acero

acero

B CP

F1 F3F2

x1 x2 x3

cobre

cobre

- 228 -

Física Universitaria: Problemas de Física Elementos de elasticidad. M27.7

7. Una barra prismática está constituida por dos materiales diferentes A y B y se somete a compresión en sus

extremos, según se indica en la figura. Determinar la variación de volumen que se produce en la barra en función de los módulos de Young y coeficientes de Poisson de los materiales.

Se trata de una compresión longitudinal pura (sin esfuerzos laterales externos) en la que ambos segmentos de la barra (A y B) están sometido a un mismo esfuerzo:

(compresor) FS

�� �

En cada uno de los segmentos, la deformación unitaria longitudinal vale

(acortamiento) zzF

E SE�

� � � �

y la deformaciones unitarias transversales son

(ensanchamiento) xx yy zzF

ES�

� � ��� � � � �

de modo que el cambio de volumen será

� (disminuye)

� (1 2 ) 1 2

�� 1 2

xx yy zz zzV F

V ESV FlV Sl

V E

� � � � � �

� � � � � � � �

� ���� � � ��� ���

En definitiva

Segmento A: � � AAA

A

(disminuye)� 1 2 FlVE

�� � �

Segmento B: � � BBB

B

(disminuye)� 1 2 FlVE

�� � �

Total: � � �A B(disminuye)� � � V V V� �

S

F

lA

lB

F

x y

z

- 229 -

Física Universitaria: Problemas de Física Elementos de elasticidad. M27.8

8. Un pilar de hormigón armado y sección cuadrada debe soportar una carga, a compresión pura, de

2.5 � 106 N. Se sabe que el 10% del área de la sección cuadrada del pilar está ocupado por los “redondos” (varillas cilíndricas) de acero y el resto por hormigón en masa. Las características máximas de trabajo permitidas y los módulos de Young tienen los valores siguientes:

H acero

2 2max max

2 2H acero

25 N mm 400 N mm

30000 N mm 200000 N mmE E

� �� �

� �

a) Calcular la deformación unitaria máxima del pilar. b) Determinar el lado de la sección cuadrada del pilar. c) Siendo cuatro los “redondos” del acero de la armadura del hormigón, determínese el diámetro de cada uno de ellos.

a) Las deformaciones máximas experimentadas en las condiciones críticas de trabajo son

hormigón: H

H

max 3max

H

25 0.83 1030000E

�� ��

� � � � �

acero: acero

acero

max 3max

acero

400 2 10200000E

�� ��

� � � � �

Como H aceromax max� �* , y ambos materiales experimentan la misma

deformación, tendrán el límite del más crítico; i.e.,

Hx3

ma 0.83 10� � �� �� �

b) La carga soportada por cada uno de los materiales será

� 6H HH acero H acero

acero acero

0.9 0.9 0.1 2.5 100.1F S E F F E E SF S E

��

��� �� � � � � � ��� ���

62

3

2.5 10 63830 mm27000 20000 ( 0.83 10 )

S �

� �� �

� � �

63830 253 mma � �

c) La sección del acero será el 10% de la del pilar 2

acero 6383mmS �

y cada uno de los cuatro redondos tendrá una sección de 2

redondo 1595.75mm 22.3 mm 44.6 mmS r� " � " < �

a

F

- 230 -

Física Universitaria: Problemas de Física Elementos de elasticidad. M27.9

9. Se cuelga una bola de 20 kg de un hilo de acero de 1 mm de diámetro y 3 m de longitud que se supone sin

peso. El sistema se comporta como una masa unida a un resorte. Cuando se le hace vibrar verticalmente, manteniéndose el hilo siempre tenso, la frecuencia resulta ser 8.14 Hz. a) Determinar la constante elástica (k) del hilo. b) Deducir la relación existente entre el módulo de Young del hilo y la constante k. c) Determinar el módulo de Young de este acero.

a) La frecuencia angular del m.a.s. que realiza la masa unida al hilo viene dada por

2 2 220 ( N52 316.2 8 48m

.14)k k mm

� � �� " � � � �

b) A partir de la definición del módulo de Young, tenemos

//

F S ES ESE F l k l kl l l

kEl

lS

��

� � " � 4 " � # �� 44

c) Aplicando la expresión anterior a los datos que nos dan en el enunciado, se sigue

210

23 2

4 4 52 316.48 3(10 )

N19.98 10m

k l klES D� � �

� �� � � � �

F

l

- 231 -

Física Universitaria: Problemas de Física Elementos de elasticidad. M27.10

10. a) Calcular el aumento de longitud que experimenta un cable de acero ordinario de 100 m de longitud

cuando se le suspende verticalmente por uno de sus extremos, de modo que se estira elásticamente bajo la acción de su propio peso. Despreciar las variaciones originadas en la sección recta y en la densidad como consecuencia del alargamiento. b) ¿Se supera el límite elástico en alguna sección recta del cable? c) ¿Qué longitud máxima de cable puede suspenderse sin que ocurra su ruptura?

Datos para el acero: densidad, 7.87 g/cm3; módulo de Young, 20�1010 N/m2; límite elástico, 25�107 N/m2; esfuerzo de ruptura, 50�107 N/m2.

a) La tensión no es constante a lo largo del todo el cable, sino que va aumentando conforme nos alejamos del punto de suspensión. La tensión y el esfuerzo tensor en un punto situado a una distancia x del punto de suspensión corresponden al peso de la porción de cable situada por debajo de dicho punto; esto es,

� �( )( ) ( ) F xF x l x Sg x l x gS

� � �� � � � �

Consideremos un elemento infinitesimal de cable, de longitud dx, situado a una distancia x del punto de suspensión. El alargamiento

unitario experimentado por dicho elemento será ddx�

�� , donde d� es el

aumento de longitud que experimenta el elemento. Aplicando la definición del módulo de Young, E� �� , resulta

� �d( ) d dd

gx E l x g l x xx E� �

� � �� � � � �

El aumento de longitud que experimenta el cable completo será la suma de las contribuciones de todos los elementos de cable,

�2 2

2 2

00

� d d2 2 2

llg g x g l gl l x x lx l l

E E E E� � � �

�� � � ���� � �� � � � � � � �� ��� � ��� �

� �

y sustituyendo los valores numéricos 3

2 7 4 310

7.87 10 9.8� 100 1.93 10 10 1.93 10 m 1.93 mm2 20 10

l � �� �� � � � � � � �

� �

b) La tensión y el esfuerzo tensor en el cable son máximos en el punto de suspensión (x=0), por lo que

3 7 2máx (0) 7.87 10 9.8 100 0.771 10 N/mgl� � �� � � � � � � �

que es considerablemente menor que el límite elástico del acero, 7 2rup 25 10 N/m� � � .

c) Puesto que el esfuerzo tensor no puede superar el valor del esfuerzo de ruptura, la longitud del cable deberá ser inferior a

7rup

máx 3

50 10 6 483 m 6.5 km7.87 10 9.8

lg�

��

� � � �� �

x

dx dx+d�

l

F(x)

- 232 -

Física Universitaria: Problemas de Física Elementos de elasticidad. M27.11

11. Un paralelepípedo de cierto material elástico (coeficiente de Poisson 0 = 0.3 y

módulo de Young E = 2�107 N/cm2), que a cierta temperatura tiene dimensiones a = 10 cm, b = 30 cm y c = 40 cm, se introduce entre dos mordazas rígidas, planas, paralelas y horizontales separadas por una distancia c. Por efecto de un aumento de temperatura, en ausencia de tensiones mecánicas, el paralelepípedo experimenta una dilatación térmica unitaria de 2.5�10-4 en cada una de las tres dimensiones dadas. a) Calcular el valor de la tensión y de la fuerza ejercida por las mordazas que impiden la dilatación térmica del material en la dirección vertical. b) Determinar las dilataciones unitarias y absolutas que, en estas condiciones, tendrán realmente cada una de las aristas. c) Hallar la variación unitaria de volumen del paralelepípedo.

a) Como consecuencia de la dilatación térmica, aparecen esfuerzos mecánicos compresores en la dirección del eje z, que contrarrestan la dilatación térmica; esto es,

11 4 7 2térm 2 10 2.5 10 5 10 N/mzz E� � �� � � � � � �

y, por ser compresor, será negativo: 7 25 10 N/mzz� � � �

La fuerza compresora ejercida sobre las caras será: 7 60.10 0.30 5 10 =1.5 10 Nzz zzF S ab� �� � � � � � �

b) Escribimos las Ecuaciones Elásticas para valorar las deformaciones unitarias debidas a los efectos puramente mecánicos:

1xx xxE� �� yy��� �

1

zzzz

yy xx

E

E

��� �

� ��

� � �

� � yy�� �

74

11

5 100.3 0.75 102 10

1

xx yyzz

zz

zz xx

E

E

� ��

�� �

� ��

�� �� � � � � � �

�� � �

� � yy��� � � 4térm 2.5 10zz

zz E�

� � �

���������������� � � � � � � ����

Las deformaciones unitarias reales serán la superposición (suma) de las de origen elástico y de origen térmico. Esto es,

* * 4 4 4

* 4 4

0.75 10 2.5 10 3.25 10

2.5 10 2.5 10 0xx yy

zz

� �

� � �

� �

�� � � � � � � ���� � � � � � ���

y las deformaciones absolutas pedidas son * * 4 5

* * 4 5

* *

� 0.1 3.25 10 3.25 10 32.5 �m

� 0.3 3.25 10 9.75 10 97.5 �m

� 0

xx

yy

zz

a ab b

c c

� �

� �

�� � � � � � � ���� � � � � � � ����� � ���

c) La variación unitaria del volumen es: *

* * * 4� 6.5 10xx yy zzVV

� � � �� � � � �

b

a c

b a

c

zz z

x

y

zz

- 233 -

Física Universitaria: Problemas de Física Elementos de elasticidad. M27.12

12. Un cuerpo, cuya forma es la de un paralelepípedo rectangular, se somete a

compresión normal uniforme sobre dos de sus caras opuestas, en tanto que se impide la expansión transversal, de modo que su anchura permanezca constante, utilizando una armadura rígida como la que se muestra en la figura adjunta. a) Determinar las deformaciones unitarias en las aristas del paralelepípedo y los esfuerzos sobre las otras caras. b) Determinar los valo-res efectivos del módulo de Young (Eef) y del coeficiente de Poisson (μef) correspondientes a esta forma de compresión (definidos por

' y 'xx zz

xx xx

E � ��

� �� � � ).

a) Escribimos las ecuaciones elásticas: 1 ( )

1 ( )

1 ( )

xx xx yy zz

yy yy xx zz

zz zz xx yy

E

E

E

� � �� ��

� � �� ��

� � �� ��

��� � � ������� � � ������� � � �����

con 0

0

yy

zz

� ������ ����

1 ( )

10 ( )

1 ( )

xx xx yy

yy xx

zz xx yy

E

E

E

� � ��

� ��

� �� ��

��� � ������� � ������� � � �����

Resolvemos este sistema de 3 ec. con 3 incógnitas ( , ,xx zz yy� � � ):

22

2

1 1( )

1 (1 )( )

xx xx xx xx

yy xx

zz xx xx xx

E E

E E

�� � � � �

� ��

� �� �� � � �

�� ��� � � ������ ������ �� � � � � �����

b) Valores efectivos del módulo de Young y del coeficiente de Poisson:

ef 2 2

ef 2

1 1

(1 )1 1

xx xx

xxxx

zz

xx

EE

E

� ��� ��

� � � ��

� � �

��� � � ��� � ������� ��� � � � ��� � ���

F

y

x F armadura

bloque

z

F

y

x F armadura

bloque

z

- 234 -

Física Universitaria: Problemas de Física Elementos de elasticidad. M27.13

13. Sobre un bloque cúbico, de 25 cm de arista, se ejerce una fuerza de 625 t, mediante

una plancha rígida. Dicho bloque se encuentra perfectamente encajado entre dos paredes rígidas. Determínense: a) Las deformaciones sufridas por las aristas (en las tres direcciones principales). b) La presión que se ejerce sobre las paredes laterales. c) La variación de volumen que experimenta el bloque.

Datos: módulo de Young, E = 1011 N/m2 ; coeficiente de Poisson, 0 = 0.4.

a) y b) Escribimos las ecuaciones elásticas: 1 ( )

1 ( )

1 ( )

xx xx yy zz

yy yy xx zz

zz zz xx yy

E

E

E

� � �� ��

� � �� ��

� � �� ��

��� � � ������� � � ������� � � �����

con

72 2

0

0

625 000 9.8 N9.8 100.25 m

xx

yy

zzFS

���� ������ ������ �� � � � � � � �����

de modo que 1[1] ( ) ( )

1[2] 0 ( )

1[3] ( )

xx yy zz yy zz

yy zz

zz zz yy

E E

E

E

�� �� �� � �

� ��

� � ��

��� � � � � � ������� � ������� � �����

Resolvemos este sistema de 3 ec. con 3 incógnitas: 7 7

2 (compresor)[2] 0.4 ( 9.8 10 ) N3.92 10myy zz� �� � � � � � � � �

711

4 40

40.4[1] ( 3.92 9.8) 1010

� 5.49 10 0.25 1.37 10 m

5.49 10

0.137 mm

xx

xxx x

� �

� � � � � �

# � � � � � � �

22 7

11

4 40

41 1 0.4[3] (1 ) ( 9.8 10 8.23)10

� ( 8.23 10 ) 0.25 2.06 10 m

10

0.206 mm

zz zz

zz

Ez z

� � �

� �

��

� � � � � �

# � � � � � � �

�� �

c) Para el cambio de volumen tenemos: 44

30

4 3 6 3

� 2.(5.49 0 8.23) 10

� ( 2.74 10 ) 0.25 4.

74 10

4.22 m 8 m0 c8 1

xx yy zzV

VV

� � � ��

� �

� � � � � � � �

# � � � � � � � ��

� �

F

z

yx

- 235 -

Física Universitaria: Problemas de Física Elementos de elasticidad. M27.14

14. Un pilar “zunchado” (i.e., recubierto de otro material que impide dilataciones transversales) soporta una

carga de 1500 N/cm2. a) ¿Qué esfuerzos transversales soportará el zuncho? b) ¿Cuál será la deformación vertical del pilar, si éste tiene 3 m de altura?

Datos: módulo de Young, 2�106 N/cm2; coeficiente de Poisson, 0.25.

Datos:

6 102 2

62 2

N N2 10 2 10cm m

0.25N N1500 15 10

cm mzz

E

��� � � � ������ ������ � � � � �����

a) Escribimos las ecuaciones elásticas, con las condiciones :xx= :yy= 0; esto es, :

1 ( ) 0

1 ( ) 0

1 ( )

xx xx yy zz

yy yy xx zz

zz zz xx yy

E

E

E

� � �� ��

� � �� ��

� � �� ��

��� � � � ������� � � � ������� � � �����

Las dos primeras nos conducen a

1xx yy zz

xx yy zxx y

zy zz

� �� ��

�� � ���

� � ��

� � ��� ��� � ���� �

y sustituyendo valores 6 6

2

0.25 ( 15 10 )0.7

N5m5

10cxx yy� �� � � � � � �

b) La tercera ecuación elástica nos permite calcular la deformación unitaria longitudinal del pilar:

146

0

1 [ ( )]

1 1[ 2 ] ( 15 2 0.25 5) 6.3 11010

02

zz zz xx yy

zz xx

E

E

� � � � ��

� �� �

� � � �

� � � � � � � � � ���

y el acortamiento total será: 4 3 1.8� 9 mm6.3 10 3 1.89 10 m=zzz z� � �� � � � � � �� �

x

y

z

zz

- 236 -

Física Universitaria: Problemas de Física Elementos de elasticidad. M27.15

15. Un pilar cilíndrico, de 20 cm de diámetro y 3 m de altura, de hormigón (E = 2.8�1010 N/m2,

� = 0.4), está revestido exteriormente con un zuncho de acero que impide las dilataciones trans-versales. a) Determinar la contracción longitudinal que experimenta el pilar cuando soporta una carga vertical de 8000 kg. b) Calcular el esfuerzo transversal que soporta el zuncho.

Escribimos las ecuaciones elásticas, imponiendo las condiciones de imposibilidad de dilataciones transversales debidas al zuncho rígido:

1 ( ) 0

1 ( ) 0

1 ( )

xx xx yy zz

yy yy xx zz

zz zz xx yy

E

E

E

� � �� ��

� � �� ��

� � �� ��

��� � � � ������� � � � ������� � � �����

de modo que disponemos de tres ecuaciones con tres incógnitas ( , ,zz xx yy� � � )

Resolviendo las dos primeras y sustituyendo los resultados en la tercera, tenemos:

1xx yy zz

xx yy zzxx yy zz

� �� �� �� � �

�� � �� �

� � ��� � ���� � � ���

Y sustituyendo en la tercera,: 21 2( ) 1

1zz

zz zz xx yyE E;0: ; 0; 0;

0� �

� � � � �� � � Sustituyendo los datos en los resultados algebraicos anteriores, tenemos:

10 2 2 214

6 2

2.8 10 N/m ; 0.4; 0.20 0.0314 m ;8000 9.8 2.50 10 N/m

0.0314xx

E S� �

� � � � � �

�� � � � �

2 65

10

5

2 0.4 2.50 101 4.16 101 0.4 2.8 10

� 4.16 10 3 0.125 mm

zz

zzx h

� �� � ��� �� � � � �� �� �� � ��

� � � � � � �

�6 6 20.4 2 2 2.50 10 1.67 10 N/m1 0.4 3 3xx yy zz zz� � � �� � � � � � � � �

zuncho

pilar

zuncho

pilar

z

yx

- 237 -

Física Universitaria: Problemas de Física Elementos de elasticidad. M27.16

16. a) Expresar el módulo de compresibilidad de un material en función del módulo de Young y del coeficiente

de Poisson del mismo. b) Calcular el incremento de presión al que debemos someter un cierto volumen de agua (K = 22 � 108 N/m2) para que su densidad aumente en un 0.1%.

a) El módulo de compresibilidad se define como �

� /pK

V V� �

Para obtener �V/V aplicamos una compresión uniforme �p sobre las seis caras de un elemento de volumen de forma paralelepipédica. Las deformaciones unitarias de las aristas, con xx = yy = zz = –�p, vienen dadas por las ecuaciones elásticas:

1 1 (1 2 )( ) (1 2 )( � ) �xx xx yy zz p pE E E

�� � �� �� �

�� � � � � � � �

y expresiones análogas para �yy y �zz . El cambio unitario en el volumen será

� 3(1 2 ) �xx yy zzV p

V E�

� � ��

� � � � �

Que, sustituida en la expresión de definición de K, nos conduce a

3(1 2 )EK�

��

b) Como la masa (constante) de un cierto volumen de agua se expresa por m = �V, siendo � la densidad, la diferencial logarítmica de esta expresión nos conduce a

d d � �0V VV V

� �� �

� � � �

que sustituida en [1] nos lleva a 8 52� � � �� 22 10 0.1 22 10 Pa 22 at0 m1V p p K

V K� �� �

�� � � � � � � � � =�

- 238 -

Física Universitaria: Problemas de Física Elementos de elasticidad. M27.17

17. Un cubo metálico que tiene de longitud de 20 cm de arista se sumerge en el mar a una profundidad

de 400 m. Conociendo el módulo de Young del metal E = 2.1�1010 N/m2, el coeficiente de Poisson 0 = 0.3 y el valor de la densidad del agua del mar % = 1.06 g/cm3, calcular la variación de volumen que experimenta el cubo sumergido.

Escribimos las ecuaciones elásticas:

1 2

1 ( )

1 1 1 2( ) ( )

1 ( )

xx xx yy zz

yy yy xx zz xx yy zz

zz zz yy xx

E

E E E

E

� � � � �

�� � � � � � � � � � � � �

� � � � �

�� � �� � � �� �� � ���� �� � �� � � � � � � � �� � �� ����� � �� � �� � �� ���

y por tratarse de una compresión uniforme debida a la presión, será � = �p, de modo que la deformación unitaria en cada arista vendrá dada por

1 2 �pE�

��

� �

y el cambio unitario en el volumen será � 3(1 2 )3 �xx yy zz

V pV E

�� � � �

�� � � � � �

con 3 5� 1.06 10 9.8 400 41.552 10 Pa 41 atmp gh�� � � � � � � �

de modo que 5 4

10

� 3(1 0.6) 41.552 10 2.37 102.1 10

VV

��� � � � � � �

4 3 36 3 1.9� 2.37 10 0.2 1 cm.9 10 mV � �� � � � � � �� �

;

;

;

- 239 -

Física Universitaria: Problemas de Física Elementos de elasticidad. M27.18

18. Un cubo macizo y de acero, de 25 cm arista se sumerge a una profundidad de 500 m en agua marina

(densidad del agua constante % = 1.05 g/cm3). Calcular la variación de volumen que experimenta el cubo. Datos para el acero: módulo de Young = 2�106 kg/cm2 y coeficiente de Poisson = 0.3.

Escribimos las ecuaciones elásticas, teniendo en cuenta que, al tratarse de una compresión uniforme debida a la presión, será �xx yy zz p� � � �� � � � � , de modo que la deformación unitaria en cada arista vendrá dada por

1 2

1 ( )1 1 2( )1 ( )

1 2 �1 ( )

xx xx yy zz

xx yy zz

yy yy xx zz

zz zz yy xx

EE E

E pE

E

� � � � ��

� � � � � � � �� � � � �

��

� � � � �

�� � �� � � �� �� � �� ��� � � � � � � �� �� �� �� �� � � � �� �� � � �� � � �� �� ���� � �� � �� � �� ���

El cambio unitario en el volumen será 3(1 2� 3 ) �xx yy zz pV

V E� � � �

�� � � �

���

con 3 5

26 4 10

2 2 2

� 1.06 10 9.8 500 51.94 10 Pa 51.3 atmkg N cm N2 10 9.8 10 19.6 10

cm kg m m

p gh

E

�� � � � � � � �

� � � � � �

de modo que 5 4

10

� 3(1 0.6) 51.19 10 3.30 1019.6 10

VV

��� � � � � � �

4 6 3 33� 3.30 10 0.25 5.2 1 5.2 cm0 mV � �� � � � � �� � �

;

;

;

- 240 -

Física Universitaria: Problemas de Física Elementos de elasticidad. M27.19

19. El esfuerzo de ruptura por cortadura para el cobre laminado ordinario es de

aproximadamente 16×107 N/m2. ¿Qué fuerza F debe aplicarse para cortar con una cizalla una tira de cobre de 60 mm de ancho y 3 mm de espesor?

Sección de corte: 3 3 6 2c 60 10 3 10 180 10 mS � � �� � � � � �

Esfuerzo de corte: cc c c c

c

F F SS

� �� �

Por consiguiente, la fuerza de corte que debe aplicarse debe ser superior a la correspondiente al esfuerzo de ruptura por cortadura; esto es,

7 6c,rupt c,rupt c 16 10 180 10 28800 N 2939 kgF S� �� � � � � � �

Plancha de cobre

CizallaF

Plancha de cobre

F

S

Cizalla

- 241 -

Física Universitaria: Problemas de Física Elementos de elasticidad. M27.20

20. El acero ordinario requiere un esfuerzo de aproximadamente 35 � 107 N/m2 para que se produzca la ruptura por cortadura. Determinar la fuerza que es preciso ejercer sobre el punzón para hacer un agujero de 20 mm de diámetro en una plancha de ese acero de 6 mm de espesor.

El esfuerzo de corte al que se encuentra sometida la plancha cuando el punzón ejerce una fuerza F, está distribuido en la superficie lateral de un cilindro de 20 mm de diámetro y 6 mm de altura, esto es, sobre una superficie S Dh�� , de modo que

corteF FS Dh

��

� �

Para conseguir el corte de la plancha, el esfuerzo de corte aplicado deberá ser superior al esfuerzo de ruptura por cortadura, de modo que

corte rupt

7min rupt 35 10 0.020 0.006

13131947 N = 464 kg

F Dh

� �

� � �

9

� � � � � � �

F

punzón

plancha

estampa

- 242 -

Física Universitaria: Problemas de Física Elementos de elasticidad. M27.21

21. Un péndulo de torsión está formado por un alambre de acero ordinario, de 80 cm de

longitud y 1 mm de diámetro, que lleva en su extremo inferior un disco homogéneo de plomo, de 12 cm de diámetro y 1 cm de espesor. Se gira el disco un cierto ángulo y después se abandona de modo que efectúe oscilaciones de rotación en un plano horizontal. El tiempo empleado en realizar 100 oscilaciones completas es 315 s. a) ¿Qué esfuerzo tensor soporta el alambre? ¿Se supera el límite elástico? b) Calcular la constante de torsión del péndulo. c) Determinar el módulo de rigidez del acero del alambre.

Datos: densidad del plomo, 11.35 g/cm3; límite elástico del acero, 25�107 N/m2; relación

entre el coeficiente de torsión y el módulo de rigidez de un alambre, 4

2R Gl�

� � .

Periodo de las oscilaciones: 315 2 rad3.15 s 1.995100 s

TT�

�� � � �

Masa del disco : �2311.35 10 0.06 0.01 1.284 kgm V� �� � � � � �

Momento de inercia del disco: 2 2 3 21 11.284 0.06 2.31 10 kg.m2 2

I mR �� � � � �

a) El esfuerzo tensor que soporta el alambre es:

�7

22 23

1.284 9.8 N1.6 10m0.5 10

lmgR

�� � �

�� � � �

� (no se supera el límite elástico)

b) El momento recuperador de torsión es proporcional al ángulo o elongación de torsión; esto es, M ��� � . Por otra parte, tenemos la ecuación fundamental de la dinámica de rotación (aplicada al disco), M I�� �� ; de modo que

0M I I�� � � ��� � � � ��� �� que es la ecuación diferencia de un m.a.s. de rotación (torsión) cuya frecuencia angular es

I�

��

de donde podemos calcular el valor del coeficiente de torsión 2 2

2 3 32 2 m.N2.31 10 9.19 103.15 rad

I IT� �

� � � �� � � �� �� �� � � � � � �� �� �� �� �� �

c) Calculamos el valor del módulo de rigidez del acero a partir del coeficiente de torsión y de las dimensiones del alambre:

43 10

4 4 2

2 2 0.80 N9.19 10 7.49 102 0.5 mR lG Gl R�

� �� �

��� � � � � � �

- 243 -

Física Universitaria: Problemas de Física Elementos de elasticidad. M27.22

22. El eje de transmisión de un automóvil es de acero y mide 1.80 m de longitud por 2.5 cm de diámetro.

a) ¿Qué ángulo se tuerce uno de sus extremos con respecto al otro cuando el eje está transmitiendo una potencia de 30 CV a 2400 r.p.m.? b) ¿Qué energía elástica está entonces almacenada en el eje?

Datos: módulo de rigidez del acero, G = 7.8�1010 N/m2; coeficiente de torsión para un eje macizo, 4

2R Gl�

� � ; 1 CV = 736 kW.

Datos:

10 2

1.80 m2.50 cm 0.025 m1.25 cm 0.0125 m2400 r.p.m. 40 r.p.s. 80 rad/s30 CV 30 736 22080 W7.8 10 N/m

lDR

PG

� �

�� �� �� � �� � � �

� �

a) Determinamos el “par” (momento) transmitido por el eje a partir de la potencia transmitida a unas “revoluciones” conocidas; esto es,

22080 87.9 m.N80

PP M M�� �

� � � �

El “par o momento de torsión” (M) es proporcional al ángulo de torsión (+), M ���

siendo < el coeficiente de torsión, cuyo valor es

�4410 30.0125

7.8 10 1.66 10 m.N/rad2 2 1.80R Gl

��� � � � � � �

Así pues, el ángulo de torsión correspondiente a las condiciones de trabajo del enunciado, vale:

3

87.9 0.0529 rad 3.03º1.66 10

M�

�� � � �

b) Energía elástica de torsión: 2 3 21 11.66 10 0.0529 2.32 J

2 2U ��� � � � �

l

- 244 -

Física Universitaria: Problemas de Física Estática de los fluidos. M29.1

1. Demostrar que el centro de presiones que un líquido ejerce sobre una pared rectangular, plana y vertical se encuentra situado una profundidad igual a dos tercios de la altura de la pared.

Aplicamos el teorema del centro de presiones:

c cpIh hS

siendo: 3 hc la profundidad a la que se encuentra el centro geométrico de la pared:

c1

2h H�

3 I el momento de áreas de segundo orden de la superficie de la pared con respecto a la línea definida por la intersección del plano de la pared con la

superficie libre del líquido: 21

3I SH�

3 hcp la profundidad a la que se encuentra el centro de presiones; de modo que

213

cp 1c 2

2 c.q.d.3

SHIh Hh S HS

� � �

H 2/3 H

- 245 -

Física Universitaria: Problemas de Física Estática de los fluidos. M29.2

2. a) Dibujar el perfil o diagrama de presiones sobre la compuerta de la figura, de dimensiones 2a y b, cuando retiene agua y aceite en el modo que se representa. b) Calcular el empuje al que se encuentra sometida la compuerta, c) Determinar la posición del centro de empuje. Aplicación numérica: a = 1 m; b = 2 m.

a) En la figura, presentamos la distribución de fuerzas de presión sobre cada una de las dos porciones de la pared. b) Determinamos el empuje sobre cada porción de la pared correspondiente a cada una de las distribuciones de fuerzas representadas en la figura y el empuje resultante.

Mitad superior de la compuerta: 2

1 1 1 11

2

28820 N con 0.67 m2 3aF g ab ga b h a� �� � � � �

Mitad inferior de la compuerta: 2

2 1 1 2

22 2 2 ¨2

1

2

1( ) 17640 N con 0.50 m22( ) 9800 N con 0.67 m

2 3

F ga ab ga b h a

aF g ab ga b h a

� �

� �

) )� � � � �

)) ))� � � � �

2 2 22 1 2 1 2

1 1

2 227 440 NF ga b ga b ga b� � � �

� ���� � � � ��� ���

Empuje total sobre la compuerta: 2

1 2 1 22

1 23 1

2 2

1

2(3 ) 36260( ) NF F F ga b ga b� � �� � �� � � � �

c) Aplicamos el teorema de Varignon, tomando momentos en A:

31 2 2 1 2

2 2 11 53 2 3 6 6a a aHF F a F a F ga b� �

� � � � � �� � �� � �) ))� � � � � � �� � �� � �� � �� � �� � �

Determinamos la posición H del centro de presiones:

� �3

1 2

21 2

1 2

1 21

2

1 11 56

(3 )

11 53 (3 )

ga bH

gaa

b

� ��

� �

��

�� �

��

y sustituyendo valores, con �1 = 0.9 y �2 = 1.0, resulta 11 0.9 5 14.91

3 (3 0.9 1) 11.11.35 mH � �

� � �� � �

Aceite (% =900kg/m3)

2a

a

ab

Agua

aceite (%1 =900kg/m3)

(%2 =1000kg/m3) agua

¨2h88

A

F

1h

¨2h82F 8

2F 88

1FH

- 246 -

Física Universitaria: Problemas de Física Estática de los fluidos. M29.3

3. Un canal rectangular de 2 m de ancho está cortado por una pared transversal que separa dos líquidos. A un lado de la pared hay agua que alcanza una altura de 1,8 m. Al otro lado un aceite de densidad 0,9 g/cm3. a) ¿Qué altura debe de alcanzar el aceite para que sobre la pared, los dos líquidos, ejerzan unos empujes equivalentes a un par de fuerza? b) Valor del par de fuerza y sentido del mismo. c) Qué fuerzas (valor, direcciones y sentidos) deben aplicarse a la pared separadora, una en B al nivel de la superficie del aceite y la otra a la de su fondo, en A, para equilibrar los empujes.

a) Para que la acción conjunta de ambos empujes sea equivalente a un par es necesario que se igualen los módulos de ambos empujes: i.e.,

agua aceite

2 21 21 1 2 2 1 1 2 2( ) ( )

2 2

E Eh hg h L g h L h h� � � �

� �

de modo que

12 1

2

11.80.9

1.897 mh h��

� � �

Cada empuje vale 2 21

1 1 1 11 1( ) 1000 9.8 1.8 2 31752 N

2 2 2hE g h L gh L� �� � � � � � �

b) Los centros de presiones distan de la solera un tercio de la profundidad, como se indica en la figura, por lo que el brazo del par de empujes y el par valen:

2 1 2 1 1.897 1.8 0.032 m3 3 3 3h h h hb � �

� � � � �

emp 0.032 31752 1030.5 NmM bE� � � �

c) Para compensar dicho par actuando sobre los puntos A y B será necesario ejercer el par opuesto, de modo que

empemp

1031AB1.897AB

543 NM

F M F� � � � � �

Eaceite Eagua

0.6324 m 0.6 m

h1h2

A

B F

F

A

aceite agua

B

- 247 -

Física Universitaria: Problemas de Física Estática de los fluidos. M29.4

4. La compuerta vertical de la figura mide 3 m de altura, está engoznada por su borde superior y separa dos recipientes que contienen agua y aceite de densidad relativa 0.85. Si la altura H del agua alcanza 2 m, calcular la altura que debe alcanzar la superficie libre del aceite para que la compuerta se mantenga vertical.

Las fuerzas que ejercen el agua y el aceite sobre la compuerta vienen dadas por

21agua 2

21aceite 2

2

2

HF g HL gH L

HF g H L gH L

� �

� �

��� � ������ )� ) ) ) )� ����� Sabemos que las líneas de acción de esas fuerzas son horizontales y están situadas a una altura sobre la solera igual a un tercio de la profundidades H y H’ respectivas. En efecto,

213

c cp cpc 2

23H

SHI Ih h h HS h S S

� " � � �

medida desde la superficie libre del líquido. Sea A la altura de la compuerta (i.e., desde la solera hasta el borde superior engoznado). Tomando momentos con respecto al eje que pasa por O, tenemos

3 2

2 21 1agua aceite 2 2

2 2 2 3

3 3 3 3

33 (3 )

30

H H H HF A F A gH L A gH L A

H H AH AHH H HA H A

� �

���

� � � �� � � �) )� �� �� �� � ) )� � � " � � � "� �� �� �� �� �� �� �� �� �� � � � � �� � )�� �� ) )� � � "�� �� ��� �

) )� � �� ) � �

En la que sustituimos los datos para obtener

2 23 32 313 3 (9 2 2 289 00.8. 5

) 00 85

H HH H ) )� �) )� � � � � " ��

Resolviendo esta ecuación cúbica se obtiene 2.2 mH ) �

H agua aceite

H’

O

- 248 -

Física Universitaria: Problemas de Física Estática de los fluidos. M29.5

5. a) Determinar la fuerza total debida a la presión del agua sobre la compuerta inclinada, de 3 m de anchura, que se muestra en la figura. b) Calcular el mo-mento de dicha fuerza respecto a la bisagra (B). c) Localizar la línea de acción de dicha fuerza resultante. d) Determinar la reacción de la solera sobre el borde inferior (A) de la compuerta.

a) Aplicamos la expresión: �cF gh S�� , con 2

c 1 0.75sen 30º 1.375 m; 1.5 3 4.5 m

1000 9.8 1.375 4.5 60 638 N 6188 kg

h S

F

� � � � � �

� � � � � �

c) Aplicamos la expresión: c cp /xxy y I S� , con

� � � �2 2 2 22 21 1cc 12 12

cp pc

1.5 2.75 1.5 2.75 7.75 m

/ 7.75 2.82 m BC 2.82 2.00 0.82 m2.75

xxxx

xx

II I SD S SS

I Syy

� � � � � � �

� � � � � ������

b) Aplicamos el Teorema de Varignon para una distribución de fuerzas paralelas, tomando momentos en B:

B pBC 0.82 60638 49723 m NM F� � � � �

d) Puesto que la compuerta está en equilibrio, los momentos en B de F y de FA deben ser iguales y opuestos; esto es,

� � BB A A

49 723BA cos30º 38 277 N 3906 kg1.5 0.866BA cos30º

MM F F� � � � � ��

agua

30º

1 m

1.5 m

B

A

agua

30º

1 m

1.5 A

B

x

1 mhc hcp

2.75 m

ycp

2 m

30º

FA

x

F

- 249 -

Física Universitaria: Problemas de Física Estática de los fluidos. M29.6

6. La compuerta representada en la figura está formada por dos superficies rectangulares unidas rígidamente entre si y puede girar alrededor del eje AA’. a) Calcular el empuje que ejerce el agua sobre la compuerta. b) Calcular el momento del par que hay que aplicar al eje AA’ para que la compuerta no se apoye en la solera.

Panel vertical: �1 C1 1

213

C1 P1 P1C1

1000 9.8 1 2 3 58800 N

2/ 4 1.3 m1 3

xx xx

F gh S

I I Sh h hS h

�� � � � � � �

� � � � ��

Panel inclinado:

� 2,hor 22 C2 2

2,vert 2

C2

2 2 2 2C2 P2

2P2

C2

sen 30º 73500 N1000 9.8 2.5 2 3 147 000 N

cos30º 127306 N

2 1 sen 30º 2.5 m1 76sen 30º con 2 5 25.3 m

12 3/ 25.3sen 30º 0.25

2.5

xx xx

xx

F FF gh S

F F

hI Ih hS S

I Shh

�� � ���� � � � � � � �� � � ���

� � � �

� � � � �

� �

�2.53 m��

Las componentes de la resultante, su módulo y dirección son: 2 2

horz

vert

132300 127306 183603 N132300N 21218 127306 N =arctg 43.9º

127306

FFF �

�� � � ��� � ��� � � �� �� �� �� ���

b) El momento del par que deberemos aplicar deberá ser igual a la diferencia de los momentos de las fuerzas F1 y F2 con respecto al eje AA’; esto es,

1

par

2 2

2 2 58800 39200 N m3 3

0.53 AP 147000 155820 N ms

116620

en 30º

N mF

MF

��� � � � � ���� ���� � � � � ��

����

��

2 m

3 m

2 m

30 º A

A’

A

xx

P F1

F2

F

O

P2

P1

C1

C2

- 250 -

Física Universitaria: Problemas de Física Estática de los fluidos. M29.7

7. Cuando sube el nivel del agua en la alberca, la compuerta rectangular, doblada en L, se abrirá automáticamente. ¿Cuál deberá ser el nivel mínimo (h) del agua por encima de la bisagra para que no se produzca la apertura? (Despreciar el peso de la compuerta).

Calculamos las fuerzas que actúan sobre cada una de las secciones de la compuerta:

�v

2h

12 2

F gh aL gLah

hF g hL gLh

� �

� �

� � ������ � ���� � ���� ��� � ��

Determinamos la posición del centro de presión de la sección vertical de la compuerta: 21

3c cp cp 1

c 2

23

xx xx ShI Ih h h hS h S hS

� � � �

medida desde la superficie libre del agua. La compuerta permanecerá cerrada mientas que el momento de la fuerza horizontal sea mayor que el de la fuerza vertical, ambos respecto de la bisagra; esto es,

2h v

13 2 2 3 2h a h aF F gLh gLah� �

� � � � � � � �� � � �� � � �. .� � � �� � � �� � � �� � � �� � � �

o sea, 21 3 1.22 3 2.11cm

3h a h a(. . � �

agua

1.22 cmh

agua

1.22 cm

Fv

Fh

h

- 251 -

Física Universitaria: Problemas de Física Estática de los fluidos. M29.8

8. Una viga de madera, de sección cuadrada de lado a, apoyada sobre una de sus aristas, bloquea el extremo de un canal de fondo plano y horizontal, alcanzando la superficie libre del agua la arista superior de la viga. Calcular el empuje que el agua ejerce sobre la viga y ángulo que éste forma con la horizontal.

2 2 2H a a a� � � Método 1 (dos caras): Cara superior:

21

2 24 4 4HF g aL g a aL ga L� � �

� �� � ���� �� � ���� ���� ��� ��

Cara inferior:

22

3 3 2 3 24 4 4HF g aL g a aL ga L� � �

� �� � ���� �� � ���� ���� ��� ��

El módulo de la resultante es:

� �2 22 2 2 21 9 58 8 4

52

F g ga La L ga L F� � �� ���� � � " ��� ���

Calculamos el ángulo: 2

1

tg 3 71.6 71.6º 45º 26.6ººFF

� � �� � " � " � ��

Método 2 (Arquímedes): Determinamos directamente las componentes horizontal y vertical de la fuerza total que actúa

sobre la viga. Empuje sobre la proyección vertical:

2hor

2 22 2H aF g HL g a L ga L� � �

� ���� � ��� ���

Empuje de Arquímedes: 2

2ver

12 2aF gV g L ga L� � �

� ��� �� � �� �� ���

Módulo de la resultante: �22 2 2 2hor ver

252

114

ga LF F F ga L F� �� ���� � � � " ��� ���

Ángulo que forma con la horizontal: ver

hor

1tg2

26.6ºFF

� �� � " �

H

F2

F1 a

� +

F

H

Fhor

Fver

a

- 252 -

Física Universitaria: Problemas de Física Estática de los fluidos. M29.9

9. Un tonel de completamente lleno de vino (de densidad, 0.92 g/cm3) tiene las dimensiones que se indica en la figura y está tumbado en la bodega. Calcular el empuje que ejerce el vino sobre cada una de las tapas del tonel y determinar la posición del centro de presiones sobre las mismas medidas respecto al centro de las tapas.

Cálculos previos Medimos las profundidades a partir del eje xx indicado en la figura. El centro geométrico (centroide) de la tapa se en-cuentra a una profundidad hc=R. Determinamos el momento de área de la tapa con respecto al eje DD (Teorema Ejes Perpendiculares):

2 2DD DD

1 122 4

I I Sr I Sr� � �

Ahora, mediante el Teorema de Steiner, determinamos el momento de área de la tapa respecto al eje xx definido en la figura:

2 2 2 2 2 2 2DD

1 1 14 4 4

xxxx

II I SR Sr SR r R S r RS

� � � �� �� �� � � � � � � �� �� �� �� �� �

La fuerza resultante sobre la tapa será

� �2 2c( )F gh S gR r gRr� � � ��� � �

La profundidad a la que se encuentra el centro de presiones se determina mediante el teorema del centro de presiones:

22 2

c cp cpc

/ 1 14 4

xxxx

I S rh h S I h r R Rh R R

� ���� � � � � ��� ���

O bien, medida respecto a centro de la tapa: 2

4rRR

�� �

Sustituyendo los valores numéricos se obtiene: 2920 9.8 0.70 0.5 4957 N 506 kgF �� � � � � � �

20.5 0.0893 m 8.93 cm4 0.7

�� � ��

y

hc

hcp

F r

xx

DD

R

F

140

cm

200 cm

100

cm

- 253 -

Física Universitaria: Problemas de Física Estática de los fluidos. M29.10

10. Un barril de cerveza tiene forma de cilindro de revolución de 1.2 m de diámetro y 1.8 m de longitud y se apoya sobre una generatriz. Si el barril contiene la mitad de su capacidad, calcular la resultante de las fuerzas de presión de la cerveza (densidad, 0.939 g/cm3) sobre la tapa del barril y determinar la posición del centro de presiones sobre la misma.

Cálculos previos Determinamos la posición del centro geométrico (centroide) de un semicírculo sirviéndonos del Segundo Teorema de Pappus-Guldin:

3 2c c

4 1 423 2 3

RV SL R R h h� � ��

� � �

Determinamos el momento de área de un semicírculo con respecto a su diámetro, sabiendo que el momento con respecto a un eje perpendicular a su plano y que pasa por su centro es igual a

212zzI SR�

de modo que, aplicando el Teorema de los Ejes Perpendiculares, resulta 212

4xx zz xxI I I SR� �

La fuerza resultante sobre la tapa será

c3

24( )3 2

23

R RF gh gg RS �� �

��� � �

La profundidad a la que se encuentra el centro de presiones se determina mediante la expresión

c cp

214

cp 43c

/ 316

0.59

xx

xx

h h S I

I S R Rhh R

R�

� � � �

Sustituyendo los valores numéricos se obtiene:

3

cp

2 939 9.8 0.6 = 135 kg30.58 0.60

1325 N

30.353 m = 5.3 cm

F

h

� � � �

� � �

x x

y

y

hchcp

F

R

- 254 -

Física Universitaria: Problemas de Física Estática de los fluidos. M29.11

11. Determinar el empuje y el centro de empuje sobre una compuerta circular de radio R, como la que se representa en la figura, cuando el nivel del líquido coincide con el punto más elevado de la misma.

Determinamos la posición del centro geométrico (centroide) de la compuerta y calculamos el momento de área de la misma con respecto al eje determinado por la intersección del plano de la compuerta con la superficie libre del líquido (eje x):

2 3c c 2

2 2 2514 4

sen 60º

xx

S R y R h R R

I SR SR SR

�� � � �

� � �

La fuerza resultante sobre la compuerta se determina multiplicando la presión en su centro por la superficie de la compuerta:

23c 2

332( )F gh S g R gR R� � � � �� � �

La posición del centro de presión o empuje se determina a partir del Teorema de Centro de Presión:

254

c cp cpc

54

xx xx SRI Iy y yS y S RS

R� � � �

60º

R

x

x

y y

h

y

F

C

P C

P

O

hc hcp

ycp

yc

- 255 -

Física Universitaria: Problemas de Física Estática de los fluidos. M29.12

12. Un depósito de base cuadrada de lado 1 m, tiene una de sus paredes inclinada 30º respecto a la vertical. a) Demostrar que el empuje sobre la pared rectangular vertical está a 1/3 de la altura h que alcanza el agua medido desde la base. b) Calcular el empuje y centro de empuje sobre la pared inclinada. c) ¿Puede producirse el vuelco del depósito al ascender el nivel del agua?

Aplicamos los dos teoremas básicos de la hidrostática en las dos paredes. a) Pared vertical:

21 c1 1

213

c1 p1 p1c1

12 2

/ 2/ 2 3

xx xx

hF gh S g ha gah

hI I Sh h h hS h h

� � ���� � � ������� � � � �����

b) Pared inclinada: La longitud de la pared inclinada bañada por el agua es

2 2 3cos30º 33 / 2 3

h h hl h� � � �

22 c2

213

c2 p2 p2c2

32 3

/ 2/ 2 3

xx xx

hF gh S g la gah

lI I Sy y y lS y l

� � ���� � � ������� � � � �����

c) Calculamos el momento neto en el punto O, ya que, de producirse el vuelco, ocurriría alrededor de ese lado de la base.

� �

2 3O 11

2 2 3O 2 22

3 3O O O2 1

2 1 1 13 3 2 6

2 3 1 2 3 3 2OP3 3 3 3 3 9

2 1 19 6 18

M h h F h gah gah

M F l l gah h gah gah

M M M gah gah

� �

� � �

� �

� ���� � � ��� ���

� ���� � � � ��� ��� � ���� � � � ��� ���

Como el momento resulta positivo, cualquiera que sea el valor de h, se producirá el vuelco en todo caso, a menos que lo impida el propio peso del depósito, que en este problema hemos supuesto despreciable.

h

a

a 30º

h

F2 l

hp2 hp1

F1

(xx)(xx)

a

O P2

P1

30º

y

aa

- 256 -

Física Universitaria: Problemas de Física Estática de los fluidos. M29.13

13. Determinar la fuerza resultante (módulo y dirección) que actúa sobre la compuer-ta cilíndrica AB, cuya sección recta es la de un cuarto de circunferencia. La an-chura de la compuerta es 1.20 m.

Determinamos las componentes horizontal y vertical del empuje hidrostático sobre la compuerta. Componente horizontal = Empuje sobre la proyección de la compuerta sobre un plano vertical:

h c cF p S gh RL�� �

y sustituyendo valores � 3

h 1000 9.8 0.60 0.45 0.90 1.20 11.1 10 N 1135 kgF � � � � � � � � �

Componente vertical = Peso del volumen de líquido situado por encima de la compuerta: 2

v encima 4 4RF gV g HR L g H R RL� �

� � �� � � ��� ���� � � � � �� �� �� ��� � �

y sustituyendo valores

3v 1000 9.8 1.50 0.90 0.90 1.20 8.40 10 N 857 kg

4F �� ���� � � � � � � � � ��� ���

La línea de acción del empuje resultante pasa por el punto O, ya que las diferentes contribuciones son perpendiculares a la superficie y, por tanto, tienen dirección radial. El módulo y la dirección del empuje hidrostático resultante se calculan fácilmente

2 2 2 2 3 3horz vert 11.1 8.40 10 13.9 10 N 1421 kgF F F� � � � � � � �

vert

horz

8.40tg 0.7554 37.1º11.1

FF

� �� � � �

A

B

60 cm

90 cm

agua

A

B

60 cm

90 cm

agua

O

Fh

Fv F

� H

CP

- 257 -

Física Universitaria: Problemas de Física Estática de los fluidos. M29.14

Fh 1m

Fv Fh

Fv

F

� O

A

14. Un canal de agua, de 2 m de ancho y 1 m de profundidad, está cerrado mediante una compuerta cilíndrica de radio igual su profundidad. a) Calcular la magnitud del empuje hidrostático sobre la compuerta y el ángulo que forma su dirección con la horizontal. b) Determinar la posición del eje de giro (normal al dibujo) de la compuerta para que dicho eje no tenga que soportar ningún momento.

Determinamos las componentes horizontal y vertical del empuje hidrostático sobre la compuerta. Componente horizontal = Empuje sobre la proyección de la compuerta sobre un plano vertical:

2 2h c h

1 1( ) 1000 9.8 2 1 9800 N2 2 2RF gh S g lR glR F� � �� � � � � � � �

Componente vertical = Empuje de Arquímedes (peso del volumen de líquido desalojado): 2 2 2

v v1 1000 9.8 2 1 15394 N4 4 4

F gV g R l glR F� �� � � �� � � � � � � �

La línea de acción del empuje resultante pasa por el punto O, ya que las diferentes contribuciones son perpendiculares a la superficie y, por tanto, tienen dirección radial. El módulo y la dirección del empuje hidrostático resultante se calculan fácilmente

2 2 2 2h v

v

h

182499800 15394

tg 1.57 57.2

N

F F FFF

�� �

� � � � �

� � � �

La posición del eje de giro de la compuerta se determina mediante la intersección de la línea de acción de la fuerza de empuje resultante con la compuerta; i.e., el punto A que se indica en la figura.

1m

- 258 -

Física Universitaria: Problemas de Física Estática de los fluidos. M29.15

15. La compuerta de la figura está compuesta de dos chapas soldadas, una rectangular y otro cilíndrica. Determinar el empuje sobre toda la compuerta y el ángulo que éste forma respecto a la horizontal cuando la compuerta contiene un líquido de densidad % .

Determinamos las componentes horizontales sobre las dos partes de la compuerta: 2

1,hor 1,c 11

2 2aF gh S g ab ga b� � �� � �

22,hor 2,c 2,proy

3( )2 2aF gh S g a ab ga b� � �� � � �

La componente vertical, dirigida hacia arriba, es igual al peso del fluido situado “encima” de la porción cilíndrica de la compuerta:

22 2

2,vert ( ) (1 )4 4aF gV g a b b ga b� �

� � �� � � � �

Las componentes horizontal y vertical de la fuerza resultante son:

2 2horz 1,horz 2,horz

vert 2,ve2

t

2

r (hacia arriba)

2

(1 )

12

4

32

gaF F F ga b ga b b

gaF bF

�� � � �

� �

El módulo y la dirección de dicha fuerza son: 2

2 2 2horz vert

v

2

ert 4

horz

(hacia arriba)

4 14

1tg 0.893

2.86

241.8º

F F F ga b

FF

ga b

�� ���� � � � � �� ���

�� � � �

F1

F2,horz

F2,vert

a

a

a

b

F2

F

Fhorz

Fvert

a

a

a

b

- 259 -

Física Universitaria: Problemas de Física Estática de los fluidos. M29.16

16. La compuerta representada en la figura tiene una anchura L y está formada por un tramo AB rectangular de altura R y la cuarta parte (BC) de una superficie cilíndrica de sección circular de radio R. La compuerta puede girar alrededor de un eje perpendicular al plano del dibujo y que pase por A. Determinar la fuerza vertical mínima F que se debe aplicar en C para mantener cerrada la compuerta.

Sobre la porción rectangular (AB) de la compuerta actúa una fuerza F1 horizontal, aplicada a una profundidad 1 2 / 3h R� (centro de presión), cuyo módulo es:

21

12 2RF g RL gR L� �

� ���� ��� ���

Sobre la porción cilíndrica (BC) de la compuerta actúa una fuerza F2 cuya línea de acción pasa por el eje de simetría de revolución del cilindro (punto O, en la figura). Las componentes horizontal y vertical de esta fuerza son

22h

3 32 2RF g RL gR L� �

� ���� ��� ���

22 2

2v 14 4RF g R L gR L� �

� �� � � ��� ���� � � � �� �� �� ��� � �

El eje fijo en A, alrededor del cual puede girar la compuerta, ejerce sobre ésta una fuerza N cuyas componentes horizontal y vertical se indican en la figura. Puesto que la compuerta debe permanecer cerrada y en equilibrio, el momento resultante sobre ella debe ser nulo; así, tomando momentos en A, podemos escribir:

A 1 2h 2v 1 2h 2v2 2

3 30M F R F R F R FR F F F F

� ���� � � � � � � ��� ���

y sustituyendo valores

2 21 2h 2v

2

3

1 3 34 313 2 4 12

F F F F gR L gR L� �� �

� � � ��� �� �� � � � � � � �� �� �� �� �� �

esto es, 23.62F gR L��

A

R F

B

C

R

O

F1

F2

F2

F2h

F2v

Nx

Ny

R

R

R

L A

C

- 260 -

Física Universitaria: Problemas de Física Estática de los fluidos. M29.17

17. Debemos obstruir un canal de sección rectangular mediante un cuarto de cilindro de radio R = 0.5 m y cuya altura coincide con la altura del agua en el canal. a) Calcular el empuje y centro de empuje en los casos ilustrados en la figura. b) ¿Cuál sería la densidad mínima del cuarto de cilindro en ambos casos para evitar el vuelco respecto al eje 00’?

Caso A. Determinamos las componentes horizontal y vertical del empuje hidrostático: 2

2 3 3h c A

23 3 3 v

v

h

1 1 (4 )( ) 612.5 N2 2 4 16

4( ) tg 2 0.429

6

4 4 2

66.5 N

23.3º

RF gh S g R gR F gR

R FF g R V g R R gRF

�� � � �

� � �� � � � �

�� �� ��� � � � � � � � �� �� �� � � �� �� ����� ��� � � � ��� � � � � � ���� ���� � �� ��

Como la línea de acción del empuje hidrostático corta al eje OO’, no se produce momento de vuelco, por lo que el peso del cuarto de cilindro es irrelevante, al menos a los efectos de vuelco. Caso B. Empuje hidrostático horizontal sobre una superficie plana

2 3B c 612.1( )

25 N

2RF gh S g R gR� � �� � � �

y el centro de presiones o de empuje se encuentra situado a una profundidad

213

c cp cp 12c

/ 2 3 cm3

3xxxx

I S Rh h S I h Rh R

� � � � �

El momento que produce el empuje hidrostático respecto del eje OO’ deberá estar contrarrestado por el que produce el peso con respecto al mismo eje: i.e.,

B B( )3 3( )R RF P R x P F

R x� � �

Determinamos x mediante el 2º teorema de Pappus-Guldin: 2

32 423 4 3

R RV LS R x x�� �

�� � �

De modo que el peso y la densidad del cuarto de cilindro deberán ser: 3 3

B

3

cil 14

33

354.1 36.27 N

0.369 g/c

kg43( ) 2 2(3 4)3( )3/ 2 0.369 =

2(3 4) (3 4)m

R RP F gR gRRR x R

m P g RV V R

�� �

��

� �� � �

� � �

� � � � �� ��

� � � � �� �

O

O’

Caso A O

O’

Caso B

R

R R

R

O

Fh

Fv FA �

hc

O

FA

hc hcp

P

x

G

- 261 -

Física Universitaria: Problemas de Física Estática de los fluidos. M29.18

18. El final de un canal está taponado por una compuerta semicilíndrica, de 1 m de radio y 2 m de longitud, cuyo diámetro está inclinado 45º. a) Calcular el empuje hidrostático sobre la compuerta, especificando su módulo y su recta de acción. b) Determinar la fuerza mínima que deberemos aplicar perpendicularmente sobre la compuerta, así como el punto de aplicación de dicha fuerza, para impedir que la compuerta vuelque sobre el eje A.

22 cos 45º 2 22

H R R R� � �

a) La componente horizontal del empuje es igual al que ejercería el agua sobre la superficie proyectada de dimensiones H�L; esto es,

� 2 2h c

1 19600 N2 2HF gh S g HL gH L gR L� � � �� � � � �

Como consecuencia del Teorema de Arquímedes, la componente vertical del empuje es igual al peso del volumen de agua “desalojado” (contenido) por la compuerta (indicado en la figura); esto es, el volumen de medio cilindro más el de un prisma de base triangular:

2 2v

1 1 2 2 1 50388 N2 2 2

F gV g R R R L g R L�� � � �

� � � �� �� �� � � � � �� �� �� �� �� �

Conocidas las componentes del empuje hidrostático, calculamos su módulo y su dirección de 2 2 2 2 2

h v 2

v

h

54061 ( 1) 2.76 55175 N

6

kg

tag 1 7.82.57082

º

F F F gR L gR LFF

�� �

�� �

� � � � � � � �

� � � � �

y su línea o recta de acción pasa por el punto O (centro de la semicircunferencia), por ser la resultante de una distribución continua de fuerzas perpendicular a la compuerta en cada uno de sus puntos (dirección radial). b) El momento del empuje E con respecto al eje de la compuerta deberá ser igual (y de signo opuesto) al de la fuerza F. La magnitud de dicha fuerza será mínima cuando su brazo, con respecto al eje A, sea máximo; esto es, cuando sea igual a R, de modo que su línea de acción está inclinada 45º. Así, tenemos

� �A cos 45º cos 67.8º 4 49841 N5º 5086 kgM ER FR F E�� � � � � � �

45º A

45ºA

Eh

R

R

2R

2H R�

Eh

Ev E

+O

F

L L

- 262 -

Física Universitaria: Problemas de Física Estática de los fluidos. M29.19

19. Un depósito contiene un líquido de densidad % hasta su borde. Una compuerta hecha con una chapa cuya forma es un cuarto de esfera de radio R está adosada a un lateral del depósito, tal como se indica en la figura. Determinar el empuje que actúa sobre la compuerta, así como la línea de acción del mismo.

Determinamos la posición del centro geométrico (centroide) de un semicírculo por medio del Segundo Teorema de Pappus-Guldin:

23

c c4 423 2 3

R RV SL R h h�� �

�� � � �

Componente horizontal del empuje = empuje sobre la superficie proyectada sobre el plano vertical:

2

h c proy324( )

3 2 3R RF gh S g gR��

� ��

� � � �

Componente vertical del empuje = peso del líquido situado encima:

3

v3

343 4

RF gV g gR��

�� �� � �

El módulo y la dirección del empuje, cuya línea de acción pasa por el centro de la esfera, serán:

2 2 2 3h v

v

h

31 43

/ 3tg

1.24

57.51.5712 / 3 2

º

F gRF F gR

FF

� �

� ��

� � � � �

� � � � �

R

Sproy

hc

Fv

Fh

F

- 263 -

Física Universitaria: Problemas de Física Estática de los fluidos. M29.20

20. En una de las paredes verticales de un acuario hay un mirador de vidrio, de forma hemisférica, de radio R = 50 cm, cuyo centro está situado a una profundidad 3R. a) Determinar el módulo (newtons) y la dirección (en grados) de la fuerza que ejerce el agua sobre el mirador, así como el punto de aplicación de dicha fuerza. b) Ídem en el caso de que el mirador fuese plano, de forma circular de radio R, contenido en el plano de la pared.

a) Determinamos directamente las componentes horizontal y vertical de la fuerza total que actúa sobre el mirador. Empuje sobre la proyección vertical (círculo):

2 3 3 3h c (3 )( ) 11.53 3 1000 9.8 0 10 N.50F gh S g R R gR� � � �� �� � � � �� � � �

Empuje de Arquímedes:

3 3 3v

32 2 2.52 1000 9.8 0.503 3

7 13

0 NF Vg R g R� � � �� �� ���� � � � � � � ��� ����

Módulo de la resultante: 2

2 2 2 3 33h v

2 11.8 133

0 N853

F F F gR gR�� ��� ���� � � � � ��� �� ��

y su línea de acción pasa por el centro de la hemisfera. Ángulo que forma con la horizontal:

v

h

2 / 3 2tg3 9

12.5ºFF

� �� � � �

b) Como antes, determinamos la fuerza horizontal que actúa sobre el mirador circular:

3h 11.5 10 NF ��

Calculamos el momento de área de la superficie del mirador con respecto al eje x:

2 2 21 37(3 )4 4xxI SR S R SR� � �

Aplicamos el Teorema del Centro de Presiones:

�2 2

2 1c cp cp 12

c

37 37 37 37 34 4 4 3 12

1.54 mxxI R Rh h R h R RS h R

� � � � � � � �

3R

R

agua

hcp

214

SR

2374

SR eje x

hc=3R

Fh

Fv F

- 264 -

Física Universitaria: Problemas de Física Estática de los fluidos. M29.21

21. Una semiesfera hueca se halla sumergida en un líquido de densidad %1, contiene en su interior otro líquido de densidad %2 (%2 � %1) y descansa sobre el fondo horizontal de un depósito. La superficie libre del líquido de menor densidad se encuentra a una distancia por encima de la semiesfera igual al radio de ésta. Ambos líquidos, no miscibles, están en contacto a través de un orificio (A) situado en la cima de la semiesfera. a) Calcular la presión en el punto B, situado en el fondo y centro de la semiesfera. b) La presión en el punto C, situado en el fondo y exterior a la semiesfera. ¿Es mayor o menor que en el punto B? c) ¿Qué masa debería tener, como mínimo, la semiesfera para que el líquido interior no la levante?

a) y b) La presión en los puntos B y C, teniendo en cuenta que la presión ejercida por el líquido externo se transmite a través del orificio A, es

� �B atm 1 2 atm 1 2

B C 2 1 B CC atm 1

02

p p gr gr p grp p gr p p

p p gr� � � �

� ��

� � � � � � ��� � � � . .�� � ���

c) En general, la presión a una profundidad h, medida desde A, en los puntos interiores y exteriores de la semiesfera son

�int atm 1 2int ext 2 1

ext atm 1 1

( )( ) ( )

( )p h p gr gh

p h p h ghp h p gr gh

� �� �

� �

� � � ��� � � ��� � � ���

de modo que la situación es equivalente a la reflejada en la figura, en la que hemos eliminado el líquido externo y hemos sustituido el líquido interno por otro de densidad (�2 - �1). En definitiva, tenemos que determinar la fuerza vertical ejercida hacia arriba por el líquido interno (�2 - �1), en ausencia del líquido externo. La distribución de presiones sobre la cara interna de la semiesfera origina una fuerza resultante vertical hacia arriba que podemos calcular mediante el Principio de Arquímedes, ya que dicha distribución es análoga a la que se produciría sobre la cara exterior (incluida la base) de una semiesfera maciza sumergida en un fluido:

�3Arq 2 1

23

F r g� � �� �

Como la semiesfera hueca no tiene base, hay que sustraer la fuerza ejercida sobre ésta, i.e.,

� �2 3base 2 1 2 1F r rg gr� � � � � �� � � �

de modo que

� � � �3 3 3 32 1 2 1 2 2 1

2 1 13 3 3

gr gr gr F gr� � � � � � �� � � �� � � � � , � �

de modo que la masa de la semiesfera deberá de ser, como mínimo,

� 3mín 2 1

13

m r� � �� �

B C

A

r

r

A

�2- �1

F

EArq

Fbase

�2- �1

- 265 -

Física Universitaria: Problemas de Física Estática de los fluidos. M29.22

22. Un bloque cúbico, homogéneo, de 20 cm de arista y densidad 0.7 g/cm3, está sumergido en un recipiente que contiene aceite de densidad 0.8 g/cm3. La cara inferior del cubo se apoya sobre una cañería de 200 cm2 de sección, que penetra 2 cm en el fondo del recipiente. La cara superior del cubo dista 40 cm de la superficie libre del aceite. Calcular la presión manométrica del aire que deberemos insuflar por la cañería para que el cubo se desprenda y comience a ascender.

3700 9.8 0.2 54.88 NP gV�� � � � �

� 21 ac 1 1 800 9.8 0.40 0.20 125.44 NF gh S�� � � � � �

�2 ac 2 2 800 9.8 0.60 0.02 94.08 NF gh S�� � � � � �

�aire aire 3 aire0.02F p S p� �

En las condiciones críticas de equilibrio deberá ser:

1 2 aireP F F F� � �

de modo que

aire 1 2 86.24 NF P F F� � � �

Así, la presión manométrica del aire insuflado a través de la cañería es

aireaire

aire

86.24 4312 Pa0.02

FpS

� � �

40 cm

20 cm 2 cm

aire

40 cm

20 cm 2 cm

aire

F1

F3 F2

P

- 266 -

Física Universitaria: Problemas de Física Estática de los fluidos. M29.23

23. Un cuerpo de 1260 kg flota en agua marina (densidad 1.05 g/cm3) emergiendo 0.24 m3 sobre la superficie libre del agua. a) Determínese la densidad del cuerpo. b) Realmente este cuerpo es poroso y está formado por una masa sólida de densidad 1.2 g/cm2 con pequeñísimas burbujas de aire atrapadas. ¿Cuál es el porcentaje en volumen del aire existente en el cuerpo?

Nota: Despreciar la densidad (peso) de aire contenido en el cuerpo.

a) Como el cuerpo está en equilibrio de flotación, podremos aplicar el principio de Arquímedes para determinar el empuje vertical, que será igual al peso del cuerpo. El empuje está relacionado con el volumen sumergido VS, del modo

m S Sm

312 1.210

m6050

mE gV mg V��

� � � � �

El volumen del cuerpo es

cuer3

po 1.2 0.24 1.44 mV � � �

y la densidad de este cuerpo es

cuerpo 3cuerp

3o

1260 kg8751.44

g0.87m

5cm

mV

� � � � �

b) El volumen de la masa sólida, Vms, viene dado por 3

msms

1260 1.05m1200

mV�

� � �

de modo que el volumen de aire viene dado por Vaire = 1.44 – 1.05 = 0.39 m3

que en porcentaje volumétrico representa

aire

total

0.39 0.271.44

27%VV

� � �

P

E

%m

- 267 -

Física Universitaria: Problemas de Física Dinámica de los fluidos ideales. M32.1

1. El géiser Old Faithful (Yellowstone Park) expulsa periódicamente un chorro de agua que alcanza una altura de hasta 40 m. a) Determinar la velocidad del agua en la base del chorro. b) Calcular la presión manométri-ca que debe existir en el interior de géiser, a una profundidad de 100 m, para que pueda proyectar el agua hasta esa altura.

a) Aplicamos Bernoulli de 2 a 3:

�2 2atm 2 atm 3 3 2

1 22

p gz v p gz v g z z� � �� � � � � �

de modo que

�3 22 2 9.8 40 28 m/s 100.8 km/hv g z z� � � � � � �

b) Aplicamos Bernoulli de 1 a 3: �1 1 atm 3 1 atm 3 10 0p gz p gz p p g z z� � �� � � � � � � �

de donde � 6

1 atm 3 1

6

1000 9.8 140 1.37 10 Pa

1.37 10 atm 13.6 atm101328

p p g z z�� � � � � � � �

�� �

1

2

3

40m

100 m

v

- 268 -

Física Universitaria: Problemas de Física Dinámica de los fluidos ideales. M32.2

2. En un tramo de una tubería horizontal existe un estrechamiento que reduce su sección a la mitad. Si por la misma circula un lí-quido y la diferencia de presión que se origina entre un punto de la tubería y el punto donde existe el estrechamiento equivale a la presión que produciría una columna del mismo líquido de altura h, ¿qué velocidad tendrá el líquido en la tubería?

Aplicamos la ecuación de Bernoulli entre los dos puntos, supuestos a la misma cota: 2 2 2 21 1 1

A A B B A B B A2 2 2 ( )p v p v p p v v gh� � � �� � � � � � �

de modo que 2 2B A 2v v gh� � [1]

Tenemos en cuenta la ecuación de continuidad:

AA A B B B A A

B

2SSv S v S v v v� � � [2]

y sustituyendo este resultado en la expresión [1], tenemos 2 2 2

A23A AA4 3 2 vv ggh hv v� � � �

AB

- 269 -

Física Universitaria: Problemas de Física Dinámica de los fluidos ideales. M32.3

3. Para medir la velocidad del agua que circula por una tubería, se intercala en ésta un venturímetro cuyos diámetros en el tramo principal y en el estrechamiento se encuentra en la relación 5:1. La diferencia de pre-sión entre el tramo principal y el estrechamiento resulta ser de 0.35 atm. ¿Cuál es la velocidad?

2 1

v2 v1

Aplicamos el teorema de Bernoulli entre los puntos 1 y 2:

�1 22 2 2 21 1 2 2 2 1

21 1 [1]2 2

p pp v p v v v� �

��

� � � � �

con

1 23

kg 101325 Pa1000 0.35atm 25331.25 Pa m 1atm

p p�� � � � �

Por otra parte, de la ecuación de continuidad, se sigue: 2 22

1 2 2 21 1 2 2 2

2 1 1 1

1 1 [2]5 25

v S D Dv S v Sv S D D

� � � ��� ���� � � � � �� ��� �� ��� � �

Resolviendo el sistema de ecuaciones dado por [1] y [2], se obtiene 2 22 1

2 11 2

m m0.34 8.4370.9

25 s svv v

v vv

�� � �� � � �� ���

- 270 -

Física Universitaria: Problemas de Física Dinámica de los fluidos ideales. M32.4

4. En el dispositivo de la figura, un fluido ideal (densidad, %) circula por una tubería de sección constante con una velocidad v. La tube-ría lleva acoplados dos tubos piezométricos, que permiten medir la presión estática y la presión dinámica, respectivamente. Sea %m la densidad del líquido manométrico. Expresar la velocidad v del flui-do en función de la diferencia de cotas h que se indica en la figura.

Aplicamos reiteradamente la ecuación de Bernoulli entre los puntos A�B�1�2�C�A y sumamos miembro a miembro todas las ecuaciones (hemos procedido a sumas parciales para mayor claridad).

A A B B

B BA A 1 1

1 m 1 2 m 2

1 1

2

A B

B 1 1 2 2 C

pp gz p gzp gz p gz

p gz

gz p gz

p gz p gz

� �

� �

� �� �

� � � � ���� � � � ��

� � �

� � �

2 C C

2C C A A

22 2 A A 1C A

12

2

p gz

p gp gz p

z pgz

gz vv

� � �� � �

�� � ������ � � �

� � � � ���

Obtenemos, después de sumar m.a.m. todas las ecuaciones,

� �

� �

2m 1 2 1 m 2

2m 1 2 1 2

2m 1 2

1 +2

12

12

gz gz gz gz v

g z z g z z v

g z z v

� � � � �

� � �

� � �

� � �

� � � �

� � �

Y, finalmente, con h = z1 – z2, resulta

2 m2 1v gh ��

� ���� � �� �� ���

v

%C

A B

%m 2

1 h

z

- 271 -

Física Universitaria: Problemas de Física Dinámica de los fluidos ideales. M32.5

5. Por una tubería circula un caudal de 208 L/s de agua. En la tubería se ha instala-do un medidor de Venturi, con mercurio en su interior, tal como se representa en la figura. Si las secciones de la tubería en 1 y 2 son 800 y 400 cm2, respectiva-mente, calcular el desnivel h que se produce en el mercurio.

Aplicamos la ec. de Bernoulli entre 1 y 2:

�2 2 2 21 1 2 2 1 2 2 1

1 1 1 [1]2 2 2

p v p v p p v v� � �� � � � � �

Aplicamos reiteradamente la ec. hidrostática en el camino 1AB2:

� 1 11 A p gz� � Ap�

�A

AA B

gz

p

��

m A Bgz p�� �

�m B

BB 2

gz

p

��

B 2 2gz p gz� �� � �

y sumando m.a.m.

� � �1 m A B A m B 2

1 2 m B A m [2]p gz gz gz gz pp p g z z gh� � � �

� � � �

� � � � �

� � � � � �

Igualamos las ecuaciones [1] y [2]

� �2 2

2 2 2 1m 2 1

m

1 [3]2 2

v vgh v v hg

�� � �

� ��

� � � ��

Aplicamos las ecuación de continuidad entre los puntos 1 y 2:

1 1 2 2 1 21 2

[4]v S v S v vS S

� � � �� �

y sustituyendo las velocidades en la ec. [3]:

2 1

2

2 2m 2

2 2

m 1

1 1 122S S

hS Sg g

�� ��

��

� �� � � �� �� �� �� �� � �� �� �� �� �� �� �� �

� ��� ��� ��� ��� �� �� � � �

Aplicación numérica: 2

2 2

1000 1 1 0.208 0.082 m13600 1000 0.04 0.08 2 9.8

82 mmh� ���� � � ��� ��� � �

1 2

h

�m

AB

1 2

h

- 272 -

Física Universitaria: Problemas de Física Dinámica de los fluidos ideales. M32.6

6. Para medir la velocidad del agua que circula por un arroyo, se dispone de un tubo en L, como se muestra en la figura adjunta. ¿Cuál será la velocidad de la corriente si el agua asciende por el tubo vertical hasta una altura de 40 cm por encima de la superficie libre del agua?

Aplicamos la ecuación de Bernuilli entre AB, BC y CD, teniendo en cuenta que C es un punto de estancamiento:

atm B

2B C

C atm

12

p p gh

p gh v p gh

p gh p gh

� � �

� �

)� �

) )� � � �

)� � �

Sumando miembro a miembro las tres ecuaciones anteriores, tenemos

21 22

v gh v gh� �� �

Y sustituyendo los valores numéricos: 2 9.8 0.40 7.84 2.8 m/sv � � � � �

h

v

h

v

A

D

C B

- 273 -

Física Universitaria: Problemas de Física Dinámica de los fluidos ideales. M32.7

7. En un pulverizador de pesticida se sopla aire sobre el extremo supe-rior de un tubito abierto por sus dos extremos, estando el extremo in-ferior sumergido en un recipiente que contiene líquido de densidad 0.92 g/cm3. ¿Cuál deberá ser la velocidad mínima del aire que pueda elevar el líquido 10 cm para ser dispersado? (Densidad del aire, 1.25 g/L).

Aplicamos la ecuación de Bernoulli entre A y B, a lo largo de la corriente de aire

2A aire atm

12

p v p�� � (1)

Aplicamos la ecuación fundamental de la estática de fluidos entre los puntos 1 y 2 (en el lí-quido)

atm 2 liqp p gh�� � (2)

Sumamos miembro a miembro (1) y (2) y tenemos en cuenta que A 2p p� ,

2aire liq

12

v gh� ��

de modo que

liq

aire

2 2 920 9.8 0.11

38 m/.

s25

ghv

�� � �

� � �

aire

líquido

A B

1

2

- 274 -

Física Universitaria: Problemas de Física Dinámica de los fluidos ideales. M32.8

8. Un tanque de grandes dimensiones abierto a la atmósfera está apoyado en el suelo y contiene agua, como se indica en la figura. Se hace un agujero de pe-queño diámetro a una altura h en la pared del tanque. a) Deducir la velocidad de salida del agua por el orificio. b) Determinar el alcance horizontal x del chorro de agua. c) Calcular la altura a la que debería hacerse el orificio para que el alcance fuese máximo.

a) Aplicamos el teorema de Torricelli para cal-cular la velocidad de salida del agua por el orifi-cio:

�0 2v g H h� �

b) Escribimos las ecuaciones paramétricas de la trayectoria, que corresponde a la de un movi-miento uniformemente acelerado; i.e., una tra-yectoria parabólica,

02

220

1 22

x v tgy h xvy h gt

� ���� � ��� � ����

El alcance se determina haciendo y = 0, de modo que

�2alc 02

0

20 22g hy h x x v h H hv g

� � � � � �

c) Puesto que el alcance es función de la posición h a la que se encuentra el orificio, determi-naremos su valor máximo imponiendo la condición de extremo (máximo o mínimo) de la fun-ción:

� �alc 1

2d 2 22 0d 2v H h H h h Hh h H h h H h

� �� � � �

� �

x

h H

h

H

xalc

v0

x

y

- 275 -

Física Universitaria: Problemas de Física Dinámica de los fluidos ideales. M32.9

9. Por la tubería de la figura, de secciones S1 = 40 cm2 y S2 = 20 cm2, circula un caudal de agua de 3�10-3 m3/s. Los tubos piezométricos están llenos de aceite de den-sidad 800 kg/m3. Se observa que el nivel del aceite en los piezómetros tiene igual cota. ¿Cuál es el desnivel h entre los dos tramos de tubería?

Nota: despreciar el valor de los radios de la tubería en la expresión de las alturas.

Datos: S1 = 40 cm2, S2 = 20 cm2, Q = 3�10-3 m3/s, %m = 800 kg/m3, % = 1000 kg/m3

11 1 2 2 2 1 1

23

1 24

(ec. continuidad) 2

0.3 1040 1

75 m/ s 1. 0 m/ s0

5

SQ v S v S v v vS

v v�

� � � �

�� � �

Aplicamos la ec. de Bernoulli entre 1 y 2:

2 2 2 21 1 2 2 2 1 2 1 1

1 1 1 3(4 ) [1]2 2 2 2

p gh v p v p v p p gh v� � � � � �� � � � � � � � �

Aplicamos la ec. de Bernoulli entre A-1 y entre B-2:

atm m 1 m2 1 m

atm m 2

[2]p gH p gh

p p ghp gH p

� ��

� � � ��� � ��� � ���

Igualando las ecuaciones [1] y [2] tenemos:

2 21 m m 1

21

m

3 3( 32( )

)2 2

gh v gh gh vhvg

� � � ���

� ��

� � � ��

de modo que 23 1000 0.75 0.43 m

2(1000 800) 9.843 cmh � �

� � �� �

Otro modo operatorio: Aplicamos la ecuación de Bernoulli a la trayectoria A-1-2-B:

atm m 1 m

2 2 21 1 2 2 2 1

2 atm m

A 11 1 11 2 (4 )2 2 2

2 B

p gH p gh

p gh v p v p v

p p gH

� �

� � � �

� � � ������ � � � � � ������ � ���

y sumamos m.a.m. estas tres ecuaciones:

2 2 21 m 1 m 1

21

m

1 1 3(4 ) (2 2 2(

)2

3)

gh v vhg

gh v gh v �� �

� � � � � � �� � � � ��

h

S1

S2

A B

1

2

H

n ivel d e referencia

- 276 -

Física Universitaria: Problemas de Física Dinámica de los fluidos ideales. M32.10

10. Un depósito de grandes dimensiones abierto a la atmósfe-ra desagua por una tubería troncocónica que lleva incor-porado un tubo en U invertida cuyo extremo inferior está sumergido en otro depósito que contiene el mismo líqui-do. Determinar la altura h a la que asciende el líquido en función de la distancia x.

Calculamos la velocidad de salida en 2 mediante el teorema de Torricelli:

2 2v gH�

Aplicamos la ecuación de Bernoulli y la condición de continuidad entre los puntos X y 2:

2 2X X atm 2

2X X 2 2 X 2

X

1 12 2X 2

p v p v

SS v S v v vS

� ���� � � ���� ��� � �����

de modo que, sustituyendo vX y v2 en la primera ecuación, tenemos 2 2

2 2 22 2X atm 2 X 22 2

X X

1 1 1 1 02 2

S Sp p v v v gHS S

� � �� � � �

� � � � � �� � � � � � � .� �� � � � �� �

Aplicamos la ecuación fundamental de la Estática de Fluidos entre B y A, teniendo en cuenta que la presión en A es igual a la presión en X:

atm X X atmp p gh p p gh� �� � � � �

Igualando las dos ecuaciones anteriores, obtenemos 2 22 22

22

X2X

2X

1 1 01S Sgh gH h HS S

Sh HS

� �� �� ��

� � � �� � � �� � � � � � � � � � �� �

� *� ��

ya que S2 < SX, por lo que realmente el agua no asciende por el tubo, sino que “desciende”, como se ilustra en la figura. Tan solo nos queda expresar SX en función de la distancia x indicada en la figura. El radio de la tubería en función de la distancia x es

1 2 1 2X 1 1 2conR R S SR R x R R

L � ��

� � � �

de modo que, después de un laborioso desarrollo, obtenemos

1 1 2 2 1 2 12 2X X 12

22

S S S S S S SS R x x S

L L�

� � �� � � �

h

H

S1S2

1 2

B

X

A

x

L

hB

X

A

- 277 -

Física Universitaria: Problemas de Física Dinámica de los fluidos ideales. M32.11

11. El depósito de grandes dimensiones de la figura está abierto a la atmós-fera y desagua por la tubería que se indica. La sección transversal de la tubería en los puntos 2 y 3 es de 300 cm2 y en el 4 de 100 cm2. Calcu-lar: a) El caudal de agua que fluye por la sección 4. b) La presión en el punto 3. c) La cota del punto 2 respecto a los puntos 3 y 4 para que la presión en aquél sea de 1.2 atm.

a) Dado que el depósito está abierto a la atmósfera y que la tubería desagua a la atmósfera, podemos aplicar directamente el Teorema de Torricelli:

4 1

44 4

34 0.15

2 2 9.80 12 235.2 15.3 m/s

100 10 15.3 3 m /s

v gz

Q S v �

� � � � � �

� � � � �

b) Aplicamos la ec. de continuidad y la ec. de Bernoulli entre 3 y 4:

43 3 4 4 3 4 4 4

3

2 2 2 2 23 3 atm 4 3 atm 4 3 4

100 1 5.1 m/s300 3

1 1 1 42 2 2 9

SS v S v v v v vS

p v p v p p v v v� � � �

��� � � � � �������� � � � � � � ����

de modo que 2

3 at 3m 44 4 1000 235.2 104 533 Pa 1.032 atm 2.0329

m9

atp p v p�� � � � � � � �

c) Aplicamos la ec. de Bernoulli entre 2 y 3: �3 2

2 2 3 2

2.032 1.2 1013281000 9

8.6 m.8

0p pp gz p zg

��

� ��� � � � �

x 1

x

x x

2

3 4

12 m

- 278 -

Física Universitaria: Problemas de Física Dinámica de los fluidos ideales. M32.12

12. Clepsidra (reloj de agua). Determinar la forma que debe darse a un recipiente con simetría de revolución alrededor de un eje vertical para que al vaciarse por un orificio situado en su fondo sea constante la veloci-dad de descenso del nivel del agua que contiene.

Aplicamos Bernoulli entre la superficie libre (1) y el orifi-cio de salida (2):

2 2 2 2atm 1 atm 2 1 2

1 1 22 2

p gz v p v gz v v� � �� � � � � �

Y la ecuación de continuidad en esos mismos puntos: 2

2 21 0 2 1 0 2 2 12

0

rSv S v r v r v v vr

� �� � �

Despejando la cota z en la primera ecuación y sustituyendo el valor de v2 obtenido en la segunda, tenemos

�2 4

2 2 412 1 4

0

1 12 2

v rz v v ar bg g r

� ��� �� � � � � �� �� ���

Puesto que la velocidad de descenso del nivel del agua es constante, esto es, independiente del tiempo, la clepsidra es un auténtico reloj de agua. Podemos utilizar cualquier software matemático para representar en 3D la clepsidra.

z

r S0

S r 1

2

- 279 -

Física Universitaria: Problemas de Física Dinámica de los fluidos ideales. M32.13

13. Un depósito abierto, cilíndrico, de eje vertical y sección recta S1 está lleno de agua hasta una altura H por encima de su fondo. Determinar el tiempo necesario para que se vacíe el depósito a través de un orificio bien perfilado, de área S2, practicado en su fondo. Aplicación numérica: S1 = 2 m2, S2 = 10 cm2, H = 3 m.

Aplicamos la ecuación de Bernoulli entre la superficie libre del agua en el depósito y el orificio de salida

2 2 2 2atm 1 atm 2 1 2

1 10 2 [1]2 2

p gz v p v gz v v� � �� � � � � � �

Aplicamos la ecuación de continuidad entre esos dos mismos pun-tos

11 1 2 2 2 1

2

[2]SS v S v v vS

� �

Eliminamos la velocidad de salida (v2) entre estas dos ecuaciones, tenemos

2 22 2 21 11 1 1 12 2 2

2 2 1 2

22 1 21

S S gzgz v v v gz v AzS S S S

� � � �� �� �� �� � � � � �� �� �� �� �� � �� �

De modo que 1ddzv Azt

� � � con 2 21 2

21

gAS S

��

Integramos la ecuación diferencial para obtener t(z):

1/2 1/2

0

d 1 1d d 2

2

z zt z

HH H

zt z z zAz A A

t H zA

�� � � � � �

# � �

� � �

Cuando se depósito se vacíe, será z = 0, lo que requiere que transcurra un tiempo t0 tal que 2

10 2

2

22 1SH HtA g S

� ��� �� � �� �� ���

Aplicación numérica:

�6 2

22 2 41 2

2 2 9.8 4.9 10 m/s1 2 10 10 1

gAS S

��� � � �

� � �

0 6

32 1565 s 26 min 5 s4.9 10

t �� � ��

H

z

S1

S2

v1

v2

- 280 -

Física Universitaria: Problemas de Física Dinámica de los fluidos ideales. M32.14

14. Un sifón es un dispositivo que se utiliza para extraer líquido de un depósito. Su forma de operar se muestra en la figura adjunta. El extremo del tubo que está sumergido en el líquido puede es-tarlo a cualquier profundidad. Naturalmente, para que el sifón funcione deberá estar inicialmen-te lleno de agua; pero una vez que está lleno, el sifón succionará líquido del depósito hasta que el nivel en éste descienda por debajo del nivel del extremo del tubo abierto al aire libre. Supon-gamos que el líquido sea agua a 15.5 �C (ps = 13 Torr) y despreciemos totalmente la fricción. a) Determinar la velocidad de salida del líquido por el extremo inferior del tubo del sifón. b) ¿Cuánto vale la presión absoluta en el punto más alto del tubo? c) ¿A qué altura máxima so-bre el extremo inferior del tubo puede estar el punto más alto del tubo sin que el sifón falle por cavitación?

a) Aplicamos la ecuación de Bernoulli entre A-C

atmp A atm0gz p�� � � 2A

10 22

v v gz�� � �

Esto es, la velocidad de salida viene dada por el Teorema de Torricelli, siendo h la diferencia de niveles entre la superficie libre del líquido (A) y la salida del tubo de desagüe (C). b) Aplicamos la ecuación de Bernoulli entre B-C

2B B

12

p gz v� �� � 2atm

102

p v�� � �

B atm B 0p p gz�# � � .

c) Se presentará cavitación, esto es, la formación de burbujas de vapor en el seno del líquido, causada por las variaciones que este experimenta en su presión, si la presión en B es inferior a la presión de vapor saturante del agua:

�B atm

B atm B s B

760 12.788 101328 10.2 m1000 9.8 760

p pp p gz p zg

��

��� � . * � �

A

B

C z=0

- 281 -

Física Universitaria: Problemas de Física Dinámica de los fluidos ideales. M32.15

15. Un depósito de grandes dimensiones desagua mediante un tubo sifón de sección S y terminado en un estrechamiento de sección S/4, como se indica en la figura. a) Determinar la presión en A. b) Calcular valor máximo de h3 para que el depósito continúe desaguando.

a) Aplicamos la ecuación de Bernoulli entre B-C: 21

atm 2 3 atm C22C 2 3

( ) 0 0

( )

p g h h p v

v g h h

� �

� � � � � �

� �

Ecuación de continuidad entre A-C:

1A C A C44

Sv S v v v� " �

Ecuación de Bernoulli entre B-A: 21

atm 2 3 A 1 2 3 A22

C1 1atm A 1 A 1 2 32 16

( ) ( )

( )4

p g h h p g h h h v

vp p gh p gh g h h

� � �

� � � �

� � � � � � � "

� ���� � � � � � � "�� ���

1 21 1A atm 1 2 3 atm 1 2 316 16( ) ( )p p gh g h h p g h h h� � �� � � � � � � �

b) Para pA = 0, (en realidad pA = ps, presión de vapor saturante), será:

atm atm11 2 3 3 1 216

16( ) 16p ph h h h h hg g� �

� � � " � � �

A

h1

h2

h3

B

Nivel de ref.

- 282 -

Física Universitaria: Problemas de Física Dinámica de los fluidos ideales. M32.16

16. Disponemos de un depósito de agua de grandes dimensiones y abierto a la atmósfera que desagua a la atmósfera a través de un tubo vertical, de sección constante, que está acoplado a su fondo, tal como se indica en la figura. a) Calcular la velocidad de salida del agua por el tubo. b) Expresar la presión en función de la cota z medida a partir del extremo inferior del tubo, representarla gráficamente y explicar la “caída de presión” en la entrada del tubo de desagüe.

a) Aplicamos la ec. de Bernoulli entre 1 y 3:

atmp atmgH p�� � 21 22

v v gH�� �

que es el mismo resultado que nos proporciona el Teorema de Torricelli. b) Aplicamos la ec. de Bernoulli entre el punto 1 y un punto genérico contenido en el depósito (de cota h < z < H):

�atm atmp gH p gz p p g H z� � �� � � � � �

Aplicamos la ec. de Bernoulli entre el punto 3 y un punto gené-rico contenido en el tubo (de cota 0 < z < h):

2 2atm atm

1 12 2

p v p gz v p p gz� � � �� � � � � �

En la representación gráfica, observamos que la presión presenta una discontinuidad (caída brusca) en la entrada del tubo, ya que

�atm

atm

( )( ) ( )

( )

p h p g H hp h p h gH

p h p gh

��

�� �

�� � � �� � ��� � ���

que es consecuencia de haber considerado despreciable la velocidad del agua en el depósito, incluso en las proximidades de la entrada del tubo, lo que es una aproximación que no se ajus-ta a la realidad.

H h

Hh

1

2

3

z z

v

Hh z

p

patm

patm-�gh

patm+�g(H-h)

�gH

- 283 -

Física Universitaria: Problemas de Física Dinámica de los fluidos ideales. M32.17

17. Calcular la longitud x del tubo de salida del agua del depósito de la figura para que la presión en B sea 1/n de la presión en A (presión atmosférica), teniendo en cuenta que SA.. SB.

Método 1 Simplemente, aplicamos el Teorema de Bernoulli entre B�C, teniendo en cuenta que la velocidad es la misma en ambos puntos:

2 2atmatm

atm

atm

1 12 2

1

1

p gx v p vn

ngx pnpnx

n g

� � �

� � � �

� �

�# �

Método 2 Aplicamos el Teorema de Bernoulli entre A�B y A�C

2atmatm

2atm atm

12

12

pp g h x gx vn

p g h x p v

� � �

� �

��� � � � � ������� � � � �����

Restando miembro a miembro estas dos ecuaciones: 2 2atm atm

atm atm1 1 1 12 2

p pn ngx v p v gx p xn n n g

� � � ��

� �� � � � � # �

Método 3 Obsérvese que la aplicación del Teorema de Bernoulli entre A y C equivale a la aplicación del Teorema de Torricelli para calcular la velocidad en el desagüe, de modo que el problema también puede plantearse de modo que sigue:

2atmatm atm

atm2

1( )2

2 ( )

pp gh v pp gh g h xnnv g h x

� �� �

��� � � ��� � � � ���� � ����

De modo que

atmatm

1 1 pn np gx xn n g

��

� �� �

C

A

B

x

z=0

h

v

- 284 -

Física Universitaria: Problemas de Física Dinámica de los fluidos ideales. M32.18

18. Desde un estanque de grandes dimensiones se desagua a un gran depósito mediante una tubería de sección S1. Desde este depósito, a su vez, se desagua mediante una tubería de sección S2, tal como se indica en la figura. a) Determinar la relación entre los diferentes da-tos para que permanezca constante el nivel del depósito intermedio. b) Ídem para que sean idénticas las velocidades de desagüe en am-bas tuberías.

Empezamos determinando las velocidades de desagüe en ambas tuberías. Aplicamos la ecuación de Bernoulli entre

20 1 2 3 B 1

B 0 3

1A-B ( )2

B-C

p g h h h p v

p p gh

� �

� � � � �

� �

����

����

que sumadas miembro a miembro, nos conducen a 2

1 2 1 1 1 21( ) 2 ( )2

g h h v v g h h� �� � � �

Aplicamos la ecuación de Bernoulli entre 2

0 4 5 0 2 2 4 51C-D ( ) 2 ( )2

p g h h p v v g h h� �� � � � � �����

Deberán ser iguales los caudales en ambos desagües; i.e., 2 2

1 1 2 2 1 1 2 2 4 5( ) ( )S v S v S h h S h h� � � �

Igualamos las velocidades; i.e.,

1 2 1 2 4 5( ) ( )v v h h h h� � � �

h1

h2

h3

h5

h4

A

C

B

D

- 285 -

Física Universitaria: Problemas de Física Dinámica de los fluidos ideales. M32.19

19. Trasvasando aceite. Dos depósitos de grandes dimensiones, abiertos a la atmósfera, contienen aceite de oliva (0.918 g/cm3), existiendo un desnivel entre las superficies libres del aceite en ellos de 10 m. Los depó-sitos están intercomunicados mediante una tubería horizontal, de 120 mm de diámetro, con entradas bien perfiladas por debajo de los niveles de aceite en cada depósito. a) Determinar el caudal que circula por la tu-bería. b) Calcular la potencia nominal de la bomba que se necesitará (70% de rendimiento) para conseguir el mismo caudal en sentido inverso.

a) Calculamos la diferencia de presiones entre los extremos de la tubería que comunica los depósitos:

0 1 22 4

0 5 4

(obvio) p gz p

p p ghp gz p�

��

�� � �� � ���� � ��

Puesto que no hay pérdidas de energía, la caída de presión entre (2) y (4) será igual al incremento de energía cinética en la tube-ría; esto es,

22 4

1 2 14 m/s2

p p gh v v gh� �� � � � �

Resultado al que también podemos llegar directamente aplican-do el teorema de Torricelli. El caudal será

2 30.06 14 0.158 m /s 158 /sQ Sv �� � � � � � � b) La bomba deberá suministrar una potencia P a la corriente fluida, dada por

P w Qw� �� �

con 4 24 2( ) ( )p pw e e gh

��

� � � � �

de modo que ( )P gh Q�� �

O sea 918 9.8 10 0.158 14214 W 14.2 kWP � � � � � � � � �

nom14.2 20.3 kW0.70

P � �

aceite

2

5

4

1

h

3

v

aceite

2

5

4

1

h

bomba

w

- 286 -

Física Universitaria: Problemas de Física Dinámica de los fluidos ideales. M32.20

20. El depósito de grandes dimensiones de la figura contiene un líquido ideal que desagua por la tubería de sección variable que se indica situada a una profundidad H. a) Calcular la veloci-dad del líquido en cada uno de los tres tramos de la tubería. b) Calcular y di-bujar la altura que alcanza el líquido en cada uno de los tubos piezomé-tricos. c) Dibujar de forma aproxi-mada la altura que tendría el líquido en los tubos piezométricos si el líquido fuese viscoso.

a) Teorema de Bernouilli entre M y N: 2

atm atm S S10 0 22

p gH p v v gH� �� � � � � �

Ecuación de continuidad:

2S S 2S

3S S 3S

S

S

1 1

2 21 1

3 3

2 2

3 2

v S v S v v gH

v S v S v v gH

��� � ������� � �����

b) Toma de presión estática en X :

X atmX atm X

p pp p gh hg

���

� � �

Teorema de Bernouilli entre M y X: 2 2

atm X X X atm X1 1

2 20 0p gH p v p p gH v� � � �� � � � � � � � �

Combinando las dos ecuaciones anteriores: 2X

X 2gvh H� �

Tubos A y B: 23S

A1

2g899

h H H H Hv� � � � �

Tubos C y D: 22S

C1

2g344

h H H H Hv� � � � �

Tubo E: 2S

A 2g0vh H H H� � � � �

H

A C D E

S

B

3S 2S

M

N

- 287 -

Física Universitaria: Problemas de Física Dinámica de los fluidos ideales. M32.21

10m

1m 2m

2m

S03S0 A 21

3S0

B21. Un depósito de grandes dimensiones desagua a la atmósfera mediante el sistema de tuberías que se representa. Determinar la velocidad y el caudal del agua en cada tramo de tubería, así como la presión en el punto A. Datos: S0 = 100 cm2.

Consideraremos los tres tramos del sistema numerados (1, 2 y 3) tal como se representa en la figura. Determinamos las velocidades en los tramos 2 y 3 aplicando el teorema de Torricelli:

2 2

3 3

15.33 m/s2 2 9.8 12

2 2 9. 16.56 /1 m s8 4

v gh

v gh

� � � � �

� � � � �

y los caudales en estos tramos son 3

2 2 0

33 3 0

153.3 L/s

165.6 L/

15.33 0.01 0.1533 m / s

16.56 0. s01 0.1656 m / s

Q v S

Q v S

� � � � �

� � � � �

El caudal y la velocidad en el tramo 1 son: 3

1 2 3

11

0

318.9 L/s0.3

10.63

189

m

/

/

s

s

m =

3

Q Q Q

QvS

� � �

� �

Aplicamos la ecuación de Bernoulli entre A y B: 2

A A A B12

p gz v p� �� � � 2B B

12

gz v� �� �

Despejando pA , teniendo en cuenta que pB = patm y que vB = 0, resulta

� 2A atm B A A

12

p p g z z v� �� � � �

�2A atm

11000 9.8 11 1000 10.63 107800-56498 = = 0.55130 06 at2

2 Pa mp p� � � � � �

A 152627 Pa = 1.506 atmp �

- 288 -

Física Universitaria: Problemas de Física Dinámica de los fluidos ideales. M32.22

22. Una bomba de 5 kW de potencia, situada en el fondo de un depósito de grandes dimensiones, eleva agua hasta los niveles A y B a través del sistema de tuberías representa-do. El agua sale en el nivel A con una velocidad de 5 m/s. a) Determinar la velocidad de salida en B. b) Calcular los caudales suministrados en A y B.

a) Aplicamos la ecuación de Bernoulli entre C-A y C-B:

2 2C C C atm A A

2 2A A B B

2 2C C C atm B B

1 1CA 1 12 21 1 2 2CB2 2

p v gz p v gzv gz v gz

p v gz p v gz

� � � �� � � �

� � � �

��� � � � � � ��� � � ���� � � � � � ����

de modo que �2 2 2 2 2

B2

B A A B B 5 2 9.8 ( 1) 5.4 m 2.3/ 2 m/s sv v g z z v v�� � � �� � � � � �

Ecuación de continuidad en la bifurcación:

A BC A B C

23 2 3.22 m/s3

v vSv Sv Sv v �� � � �

b) La potencia proporcionada por la bomba será P =�wQ, donde Q es el caudal total (en C) y w es el trabajo específico realizado sobre el fluido, tal que, aplicando la ec. de Bernoulli (energías específicas) entre O y A, i.e., 0 Ae w e� � , incluyendo la bomba, tenemos

� 2 2A 0 atm atm A 0

1 12

w e e p p v v�

� � � � � � � � � �

2A 0 C 0 A A 0

2 2A A 0

1 12

1 11000 5 1000 9.8 5 61500 Pa2 2

g z z p p v g z z

w v g z z

� � �

� � � � � � �

� � � � � � � � �

El caudal principal será 3

C5000 0.0813 m /s 81.3 L/s61500

PQw�

� � � �

La sección de la tubería principal será: 2 3 2C

CC

0.08133 0.0253 m 25.3 10 m3.22

QS Sv

�� � � � � �

De modo que las secciones y caudales en las tuberías secundarias son: 3 2 3 2

A A A A3 2 3 2

B B B B

8.42 10 m 42.1 10 m

16.

42.1 L/s

38 10 m 39.1 1 9.1 L/s0 m

S Q S vS Q S v

� �

� �

� � � � � �

� � � � � �

6 m5 m

3S

S 2S 15 m

A B

C

O

N.R.

- 289 -

Física Universitaria: Problemas de Física Dinámica de los fluidos reales. M33.1

1. Una lámina de aluminio (% = 2.70 g/cm3) desliza por un plano inclinado bajo la acción de la gravedad y de la fuerza viscosa ejercida sobre ella por una fina película de aceite lubricante SAE-30 (> = 250 cP) de 0.25 mm de espesor, depositada sobre el plano. Las dimensiones de la lámina son 10�4�1 cm3 y el ángulo de inclinación del plano respecto de la horizontal es de 5º. Calcular la velocidad límite que alcanza la lámina en su descenso.

Datos: �Al = 2.70 g/cm3

h = 9.25 mm = 0.025 cm S = 4�10 = 40 cm2

� = 250 cP = 2.5 P

El flujo del líquido en el espacio comprendido entre las dos superficies se reduce a un flujo de Couette, como se ilustra en la figura inferior. Aplicamos la ley de la viscosidad de Newton:.

visc= v vf S Sh h

; > ; >� � �

ya que el esfuerzo cortante está uniformemente distribuido sobre la cara de la lámina en contacto con el aceite. Bajo la acción de las fuerzas que actúan sobre la lámina (indicadas en la figura superior), ésta se acelera, aumentando continuamente su velocidad de bajada y la resistencia viscosa a su movimiento, hasta que finalmente se alcanza una cierta velocidad límite. Las ec. del movimiento se escribe en la forma:

límvisc lím

sensen 0 senv mghmg f ma mg S vh S

�� � >>

� � � � � � �

con Al 2.7 (10 4 1) 108 gm V%� � � � � � , de modo que

lím108 980 0.025 sen 5º 2.31 cm

2.5 (10 4)v � � �

� �� �

v

h

v

mg

NFvisc

- 290 -

Física Universitaria: Problemas de Física Dinámica de los fluidos reales. M33.2

2. Un cilindro macizo de radio R = 10 cm y altura h = 10 cm gira en el interior de otro cilindro hueco de radio R + 2 mm y altura h + 4 mm. En el espacio entre ambos cilindros existe un líquido de viscosidad > = 84 cP que rodea totalmente el cilindro interior. Determínese el momento que hay que aplicar al eje del cilindro móvil y potencia necesaria para mantener una velocidad de rotación constante de 1000 r.p.m.

Datos: 21000 104.72 rad/s60104.72 10 1047.2 cm/sv R

��

� �

� � � �

El flujo del líquido en el espacio entre los dos cilindros se reduce a un flujo de Couette, como se ilustra en la imagen inferior. Aplicamos la ley de la viscosidad de Newton:

21047.2= 0.84 4398.23 dyn/cm0.2

vR

� � � �4

Este esfuerzo cortante está uniformemente distribuido sobre la superficie lateral del cilindro interior móvil. En consecuencia, el momento dinámico con respecto al eje de rotación será:

2

7

2 7(2 ) 2 2 10 10 4398.23 2.76 10 dyn c2.76 10 dyn cm 2

m.76 N mM

M Rh R R h� � � � �

� � � � � �

� �

� � �

# � �

que es igual al momento que deberemos aplicar para mantener el cilindro interior en movimiento. La potencia necesaria para mantener constante la velocidad de rotación será:

2.76 104.72 289 WP M�� � � �

R

R+�R

v=�R

- 291 -

Física Universitaria: Problemas de Física Flujo viscoso. M34.1

1. a) Determinar la dirección del flujo en la tubería de sección cons-tante representada en la figura adjunta. b) Calcular el caudal y el número de Reynolds del flujo. El fluido es aceite de oliva a 20 ºC.

Datos: densidad, 0.918 g/cm3; viscosidad, 84 cP;

Datos: 42

kg1 9.8 10 Pa; 0.84 P 0.084 Pa.scm

�� � � �

a) Determinamos la pérdida de carga entre los puntos 1 y 2:

�2 21 21 2 1 2

12l

p pH e e gh v v��

� � � � � �

1 2

41 2 9.8 10 J9.8 12sen 30º 106.81 58.84 47.97918 kg

p p gh��

� � �

� � �� � � � � � � �

La dirección del flujo es la de la pérdida de carga (disminución de energía); por consiguiente se dirige desde abajo hacia arriba, (2) � (1) . b) Expresamos la pérdida de carga (Hl) en términos de presión (pérdida de presión, Hp):

2

kg�' 918 47.97 44039 Pa 0.45cmp lH H�� � > � � � �

Aplicamos la ley de Hagen-Pouiseuille 4 4 3

3� ' 0.045 44039 m L4.4 10 4.4128 128 0.084 12 s

DQL s

� ��

�> �� � � � �

La velocidad media del fluido en la tubería es: 2

2

4 2.76 m/s4D QQ SV V V

D�

�� � � �

Calculamos el número de Reynolds del flujo:

1360DV��

� �R

de modo que se trata de un flujo laminar, por ser R < 2300.

1 1

1 22 2

98000 918 9.8 6 151978 Pa� 44022 Pa

196000 Pa p

p ghH

p��> � � � � � � ��� ) � > �> � > ���> � ���

1 kg/cm2

2 kg/cm2

12 m

? 45 mm

30º

1

2

- 292 -

Física Universitaria: Problemas de Física Flujo viscoso. M34.2

2. El líquido de un depósito de grandes dimensiones se vacía por medio de un tubo horizontal de 250 m de largo y 20 mm2 de sección, que está situado a 15 m por debajo del nivel del líquido. Sabiendo que la densi-dad del líquido es 1 g/cm3 y su velocidad de salida es de 4.67 cm/s, calcúlese su viscosidad. Determinar si el flujo es laminar.

Datos: S = 0.20 cm2

; 4 / 4 0.2 / 0.50 cmD S � �� � � �

v = 4.67 cm/s; 30.934 cm /sQ Sv� �

l = 250 m = 25�103 cm; h = 15 m = 1.5�103 cm

Método 1 (simple): Calculamos la caída de presión en el tubo:

3 6� 1 980 1.5 10 1.470 10 bariasp gh�� � � � � � � Aplicamos la ecuación de Poiseuille:

4 4 4 6

3

� � 0.50 1.47 10 0.097 P 9.7 cP128 128 128 0.934 25 10D p D pQ

l Ql� � �

��

� � �� � � � �

� � �

Calculamos el número de Reynolds:

(laminar)1 0.50 4.67 24 2300

0.097DV��

� �� � � �@

Método 2 (avanzado): Aplicamos la ecuación de Bernoulli (sin pérdidas) entre 0 y 1:

2atm 0

12

p gh v� �� � 2 21 1 atm

1 12 2

p v p p gh v� � �� � � � �

Como el tubo horizontal desagua a la atmósfera, p2 = patm, de modo que la caída de presión entre los extremos del tubo de desagüe es,

21 2 1 atm

23

6 6

12

1 4.671 980 1.5 102

1.47 10 11 1.47 10 barias

p p p p gh v� �� � � � � �

�� � � � � �

� � � � �

Aplicamos la ecuación Poiseuille (flujo viscoso) entre 1 y 2: 4 4

1 2�128 128

p pD p DQl l

� �� �

) �� �

4 4 61 2

3

0.5 1.47 10 0.097 P 9.7 cP128 128 0.934 25 10

p pDQ l

� ��

� � � �� � � �

� � �

20 mm2

15 m

250 m

1

0

2

15m

250 m

- 293 -

Física Universitaria: Problemas de Física Flujo viscoso. M34.3

3. El agua de un depósito de grandes dimensiones se vierte por medio de un tubo horizontal de 250 m de largo y 20 mm2 de sección situado a 15 m por debajo del nivel del agua en el depósito. Calcular la velocidad y el caudal de salida del agua. Datos: viscosidad del agua = 1 mPa�s

(1) Aplicamos la ecuación de Bernoulli (sin pérdidas) entre 0 y 1:

2atm 0

12

p gh v� �� � 2 21 1 atm

1 12 2

p v p p gh v� � �� � � � �

(2) Aplicamos la ecuación Poiseuille (flujo viscoso) entre 1 y 2: 4 4

1 2

128 128p pD p DQ

L L� � �� �

) �� �

con p2 = patm, esto es, 2

1 212

p p gh v� �� � �

de modo que 4

21128 2

DQ gh vL

�� �

�� ���� � �� ���

Teniendo en cuenta que 2

2 42

4 16S SD D

QQ vS vS

� �� �

� �

resulta 2 2 2 2

2 2 2

22 2 2 2

2 2

16 1 1128 2 8 2

8 1 16 162 2 02

S Q S QQ gh ghL S L S

L Q L LQ gh Q S gh Q Q Q S ghS S

� �� �

� � ��

�� �� ��� � �

� � � �� �� �� �� � � � � �� �� �� �� �� �

� � � � � � �

y sustituyendo valores, queda la ecuación

�3 22 6

2 3 9

16 10 250 2 20 10 9.8 15 01000

12.57 10 117.6 10 0

Q Q

Q Q

���

� �

� �� � � � �

� � � � �

Y resolviendo esta ecuación de segundo grado resulta 6 3 39.34 10 m /s 9.34 cm /s 0.467 m/sQQ v

S�� � � � �

250 m

15m

1

0

2

15m

250 m

- 294 -

Física Universitaria: Problemas de Física Flujo viscoso. M34.4

4. El agua de un recipiente cilíndrico de 5 cm de diámetro abierto a la atmósfera se vacía a través de un tubo delga-do de 0.5 mm de diámetro y 20 cm de longitud. Inicial-mente la altura del agua en el depósito es de 10 cm. a) Considerando el agua como un fluido ideal, calcular la velocidad de salida del líquido en el instante inicial y el tiempo necesario para que el nivel del líquido des-cienda a 5 cm. b) Siendo la viscosidad del agua de 1 mPa�s, determinar la velocidad real de salida por el tu-bo, en el instante inicial, despreciando la velocidad del líquido en el depósito.

a) La secciones del recipiente y del tubo son: 2 2 3 21 1

A A4 4 4A2 3 2 4 21 1

BB B4 4

0.05 1.96 10 m10

(0.5 10 ) 1.96 10 m

S D SSS D

� �

� �

� �

��� � � � � � ���� � � � � � ��

Bernoulli entre A-B: 2 2 2 21 1

atm A atm B B A2 20 ]2 [1p gH v p v v v gH� � �� � � � � � �

Ecuación de continuidad entre A-B: 4B

A A B B A B BA

10S

[2]Sv S v S v v v�� � �

Sustituimos [2] en [1]:

2 8 2B B B 810 2 2 9.8 0.10

1 101.40 m/s2gHv v gH v�

�� � � � � � ��

En un instante genérico, cuando la altura del agua en el recipiente sea z, la expresión [1] se escribe en la forma:

2 2 8 4A B A A 8

4 1/2 4 1/2

0

2 d2 10 2 10 210 1 d

10 2 d d 10 2 d d 2 2( )t z

z

HH

gz zv v gz v gz v gzt

g t z z g t z z z H z

� � � �

� � � � � � � � �

� �� � � � � � � �� �� � �

4 42 10 ( ) 2 10 ( 0.10 0.05)2 2 9.8

418 sH ztg

� � � �# � � �

b) Aplicamos Bernoulli “con pérdidas” entre A-B y sustituimos la pérdida de presión por la expresión dada por la fórmula de Hagen-Poiseuille:

atmp 21A2gH v% % atmp�

4

2 21B visc B B B2

642 2B B B B

2B B B

412

34

3

128

64 10 0.20

1000 0.5 10

3

0 ( ) ( )

2 0 2 9.8 0.10 0

51.2 1.9 .83 cm/s6 0 0.0383 m/s =

[3]

lD

lD

v p gH v S v

v v gH v v

v v v

>%

>�

% % %

� �� � �

� �

� �

8 , � �

� � � � � � � �

� � � � �

Obsérvese la gran importancia de los efectos viscosos en los tubos de descarga muy delgados. Además, puesto que 2 2 3

B 0.0383 1.467 10 1.96 2v gH�� � � �� , podemos despreciar el tér-mino 21

B2 v� en la expresión [3], lo que simplifica notablemente los cálculos.

10 cm

20 cm

A

B

- 295 -

Física Universitaria: Problemas de Física Flujo viscoso. M34.5

5. Un aparato estándar para hacer demostraciones acerca de la pérdida de carga a lo largo de una tubería está constituido por un depósito de grandes dimensiones que desagua a la atmósfera a través de una tubería horizontal de longitud L y sección constante, de 8 mm de diámetro interno. La entrada de la tubería está bien perfilada y pue-den despreciarse las pérdidas menores en la misma. A lo largo de la tubería se han dispuesto dos tubos manométricos verticales, como se ilustra en la figura adjunta. En el instante en que el nivel de agua en el depósito se encuentra a 25 cm por encima de la entrada de la tubería y los manómetros indican 15 cm y 11 cm, respectivamente. a) ¿Cuál es la longitud de la tubería? b) En el instante mencionado, ¿cuál es el cau-dal en el desagüe? c) Calcular el número de Reynolds del flujo en la tubería.

Datos: densidad, 1 g/cm3; viscosidad, 1.002 cP.

a) Caída lineal de presiones. Una simple relación de seme-janza de triángulos en la figura nos permite determinar la lon-gitud de la tubería.

15 1211 24

15 360 11 132228 57 cm4

LL

L L

L

��

�� � �

� �

b) La presión estática existente a la profundidad de entrada de la tubería es

2 atm 2p p gh�� �

Aplicamos la ec. de Bernouilli sin pérdidas entre 1-2: 21

atm 1 2 2p gh p v� �� � �

Sumando m.a.m. estas dos ecuaciones:

�211 2 1 22 2 2 9.8 0.06 1.085 m/sgh gh v v g h h� � �� � � � � � � �

La disminución de “altura piezométrica” entre 1 y 2 está asociada al aumento de velocidad entre esos puntos. El caudal de salida es

2 2 3 360.008 m cm1.085 54.5 10 54.5

4 4 s sDQ v� � ��

� � � � � �

c) Número de Reynolds: 3

3 (turbulento)1000 8 10 1.085` 8663 2300

1.002 10DV��

� �� � � .

�@

La ley de Hagen-Pouiseuille no conduciría a

4 4 3 36

3

� ' 0.008 1000 9.8 0.04 m cm3.27 10 327128 128 1.002 10 0.12 s s

DQl

� ��

��

� �> � � ���� � � � ��� ��� � �

No es aplicable por tratarse de un flujo turbulento.

15 cm 11 cm

25 cm

12 cm 12 cm

15 cm 11 cm

25 cm

12 cm 12 cm

2

1

3 4 5 L

h2

h3 h4

- 296 -

Física Universitaria: Problemas de Física Flujo viscoso. M34.6

6. Un depósito de grandes dimensiones abierto a la atmósfera contiene petróleo de densidad 860 kg/m3 y vis-cosidad 7.2 mPa�s. El petróleo debe transportarse mediante una tubería lisa de acero de diámetro 0.12 m y 1 km de longitud que descarga a la atmósfera. Determinar la potencia que debe suministrar la bomba que impulse el líquido para mantener un caudal de 0.06 m3/s en los siguientes casos: a) El extremo por el que descarga la tubería está a la misma cota que el nivel del petróleo en el depósito. b) Dicho extremo está si-tuado a 30 m por encima del nivel del petróleo en el depósito.

Mediante la ecuación de Poiseuille, calculamos la caída de presión debida a la fricción (visco-sidad) en la conducción:

�f 44

128 128 0.0072 1000� 0.06 84883 Pa0.12

lp QD�

� �

� �� � � � %

Velocidad de circulación en la conducción:

2 2 2

0.06 5.3 m/s0.06

QQ v S vS �

� " � � �

Número de Reynolds del flujo:

(turbulento)860 0.12 5.31 76041

0.0072DV��

� �� � �R

(a pesar de ello, continuamos con el problema, aunque los resultados discreparán notablemete de la realidad). Estableciendo el balance energético entre los puntos 1 y 2, llamando w al trabajo específico (valor absoluto) que suministra la bomba y qvisc a la pérdida de energía específica (valor abso-luto) debido a la fricción (viscosidad) en la conducción, podemos escribir:

1 2 visc 2 1 visce w e q w e e q P w Qw� �� � � � � � � �

con

2 12 1

p pe e��

� � 2 22 1 2 1

1( ) (2

g z z v v� � � � 1 2 0 2 1 1) con - y 0p p p z z h v� � � =

22

2 1 2ve e gh# � � � f

visc� 84883 J98.70

860 kgpq�

� � �

a) En el primer apartado es h=0, por lo que: 2 22

visc5.31 J98.70 14.07 98.70 112.77

2 2 kgvw q� � � � � � �

860 0.06 112.77 5819 5.8 kWWP Qw�� � � � � % � b) En este caso es h = 30 m, por lo que

2 22

visc5.31 J9.8 30 98.70 294 14.07 98.70 406.77

2 2 kgvw gh q� � � � � � � � � � �

860 0.06 406.77 20 98 219 kWWP Qw�� %� � � � �

1 2

- 297 -

Física Universitaria: Problemas de Física Flujo viscoso. M34.7

7. a) Determinar la velocidad límite de una esferita de acero ( 7 = 7.87 g/cm3) de 2 mm de diámetro que cae en un recipiente que contiene glicerina a 20 ºC ( % = 1.26 g/cm3 , > = 1.49 Pa�s). b) Calcular el valor del nú-mero de Reynolds correspondiente a esa velocidad límite para asegurarse de que fue correcto utilizar la ley de Stokes en el apartado anterior. c) Determinar el valor máximo del diámetro de la esferita de acero que aún permita utilizar la ley de Stokes.

Ley de Stokes : 3F D v��� . Número de Reynolds: D v��

? � . Número de Reynolds crítico: crítico 1? �

a) Expresamos las tres fuerzas que actúan sobre la esferita en su movimiento de caída en el seno de la glicerina, sien E el empuje de Arquímedes y F la resistencia viscosa al movimiento:

3 34 4 63 3

mg R g E R g F Rv� � � � ��� � �

Inicialmente el movimiento es acelerado; pero cuando la esferita al-canza una cierta velocidad, la resistencia viscosa es suficientemente intensa como para, sumada el empuje de Arquímedes, compensar el peso de la esferita. A partir de ese instante la velocidad no se incre-mentará (aceleración nula):

0mg E F ma mg E F� � � � � � de modo que

3 3 2 2lím lím

4 4 263 3 9 18

R g R g Rv v gR gD� � � �� � � � ��

� �� �

� � � �

� �3

23lím

7.87 1.26 109.8 2 10

18 1.49.66 mm/s

9v �� �

� � � � ��

b) El valor del número de Reynolds en estas condiciones de flujo externo es 3 3 3

crít1.26 10 2 10 9.66 10

1.490.016

� �� � � � �? � � * ?

que, al ser muy inferior al valor crítico, nos asegura que fue correcto utilizar la ley de Stokes en el apartado anterior. c) Sustituimos la expresión de la velocidad límite en la expresión del número de Reynolds

� �

1/323 críticolím

crítico 2

1818

D v gD Dg

� � � ��� � � � �

� �� ? �� �? � � � � �� ��� ��

de modo que el valor máximo del diámetro de la esferita será:

1/32

6

18 1.49 1 7.88 mm1.26 7.89 1.26 10 9.8

8 mmD� �� � �� �� � � =�� ��� � � ��

mg

E F

v

- 298 -

Física Universitaria: Problemas de Física Ondas mecánicas. M35.1

1. Una onda esférica se propaga en un medio absorbente (1 = 0.0231 m-1), homogéneo e isótropo. La intensi-

dad de la onda a una distancia de 10 m de la fuente es de 100 nW/m2. a) ¿Cuál será la intensidad a una dis-tancia de 20 m de la fuente? b) ¿Ídem a 100 m? c) Calcular la potencia de la fuente. d) Evaluar los apartados anteriores en ausencia de absorción a fin de resaltar la importancia de ésta.

a) La intensidad de una onda esférica que se propaga en un medio absorbente viene dada por

24e rPI

r�

���

siendo P la potencia emitida por el foco y r la distancia al mismo. Aplicando esta expresión a dos distancias diferentes y dividiendo miembro a miembro:

20

20 10 20 10

10

20 2 2 220 ( ) ( )20 10 10

20 102 210 20 20

10 210

4

4

ee e

e

r

r r r r

r

PIr I r rI I

P I r rIr

� �

� � � �

��� ���� � ���� �����

20.0231(20 10) 0.23

0 21

210 nW19.84

m100 25

20e eI � � �� ���� � ��� ���

b) Análogamente, a una distancia de 100 m del foco, será: 2

0.0231(100 12

0) 2.08100

10100 nW0.125m100

e eI � � �� ���� � ��� ���

c) La potencia de la fuente se calcula directamente a partir de la expresión de la intensidad: 2 2 0.0231 10 34 154 10 100 158 10 nW= 8 We erP r I �� � �� �� � � � � � �

d’) Supongamos que se trata de la misma fuente, i.e., emitiendo una potencia de 158 �W. La intensidad vendrá dada por

2

20 102

10 204I rPI

r I r�

� ��� �� � � �� ���

a’) Intensidad a una distancia de 10 m: 2 2

1584 10

nW126m

I�

� ��

Intensidad a una distancia de 20 m: 2

2 nW3110 1262

.0

5m

I� ���� � ��� ���

b’) Intensidad a una distancia de 100 m: 2

100 2

nW1.26m

10 126100

I� ���� � ��� ���

d’’) Supongamos que la intensidad de la onda a una distancia de 10 m de la fuente es de 100 nW/m2.

a’’) Intensidad a una distancia de 20 m: 2

20 2

nW210 10020

5m

I� ���� ��� ���

b’’) Intensidad a una distancia de 100 m: 2

100 2

10 100100

nW1m

I� ���� ��� ���

c’’) Potencia de la fuente: 2 2 34 4 10 100 126 1 120 W 6n �W=P r I� �� � � � � �

- 299 -

Física Universitaria: Problemas de Física Ondas mecánicas. M35.2

2. La función y(x,t) correspondiente a cierta onda estacionaria sobre una cuerda tensa es

y = 0.05 sen (2.5 x) cos (500 t) (SI) a) Hallar la amplitud y velocidad de propagación de las ondas que originan esta onda estacionaria. b) ¿Cuál es la distancia entre nodos sucesivos? ¿Cuál es la longitud más corta posible de la cuerda?

Cuando dos ondas que se propagan en direcciones opuestas se superponen, dan lugar a una onda estacionaria, cuya función de onda es

01 0 1

2 0 1

( , ) (2 )( , ) sen( )( , ) sen(

cos( )sen( ))

y x t A t kty x t A t kt

y x t A kx t� �

�� � �

�� � � ����� �� �

��

� �

a) Identificando los valores numéricos de la expresión dada en el problema, se obtiene -1 -12 0.05 m 2.5 m = 500 sA k �� �

de donde 5 m25 mm 200

. s00

2 5A c

k�

� � � �

b) La longitud de onda viene dada por 2 2 2.513 m

2.5k� �

�� � �

y tanto la distancia entre nodos sucesivos como la longitud mínima de la cuerda serán iguales a media longitud de onda; esto es

min 1.257 m2

L �� �

N NV

L

- 300 -

Física Universitaria: Problemas de Física Ondas mecánicas. M35.3

3. Las cuerdas de una guitarra tienen una longitud útil de 66 cm. La quinta cuerda tiene una densidad lineal de

3.1 g/m y su frecuencia fundamental es de 440 Hz (La3) cuando el instrumento está bien afinado. a) Calcular la velocidad de propagación de las ondas en esa cuerda y la tensión de la misma. b) Escribir una expresión general que nos relacione el incremento unitario en la tensión con el cambio unitario en la frecuencia. Utili-zar esa relación para calcular la tensión necesaria para incrementar la frecuencia fundamental en un 2%. c) Si la tensión de ruptura de la cuerda es de 200 kg, ¿cuál será la frecuencia fundamental más alta a la que podemos tensar la cuerda? d) Escribir la función de onda estacionaria - esto es, y(x,t) – para el armónico n-ésimo de una cuerda tensa, de longitud L, sujeta por ambos extremos. Particularizar para el primer y se-gundo armónicos de la quinta cuerda de la guitarra, si la amplitud de los vientres o antinodos es de 2 mm y 1.5 mm, respectivamente.

a) La longitud de la cuerda debe contener un número entero de semilongitudes de onda:

22 2

nn n

n

L c cL n K nn L

�� �

�� " � � " � (1)

De donde se sigue que 2 2 0.66 440

1580.8 m snLc

n� � �

� � �

Por otra parte: 2 3 23.1 10 580.8 1071 4 = kN g0 6F c� �� � � � �

b) La expresión (1) la escribimos en la forma 2

22

d d22

24

nn n

n

n

n

n F n FL FF

FFL

�� �

� � �����

� " � " � �"

Para � 2%n

n

�� � será � 4%F

F � , de modo que

� 0.04 107 kg 4 kg 111 kg'F F� � � " �

c) La frecuencia fundamental correspondiente a la tensión de ruptura será

rup1 3

1 1 200 9.82 2 0.66 3.1 10

602 HzF

L�

� �

�� � �

� �

d) La función general de onda estacionaria es:

2 22, sen sen con

2 22

nn

n n n n

n n

nkL n Ly x t A k x t

nc n cL L

� � ���

�� �� �

��� � � ����� ��� � � �����

o sea

�, sen senn nn ny x t A x ctL L� �� � � �� �� �� � �� �� �� �� �

� � � � � �

1

2

1 , 0.002sen 4.76 sen 2765

2 , 0.0015sen 9.52 sen 5529

n y x t x t

n y x t x t

� " �

� " �

primer armónico fundamental (n = 1)

segundo armónico (n = 2)

- 301 -

Física Universitaria: Problemas de Física Ondas mecánicas. M35.4

4. Una cuerda con una densidad lineal de 4 g/m está sometida a una tensión de 360 N y está fija en ambos

extremos. Una de sus frecuencias de resonancia es 375 Hz; la siguiente frecuencia más alta es de 450 Hz. a) ¿Cuál es la frecuencia de resonancia fundamental? b) ¿Qué armónicos son los que se dan en el enunciado de este problema? c) ¿Cuál es la longitud de la cuerda?

a) Las frecuencias de las ondas estacionarias que pueden residir en la cuerda son múltiplo de la frecuencia fundamental o primer armónico.

�11 1

1 1( 1)n

n nn

nn

� �� � �� � �

�� �� � ��� � ���

En consecuencia, la frecuencia fundamental es:

1 1450 375 Hz 75 Hz � �� � " �

b) En el enunciado del problema se dan el quinto y el sexto armónico, ya que:

1

375 4505 675 75

nn ��

� " � �

c) La longitud de la cuerda es la mitad de la longitud de onda del primer armónico

3

360 300 m/s4 10

Fc� �� � �

1 11

300 4 m 2 8 m75

c c L� � �� �

� " � � � " � �

1 2 m2

L L�� " �

primer armónico fundamental (n = 1)

segundo armónico (n = 2)

tercer armónico (n = 3)

cuarto armónico (n = 4)

- 302 -

Física Universitaria: Problemas de Física Ondas mecánicas. M35.5

5. Un altavoz genera en un concierto de rock 10-2 W/m2 a 20 m a una frecuencia de 1 kHz. Supongamos que la

energía procedente del altavoz se extienda uniformemente en todas las direcciones. a) ¿Cuál es el nivel de intensidad a 20 m? b) ¿Cuál es la potencia acústica total generada por el altavoz? c) ¿A qué distancia alcan-zará la intensidad el umbral de dolor de 120 dB? d) ¿Cuál es el nivel de intensidad a 30 m?

a) Por definición del nivel de intensidad, se sigue 2

102020 12

umb

1010 log 10 log 10 log 10 10 101

0 d0

10 BII

��

�� � � � � �

b) Por tratarse de una onda esférica, la potencia se reparte sobre frentes de onda cada vez más extensos, a medida que nos apartamos del foco.

2 220 20 10 4 50 .2 0 3 WP I S ��� � � � �

c) Deberemos disponer de una intensidad de 1 W/m2, ya que entonces será 12

12

110 log 10 log10 120 dB10

� �� � �

Como P = IS = 4�R2I, sera

50.3 4.004 4

21

mPRI� �

� � � ��

d) De nuevo tenemos en cuenta que se trata de una onda esférica. 3 2

30 230

50.3 4.44 10 W/m4 30

PIS �

�� � � ��

39

30 12

4.44 1010 log 10 log 4.44 10 96.6 dB10

��

�� � � �

- 303 -

Física Universitaria: Problemas de Física Ondas mecánicas. M35.6

6. Una onda sonora disminuye su nivel de intensidad en 30 dB cuando avanza 50 m. Determínese el coeficien-

te de absorción de dicha onda en el medio en el que se propaga en los siguientes supuestos: a) Se trata de una onda plana. b) Se trata de una onda esférica a 100 m de su foco.

Sea � el nivel de intensidad de la onda sonora definido por 0

10lg II

� � , siendo I la intensi-

dad e I0 la intensidad umbral de referencia. Determinamos la relación existente entre la varia-ciones del nivel de intensidad y los cocientes de intensidades:

� /10 30/10 32 22 1

1 1

� 10 lg 10 10 10I II I

�� � � � �� � � � � �

a) Onda plana: 0 e xI I ��� 1

2 1

2

1 0 ( ) �2

1

2

12 0

1 ln�

e ex

x x xx

I I e III e

Ix II

�� �

��

�� � �

�� �

��� � " � � ��� ��

y sustituyendo valores 31 -1

2

1 1ln ln10� 50

0.138 mIx I

�� � �

b) Onda esférica:

2 1

2 2- ( ) - �1 2 2

2 1 2 22 1

22

1

22

1 1

e e 1 ln�

r r rr I rI IIr

rrr r II

� � �� � �� �

y sustituyendo valores 2

3 -11 150ln 1050 100

0.122 m� �� �� �� ���� � ���� ���� � ��

I1

r1 =100 m

I2

r2

- 304 -

Física Universitaria: Problemas de Física Ondas mecánicas. M35.7

7. Una onda plana se propaga por un medio cuyo coeficiente de absorción es � =0.01 m-1. ¿Qué distancia

tendrá que avanzar en dicho medio para que su intensidad disminuya 10 dB?

De acuerdo con la definición del decibelio (dB)

010 10

0 0

10 log 10 dB log 110II I I

I I� � � � �

de modo que la intensidad debe reducirse a la décima parte de su valor inicial. La intensidad de una onda plana que se propaga en un medio absorbente decrece exponen-cialmente conforme avanza en dicho medio:

00

0 0

1 1ln lnx x II II I e e xI I I

� �

� �� �� � � � �

donde x es el trayecto recorrido. Sustituyendo los datos en esta expresión 1 ln10

0.3 m

012 0x � �

- 305 -

Física Universitaria: Problemas de Física Ondas mecánicas. M35.8

8. Para determinar la densidad lineal de un hilo tomamos un trozo del mismo, de 1 m de longitud, fijamos un

extremo y del otro colgamos una pesa de 50 kg, al someterla a vibraciones observamos que a 70 Hz se esta-blece una onda estacionaria con un vientre entre sus extremos. a) Determinar la densidad lineal de dicho hilo. b) A qué frecuencia conseguiremos observar dos nodos entre sus extremos. c) Qué observaríamos a una frecuencia de 100 Hz.

a) La longitud L de la cuerda corresponde a media longitud de onda (distancia internodal); por consiguiente

11 2 2 1 2 m

2L L�

�� � � � �

Determinamos la velocidad de fase a partir de la relación existente entre la longitud de onda y la frecuencia:

1 1 2 70 140 m/sc � �� � � �

La tensión del hilo será: 50 kg = 50 9.8 490 NF � � � La velocidad de los ondas transversales en el hilo viene dada por:

2 2

490 kg0.025140

gm

25m

F Fcc

��

� � � � �

b) Ahora, la longitud de la cuerda corresponde a tres distancias internodales (tercer armónico):

3 23 3

33

23 0.66 m

210H

2 31 490

0.0250 6z

. 6

L L

c

��

��

� � � �

� � �

lo que resulta obvio, ya que para el tercer armónico es 3 13� �� .

c) La frecuencia de 100 Hz no es múltiplo de 70 Hz, por lo que no corresponde a ningún ar-mónico y la cuerda no mostrará vientres ni nodos. Esa frecuencia no se establecerá en la cuer-da como una onda estacionaria, sino como una onda progresiva o viajera a lo largo de la cuer-da.

L

NN

3 / 2� 3 / 2� 3 / 2�

- 306 -

Física Universitaria: Problemas de Física Termología - T00.1

1. Un mol de agua a 25 ºC se calienta en recipiente abierto a la atmósfera hasta 100 ºC. Los valores medios del coeficiente de dilatación y del calor específico del agua en ese intervalo de temperaturas son 4.0 � 10-4 K-1 y 1.0 cal/g·K, respectivamente. Calcular las variaciones de la energía interna, entalpía y entropía en el proceso.

Durante el proceso a presión constante, el agua absorbe una cantidad de calor � 18 1 (100 25) 1350 cal = 5643 JQ mc T� � � � � �

y realiza un trabajo de expansión

atmW p V��

siendo �V el incremento de volumen que experimenta el agua al incrementarse su temperatura. De la definición del coeficiente de dilatación, se sigue:

00

0

� / � � ��V V mV V T T

T� � �

�� � �

y tomando �0 � 1 g/cm3, 4 3 418� 4.0 10 75 0.54 cm 5.4 10 L

1V � �� � � � � � �

de modo que 4 41atm 5.4 10 L = 5.4 10 atm L 0.013 cal 0.055 JW � �� � � � � � �

que es despreciable en comparación con el calor absorbido. Las variaciones de energía interna y de entropía son:

1350 cal 56� 13 43 J50 0.01350 cal 5643 J

13� p

U Q WH Q

� � � � = �� � �

La variación de entropía se calcula imaginando un proceso isobárico reversible entre el estado inicial y el final; esto es,

f

i

f

i

� d 373� ln 1 c8 1 ln2

al J4.04 16.9K98 K

T

T

TQ TS mc mcT T T

� � � � � � � �� �

- 307 -

Física Universitaria: Problemas de Física Termología - T00.2

2. En un calorímetro disponemos de 200 g de hielo a 0ºC. a) Determinar la cantidad de agua a 40ºC que deberemos añadir al calorímetro para que justamente funda todo el hielo. b) Calcular el la variación de entropía que experimenta el sistema (hielo+agua) durante el proceso de fusión del hielo. Discutir y explicar el signo de dicha variación. ¿Se trata de un proceso reversible? Explíquese.

a) Para fundir, el hielo deberá absorber una cantidad de calor

hielo h f 200 80 16000 calQ m� � � ��

que deberá ser cedida por el agua al enfriarse desde 30ºC a 0ºC; esto es

aguaagua a a a

a

16000� 400 g� 1 40

QQ m c t m

c t� � � �

b) El proceso de fusión del hielo tiene lugar a temperatura constante (0ºC = 273 K) con aumento de entropía:

hielohielo

f

16000 cal� 58.6273 K

QST

�� � � �

Proceso de enfriamiento del agua desde 40ºC ( = 313 K) hasta 0ºC (=273 K) con aumento de entropía:

2

1

2agua a a a a

1

� d 273 cal� ln 400 1 ln 54.7313 K

T

T

TQ TS m c m cT T T

� � � � � � � �� �

La variación de entropía del sistema es:

h acal� � � 58.6 54.7 3.9K

S S S� � � � � �

Donde el signo positivo nos indica que este proceso es espontáneo y que ocurre siempre en este sentido y nunca en el sentido opuesto (600 g de agua a 0ºC � 200 g de hielo a 0ºC + 400 g de agua a 40 g). Se trata claramente de un proceso irreversible.

hielo a 0ºC

agua a 40ºC agua a

0ºC + =

- 308 -

Física Universitaria: Problemas de Física Termología - T00.3

3. En un recipiente abierto a la atmósfera, colocamos 1 kg de hielo a 0 ºC y 2 kg de agua a 90 ºC. Determinar: a) La temperatura final de equilibrio. b) El trabajo realizado por cada sistema. c) Los cambios de energía interna, entalpía y entropía de cada unos de los sistemas en la transformación.

Datos: Calor latente de fusión del hielo, 80 cal/g; calor específico del agua, 1 cal/g.ºC; densidad del hielo, 0.9 g/cm3; densidad del agua (0 a 100ºC), 1g/cm3.

Comenzamos tanteando el estado final del sistema:

1 h f

3,máx a

1 80 80 kcal2 1 90 180 kcal

Q m lQ m c t

� � � �

� 4 � � � �

Por consiguiente, al enfriarse el agua es capaz de ceder al hielo calor más que suficiente para su fusión. El estado final del sistema será el de agua (fase líquida) a una cierta temperatura t. Por tratarse de un sistema cerrado, será

1 2 3 0Q Q Q� � � , de modo que

h f h 0,h a 0,a h f 0,h a 0,a( ) ( ) 0 ( ) ( )m l m c t t m c t t m l c t t m c t t� �� � � � � � � � ��

y sustituyendo los datos 1 21 80 1 ( 0) 2 1 ( 90) 0 80 3 180 0 306.5 K33.3 ºCt t t t� � � � � � � � � � � � � �

Trabajo Tan solo se produce cambio de volumen en el proceso de fusión del hielo, a presión constante, por lo que el trabajo asociado al mismo será

�a h ha h

1 1

1000 900

1 1 101328 1 1 2.1. 69 cal26 JW p V p V V pm� �

�� � � ��� ���� 4 � � � � � � � � � �� ��� ��� � ��

��

Entalpía Puesto que el proceso tiene lugar a presión constante, los cambios de entalpía coinciden con los calores absorbidos o cedidos. Esto es,

�hielo

agua

113.3 k1 80 1 cal113.3 kc

1 33.3 80 kcal 33.a

3 kcal0

2 1 3 l3.3 90H

HH

�4 � � � � � � � � � ��4 ���4 � � � � � ���

Energía interna El cambio de la energía interna, en cada sistema será U Q W4 � � , de modo que

�hielo agua113.3 k1 cal13.3 kcal 2.7 cal 2.7113 cal.3 kcalU U U4 � � � � 4 � 4� �

Entropía

f

0

1 h fhielo h h

f f f

agua a a0

d 80 306.48 ln 1 1 ln 0.29 0.12 0.

kcalK

kcal0.

41273 273.15

d 306.48 ln 2 1 ln

363.1534

Kkcal0.07K

T

T

T

T

Q m lT TS m c m c

T T T T

T TS m c m c

T TS�

4 � � � � � � � � � � � �

4 � � � � � � 4 � �

mh mh+ma

ma

Q2

Q3

Q1

tiempo

tempe

ratu

ra (º

C)

0

90

t

- 309 -

Física Universitaria: Problemas de Física Termología - T00.4

4. Un recipiente adiabático contiene 40 g de hielo a una temperatura de -20ºC. Añadimos 30 g de agua a 50ºC, removemos y esperamos a que se alcance el equilibrio térmico. a) Determinar la composición de la mezcla y la temperatura de la misma. b) Calcular los cambios de entropía que experimentan el hielo y el agua. c) ¿Varía la entropía del sistema hielo-agua en este proceso? ¿Aumenta o disminuye? ¿Por qué?

Datos: calor específico del agua, 1.00 cal/g.K; calor específico del hielo, 0.5 cal/g.K; calor latente de fusión del hielo, 80 cal/g.

a) Balance calorífico para pasar 40 g de hielo a –20ºC hasta agua a 0ºC:

1hielo

f f

calentamiento:

fusi n:

� 40 0.5 (0 ( 20) 400 cal3600 cal

40 80 3200Q mc T

QQ m

�� � � � � � � �� ���� � � � ���ó

Balance calorífico cuando se enfría el agua desde 50ºC hasta 0ºC: enfriamiento: � 30 1.0 (0 50) 1500 calQ mc T� � � � � � �

Resulta obvio que no puede fundirse todo el hielo, por lo que en el estado final tendremos hielo y agua en equilibrio a la temperatura de fusión (0ºC). Sea mf la masa de hielo que funde. Escribimos la ecuación del balance calorífico:

f f110080 (1500 400) 1100 13.75 g

80m m� � � � �

de modo que la mezcla final estará compuesta por (30 + 13.75) = 43.75 g de agua líquida y (40-13.75 )= 26,25 g de hielo. b) Determinamos los cambios de entropía que experimentan el hielo y el agua:

273h h f f

hielo 253

273a a

agua 323

d 273 13.75 80 cal� 40 0.5 ln 1.52 4.03 5.55273 253 273 K

d 273 cal� 30 1 ln 5.04323 K

m c T m lST

m c TST

�� � � � � � � � �

� � � � � �

c) El cambio de entropía que experimenta el sistema hielo+agua será

hielo agua� � c� al0.50K

S S S �� � �

i.e., la entropía del sistema (aislado) aumenta porque el proceso que ha tenido lugar en el es espontáneo e irreversible.

- 310 -

Física Universitaria: Problemas de Física Termología - T00.5

5. En un recinto adiabático disponemos de 1 kg de hielo a 0 ºC y exceso de vapor de agua a 100 ºC. Calcúlense la cantidad de vapor que se condensará y los cambios de entalpía y entropía experimentados por cada componente. Datos: lf = 80 cal/g, lv = 540 cal/g , c = 1 cal/g.K.

Puesto que tenemos exceso de vapor, el estado final de la mezcla corresponde a la temperatura de éste (100 ºC), por lo que tendremos agua y vapor a 100º C. Balance calorífico Calor absorbido por el hielo para fundir + calor absorbido por el agua al calentarse desde 0 ºC hasta 1000 ºC = Calor cedido por el vapor de agua al condensarse; i.e.,

fh f h v v v h

v

0.3� 80 1 100 1� 1540 3

33 kgl c Tm l m c T m l m ml

� � �� � � � � � �

Cambios de entalpía

h h f

agua h

v v v

total

80 kcal100 kcal

� 1 80� � 1 1 (100)

� 0.33 180 k3 540� 80

cal100 180 0

H m lH m c TH m lH

� � � �

� � � � �

� � � � �

� � � �

Cambios de entropía

h

fagua h

i

v

total

kcal0.29K

kcal

80�273

373� ln 1 1 ln273

180�37

0.31K

kcal0.48K3

� 0.29 0.31 0.4 kcal0.12K

8

QST

TS m cT

QST

H

� � �

� � � � �

�� � �

� �

� ��

de modo que la entropía crece en el proceso, como era de esperar por tratarse de un proceso espontáneo e irreversible.

- 311 -

Física Universitaria: Problemas de Física Termología - T00.6

6. A presión constante, se suministran 500 J en forma de calor a 2 moles de un gas ideal biatómico. Calcular: a) El incremento de temperatura. b) El trabajo realizado por el gas. c) El cociente entre el volumen final y el inicial del gas si la temperatura inicial es 20 ºC. d) La variación de entropía. Dato: R = 8.31451 J/mol K.

Por tratarse de un gas biatómico, serán

5 20.786 J/ mol K27 29.101J/ mol K2

V V

p p

C R C

C R C

��� � " � ������ � " � ����

a) Se trata de un proceso de expansión isobárico, por lo que Q = n Cp ,T 500� 8.59 K

2 29.1� 8.6

01K

p

QTnC

T� �� � "�

b) Calculamos la variación de la energía interna: � � 2 20.786 8.59 � 357 JVU nC T U� � � � " �

500 357 143 JW Q U�� � � � � c)

1 2 2 2 1 2

1 2 1 1 1 1

293. 1.0315 8.59 301.74293.15 293.15

pV pV V T T T VT T V T T V

�� �� � � � � �

d) Calculamos la variación de entropía para un proceso isóbaro 2

1

2

1

d d 301.59� ln 2 29.101 ln293.1

J1.67K5

Tp ppT

Q nC T TS nCT T T

� � � � � � �� �

- 312 -

Física Universitaria: Problemas de Física Termología - T00.7

7. Dos moles de un gas perfecto, que inicialmente ocupan 44.8 L a 1 atm de presión, se someten a una transformación isoterma reversible en la que su entropía disminuye 2.75 cal/K. Determinar el estado final del gas (p, V y T) así como el trabajo realizado y el calor absorbido durante el proceso.

La ecuación de estado del gas perfecto nos permite determinar la temperatura.

1 44.8 273 K2 0.08205

pVpV nRT TnR

�� � � �

La variación de entropía en una transformación isoterma viene dada por

2

1

� ln VQS nRT V

� �

de donde se sigue

�� 273 2.75 751cal

Q T S

W Q U

� � � � � �

� � 751cal 3138 J

����� � � � ���

La expresión de la variación de entropía también nos permite determinar el volumen en el estado final

� 2.75 2 1.987 0.69222

10.50 22.4 LS nRV e e e V

V� � �� � � � �

Ahora determinamos la presión en el estado final

11 1 2 2 2 1

2

2 1 2 atmVp V p V p pV

� � � � �

p

V10 20 30 40 50 60

1

2

3

273 K1

2

- 313 -

Física Universitaria: Problemas de Física Termología - T00.8

8. Cinco moles de un gas biatómico, que se encuentran inicialmente a presión atmosférica y temperatura de 0ºC, se someten a las transformaciones indicadas en el diagrama adjunto. a) Determinar la temperatura final del gas. b) Calcular el trabajo realizado por el sistema, el calor absorbido y los cambios de energía interna, de entalpía y de entropía.

Datos: R = 1.987 cal/mol�K = 0.08205 atm�L/mol�K

En un gas biatómico (f = 5): 2 cal6.95

2 mol Kcal4.97

2 mol K

p

V

fC R

fC R

�� �

� ��

a) Aplicando la ecuación de estado de los gases perfectos a los puntos 1 y 4:

1 1 4 4 4 4 1 14 1 1

1 4 1 1 1 1

4 1

2 2

4 4 273 1092 K

p V p V p V p VT T TT T p V p V

T T

�� " � �

� � � �

b) El trabajo realizado en la transformación completa será:

� �2 3 4

1,4 1 2 1 4 4 31 2 3

atm 1 1 atm 1 1 1 atm 1 atm

d d d

(3 ) 2 (2 3 ) 2 2 0

W p V p V p V p V V p V V

p V V p V V V p V p

� � � � � � � �

� � � � � � �

� � �

El calor absorbido por el sistema, según el primer principio de la termodinámica será:

�Q U W� � � 23353 calU� � La variación de energía interna es:

� � 5 4.97 (1092 273) 23353 calVU nC T� � � � � �

La variación de entalpía será: � � 5 6.95 (1092 273) 28460 calpH nC T� � � � � �

La variación de entropía será: 4 4 4 4 4

4 41,4 1 1 1 1 1

1 1

d� d d d d� ln lnV VT VQ U p V T VS nC nR nC nR

T T T T V T V� � � � � � � �� � � � �

� � cal 5 4.97 ln 4 + 5 1.987 ln 2 41.34K

� � � � � �

p

V

2patm

patm

V1 2V1 3V1

1 2

3 4

- 314 -

Física Universitaria: Problemas de Física Termología - T00.9

9. Una corriente ascendente de aire seco tiene una temperatura de 27 ºC, una presión de 101.3 kPa y una densidad de 1.18 kg/m3 a nivel del suelo. Consideremos que asciende a 100 m de altura sin intercambio de calor con su entorno. a) Determinar la presión atmosférica a la altura de 100 m. b) Calcular la temperatura que tendrá la corriente de aire a esa altura. c) Explicar la causa de esa variación de temperatura de la corriente de aire. Cuantifíquese para 1 mol de aire.

Nota: A los solos efectos del apartado a), considérese que la densidad de la atmósfera no varía significativamente en una diferencia de alturas de 100 m.

a) Calculamos la presión atmosférica a una altura de 100 m sobre el suelo despreciando el cambio de densidad del aire con la altura (� = cte); esto es, 0 100p p gh�� �

100 0 101300 1.18 9.8 100 101300 1156 100144 Pap p gh�� � � � � � � � �

b) Consideremos una cierta cantidad de aire (digamos, n moles). Durante el ascenso, experimenta un proceso termodinámico en el que disminuye su presión, en tanto que no intercambia calor con su entorno. En definitiva, experimenta una expansión adiabática durante el ascenso, lo que implica una disminución de la temperatura (vide gráfica). A partir de la ec. de la adiabática en función de (p,V) y de la ec. de estado del gas perfecto, eliminamos el volumen para obtener la ec. de la adiabática en función de (p,T).

11

cte

cte cte cte

pVTpV nRT V nRp

Tp p T p Tp

� �� � �

��

�� ������ � �����

� ��� � � � �� �� ���

Puesto que el aire se comporta como un gas biatómico, será

2 5 2 7 1 21.45 5 7

p

V

C fC f

��

�� � �

� � � � � � �

Entonces, aplicando la ec. de la adiabática entre los estados inicial y final, será

� �

1

1 10

100 100 0 0 100 0100

pp T p T T Tp

��

� �� �

� � � ��� �� � � �� ���

2/72/7

100101300300 300 1.0115 300 0.997 299 K 26 ºC100144

T�

�� ��� �� � � � � � �� �� ���

de modo que la corriente de aire ascendente se ha enfriado un grado. c) Aplicando el Primer Principio al proceso adiabático, �U Q� W W� � � , de modo que el trabajo asociado a la expansión adiabática del aire da lugar a la disminución de la energía interna de éste y, por ende, de su temperatura. Así, para 1 mol de aire será:

�5� � 1 8.3143 1 20.8 J2VW U nC T

� ���� � � � � � � � � � ��� ���

V

p

T100

T0p100

p0 suelo

100 m

- 315 -

Física Universitaria: Problemas de Física Termología - T00.10

10. Consideremos un tubo en U, de sección uniforme S, con uno de sus extremos taponado y el otro abierto a la atmósfera. Inicialmente, la columna de aire atrapada en la rama cerrada tiene longitud H y se encuentra a la presión atmosférica p0. Sean L la longitud y � la densidad de la columna líquida. Determina la frecuencia de las pequeñas oscilaciones de la columna líquida.

Iniciadas las oscilaciones de la columna líquida, para una elongación genérica x, las fuerzas que actúan sobre la columna líquida, de masa

m SL�� son las que se indican en la figura de la derecha, que corresponden a la presión ejercida por el aire sobre las superficies libres de la columna líquida y al peso de la porción de columna líquida en exceso en la rama derecha. De este modo, la fuerza neta en la dirección del desplazamiento es

� �neta 0 02 2F p S pS S x g gSx p p S� �� � � � � � �

Si suponemos que las oscilaciones son suficientemente rápidas, el aire atrapado en la rama de la derecha experimenta compresiones y expansiones aproximadamente adiabáticas, por lo que podemos escribir

� 1 2 00 0 0 0( ) 1 1 px xp SH p S H x p p p p p x

H H H

�� � �

��� � � �� �� �� � � � = � � =� �� �� �� �� �

donde hemos supuesto que las oscilaciones son pequeñas en comparación con la longitud de la columna de aire �, i.e., x H� . De este modo, la fuerza neta que actúa sobre la columna de líquido es

0 0neta 2 2p pF gSx Sx gS S x

H H� �

� �� ���� � � � � � �� ���

Y aplicando la ec. fundamental de la dinámica, netaF mx� �� , tenemos

0 022 0p pggS S x SL x x xH L HL� �

� ��

� �� � ����� � � � � � �� �� ��� � ��� � �� ��

Que es la ecuación diferencial correspondiente a un m.a.s. cuya frecuencia angular es

0

0

2 2 22

pg LTL HL g p H

� �� �

� � � �� � � �

� Si 1�� , es � �1 1n n� �� = � , como se demuestra fácilmente a partir del desarrollo del

binomio de Newton � 2 31 0 1 2 3n n n n n� � � �

� � � � � � � �� � � �� � � �� � � � � �� � � �� � � �� � � �� � � �� � � � .

p0 H

p0

�S(2x)g p0S

x 2x

pS

- 316 -

Física Universitaria: Problemas de Física Termología - T00.11

11. Un mol de oxígeno a 300 K se mezcla con 2 moles de oxígeno a 400 K, ambos a la presión atmosférica. Determinar la temperatura final de la mezcla, así como las variaciones de energía interna, de entalpía y de entropía durante el proceso.

En el proceso de mezcla se conserva la energía interna del sistema:

� � � �

1 1 2 2

1 1 2 2

0V V

V

U n C T T n C T T

C n T T n T T

� � � � �

� �� � � � ��

Esta expresión nos permite determinar la temperatura final de la mezcla:

1 1 2 2

1 2

1 300 2 400 367 K1 2

n T n TTn n

� � � �� � �

� �

El cambio que experimenta la entalpía del sistema será:

� � � �1 1 2 2 1 1 2 2� 0p p pH n C T T n C T T C n T T n T T� �� � � � � � � � ��

El cambio que experimenta la entropía del sistema será la suma de los cambios experimentados por cada uno de los gases en los procesos isobáricos correspondientes:

1

2

1 1 11

2 2 22

1 2

� d 7 367 cal� ln 1 1.987 ln 1.412 300 K

� d 7 367 cal� ln 2 1.987 ln 1.212 400 K

cal J� � � 0.20 0.84K K

T

p pT

T

p pT

Q T TS n C n CT T TQ T TS n C n CT T T

S S S

� � � � � �

� � � � � � �

� � � �

� �

� �

Resultando un aumento de entropía por tratarse de un proceso irreversible.

Otro método:

1 11 1 2

1

2 22

2

1 0.08205 300 24.62 L 90.26 L11 90.262 0.08205 400 367 K65.64 L 3 0.082051

n RTV V V VppVn RT TV nRp

�� � ��� � � � � � ��� �� �� � � �� �� � � � �� �� � � � � �����

n2=2molp2=1atm

V2=65.6 L T2=400K

n=3mol p=1atm

V1=90.2 L T=367K

n1=1molp1=1atm

V1=24.6 L T1=300K

- 317 -

Física Universitaria: Problemas de Física Termología - T00.12

12. En dos compartimentos de un recipiente adiabático de paredes rígidas mantenemos separados, mediante un tabique rígido y adiabático, 1 mol de helio a 1 atm y -100 ºC y 2 moles de hidrógeno a 1 atm y 27 ºC. a) Determinar la temperatura y la presión final de la mezcla cuando se retira el tabique. b) Calcular la presión parcial de cada componente en la mezcla. c) Evaluar el cambio de entropía en este proceso de mezcla.

1 mol de He a 1 atm y 100 K ocupa 1 11

1

1 0.08205 100 8.21 L1

n RTVp

� �� � �

2 mol de H2 a 1 atm y 300 K ocupan 2 22

2

2 0.08205 300 49.23 L1

n RTVp

� �� � �

El volumen del recipiente es: 1 2 57.44 LV V V� � �

El proceso de mezcla es adiabático (Q=0), isócoro (V=cte; W=0) e irreversible. El Primer Principio de la Termodinámica establece que 0 0 0U Q W� � � � � � , de modo que la energía interna del sistema permanece constante, lo que nos permite determinar la temperatura final de la mezcla

� � � �

� �1 2 1 1 1 2 2 2

1 1 1 2 2 2

1 1 2 2

� � � ( ) ( ) 01 3 100 2 5 300 3 100 10 300

254 K= 19ºC1 3 2 5 3 10

3 0.08205 254 1.09 atm57.44

V V

V V

V V

U U U n C T T n C T T

n C T n C TTn C n C

nRTpV

� � � � � � �

� � � � � � � ��� � � � �

� � � � �

� � � � �

b) Las presiones parciales se calculan multiplicando la presión por las fracciones molares de cada componente:

1 21 1 1 23 30.36 atm 0.73 atmp p p p p p� �� � � � � �

c) El cambio de entropía en el proceso de mezcla, para cada uno de los gases, viene dado por f f f f f f

i i i i i i

f f

i i

f f

i i

� d � d � d d

d d

ln ln

V

V V

Q U W U W T p VnC

T T T T T T

T VnC nR

T V

S

T VnC n RT V

�� � � � � � �

� � � �

� � � � � �

� �

31 2

52 2

254 57.44 cal� 1 1.987 ln 1 1.987 ln 2.78 3.87 6.64cal100 8.21 K � 9.63

254 57.44 cal K� 2 1.987 ln 2 1.987 ln 1.66 4.64 2.99300 49.2 K

SS

S

��� � � � � � � � � � ���� ���� � � � � � � � � � �����

1 mol p1=1atm V1=8.21L T1=100K

2 mol p2=1atm V2=49.23L T2=300K

3 mol p=1.09atm V=57.44L T=254K

- 318 -

Física Universitaria: Problemas de Física Termología - T00.13

13. Un sistema termodinámico está formado por 2 moles de hidrógeno a 2 atm y 1 mol de helio a 1 atm que están contenidos en un cilindro rígido y adiabático, a diferente lado de un pistón buen conductor del calor bloqueado que los separa. Inicialmente, el sistema se encuentra en equilibrio térmico a la temperatura de 0ºC. En un instante dado desbloqueamos el pistón, de modo que el sistema evoluciona hasta que finalmente alcanza el equilibrio. En toda esta transformación termodinámica: a) ¿Se conservará inalterada alguna variable de estado del sistema? ¿Cuál será la temperatura final del sistema? ¿Es reversible esta transformación? Razónense las respuestas. b) Calcular los cambios de entropía experimentados por cada uno de los dos gases y por el sistema.

a) Por tratarse de un sistema aislado, que no intercambia energía con el exterior en forma de calor o de trabajo, la energía interna del sistema permanece constante. Puesto que en el estado final de equilibrio ambos gases tendrán la misma temperatura (y la misma presión), podemos escribir

�H He H ,H f 0 He ,He f 0

H ,H He ,He f f 00

cte. � � � ( ) )

0) 0 º

(

( CV V

V V

U U U U n C T T n C T T

n C n C T T T T

� � � � � � � �

� � � � ��

de modo que la temperatura final es la misma que la inicial. El proceso es irreversible, ya que al expansionarse un gas y comprimirse el otro, contra una presión diferente a la suya propia, hasta que adquieren una presión común pf, los estados intermedios no serán estados de equilibrio. b) Como los procesos implicados son irreversibles, para calcular ,S debemos imaginar unas transformaciones reversibles que lleve al sistema del estado inicial al final. Puesto que, para cada gas, la temperatura inicial es igual a la final, podemos considerar un proceso isotermo:

f

0

f

0rev

cte d � d 0 � d� d d� ln

V

V

T U Q p V Q p VVQ p V VS nR nR

T T V V

� " � � � " �

� � � �� � �

Determinamos los valores iniciales y finales ocupados por cada gas H 0 0 He 0 0

H 0 He 0 H He 0H He

2 122.4 22.4 2 44.8 2 1

n RT RT n RT RTV RT V RT V V V RTp p

� � � � � � � � � � � �� � �

Al final tenemos la misma presión en ambos gases

'' ' H 0

H HeHe fH ff ' ' ' ' ' '

'H He H He H He 0He 0

4

3

2

3

29.9 22 12 14.9

V RTV Vn RTn RTpV V V V V V RT V RT

��� � ���� � �� �� � � � �� �� �� �� � �����

H

He

4 cal J� 2 ln 1.14 4.78cal J3 K K � 0.34 1.41

2 cal J K K� ln 0.81 3.373 K K

S RS

S R

���� � � � ��� � � � ����� � � � � ����

de modo que el sistema experimenta un incremento de entropía, ya que evoluciona espontáneamente hacia un nuevo estado de equilibrio.

He H2

He H2

estado inicial

estado final

- 319 -

Física Universitaria: Problemas de Física Termología - T00.14

14. Una mezcla de gases constituida por 1 mol de helio y dos moles de oxígeno se encuentra en un recinto de 67.2 L a 1 atm de presión. Si la mezcla se calienta isobáricamente hasta duplicar su volumen, determínense los balances de calor y trabajo y las variaciones de energía interna, entalpía y entropía de la mezcla.

3 52 2

5 72 2 2

He (monoatómico): cal1.987mol KO (biatómico):

V p

V p

C R C RR

C R C R

�� � �� ��� �� � ��

Calculamos las temperaturas inicial y final de la mezcla:

1 11 1 1

2 22 2 2

1 67.21atm 67.2 L3 0.08205 � 273 K

1 134.41atm 134.4 L3 0.08205

273 K

546 K

p Vp V TnR Tp Vp V TnR

�������������

�� � � � �

� ��

� � � � ��

El trabajo realizado en el proceso de calentamiento isobárico será: � 1 67.2 67.2 atm 6809 JL =W p V� � � � �

El balance calorífico y las variaciones de energía interna, entalpía y entropía son: 5 72 2

3 52 2

5 72 2

� (1 1.987 273) (2 1.987 273) 1356 3797 5153 cal=

� � (1 1.987 273) (2 1.987 273) 814 2712 3526 cal=

� � (1 1.987 273) (2 1.987 273) 1356 3797 5153 cal

21541 J

14739 J

1 4= 2 5

p

V

p

Q nC T

U nC T

H nC T

� � � � � � � � � � � �

� � � � � � � � � � � �

� � � � � � � � � � � �

+++

2 5 72 2

1

cal� ln (1 1.987 ln 2) (2 1.987 ln 2) 3.44 9.64

1 J

J513.08 4.7K

=Kp

TS nCT

� � � � � � � � � � � �+

- 320 -

Física Universitaria: Problemas de Física Termología - T00.15

15. Una botella de acero, cerrada y de volumen constante, contiene en su interior 0.5 kg de oxígeno a la presión de 10 atm y temperatura de 20 ºC. Hállense: a) La capacidad de la botella de acero. b) La presión que se alcanzará en su interior, cuando se calienta el oxígeno hasta 80 ºC . c) El incremento de entropía del oxígeno al realizarse el calentamiento descrito en el apartado b).

Los 0.5 kg de oxígeno representan 500/32 = 15.625 mol y, puesto que se trata de un gas (ideal) biatómico, será

52VC R�

a) Determinamos el volumen ocupado por el gas (capacidad de la botella) a partir de la ec. de estado de los gases perfectos:

11 1

1

15.625 0.08206 293 37.57 L10

nRTp V nRT Vp

� �� � � � �

b) Cuando lo calentamos a volumen constante hasta 80 ºC = 353 K, la presión aumenta:

1 2 22 1

1 2 1

353 10 12.05 atm293

p p Tp pT T T

� � � � �

c) En el proceso isocoro (a volumen constante) el incremento de entropía será 2

1

2

1

� d 5 353 cal� ln 15.625 1.987 ln 14.462 293 K

T

V VT

TQ TS nC nCT T T

� � � � � � � �� �

p1

p2

V

- 321 -

Física Universitaria: Problemas de Física Termología - T00.16

16. Un recinto de paredes rígidas y adiabáticas está dividido en dos compartimentos mediante un tabique ligero y móvil. Uno de los compartimentos contiene un mol de anhídrido carbónico a una presión inicial de 50 atm y una temperatura de 300 K; en el otro compartimiento existe el vacío. Permitimos que el gas se expansione espontáneamente hasta que su volumen se hace 20 veces superior a su volumen inicial. Supongamos que el CO2 se comporte como un gas perfecto. a) Explicar si el proceso es reversible o no. ¿Se intercambia calor? ¿Se realiza trabajo? b) Calcúlense los cambios de energía interna, de temperatura y de entropía, experimentados durante el proceso.

Datos: R = 1.987 cal/(mol�K) = 0.082057 (atm�L)/(mol�K).

a) El proceso es irreversible, espontáneo, ya que al expansionarse contra el vacío los estados intermedios no serán estados de equilibrio. Por consiguientes:

3 No se intercambia calor (paredes adiabáticas) 3 No se realiza trabajo, por tratarse de una expansión contra el vacío.

b) Según el Primer Principio de la Termodinámica �U Q W� �

y, puesto que no se intercambia calor ni trabajo, es 0U� � . Entonces, dado que la energía interna del gas ideal es tan solo función de la temperatura, será

� � 0 � 0VU nC T T� � " �

y la temperatura permanece constante (efecto de Joule-Kelvin). La variación de entropía la calculamos como

rev

�� QST

� �

Como el proceso que se describe en el enunciado es irreversible, para calcular ,S debemos imaginar una transformación reversible que lleve al sistema del estado inicial al final. Puesto que la temperatura inicial es igual a la final podemos considerar un proceso isotermo:

cte d � d 0 � dT U Q p V Q p V� " � � � " � de modo que

0

0

200

0rev

20� d d� ln ln 20

cal� 1mol 1.987 ln 20 � 5.95 cal/Kmol K

V

V

VQ p V VS nR nR nRT T V V

S S

� � � � �

� � � " ��

� � �

vacío CO2

- 322 -

Física Universitaria: Problemas de Física Termología - T00.17

17. Un mol de hidrógeno a una presión de 1 atm ocupa 22.4 L y evoluciona según las transformaciones reversibles siguientes: 1º Un calentamiento isobárico hasta una temperatura de 819 K; 2º Un enfriamiento isócoro hasta la temperatura inicial; 3º Una transformación que cierra el ciclo, cuya representación en un diagrama pV es una recta. a) Calcular el trabajo neto y el balance calorífico en el ciclo completo, así como las variaciones de energía interna, entalpía y entropía en el mismo. b) El intercambio de calor y de trabajo y las variaciones de energía interna, de entalpía y de entropía en la tercera transformación.

Estado A: A AA A A A

1 22.4 273 K1 0.082

p Vp V nRT TnR

�� � � �

Estado B: Bp B A

B

V pT

� A BB A A A

A A

819 3 67.2 L273

V TV V V VT T

� � � �

Estado C: C Cp V B B

C

p VT

� CC B B

B B

273 0.33 atm819

Tp p pT T

� � ��

a) En el proceso cíclico, las variaciones de las funciones termodinámicas son nulas y los balances de calor y de trabajo coinciden y vienen representados por el área del ciclo en el diagrama de Clayperon (pV). Así pues:

361.6 cal = 151

1 1 2AB BC 44.8 14.9 atm L2 2 3

� 0 � 0 � 0 2 J

Q W

U H S

� � � � � � � �

�� � �

b) Transformación no-politrópica CA. Puesto que en esta transformación a los estados inicial (C) y final (A) les corresponde la misma temperatura, serán �U = 0, �H = 0 y, de acuerdo con el Primer Principio, Q = W, viniendo representado éste último por el área del trapecio ACDE que se indica en el diagrama de Clayperon:

3

CA CA

(AB AE) 44.9 1 14.9 29.723.2 cal = 3023 J

9 atm×L29.9 atm L =

W WQ W - -� � � � � � �

# � � � �

La variación de la entropía la calculamos a través de una transformación isoterma que conecte los estados C y A:

A A A AA

C C C CC

d d d d� ln

cal J2.111 1.987 8 9.1l 2K

nK3

VQ W p V VS nR nRT T T V V

� � � � � �

� � � � �� �

� � � �

p (atm)

V (L)

T (K)

A 1.00 22.4 273

B 1.00 67.2 819

C 0.33 67.2 273

p (atm)

C

B

W

D E

1

1/3

V (L)22.4 67.2

819 K

273 K

A

W3

- 323 -

Física Universitaria: Problemas de Física Termología - T00.18

18. Disponemos de una cierta cantidad de gas perfecto, a 1 atm de presión y 27º C de temperatura, y lo sometemos a los siguientes procesos: 1º Una compresión isotérmica hasta que su volumen se reduce a la cuarta parte de su valor inicial. 2º Una expansión adiabática hasta devolverle su volumen inicial; entonces, su presión es 0.4 atm. a) Determinar la atomicidad del gas (nº de átomos de la molécula). b) Calcular los cambios molares de energía interna, entalpía y entropía en el proceso total.

12 Transformación isoterma, 1 1 2 2p V p V� :

12 1

2

4 1 4 atmVp pV

� � � �

23 Transformación adiabática, 2 2 3 3p V p V� �� :

3 3 22

3 2 2 3

ln( ) ln(0.4 / 4) 1.66ln( ) ln(1/ 4)

p p pVV p V V

�� ��� � � � � �� �� ���

Entre el valor del coeficiente adiabático y el número de grados de libertad de la molécula (mono o biatómicas) están relacionados por

2 2 2 31 1.66 1

f ff

��

�� � � =

� �

que corresponde a moléculas monoatómicas. Entonces cal cal 2 cal1.987 2.98 4.98

K mol 2 K mol K molV pf fR C R C R

R�

� � � � �

b) Determinamos la temperatura correspondiente al estado 3:

3 3 3 31 13 1

3 1 1 1

0.4 300 120 K1

p V p Vp V T TT T p V

� � � � �

Los cambios ,U y ,H tan solo dependen de las temperaturas inicial y final, de modo que � �

� � 2.98 120 300 536.4 cal

� � 4.98 120 300 892.8 calV

p

U C T

H C T

� � � � � �

� � � � � �

El cambio ,S en el proceso total se calcula a través de la transformación isocora 1�3

3

1

120� ln 2.98 ln 2.73 cal/K300V

T

TS C

� � � �� �� �� � � � �� �� �� ���� � �

p

2

1

3

V1/4 V1 V

0.4 1.

4.

300 K

120 K

- 324 -

Física Universitaria: Problemas de Física Termología - T00.19

19. Tres moles de N2 a presión atmosférica y temperatura de 300 K se comprimen isotérmica y reversiblemente hasta duplicar la presión. Desde ese nuevo estado se expansiona adiabática y reversiblemente hasta alcanzar la presión original. a) Dibujar la transformación indicando los estados inicial (1), intermedio (2) y final (3). b) Calcular la temperatura, la presión y los volúmenes desconocidos (resumirlo en un cuadro de resultados, con sus unidades). c) Calcular los trabajos, calores y variaciones de energía interna, entalpía y entropía en cada una de las transformaciones elementales y en su conjunto (resumirlo en un cuadro de resultados, con sus unidades

Por tratarse de un gas biatómico, serán: 7 cal3 mol 6.95 2 mol K7 5 cal1.40 4.97 5 2 mol K

p

V

n C R

C R�

���� � ��� ���� � � � ��� ��

(1) 11

1

3 0.08205 73.0 8 L1

53 0nRTVp

� �� � �

(2) 11 1 2 2 2 1

2

73 36.92. 52

L8pp V p V V Vp

� " � � �

(3) 1/

22 2 3 3 3 2

3

pp V p V V Vp

� �� ��� �� " � � �� ���

1/1.43 36.92 2 36.92 1.64 60.58 LV# � � � � �

3 33

1 60.583 0.08205

246.10Kp VTnR

�� � �

Proceso 1-2 .- Isotermo (T = cte.) 2

1

212

1

12

d 36.92d ln 3 1.987 300ln 1240 cal73.82

51 J

81V

V

VVW p V nRT nRTV V

Q U

� ���� � � � � � � � ��� ����

� �

1212 12 12

5181 JJ0

1240 cal1240 cal� � � � � 4.13300 K

0 17.3KV p

WQU nC T H nC T ST

� � � �

�� � � � �� �

� � �

Proceso 2-3.- Adiabático (Q = 0) �U Q# � y � 0W S� �

� � � �

23 3 2

23 3 2

23 23 23

� 3 4.97 246.10 300 804 cal

� 3 6.95 246.10 300 1124

3360 J

4698 J

3

cal

� 804 ca 360 Jl �0 0

V

p

U nC T T

H nC T TW U Q S

� � � � � � � �

� � � � � � � �

� � � � �

p (atm) V (L) T (K)

1 1 73.85 300

2 2 36.92 300

3 1 60.58 246.1

V2 V3 V1 V

p2

p1

p

1

2

3

- 325 -

Física Universitaria: Problemas de Física Termología - T00.20

20. Disponemos de 5 moles de nitrógeno que se encuentran inicialmente a la presión atmosférica y temperatura de 0 ºC. Se les someten a un proceso ciclo compuesto por las siguientes etapas reversibles: 1º expansión isóbara, 2º compresión isotérmica y 3ª enfriamiento isócoro. Sabiendo que al expandirse se realiza un trabajo de 11346 J, calcular las variaciones de energía interna, de entalpía y de entropía, así como el trabajo y calor producidos o consumidos en cada transformación y en el ciclo.

Datos: R = 1.987 cal/mol K = 0.08205 atm�L /mol K.

Por tratarse de un gas biatómico, será 5 cal

2 mol.K7 cal

2 mol.K

4.9675

6.9545

V

p

C R

C R

� �

� �

Determinamos las coordenadas termodinámicas de los tres estados:

11

1

1 2 22 1

1 2 1

22 2 3 3 3 2

3

5 0.082 273 112 L1

224 273 546 K112

224 1 2 atm112

nRTVp

V V VT TT T V

Vp V p V p pV

� �� � �

� � � � �

� � � � �

12 Proceso isobárico:

2

1

� � 5 4.9675 (546 273) 6780 cal� � 5 6.9545 (546 273) 9493 cal

546 cal� ln 5 6.9545 ln 24.1273 K

V

p

p

U nC TH nC T

TS nCT

� � � � � �

� � � � � �

� � � � �

23 Proceso isotérmico:

2

1

� 0 � 0112 cal� ln 5 1.987 ln 6.9224 K

U HVS nRV

� �

� � � � � �

31 Proceso isocoro:

2

1

� � 5 4.9675 (273 546) 6780 cal� � 5 6.9545 (273 546) 9493 cal

273 cal� ln 5 4.9675 ln 17.2546 K

V

p

V

U nC TH nC T

TS nCT

� � � � � � �

� � � � � � �

� � � � � �

Para el ciclo completo: � 0 � 0 � 0U H S� � �

estado p (atm) V (L) T (K)

1 1 112 273

2 1 224 546

3 2 112 546

546 K1

224 V (L)

p (atm)

112

2

3

1

2

- 326 -

Física Universitaria: Problemas de Física Termología - T00.21

21. Dos litros de nitrógeno (gas biatómico), inicialmente a presión atmosférica y 27º C, evolucionan según las transformaciones reversibles siguientes: 1) Un calentamiento a presión constante hasta duplicar el volumen inicial. 2) Una expansión adiabática hasta alcanzar la temperatura inicial. 3) Una transformación que cierra el ciclo, y cuya representación es una recta en el diagrama p-V. Calcular los balances de calor y trabajo y los cambios de energía interna, entalpía y entropía en cada una de las transformaciones.

Datos: R = 1.987 cal/mol�K = 0.08205 atm�l/mol�K.

5 7 7 1.42 2 5

pV p

V

CC R C R

C�� � � � �

Determinamos el número de moles: 1 2 0.08 mol

0.08205 300pVpV nRT nRT

�� � � �

� Determinación de los procesos (12) Proceso de expansión isobárica:

1 2 22 1

1 2 1

4 300 600 K2

V V VT TT T V

� � � �

(23) Proceso de expansión adiabática: 1cte. cte.pV TV� ��� �

1/0.40.4 0.4 1/0.42

2 2 3 3 3 23

4 2 22.63 LTT V T V V VT

� ��� �� � � � �� �� ���

(31) Proceso de compresión no-politrópico:

13 3 1 1 3 1

3

2 1 0.088 atm22.63

Vp V p V p pV

� � � � �

Balances energéticos (12) Proceso de expansión isobárica:

12 2 1

12 1 2 1

12 2 1

12 2 1

212

1

7( ) 0.08 1.987 300 170 cal 709 J2

( ) 1 2 2 atm L 48 cal 203 J5� ( ) 0.08 1.987 300 121cal 506 J27� ( ) 0.08 1.987 300 170 cal 709 J2

7� ln 0.08 1.987 ln2

p

V

p

p

Q nC T T

W p V V

U nC T T

H nC T T

TS nCT

� � � � � � � �

� � � � � � � �

� � � � � � � �

� � � � � � � �

� � � � �cal J2 0.39 1.64K K

� �

(sigue…)

� p (atm) V (L) T (K)

1 1 2 300

2 1 4 600

3 0.088 22.63 300

1 1 2

3

600 K

300 K

2 4 22.63

p (atm)

V(L)

0.088

- 327 -

Física Universitaria: Problemas de Física Termología - T00.22

(23) Proceso de expansión adiabática:

23

23 23

23 3 2

23 3 2

23

0� 121cal 506 J

5� ( ) 0.08 1.987 300 121cal 506 J27� ( ) 0.08 1.987 300 170 cal 709 J2

� 0

V

p

QW U

U nC T T

H nC T T

S

� � � � � �

� � � � � � � � � � �

� � � � � � � � � � �

(31) Proceso de compresión no-politrópico:

� �

311 3

31

31 31 31 31 31

31 31

31 123

área del trapecio

� 0 ya que

� 0� 0

1 = 1 0.088 22.63- 2 11.23 atm L 272 cal 1136 J2

cal J� � 0.39 1.64K K

UT T

HU Q W Q W

Q W

S S

�� �� ���� ��� � � �

� � � � � � �

� � � � � �

- 328 -

Física Universitaria: Problemas de Física Termología - T00.23

22. Un mol de gas perfecto biatómico, a 27ºC de temperatura y 10 atm de presión, se somete a los siguientes procesos reversibles: 1º. Un calentamiento isobárico hasta que alcanza una temperatura de 227ºC. 2º. Un enfriamiento adiabático hasta que se recupera la temperatura inicial. 3º. Una compresión isotérmica que cierra el ciclo. a) Representar gráficamente el ciclo en un diagrama de Clayperon. b) Determinar el rendimiento de una hipotética máquina que funcionase de acuerdo con el ciclo descrito y compararlo con el de una máquina reversible de Carnot que funcionase entre las mismas dos temperaturas extremas (i.e., 27ºC y 227ºC).

7 7 51 mol 1.405 2 2p Vn C R C R�� � � � �

a) En la figura adjunta hemos representado el ciclo en un diagrama p-V (Clayperon), recorrido en el sentido horario.

(A) AA

A

1 0.08205 2.400

L1

63 0nRTVp

� �� � �

(B) A B BA

A B A

500 2.46300

4.10LBV V TV VT T T

� " � � �

(C)

11

1 1 BB B C C C B

C

TT V T V V VT

�� �

�� �

� ��� �� � � �� ���

2.5

C

A AA A C C C

C

14.5004.103

71 L

1.6 a m

00

7 t

V

p Vp V p V pV

� ���� � ��� ���

� � �

b) El área encerrada por el ciclo representa el trabajo neto realizado (positivo). En la transformación isobárica AB el sistema absorbe una cantidad de calor Q1 (positivo). En proceso adiabático BC el sistema no intercambia energía en forma de calor. En el proceso de compresión isotermo CA el sistema cede una cantidad de calor Q3 (negativo).

A

C

1 B A

A3 CA

C

7( ) 1 1.987 500 300 1391 cal2

d 2.46d ln 1 2 300 ln 1073 cal14.7

p

V

V

Q nC T T

VVQ W p V nRT nRTV V

��� � � � � � � ������� � � � � � � � � ������ �

Puesto que se trata de un ciclo (,U = 0), el Primer Principio de la Termodinámica nos permite escribir

1 3 1391 1073 318 calW Q Q� � � � �

b) El rendimiento de esa máquina térmica sería: 1

3181391

23%WQ

�� � �

El rendimiento de la máquina de Carnot sería: 1 2Carnot

1

500 300 2500

05

4 %T TT

�� �

� � � �

p (atm) V (L) T (K)

A 10 2.46 300

B 10 4.10 500

C 1.67 14.7 300

V

p

300K500K

AQ1

Q2

B

C

W

- 329 -

Física Universitaria: Problemas de Física Termología - T00.24

23. Un mol de gas perfecto biatómico, a 27ºC de temperatura y 10 atm de presión, se somete a los siguientes procesos reversibles: 1º. Un calentamiento isocórico hasta que alcanza una temperatura de 227ºC. 2º. Un enfriamiento adiabático hasta que se recupera la temperatura inicial. 3º. Una compresión isotérmica que cierra el ciclo. a) Representar gráficamente el ciclo en un diagrama de Clayperon. b) Determinar el rendimiento de una hipotética máquina que funcionase de acuerdo con el ciclo descrito y compararlo con el de una máquina reversible de Carnot que funcionase entre las mismas dos temperaturas extremas (i.e., 27ºC y 227ºC).

Por tratarse de un gas biatómico: 7 5 7 1.402 2 5p VC R C R �� � � �

a) En la figura adjunta hemos representado el ciclo en un diagrama p-V (Clayperon), recorrido en el sentido horario. Determinamos los estados A, B y C:

(A) AA

A

1 0.08205 2.400

L1

63 0nRTVp

� �� � �

(B) A B BA

A B A

50010300

16.67 atmBp p Tp pT T T

� " � � �

(C)

11

1 1 BB B C C C B

C

TT V T V V VT

�� �

�� �

� ��� �� � � �� ���

2.5

C

A AA A C C C

C

8.85002.4630

2 L

2.79 at

0

m

V

p Vp V p V pV

� ���� � ��� ���

� � �

b) El área encerrada por el ciclo representa el trabajo neto realizado (positivo). En la transformación isocórica AB el sistema absorbe una cantidad de calor Q1 (positivo). En proceso adiabático BC el sistema no intercambia energía en forma de calor. En el proceso de compresión isotermo CA el sistema cede una cantidad de calor Q3 (negativo).

A

C

1 B A

A3 CA

C

5( ) 1 1.987 500 300 993.5 cal2

d 2.46d ln 1 1.987 300 ln 761.1cal8.82

V

V

V

Q nC T T

VVQ W p V nRT nRTV V

��� � � � � � � ������� � � � � � � � � ������ �

Puesto que se trata de un ciclo (,U = 0), el Primer Principio de la Termodinámica nos permite escribir

1 3 993.5 761.1 232.4 calW Q Q� � � � �

b) El rendimiento de esa máquina térmica sería: 1

232.4993.5

23%WQ

�� � �

El rendimiento de la máquina de Carnot sería: 1 2Carnot

1

500 300 2500

05

4 %T TT

�� �

� � � �

p (atm) V (L) T (K)

A 10 2.46 300

B 16.67 2.46 500

C 2.79 8.82 300

V

p

300K500K

A

pC

Q1

Q3

pA

B

C

W

pB

- 330 -

Física Universitaria: Problemas de Física Termología - T00.25

24. Tres moles de nitrógeno inicialmente a 27ºC y presión atmosférica se someten al ciclo constituido por las tres transformaciones siguientes: 1) calentamiento a presión constante hasta duplicar el volumen inicial. 2) una compresión isoterma hasta alcanzar el volumen inicial. 3) un enfriamiento isócoro que cierra el ciclo. Determinar el trabajo, el calor, y las variaciones de energía interna, entalpía y entropía en cada transformación y en el ciclo.

Datos: 3 moles de gas biatómico. cal cal

mol K mol Kcal

mol K

7 56.95 4.972 2

1atm L 101.328 cal 1.987

p VC R C R

R

� �

� � � �

� � �

Determinación de los estados 1, 2 y 3:

11

1

1 2 22 1

1 2 1

22 2 3 3 3 2

3

3 0.08205 300(1) 73.85 L1

(2) 2 300 600 K

(3) 2 1 2 atm

nRTVp

V V VT TT T V

Vp V p V p pV

� �� � �

� � � � �

� � � � �

Balances energéticos: (12) Proceso de expansión isobárica (�p=0)

12

12

12

� 1 (147.69 73.85) 73.85 atm L 1788 cal 427 J12 1� 3 6.95 (600 300) 6259 cal 1496 J12

� � 3 4.97 (600 300) 4471 cal 1069 J

� � 3 6.95 (600 300) 6259 cal 1496 J

6002� ln 3 6.95 ln301

W p V

Q nC TpU nC TVH nC Tp

TS nCp T

� � � � � � � �

� � � � � � �

� � � � � � �

� � � � � � �

� � � �cal J14.46 3.46

0 K K� �

(23) Proceso compresión isoterma (�T=0)

23

23

23

3ln 3 1.987 600 ln 2 2479 cal 592 J2322479 cal 592 J23 23

� � 0

� � 0

2479 cal J23� 4.13 0.99600 K K2

VW nRT

V

Q W

U nC TVH nC Tp

QS

T

� � � � � � � � �

� � � � �

� �

� �

�� � � � � �

p (atm) V (L) T (K)

1 1 74 300

2 1 148 600

3 2 74 600

600 K300 K

1 atm

2 atm

74 L 148 L

(3)

(1)(2)

- 331 -

Física Universitaria: Problemas de Física Termología - T00.26

(31) Proceso isocoro (�V=0)

31

31

31

031� 3 4.97 (300 600) 4471cal 1069 J31

� � 3 4.97 (300 600) 4471cal 1069 J

� � 3 6.95 (300 600) 6259 cal 1496 J

300 cal J1� ln 3 4.97 ln 10.33 2.47600 K K3

W

Q nC TVU nC TVH nC Tp

TS nCV T

� ������ � � � � � � � � ����� � � � � � � � � ����� � � � � � � � �����

� � � � � � � �

Balances energéticos en el ciclo completo:

12 23 31

12 23 31

691cal 165 J691cal 165 J

� 0 � 0 � 0

W W W WQ Q Q Q

U H S

� � � � � � �

� � � � � � �

� � �

- 332 -

Física Universitaria: Problemas de Física Termología - T00.27

25. Dos litros de helio (gas monoatómico) a la presión de 16 atm y 600 K se expansionan isotérmicamente hasta que su volumen es de 8 L y luego se comprime a presión constante hasta que su volumen y temperatura son tales que puede cerrarse el ciclo mediante una compresión adiabática. a) Dibujar el ciclo termodinámico reversible en un diagrama (p-V) b) Calcular la temperatura, presión y volumen en los estados que no son conocidos. c) Calcular los trabajos, calor y variaciones de energía interna, entalpía y entropía en cada una de las transformaciones elementales y en el ciclo.

Datos: Puesto que se trata de un gas ideal monoatómico (tres grados de libertad), serán:

1 1

1

cal

mol.Kcal

mol.K

2 5 4.972 2

3 2.98 2 2

2 3 2 1,63

16 2 0.65 moles0.08205 600

p

V

p

V

fC R R

fC R R

C fC fp VnRT

�� � �

� � �

� �� � � �

�� � �

Determinación de los estados 1, 2 y 3: 12 Isoterma:

1 11 1 2 2 2

2

418

a2 t6 mp Vp V p V pV

�� � � �

23 Isobara: 3 323 2

2 3 2

4.59600 344.6 K 8

V VV T TT T V

� � � �

13 Adiabática: 1 1 3 3p V p V� ��

31 35 53 1 1

3 11 3 3

1624

4.59 LV p pV VV p p

�� � � � � �� �� � ��� �� � � �� � ��� � �� � �� �� � � � �

Balances energéticos: (12) Proceso de expansión isotérmica (�T = 0):

22

12 12 111

1212 12 12

1

8d ln 0.65 1.987 600 ln 1074 cal2

1074 cal� � 0 � 1.79 K600

VQ W p V nRTV

QU H ST

� � � � � � �

� � � � �

(23) Proceso de compresión isobárica (�p = 0): � �

� � � �

23 3 2

23 2 3 2

23 3 2

23 23

3 33

23 2 22

0.65 4.97 345 600 825 cal

4 3.41 13.64 atm L 330.3 cal

� 0.65 2.98 345 600 495 cal� 825 cal

d d 345 cal� ln 0.65 4.97 ln 1.79 K600

p

V

p p

Q nC T T

W p V V

U nC T TH Q

TQ TS nC nCT T T

� � � � � � � �

� � � � � � � � � �

� � � � � � � �

� � �

� � � � � � �� �

p (atm)

V (L)

T (K)

1 16 2 600

2 4 8 600

3 4 4.6 345

2 3

1

4

16

8 2 V (L)

p (atm)

600 K

- 333 -

Física Universitaria: Problemas de Física Termología - T00.28

(31) Proceso de compresión adiabática (Q = 0):

� � � �

31

31 2 1 3

31 1 3

31 31

31

04 3.41 13.64 atm L 330.3 cal

� 0.65 2.98 345 600 495 cal� 825 cal� 0

V

QW p V V

U nC T TH QS

� � � � � � � � � �

� � � � � � � �

� � �

Q (cal) W (cal) ,U (cal) ,H (cal) ,S (cal/K)

1�2 (T = cte) 1074 1074 0 0 1.79

2�3 (p =cte) -825 -330 -495 -825 -1.79

3�1 (Q = 0) 0 -495 495 825 0

Ciclo 249 249 0 0 0

- 334 -

Física Universitaria: Problemas de Física Termología - T00.29

26. Un mol de un gas ideal monoatómico está inicialmente a 273 K de temperatura y 1 atm de presión. El gas experimenta una expansión isotérmica hasta que alcanza un volumen 2.5 veces el inicial. Luego se extrae calor a volumen constante, disminuyendo la presión. Finalmente se comprime adiabáticamente hasta volver al estado inicial. a) Dibujar la transformación en un diagrama pV. b) Determinar en cada transformación el calor y el trabajo intercambiados así como los incrementos de energía interna, entalpía y entropía. (Presentar los resultados en una tabla y en unidades del sistema internacional).

Gas Monoatómico: � �3 / 2 5 / 2 5 / 3V pC R C R �� � �

Determinación de los estados: Estado (1):

1 0.08205 273 22.4 L1

nRTpv nRT Vp

� �� � � �

Estado (2):

2 1

11 1 2 2 2 1

2

2.5 56.0 L

0.40 atm

V VVp V p V p pV

� �

� � �

Estado (3): 5/3

13 3 1 1 3 1

3

3 33

1 0.22 atm2.5

0.22 56.0 148 K0.08205

Vp V p V p pV

p VTnR

� �� � � ��� ���� � � �� ��� �� ��� � �

�� � �

Balances energéticos: Proceso (12) Expansión isotérmica:

212

1

12 12

1212

1

ln 497 cal 2078 J

� � 0

� 1.8 cal/K 7.6 J/K

VQ W nRTV

U HQST

� � � �

� �

� � �

Proceso (23) Enfriamiento isocórico: �

23 3 2

23 23

23 3 2

323

2

375 cal 1568 J0

� 375 cal 1568 J� 625 cal 2613 J

� ln 1.8 cal/K 7.6 J/K

V

p

V

Q nC T TW

U QH nC T T

TS nCT

� � � � � ��

� � � � �� � � � � �

� � � �

Proceso (31) Compresión adiabática:

� �

31

31 31

31 1 3

31 1 3

31

0� 375 cal 1568 J

� 375 cal 1568 J� 625 cal 2613 J� 0

V

p

QW U

U nC T TH nC T TS

�� � � � � �

� � � �� � � �

Ciclo completo: 122 cal 510 J122 cal 510 J

� 0� 0� 0

QW

UHS

� �� �

��

Estados p (atm) V (L) T (K)

(1) 1.00 22.4 273

(2) 0.40 56.0 273

(3) 0.22 56.0 148

V

p 1

2

3

273 K

- 335 -

Física Universitaria: Problemas de Física Termología - T00.30

27. Un mol de N2 , inicialmente a una temperatura de 20 ºC y una presión de 5 atm, es sometido a los siguientes procesos reversibles sucesivos: (i) expansión adiabática hasta una presión de 1 atm; (ii) calentamiento a presión constante hasta alcanzar de nuevo una temperatura de 20 ºC; (iii) calentamiento a volumen constante hasta que la presión es de 5 atm; (iv) enfriamiento a presión constante hasta el estado inicial. a) Dibujar el diagrama pV del ciclo. b) Calcular el trabajo realizado en el ciclo. c) Determinar las cantidades de calor absorbidos o cedidos en cada etapa y en el ciclo completo. d) Calcular las variaciones de entropía en cada etapa y en el ciclo completo.

Datos: 1 mol de gas biatómico.

J Jmol K mol K

J Jmol K mol K

5 5 8.314 20.792 27 7 8.314 29.102 2

7 1.45

V

p

p

V

C R

C R

CC

� �

� �

��� � � ������� � � �����

� � �

:

Determinación de los estados:

(1) 11

1

1 0.08205 293 4.81L5

nRTVp

� �� � �

(2) 1 1/1.4

11 1 2 2 2 1

2

54.81 15.17 L1

pp V p V V Vp

�� �

� � � ��� ���� � � �� ��� �� ��� � �

2 22

1 15.17 185K0.08205

p VTnR

�� � �

(3) 33

3

0.08205 293 24.04L1

nRTVp

�� � �

(4) 4 44

5 24.04 1465K0.08205

p VTnR

�� � �

Balances energéticos: (1�2) Expansión adiabática (Q=0):

12 120 � 0Q S� �

(2�3) Calentamiento isobaro (�p=0): � �23 3 2

3 33

23 2 22

29.10 293 185 3143 J

� d 293 J� ln 29.10 ln 13.38 K185

p

p p

Q nC T TTQ TS nC nC

T T T

� � � � � �

� � � � � �� �

(3�4) Calentamiento isocoro (�V=0):

p(atm) V (L) T (K)

1 5 4.81 293

2 1 15.17 185

3 1 24.04 293

4 5 24.04 1465

Q =0 T=293 K

V (L)

23

1

p(atm)

4 5

1

- 336 -

Física Universitaria: Problemas de Física Termología - T00.31

� �34 4 3

4 44

34 3 33

20.79 1565 293 24 360 J� d 1465 J� ln 20.79 ln 2033.45 K293

V

V V

Q nC T TTQ TS nC nC

T T T

� � � � � �

� � � � � �� �

(4�1) Enfriamiento isobaro (�p=0) �

1

4

41 1 4

11

41 44

34104 J

� d 293 J� ln 29.19 ln 46.83 K1465

p

T

p pT

Q nC T TTQ TS nC nC

T T T

� � � �

� � � � � � �� �

Ciclo completo:

ciclo

ciclo ciclo ciclo ciclo ciclo

ciclo

6601.3 J� 0 6601.3 J� 0

QU Q W W QS

� �

� � � � � �

- 337 -

Física Universitaria: Problemas de Física Termología - T00.32

28. Una máquina térmica ideal está constituida por un cuerpo de bomba que encierra un gas perfecto que evoluciona según un ciclo de Carnot. a) Deducir la expresión de su rendimiento en función de las temperaturas T y T’. b) ¿Son necesariamente iguales las áreas 233’2’2 y 144’1’1? ¿Por qué?

a) El rendimiento de un ciclo de Carnot viene dado por la expresión

WQ

��

en la que W es el trabajo realizado y Q el calor intercambiado con la fuente caliente. Formulando el Primer y Segundo Principios de la Termodinámica, escribiremos:

11

0

TQW Q QQ T

TQ Q Q TT T Q

TT

T

��

� )�� )� � � ��� )� � � ��� ) ) )� � � � ����

)�

� )

que es la expresión pedida. b) Las dos áreas indicadas son necesariamente iguales. Para demostrarlo, prestemos atención a que las transformaciones 2�3 y 4�1 tienen lugar entre las mismas temperaturas extremas y por lo tanto

23 41� �U U� �

y además, por ser adiabáticas, serán

23 41 0Q Q� �

por lo que, en virtud del Primer Principio �U Q� 23 41W W W� � y, como estos trabajos vienen representados en un diagrama de Clayperon por las áreas 233’2’2 y 144’1’1, dichas áreas serán igual (c.q.d.).

1

2

4 3

1’ 4’ 2’ 3’

T

T’

V

p

W

T

T’

Q’

Q

- 338 -

Física Universitaria: Problemas de Física Termología - T00.33

29. Una máquina térmica reversible, trabaja según un ciclo formado por las siguientes transformaciones reversibles:

1. Expansión isotérmica a 300 K 2. Expansión isoentrópica (adiabática) a 3500 cal/K 3. Compresión isotérmica a 200 K 4. Compresión isoentrópica a 1500 cal/K a) Representar el ciclo en un diagrama TS. b) Determinar el calor absorbido en la primera transformación. c) Ídem en la tercera. d) Calcular el trabajo realizado en dicho ciclo.

a) En el diagrama entrópico TS, el ciclo se representa por un rectángulo, recorrido en el sentido que se indica en la figura. En realidad, se trata de un ciclo reversible de Carnot, ya que está constituido por dos isotermas y dos adiabáticas. De la definición de entropía, se sigue que el calor intercambiado es

dd d �QS Q T S T ST

� � ��

b) En la expansión 12 el sistema cede energía en forma de calor:

� 512 1 12� 300 3500 1500 300 2000 6 10 calQ T S� � � � � � � �

c) En la compresión 34 el sistema absorbe energía en forma de calor: � 5

34 3 34� 200 1500 3500 200 2000 4 10 calQ T S� � � � � � � � � �

b) En el ciclo completo, la energía interna del sistema permanece constante, por lo que el Primer Principio de la Termodinámica nos permite calcular fácilmente el trabajo realizado:

512 34� 0 2 10 calU Q W W Q Q Q� � � � � � � �

S

T

300 K

200 K

3500 cal/K1500 cal/K

1 2

3 4

- 339 -

Física Universitaria: Problemas de Física Termología - T00.34

30. La figura muestra el ciclo teórico de una máquina térmica ideal en el diagrama T-S. El calor absorbido en la fuente caliente es de 924 cal y se cede calor a una fuente térmica a 0 ºC. Calcular: a) La temperatura T de la fuente caliente. b) El calor cedido a la fuente fría. c) El trabajo proporcionado por dicha máquina. d) El rendimiento de la máquina.

Se trata de un ciclo de Carnot, constituido por dos isotermas y dos adiabáticas. a) En el proceso isotérmico 1�2 será

1212

924� 308 K� 3

Q QS TT S

� � � �

b) Aplicamos la relación 2730 924 819 cal308

Q Q TQ QT T T

) ))� � � � � � � � �

)

c) A partir del Primer Principio de la Termodinámica, 924 819 105 cal 439 JW Q Q)� � � � � �

d) El rendimiento de una máquina está definido como el cociente entre el beneficio y el coste:

105 0.114 11.4%924

WQ

�� � � �

T’

T

Q

Q’

Wmáquina térmica

refrigerante

hogar

S(cal/K)5 2

3 4

1 2

- 340 -

Física Universitaria: Problemas de Física Termología - T00.35

31. Un acondicionador de aire funciona según un ciclo reversible de Carnot. Su potencia frigorífica es de 10 kilofrigorías/hora cuando extrae calor de un local a 24 ºC y lo cede al exterior a 35 ºC. a) Calcular la potencia que debe suministrar el motor. b) Con esa misma potencia del motor, calcular la potencia frigorífica cuando el local se encuentra a 21 ºC y el exterior a 40 ºC.

Datos: 2Mcal1 kfrigoría 1 Mcal 10

hQ� ��

a) Aplicamos el Primer y Segundo Principios de la Termodinámica a la máquina reversible:

1 2

1 2

1 2

11 2

2

0

Mcal10.37 10 0.37h

308 Mcal10 10.37297 h

Q Q W

Q QT T

W

TQ QT

�� � ���� �� � �������� � � � � ������� � � � � � � �����

� � �

� �

� �

De modo que la potencia del motor es: Mcal 4.18 MJ 1 h0.37 430 W

h 1 Mcal 3600 sP W� � � � ��

b) De nuevo, aplicamos el Primer y Segundo Principios de la Termodinámica:

1 21 2

1 21 2 1 2

1 2

11

1 2

22

2 1

0

313 Mcal0.37) 6.1313 294 h

294 Mcal0.37) 5.7294 313 h

Q Q WQ Q W

Q Q T T T TT T

TQ WT T

TQ WT T

�� � ��� �� � ��� �� �������� � � � � ��� � ����� � � � � ��� � ���

� � �� � �

� �

� �

� �

De modo que la potencia frigorífica es:

2Mcal kfrigorías5.7 5.7

h hQ � ��

W

T1=308 K

Q2

T2=297 K

Q1

W

T1=313 K

Q2

T2=294 K

Q1

- 341 -

Física Universitaria: Problemas de Física Termología - T00.36

32. Una máquina térmica reversible funciona intercambiando calor con tres focos térmicos cuyas temperaturas son: T1 = 500 K, T2 = 400 K y T3 = 300 K. La máquina toma una cantidad de calor Q1 = 700 kcal del primer foco y realiza un trabajo de 1 kWh. a) Calcular las cantidades de calor intercambiadas con los otros focos. b) Determinar el rendimiento de la máquina. c) Calcular los cambios de entropía en los distintos niveles térmicos y el total.

61 kWh 1000 3600 3.6 10 J 860 kcalW � � � � � � a) Aplicamos el Primer y el Segundo Principio de la Termodinámica:

1 2 3 2 3 1

3 31 2 2 1

1 2 3 2 3 1

0

Q Q Q W Q Q W QQ QQ Q Q Q

T T T T T T

� �� � � � � �� �� �� �� � � �� �� � � � � �� �� �� �� �

de modo que disponemos de dos ecuaciones con dos incógnitas (Q2, Q3), que nos conducen a

2 3 1 2 3

1 13 32 24 3

160 1601 1 1 1 1

700 3 4 12140400 300 500 4 3

Q Q W Q Q QQ QQ Q �

� �� � � � � �� �� �� � � � � �� �� �� � � � � �� �� �� �

2 31 13 4

160 1 1 16012 2320 kcal 12 2160 kcal

140 140Q Q� � � � �

� � b) Por definición de rendimiento de una máquina

abs 1 2

860 860 28.5%700 2320 3020

W WQ Q Q

> � � � � �

c) Cambios de entropía en cada nivel térmico:

11

1

22

2

33

3

700 kcal� 1.4500 K

2320 kcal� 5.8 � 0400 K

2160 kcal� 7.2300 K

QST

QS ST

QST

���� � � � ��������� � � � �������� �� � � � �����

T2=400K

T1=500K

T3=300K

Q3

Q2

Q1=700kcal

W=860kcal

- 342 -

Física Universitaria: Problemas de Física Termología - T00.37

33. Dos máquinas térmicas reversibles funcionan acopladas: una como máquina térmica y la otra como máquina frigorífica. La primera máquina absorbe 30 kcal de un foco a 600 K y cede calor a otro foco a 200 K. El trabajo producido por la máquina térmica se le suministra a la máquina frigorífica, que intercambia calor con dos focos a 200 K y 300 K. Determinar todos los intercambios de calor de las máquinas con sus focos caloríficos.

Aplicamos el Primer y Segundo Principios de la Termodinámica a la primera máquina:

1 2 2

1 2 22

1 2

30 20 kcal300 10 kcal0600 200

Q Q W Q W WQ Q Q QT T

� � � �� � ��� �� � ��� �� � � � �� � �� � � �� � ��� �� ����

y a la segunda, teniendo en cuenta que W' = -W = -20 kcal, de modo que

1 2 1 21

1 2 1 22

1 2

20 60 kcal0 40 kcal0

300 200

Q Q W Q Q QQ Q Q Q QT T

� ) ) ) �� �� ) )� � ��� �� ) � ��� �� � � ) )� � �) )� � � )� � � �� �� � ��) )� �����

W

600 K

200 K 200 K

300 K

W’

Q1

Q2

Q1’

Q2’

- 343 -

Física Universitaria: Problemas de Física Termología - T00.38

34. Disponemos de 100 g de nitrógeno (N2) a 25ºC y 30 atm. Los sometemos a una expansión adiabática brusca contra una presión exterior constante de 10 atm, hasta que el gas alcanza esta presión. Admítase que el gas tiene un comportamiento ideal. a) Determinar la temperatura final del gas. b) Calcular los cambios que experimentan la energía interna y la entropía del gas en el proceso de expansión. Datos: R = 0.08205 atm.L/(mol.K) = 1.987 cal/(mol.K ), M(N2)=28 g/mol.

Se trata de una expansión adiabática irreversible, contra una presión exterior constante, ya que no existe equilibrio entre la presión del gas y la presión exterior, de modo que tan sólo los estados inicial (i) y final (f) del sistema son estados de equilibrio.. a) Calcularemos la temperatura final (que será distinta de la que se obtendría en un proceso reversible) a partir del Primer Principio de la Termodinámica, con Q = 0, por tratarse de un proceso adiabático:

0U Q W W W� � � � � � � Por tratarse de una gas ideal que se expande contra una presión exterior constante, tendremos:

f i

f i fext f i f f i

f i i

� ( )

( )

VU nC T T

nRT nRT pW p V V p nR T Tp p p

� � ����� � � � � "� � �� �� � �� � � � � �� �� �� � �� �� �� � � ��

f ff i f i f i

i i

f

if i( ) ( )

V

pV V V

p pnC T T nR T T C R T C R Tp p

pC RpT T

C� � � �� �� �� �

��� � � � � � � � �� �� �� �� �� �

f

5 10 2.832 30 298 2987 3.2

541 K 32 º

02C

R RT

R�

�� �� � � �

b) La variación de energía interna en el proceso será:

f i100 5� ( ) 1.987 (241 298) 1011cal 4227 J28 2VU nC T T� � � � � � � � � � �

Para calcular la variación de entropía, partimos de la formulación del Segundo Principio en función de la entalpía:

� d d� d d dp pQ T pQ nC T V p S nC nRT T p

� � � � �

de modo que, para un proceso entre los estados inicial (i) y final (f), será:

f ff i

i i

100 7 241 10ln ln 1.987 ln lnT 28 2 29

cal J2.52 10.55K8 3 K0p

T pS S S nC nRp

� ��, � � � � � � � � �� �

produciéndose un incremento de entropía, por tratarse de un proceso de expansión espontánea (irreversible).

p

V

298 K

241 K f

i 30 atm

10 atm

Proceso irreversible

- 344 -

Física Universitaria: Problemas de Física Termología - T00.39

35. Dos moles de nitrógeno a 27ºC están contenidos en un cilindro cerrado por un émbolo móvil y aislado térmicamente del exterior. Inicialmente el gas está a una presión de 4 atm debido a una pesa que hay sobre el émbolo. Cuando se retira la pesa, el gas se expande bruscamente contra una presión exterior constante de 1 atm. a) Determinar el estado final del gas. b) Calcular el trabajo realizado por el gas y las variaciones de energía interna, entalpía y entropía. Datos: R = 0.08205 atm�L/mol�K = 1.987 cal/mol.K.

Se trata de una expansión adiabática irreversible, contra una presión exterior constante, ya que no existe equilibrio entre la presión del gas y la presión exterior, de modo que tan sólo los estados inicial (i) y final (f) del sistema son estados de equilibrio.. a) Calcularemos la temperatura final a partir del Primer Principio, con Q = 0 (proceso adiabático): � 0U Q W W W� � � � � � Por tratarse de una gas ideal que se expande contra una presión exterior constante, tendremos:

f

i

f i

f i fext f i f f i

f i i

f ff i f f ii f i

i i

� ( )

( )

( ) ( )V

p

V

V V V

U nC T T

nRT nRT pW p V V p nR T T

p p p

p pnC T T nR T T C R T C R T

p

pC

pT T

p

R

C

� �

"� � � � � �

� � � � � � �

����� � � � �� � �� �� � �� �� � �� �� �� � � ��

� � � �� �� �� �� �� �� �� �� �

o sea, f

5 1 2.752 4 300 3007 3.236 K 37

50º

2C

R RT

R�

�� �� � � �

El volumen final será: ff

f

2 0.08205 38.7 L2361

nRTV

p� �

� � �

b) Las variaciones de energía interna y de entalpía en el proceso serán:

f i

f i

5� ( ) 2 1.987 (236 300)

27

� ( )

639 cal 2673 J

895 cal 372 1.987 (236 30 )2

42 J0

V

p

U nC T T

H nC T T

� � � � � � � �

� � �

� � �

� � �� � � � �

El trabajo realizado durante el proceso de expansión es 2 73� 6 JW U� � � Para calcular la variación de entropía, partimos de la formulación del Segundo Principio en función de la entalpía:

� d d� d d dp pQ T pQ nC T v p S nC nRT T p

� � � � �

f ff i

i i

7 236 1� ln ln 2 1.987 ln ln2 300 4

cal J2.15 9.01K Kp

T pS S S nC nR

T p� ���� � � � � � � � ��� ����

produciéndose un incremento de entropía por tratarse de un proceso de expansión adiabática espontánea (irreversible).

p

V

300 K

236 Kf

i4 atm

1 atm

Proceso irreversible

38.7 L 12.3 L

- 345 -

Física Universitaria: Problemas de Física Termología - T00.40

V

2

1 3

473 K

373 K

857 K

8.71 atm

p

20 atm

3.88 L 7.03 L

36. Dos moles de un gas biatómico a 200ºC y 20 atm, se expansionan adiabática y reversiblemente hasta una temperatura final de 100ºC. a) Calcular la presión y el volumen finales. b) Si el proceso hubiese sido no adiabático y constituido por una transformación isobárica seguida de otra isocórica, calcular la cantidad de calor que intercambiaría el sistema hasta alcanzarse el mismo estado final. c) Por último, considérese un proceso de expansión adiabático y no reversible contra una presión exterior constante igual a la presión final del proceso en los apartados anteriores; ¿cuál será la temperatura final alcanzada? Determinar la variación de entropía en este proceso.

Gas biatómico: 5 7 7 1.42 2 5V pC R C R �� � � �

11

1

2 0.08205 473.15 3.88 L20

nRTVp

� �� � �

a) Por tratarse de un proceso adiabático, será 1 1

1 1 2 2p T p T� � � �� �� , de modo que

3.511

2 12

22

2

473.1520373.15

2 0.082

8.71atm

7.0305 3738.7

L1

Tp pT

nRTVp

�� ��� � � ��� ���� � �� ��� �� ��� � �

� �� � �

b) Determinamos la temperatura correspondiente al punto 3:

3 313 1

1 3 1

7.03 473.15 856.62 K3.88

V VV T TT T V

� " � � �

3 1

2 3

7252

( ) 2 1.987 (856.62 473.15)

( ) 2 1.987 (373.15 856.62)

5333.66 cal

4803.26 c530.

al40 calp p

V V

p V

Q nC T T

Q nC T TQ Q Q

� � � � � � �

� � � � � � �

��� � � ��� ���

c) Por tratarse de un proceso adiabático, será Q = 0; por tratarse de un proceso de expansión contra una presión externa constante, será W = p2 (Vf - V1). Entonces, aplicando el Primer Principio al proceso irreversible y utilizando la ecuación de estado en el estado final, se sigue

f 1 ext f 11 ext 1

ffext f f f

ext

� ( ) ( )VV

p

U Q W nC T T p V VnC T p VTnRTp V nRT V nC

p

� � � " � � � ��� ��� " ��� � " �����

5 1.9872 0.08205

f 72

f

3962 1.987 473.15 8.7 .85 K1 3.882 1.987

2 0.08285 396.8 7.4.71

L58

8

T

V

� � � � �� �

� �� �

El cambio que experimenta la entropía se calcula mediante una expresión general que relaciona los dos estados, inicial y final, de equilibrio; i.e.,

f f 52

1 1

396.85 7.48ln ln 2 1.987 ln 2 1.987 ln473

cal0.86. K15 3.88V

T VS nC nRT V

4 � � � � � � � � ��

- 346 -

Física Universitaria: Problemas de Física Termología - T00.41

37. En un sistema aislado, un foco a 127 ºC transfiere una cantidad de calor de 600 J a otro foco a 27 ºC, sin la obtención de trabajo y manteniéndose constantes las temperaturas de los focos. a) ¿Cuál es la variación de entropía del sistema (o del universo) en este proceso?. b) ¿Cuánto calor, como máximo, se puede transformar en trabajo? c) En el caso anterior, ¿cuál sería la variación de entropía del sistema (o del universo)?

Datos: T1 = 400 K, T2 =300 K, Q = 600 J a) En la figura se esquematiza el proceso mediante el cual el foco o subsistema caliente cede calor al foco o subsistema frío, manteniéndose constantes las temperaturas respectivas. Obviamente, se trata de un proceso irreversible en el que el foco caliente cede una cantidad de calor Q1 = -600 J y el foco frío recibe una cantidad de calor Q2 = +600 J. Así, la variación de entropía durante el proceso será

1 2

1 2

600 600� 1.5 2.0400 300

J0.5K

i

i i

Q Q QST T T

�� � � � � � � � �+

de modo que la entropía del sistema aumenta, como corresponde al proceso irreversible que tiene lugar en su seno. Puesto que el sistema está aislado, no intercambiando calor ni trabajo con su entorno, la variación de entropía del Universo coincide con la del sistema. b) Para calcular el .máximo rendimiento de la conversión de calor en trabajo, imaginamos una máquina reversible de Carnot que opere entre esas temperaturas, de modo que aplicando el Primer y Segundo Principios de la Termodinámica:

1 2 1 2

1 2 22 1

1 2 1

600 450 153

0 J

450 J000 600400

Q Q W W Q QQ Q TQ QT T T

� �� � � � � � �� �� �� �� � � �� �� � � � � � ��������� �

Se pueden transformar 150 J por cada 600 J que se toman del foco caliente, cediéndose 450 J al foco frío. c) En las condiciones del apartado anterior, al tratarse de un rendimiento máximo en la conversión de calor en trabajo, la variación de entropía será nula, como queda bien patente en la segunda de las ecuaciones (2º Principio) que hemos utilizado para calcularlo.

Entorno del sistema

T1 = 400K

T2 = 300K

Q Sistema aislado

T1 = 400K

T2 = 300K

Q1

Q2

W

- 347 -

Física Universitaria: Problemas de Física Termología - T00.42

38. a) Una máquina frigorífica irreversible, movida por un motor de 1 kW, extrae 50 kcal/min de un recinto frío. ¿Cuál es su rendimiento o eficacia? ¿Qué cantidad de calor recibe el foco caliente? b) Si el rendimiento de la máquina fuese la mitad del correspondiente a un ciclo de Carnot que operase entre los mismos límites de temperatura (-20ºC y 130ºC), ¿cuál sería el trabajo recibido por minuto por el sistema activo? ¿qué cantidad de calor se cedería entonces al foco caliente?

Datos: 250000 4.181000 W 3483 W

60W Q �

� � ���

a) Primer Principio:

1 2 348 4483 1 W000 3Q Q W� � � � �� � �

Eficacia de la máquina frigorífica:

2f

3483 3.48310

3 %00

48Q

W� � � � �

Rendimiento funcionando como máquina térmica:

1

1000 0.22 22 %4483

W

Q�� � � �

Obsérvese que se trata de una máquina irreversible, de modo que:

1 2

1 2

1 2

1 2

(irreversible)

(teorema de Clausius)

4483 3483 11.12 13.77 2.64 W/K 403 253

0

Q QT T

Q QT T

� � � � � � �

� -

� �

� �

b) Rendimiento de un ciclo reversible de Carnot que trabaje entre las mismas temperaturas:

1 2 2

1 1

2531 1 0.37 37 % 0.19403 2

T T TT T

�� �

� )� � � � � � � � �

Del Primer Principio de la definición del rendimiento se sigue:

1 2

1 2

11

(1 )

Q Q W

Q QWW Q

Q

�� �

�� ) ) )� ����� ) ) ) � � )�� ) ) ) )� ��� )���

� � �

� ����

21

50000 kcal61.43(1 ) 1 0.19 min

0.19

4280 W

94 8 W0 7 62

QQ

W

�� )�� ) � � � ��� )� ����� ) � � ����

��

W

T1

Q2

T2

Q1

W

T1=403K

Q2

T2=253K

Q1

W’

T1

Q’2

T2

Q’1

- 348 -

Física Universitaria: Problemas de Física Campo eléctrico. E01.1

1. Una esfera sólida, no conductora y de radio R, posee una densidad volumétrica de carga proporcional a la distancia desde su centro, i.e., % = Ar, siendo A una constante. Determinar el valor del campo eléctrico en el interior y en el exterior de la distribución de carga y representar su magnitud en función de r.

Aplicaremos el teorema de Gauss a una superficie gaussiana de radio r concéntrica con la esfera sólida:

02

2

0 0

d d d 4 14S S S

q qE S E S S qEr

E E r��� � �

� � � � � �� � �E S�� � �

siendo q la carga eléctrica contenida en la superficie gaussiana, que calcularemos descomponiendo la esfera sólida cargada en capas esféricas. En efecto la carga que pose una de tales capas, de radio r y espesor dr, será

2 3 3 4

0d d 4 d 4 d d 4 d

rq V Ar r r Ar r q q A r r Ar� � � � �� � � � � �� �

a) En el interior de la esfera, para r � R, será 4

2

02

0

14 4

ArE A rr�

� ��� �

b) En el exterior de la esfera, para r � R, la carga que contribuye al campo es 4Q AR�� , con independencia del valor de r, de modo que será

4

20

4

20

1 144

AREr

ARr�

���

� �

c) Justamente en la superficie de la esfera, para r = R, ambas expresiones anteriores conducen a

2

04E AR

��

dr

r

dq

E dS

E

rR

2

04AR�

- 349 -

Física Universitaria: Problemas de Física Campo eléctrico. E01.2

2. Un conductor esférico hueco tiene un radio interno R1 y un radio externo R2. En el centro de la cavidad existe otro conductor esférico macizo, de radio R0, que posee una carga +Q. a) Determinar el campo eléctrico y el potencial en un punto cualquiera P en función de la distancia r al centro de las esferas, desde r = 0 hasta r = �. b) Representar gráficamente E(r) y V(r) en función de r. Nota: No es necesario hacer los cálculos detallados para cada etapa.

Campo eléctrico Como es bien sabido (demostrable mediante el teorema de Gauss), a efecto del cálculo del campo eléctrico es como si toda la carga estuviese en el centro del conductor cargado.

Obviamente, el campo es nulo en el interior de cada conductor.

0

0 20 0

0 1 20

1

1 2

2 20 2

2 20

01

41

400

14

14

r R EQr R ERQR r R Er

r R ER r R E

Qr R ERQR r Er

��

��

��

��

* �

� �

* * �

� �

* * �

� �

* * ! �

Potencial eléctrico Para determinar el potencial, con un referencial nulo para una distancia infinita, procederemos en cada una de las regiones indicadas en la figura.

1 1 1

1

220 0 0

1 20 2

20 2 0 0 2 0

0 10 2 1

0

d 1 1( ) d4 4 4

14

1 d 1 1( ) d4 4 4 4

1 1 14

1( )4

rrr r

R R R

Rrr r

Q r Q QV r V R rr r r

QV R r RR

Q Q r Q QV r VR r R r

Q R r RR R r

QV rR

�� �� ��

��

�� �� �� ��

��

��

!! !

!

� �� �� � � � � � - * !� ��

� - -

� �� �� � � � � � � �� ��

� ��� �� � � - -� �� ���

� �

� �

E r

E r

� 02 1 0

1 1 r RR R

� ��� �� � -� �� ���

R0 R1

R2 O

V

r R0 R1 R2

E

r R0 R1 R2

1 23 4

- 350 -

Física Universitaria: Problemas de Física Campo eléctrico. E01.3

3. Una esfera conductora cargada se encuentra a un potencial de 300 V; a 10 cm de su superficie el potencial eléctrico es de 200 V. a) Determinar el radio la esfera conductora b) ¿Cuál es su carga? c) Calcular el valor del campo eléctrico en la proximidad de la superficie de la esfera.

Determinamos el potencial en la superficie de la esfera (V0) y a una distancia h de la misma (V):

00 0

0

14�1

4�

QVR V R hQ V RV

R h

$

��� ��� �� ;; ���� ��� ���

a) Determinamos el radio de la esfera:

20 c300 10 3 2 20200

mR R RRR

��

� " � � "

b) Calculamos su carga: 9

0 0 9

0.20 3004� 6.67 10 C =9 1

60

.67 nCRVQ � ��� � � �

c) Determinamos el campo eléctrico en la superficie de la esfera:

00

02

V15001 30m

04� 0.20

VQR R

E�

� � � �

300 V

R+h

200 V

- 351 -

Física Universitaria: Problemas de Física Campo eléctrico. E01.4

V1 V2

Q1 Q2

Antes de ponerlas en contacto

Después de haber estado en contacto

V’

Q1’ Q2’

4. Sobre dos esferas conductoras, de radios 0.10 m y 0.15 m, se depositan cargas eléctricas de +100 nC y +200 nC, respectivamente. Ponemos las esferas en contacto y luego las separamos de nuevo. Calcular la carga final y el potencial de cada esfera.

Durante este proceso 3 La carga neta total permanece constante 3 Se transfiere carga hasta que finalmente se igualan los potenciales

Escribimos las ecuaciones correspondientes:

n 1 2 1 2 1 2 n

1 2 1 2

0 1 0 2 1 2

(2 ec. 2 incogn.)1 1

4 4

Q Q Q Q Q Q Q QQ Q Q QVR R R R�� ��

� �) ) ) )� � � � � �� �� �� �� � ) ) ) )� �)� �� � �� �� ��� ��

Resolvemos1 las dos ecuaciones anteriores

1 2 n

1 2 1 2

Q Q QR R R R

) )� �

y sustituimos los valores dados en el enunciado, de modo que

11 n

1 2

22 n

1 2

10 300 120 �C2515 300 180 �C25

RQ QR R

RQ QR R

��� ) � � ��� ����� ) � � ��� ���

El potencial final, común a las dos esferas conductoras, es 9

911

0 1

1 120 109 10 10.8 kV4 10

QVR��

) �) � � � �

1 Hemos tenido en cuenta la conocida propiedad de la suma de fracciones: a c a cb d b d

3� �

3

- 352 -

Física Universitaria: Problemas de Física Campo eléctrico. E01.5

V1 V2

Q1 Q2

Antes de ponerlas en contacto

Después de haber estado en contacto

V’

Q1’ Q2’

V’

5. Sobre dos esferas conductoras, de radios 10 cm y 15 cm, se depositan cargas eléctricas de -100 nC y +350 nC, respectivamente. a) Calcular el potencial de cada esfera cuando están muy alejadas entre sí. b) Ponemos las esferas en contacto y luego las separamos de nuevo. Calcular la carga final y el potencial de cada esfera.

a) Calculamos los potenciales: 9

911 1

0 1

992

2 20 2

1 100 109 10 9 kV4 10

1 350 109 10 21 kV4 15 10

QVRQVR

��

��

� �� � � � �

�� � � � �

b) Cuando se ponen en contacto: 3 La carga neta total permanece constante 3 Se transfiere carga hasta que finalmente se igualan los potenciales

Escribimos las ecuaciones correspondientes:

1 2 1 2 1 2 1 2

1 2 1 1

0 1 0 2 2 2

(2 ec. 2 incogn.)1 1

4 4

Q Q Q Q Q Q Q QQ Q Q RVR R Q R�� ��

� �) ) ) )� � � � � �� �� �� �� � ) ) )� �)� �� � �� � )� ��� ��

Resolvemos1 las dos ecuaciones anteriores

1 2 n

1 2 1 2

Q Q QR R R R

) )� �

y sustituimos los valores dados en el enunciado, de modo que

11 n

1 2

22 n

1 2

10 250 100 nC2515 250 150 �C25

RQ QR R

RQ QR R

��� ) � � � ��� ����� ) � � � ��� ���

El potencial final, común a las dos esferas conductoras, es 9

911

0 1

1 100 109 10 9 kV4 10

QVR��

) �) � � � � �

1 Hemos tenido en cuenta la conocida propiedad de la suma de fracciones: a c a c

b d b d3

� �3

- 353 -

Física Universitaria: Problemas de Física Campo eléctrico. E01.6

6. Una carga eléctrica puntual, +q, está situada a una distancia D del centro de una esfera conductora de radio R. a) Determinar el potencial eléctrico al que se encuentra la esfera. b) Unimos la esfera a tierra mediante un hilo conductor largo y delgado (de influencia despreciable). Calcular la magnitud de la carga eléctrica inducida sobre la esfera. (Explicar y hacer los esquemas gráficos oportunos para cada apartado, indicando la posición de las cargas inducidas sobre la esfera.)

a) Todos los puntos de la esfera conductora en equilibrio están al mismo potencial, por lo que basta con determinar el potencial al que se encuentra su centro (O). Este potencial será igual al creado en O por la carga puntual +q; i.e.,

0

14

qVD��

�� [1]

más el creado por las cargas inducidas (de uno y otro signo) sobre la superficie de la esfera conductora, como se ilustra en la figura. Como la carga neta inducida es nula y todos los elementos de dicha carga se encuentran a la misma distancia del centro de la esfera conductora, su contribución al potencial en O será nula. Así el potencial de la esfera será el expresado por [1]. b) La esfera conductora, después de conectarla a tierra, quedará con una carga inducida negativa. Esto es así porque la toma a tierra suministra carga negativa que “neutraliza” la carga positiva inducida en la esfera, como se muestra en la figura. Cuando suprimamos la toma a tierra, la carga negativa (-Q) quedará “atrapada” sobre la superficie de la esfera (carga por inducción). El potencial de la esfera será ahora

0 0 0

1 1 1 04 4 4

q Q q QVD R D R�� �� ��

� �� � ��� � � � ��� ���

puesto que está conectada a tierra (referencial nulo de potenciales). En consecuencia, la carga negativa inducida será

q QD R

RQ qD

� �

disminuyendo, como era de esperar, cuando aumenta la distancia D.

D R

+q

D R

+q+Q-Q

Flujo de cargas negativas

- 354 -

Física Universitaria: Problemas de Física Campo eléctrico. E01.7

7. Disponemos de una esfera conductora, maciza, de radio R1, que posee una carga eléctrica neta +Q1, y de otra esfera conductora, hueca, de radios interior R2 y exterior R3, que posee una carga eléctrica neta +Q2. Inicialmente, las esferas están separadas y muy distantes la una de la otra. a) Dibujar esquemáticamente la distribución de carga en cada una de las esferas. b) Calcular el potencial al que se encuentra cada esfera y la d.d.p. entre ellas. c) Ahora, colocamos la esfera maciza en el interior de la hueca, en posición concéntrica, como se ilustra en la figura 2. Dibujar esquemáticamente la distribución de carga en cada una de las esferas. d) Determinar el campo eléctrico en la región comprendida entre ambos conductores concéntricos y calcular la d.d.p. entre ellos.

a) Tanto en la esfera maciza como en la hueca, la carga eléctrica se distribuye uniformemente sobre la superficie externa de las esferas, no existiendo carga eléctrica ni en el interior de los conductores ni en la superficie interna del conductor hueco. b) El potencial al que se encuentra la esfera maciza es el de su superficie (el mismo que en su interior) y viene dado por

11

0 1

14

QVR��

Del mismo modo el potencial al que se encuentra la esfera hueca (el mismo que en su interior, incluido el hueco) viene dado por

22

0 3

14

QVR��

y la d.d.p. entre ellas es

1 212 1 2

0 1 3

14

Q QV V VR R��

� ��� �� � � �� �� ���

c) Aparece una carga eléctrica inducida, –Q1, sobre la superficie interior del conductor hueco, de modo que el campo eléctrico en el interior del conductor hueco (al igual que en el interior del macizo) sea nulo (Teorema de Gauss). La carga eléctrica sobre la superficie exterior del conductor hueco será ahora Q2* = Q1 + Q2, ya que su carga neta de este conductor debe permanecer constante (Q2 = -Q1 + Q2*). d) Como consecuencia del Teorema de Gauss, la intensidad del campo eléctrico en un punto situado a una distancia r del centro común de las esferas, con R1 < r < R2, tan solo está determinado por la carga Q1, tiene dirección radial y vale

12

0

14

QEr��

La d.d.p. entre los dos conductores se calcula como la circulación del campo eléctrico entre los puntos 1 y 2, indicados en la figura, a lo largo de una línea de campo:

22

11

2 21 1 1

12 21 10 0 0 1 2

d 1 1 1d d 04 4 4

RR

RR

Q Q QrV E rr r R R�� �� ��

� �� � �� �� �� � � � � � � .� �� ��� � � � � � �E r�

de modo que la esfera maciza siempre está a mayor potencial que la hueca, con independencia de las cargas de una y otra.

R2

R3 Q2

R1

Q1

R1

Figura 1

R2

R3 Q2

Figura 2

R2

R3 Q2*

R1

+Q1 + +

+ +

+

+

+

+

+

+

+

++ +

+

+

+

+

+

+ +

-

-

- -

-

-

-

--Q1

1 2E

R1

R2

R3 Q2

+

+

++

+

+

++Q1

+ +

+

+

+

+

+

+

+

+

+

- 355 -

Física Universitaria: Problemas de Física Campo eléctrico. E01.8

8. Considérese un conductor aislado, como el que se indica en la figura, que posee una carga eléctrica +Q en equilibrio. a) ¿Cuánto vale el campo eléctrico en el interior del conductor? Razonar la respuesta. b) ¿Dónde se sitúa la carga eléctrica? Hacer un esquema de la distribución de la carga eléctrica en el conductor. ¿En que zonas de la superficie del conductor es mayor la densidad de carga? Razonar las respuestas. c) Aplicar el Teorema de Gauss para calcular la intensidad del campo eléctrico en las proximidades de la superficie exterior del conductor ¿En que zonas de la superficie del conductor es más intenso el campo eléctrico?

a) El campo eléctrico en el interior de un conductor en equilibrio es nulo. Esto es así porque, si el campo no fuese nulo, las cargas eléctricas se moverían bajo la acción del mismo y el conductor no estaría en equilibrio. b) La carga eléctrica se distribuye en la superficie del conductor. Este resultado es una consecuencia inmediata del teorema de Gauss, al aplicarlo a una superficie gaussiana en el interior del conductor. La densidad de carga es mayor en las zonas de mayor curvatura, tal como se ilustra en la figura. Es decir, tiende a acumularse en las zonas más “puntiagudas. A este efecto se le conoce como “poder de las puntas”. c) Consideramos una superficie gaussiana de forma cilíndrica, con una de sus bases en el interior de conductor y la otra fuera del mismo. Por ser el campo nulo en el interior del conducto, no hay flujo a través de la base interior de la superficie gaussiana. En el exterior del conductor, cerca de su superficie, el campo es normal a la superficie por ser ésta una superficie equipotencial, por lo que tampoco existe flujo a través de la superficie lateral del cilindro gaussiano. Siendo la densidad superficial de carga, el cálculo del flujo a través de la base exterior del cilindro y el teorema de Gauss nos permiten escribir:

0 0

dd d S EE S ��

���

� � �

Como consecuencia, el campo eléctrico será más intenso donde sea mayor la densidad de carga, esto es, en las zonas de mayor curvatura.

+

+

+

+

+ +

+ + + +++

Poder de las puntas

+ +

++

� E dS

E=0

conductor

Pie aislante

- 356 -

Física Universitaria: Problemas de Física Campo eléctrico. E01.9

9. Una esfera dieléctrica homogéneamente cargada (densidad de carga, %), de radio R y centrada en el sistema de coordenadas posee una oquedad también esférica de radio R/2, dispuesta tal como indica la sección transversal representada. Determinar: a) El campo eléctrico y el potencial en los puntos P del eje Oy. b) La velocidad mínima que se debe imprimir a una carga –q y masa m, situada en A, para que dicha carga salga del campo de acción del campo creado por la esfera.

a) Podemos considerar una esfera de radio R, sin oquedad, con carga positiva Q1, y otra esfera de radio R/2, correspondiente a la oquedad, con carga negativa –Q2, siendo

�3

31 2

4 423 3

RQ R Q� � � �� �

Calculamos el campo y el potencial eléctricos creados por las distribuciones esféricas y homogéneas de cargas Q1 y Q2 del mismo que si fuesen cargas puntuales ubicadas en el centro de las correspondientes esferas.

3 31

1P 2 2 20 0 0

3 31

1P0 0 0

41 1 134 4 3

41 1 134 4 3

RQ REy y y

RQ RVy y y

� � ��� �� �

� � ��� �� �

���� � � ��������� � � ������

� �

� �

� �

� �

33

22P 2 2 2

0 0 0

33

22P

0 0 0

41 1 13 24 4 3 82 2 2

41 1 13 24 4 3 82 2 2

RQ RER R Ry y y

RQ RVR R Ry y y

� � ��� �� �

� � ��� �� �

��� ���� � � � ���� � � ������ �� �� � � � ���� � � ����

Aplicamos el Principio de Superposición:

�P 1P

3

220

2P1 1

3 8 2

E Ry R

Ey

E ��

� �� �� ��� �

�� ��

� � � �0

P

3

1P 2P1 1

3 8 2

RV VR

Vy y

��

� �� �

�� � � �� ��� ��

b) Aplicando el Principio de conservación de la energía (potencial electrostática +cinética), y teniendo en cuenta que 0 y 0v V! !� � , se sigue

2 2 2 AA

1 1 02 2

2qVqV mv qV mv vm! !� � � � � � �

con �

3 2

A0 0

1 13 48 2

R RVRR R

� �� �

� �� �

� � �� �� ��� ��

de donde 2

02R

mv q�

��

P A y

R z

- 357 -

Física Universitaria: Problemas de Física Campo eléctrico. E01.10

10. Un plano infinito situado en el plano de coordenadas xy posee una densidad de carga superficial uniforme ;1 = 354 nC/m2. Un segundo plano también infinito está situado en el plano xz y su densidad de carga es ;2 = 177 nC/m2. Determinar el vector campo eléctrico en un punto genérico P de coordenadas (x, y, z), utilizando el teorema de Gauss.

a) Consideramos una superficie gaussiana de forma cilíndrica, de generatriz infinitesimal y perpendicular al plano cargado y con cada una de sus bases a cada lado del plano, como se ilustra en la figura. Por ser el campo perpendicular al plano (razones de simetría), no hay flujo a través de la superficie lateral del cilindro gaussiano. Siendo la densidad superficial de carga, el cálculo del flujo saliente (+) a través de cada una de las bases del cilindro y el teorema de Gauss nos permiten escribir:

0 0

dd d d 2 d2

SE S E S E S E� ��

� ��� � � �

que es la expresión del campo eléctrico creado por un plano infinito cargado con una densidad de carga ; vemos que el campo es independiente de la distancia al plano. b) El campo creado en un punto cualquiera del espacio viene dado por la superposición de los campos creados en dicho punto por cada uno de los planos:

1 21 2

0 0

2 2� �� �

� �E j E k

�1 21 2 1 2

0 0 0

12 2 2� �

� �� � �

� � � � � �E E E j k j k

con ;1 = 354�10-9 C/m2, ;2 = 177�10-9 C/m2 y :0 = 8.85�10-12 C2/N.m2, de modo que

� �912

1 354 1 kV20 1077 102 8.85 10 m

��� � � �

��

�E kj k j

o bien kV 1022.36 arctg 26.6ºm 20

E �� � �

en todos los puntos del espacio.

z

y

x

;1

;2

P(x,y,z)

+ + + + +

+ +

+

+

+

E

dS

+

+ +

z

y

x

;1

;2

P(x,y,z)+

+ + + +

+ +

+

+

E2

E1

E

- 358 -

Física Universitaria: Problemas de Física Campo eléctrico. E01.11

11. Dos hilos conductores rectilíneos, indefinidos y paralelos, separados por una distancia de 1 m en un medio de constante dieléctrica 3 (� r=3) se ejercen una fuerza atractiva de 1.08�10-7 N por cada metro de longitud. Sabiendo que uno de ellos tiene una distribución uniforme de carga � = 3�10-9 C/m, calcular la carga existente en un tramo de 5 m del otro hilo.

Mediante el teorema de Gauss determinamos el campo eléctrico creado por uno cualquiera de los hilos, digamos el primero, a una distancia h del mismo:

0

(Teorema de Gauss) d q�

�� E S��

1 11

11 r 0 0

1

122

con y 3

qE hl Eh

ql

��

� ��

� � � � �

� �

� � �

La fuerza que actúa sobre un elemento de longitud l2 y carga q2 =�2l2 en el segundo hilo es:

1 1 22 2 2

2

1 12 2

FF q E lh l h� ���

�� ��� � �

i.e., la fuerza por unidad de longitud entre ambos hilos.

Despejando �2 de la expresión anterior, ...

�12

7 92 9

1 2

2 2 3 8.85 10 1 1.08 10 6 10 C/m =3 1

6 n0

C/mh Fl

�� ��

�� �

� � � �� � � � �� � � �

y, en 5 m de hilo, tendremos

2 6 5 30 nCq �� � � �

E

h

E

F/l2l2

- 359 -

Física Universitaria: Problemas de Física Campo eléctrico. E01.12

12. Tres distribuciones lineales de carga, indefinidamente largas, y paralelas al eje Z, poseen unas densidades lineales de carga uniformes de valores �, � y -�. Sabiendo que las rectas pasan por los puntos A, B y C respectivamente. Determinar: a) Campo eléctrico en el origen de coordenadas. b) Trabajo necesario para llevar un electrón desde el origen de coordenadas hasta el punto de coordenadas (0, 0,1).

a) Mediante el Teorema de Gauss, determinamos el campo eléctrico creado por uno cualquiera de los hilos a una distancia r del mismo:

0

d d d qE S E S�

� � �� � �E S�� � �

0 0

1 224

qE rl Er�

�� ��

� �

La dirección del campo eléctrico creado en O por cada una de las distribuciones de carga se indica en la figura y sus módulos son:

A B C0 0

1 2 1 24 4 12

E E E� ��: �:

� � �

El campo eléctrico resultante (E) en el origen de coordenadas viene dado por la suma vectorial

A B C� � �E E E E

tiene la dirección del eje x negativo y su módulo es

A B C0 0 0

2 1 2 1 2 2cos 45º cos 45 1 24 2 2 42 2

E E E E � � ��� �� ��

� ��� �� � � � � � � � �� �� ����

0

��:

� �# E i

b) Puesto que el campo eléctrico E es perpendicular al eje z en todos los puntos de éste (además, E es constante), la circulación de E entre dos puntos cualquiera del eje z será nula, lo que significa que el eje z es equipotencial; i.e.,

(0,0,1)

0 (0,0,0)d 0 cte.V V V� � � � �� E r�

En consecuencia, el trabajo realizado por el campo sobre un electrón (o cualquier otra carga) al desplazarse a lo largo de dicho eje será nulo; i.e.,

0( ) 0W q V V� � �

� A(1,1)

B(1,-1)

C(-1,0)

-�

y

x

E

r

dS

l

� A(1,1)

B(1,-1)

C(-1,0)

-�

y

x

EB

EA

EC

2

2

- 360 -

Física Universitaria: Problemas de Física Campo eléctrico. E01.13

13. Sea una varilla recta y delgada de longitud l, uniformemente cargada con una carga eléctrica +q. a) Encontrar la expresión del potencial eléctrico en el punto P en función de su distancia x al extremo de la varilla. b) A partir de la expresión anterior, obtener la expresión del campo eléctrico en P en función de dicha distancia x.

Descomponemos la varilla en elementos de longitud dA, que soportan una carga elemental dq, situados a una distancia A del extremo izquierdo de la varilla, tal como se indica en la figura. Siendo � la densidad lineal de carga, podemos escribir

d d con qql

� A �� �

a) Cálculo del potencial. El potencial en P se obtiene como suma (escalar) de las contribuciones de todos loe elementos a lo largo de la varilla. Procedemos por integración:

1 200 0 0 00

1 d d ln( ) ln4 4 4 4

llq l xV l x

l x l x x� � � �

��� � �� � �� ��

�� � � � � � �

� � � �� �

0

ln4

q l xVl x��

��

b) Cálculo del campo. Expresamos el campo eléctrico como el gradiente del potencial eléctrico con signo negativo; esto es,

d ya que ( )dVV E V V xx

� �� � � �E

Procedemos por derivación:

0 0 0

d d 1 1lnd 4 d 4 4 ( )V q l x q q lEx l x x l l x x l x l x�� �� ��

� �� �� ����� � � � � � � � � �� �� ��� � ��� � ��

0

14 ( )

qEx l x�:

# �

P

l x

l x

dA A P

- 361 -

Física Universitaria: Problemas de Física Campo eléctrico. E01.14

14. Consideremos tres varillas no conductoras, cada una de ellas de longitud l y cargadas uniformemente con cargas eléctricas +Q, +Q y –Q, respectivamente, como se indica en la figura. Determinar el campo eléctrico en el punto P equidistante de las tres varillas.

Por razones de simetría, los campos eléctricos creados en P por cada una de las tres varillas cargadas tienen todos la misma magnitud (E1 = E2 = E3 = E) y las direcciones indicadas en la figura. Así, el campo eléctrico resultante tiene una intensidad:

2 2T 2 1 5E E E� �

y forma un ángulo 1arctg 26.57º2

�� � con la horizontal.

Para calcular el valor de E, descomponemos la varilla en elementos de longitud dx, que poseen una carga dq = �dx, siendo � =Q/l la densidad lineal de carga. El campo elemental creado por tal elemento en P será

22 2 2

0 0 0 0

2

1 d 1 d 1 d 1d cos d4 4 4 4

dcon cos / cos ; tg dcos

q x xEs s h h

h s s h x h x h

� � �� �

�� �� �� ��

�� � �

��� � � � ������� � � � �����

En tanto que calculemos el campo en puntos de la mediatriz de la varilla, las componentes paralelas a ésta se compensarán por simetría, y el campo será perpendicular a la varilla, de modo que, al integrar todas las contribuciones elementales, tendremos:

1 2 /44/4

0 0 0 04

2 2 2d cos cos d sen4 4 4 2 2 4

E Eh h h h

��

� �

� � � �� � � �

�� �� �� ����

� ��� �� � � � � �� �� ���� � �

con h = l/2 y � =Q/l, de modo que

20

22

QEl�:

T 20

1052

QE El�:

� �

P -Q+Q

+Q

s

x

xdx

dE dE cos�

h=l/2

l

P

P -Q +Q

+Q

E2

E1 E3

ET

- 362 -

Física Universitaria: Problemas de Física Campo eléctrico. E01.15

15. Un anillo de alambre fino, de radio R, posee cierta carga eléctrica +Q. ¿Cómo se moverá una carga puntual, de masa m y carga eléctrica –q, que se encuentra inicialmente en reposo en un punto situado sobre el eje del anillo, a una distancia x << R de su centro?

Por razones de simetría, la fuerza que ejerce la carga +Q del anillo sobre la carga puntual –q es

20

1 cos4

QqFr

���

� �

donde el signo negativo indica que la fuerza es atractiva, tal como se muestra en la figura, con cos /x r�� , de modo que

�3/23 2 20 0

14 4

Qq Qq xF xr R x�� ��

� � � ��

Si consideramos tan sólo distancias x << R, de modo que la carga puntual se encuentra siempre cerca del centro del anillo, la expresión anterior se reduce a

3 3 30 0 0

con 4 4 4Qq x Qq QqF x kx k

R R R�� �� ��

� ��� �� � � � � � �� �� ���

Así pues, se trata de una fuerza regida por la ley de Hooke (fuerza dirigida hacia la posición de equilibrio cuyo módulo es proporcional a la elongación). Bajo la acción de dicha fuerza, la ecuación del movimiento de la carga puntual es

0F mx kx mx mx kx� � � � ��� �� �� ¨ que es la ecuación diferencial de un movimiento armónico simple de frecuencia

304

k Qqm mR

���

� �

y periodo 3

042 2 mRTQq���

��

� �

Método de la energía potencial del sistema:

1/22 2

1/2 2 22 20 0 00

1 11 14 4 4 24

Q q Qq Qq x Qq xEr R R R RR x�� �� ����

�� � � �� � �� �� �� � � � � � = � �� �� �� �� �� �� � �

de modo que 2 30 0

dd 4 4E Qq x QqF xx R R R�� ��

� ��� �� � = � � �� �� ��� , y sigue como antes…

+Q

-q

r

R

x

E

F

x

- 363 -

Física Universitaria: Problemas de Física Campo eléctrico. E01.16

16. Consideremos un disco de radio R y espesor despreciable con densidad superficial de carga eléctrica uniforme en una de sus caras. a) Determinar el potencial en un punto del eje de simetría de revolución del disco que está situado a una distancia z del mismo. b) A partir del resultado del apartado anterior, sin recurrir a métodos de integración, determinar el campo eléctrico en los puntos de dicho eje. c) Verificar los resultados para z � 0 y para z � �.

a) Descomponemos el disco en coronas circulares concéntricas, de radio r y ancho dr, como se ilustra en la figura. Cada una de esas coronas posee una carga eléctrica infinitesimal, d d 2 dq S r r� � �� � equidistante del punto P donde calculamos el potencial:

0 0 0 0

1 d 1 2 d dd d4 4 2 2

q r r r rV ss s s

�� � ��� �� � �

� � � �

donde hemos tenido en cuenta que 2 2 2 d ds r z s s r r� � � .

Integrando �2 2

2 2

0 0

d2 2

R z

z

V s R z z� �� �

� � � ��

b) Obtenemos el campo a partir del potencial mediante la operación gradiente, i.e., V� ��E . Por la simetría que presenta el problema, el campo en los puntos del eje z está

dirigido a lo largo de dicho eje, por lo que podemos escribir: ddz

V VEz z

�� � � �

de modo que, efectuando la derivada, es

�2 2

2 20 0

d 12 d 2

zE R z zz R z

� �� �

� ��� ��� � � � � � �� ���� �

c)

0 0

(plano cargado infinito)para 02 2

para 0 0

Rz V E

z V E

� �� �

��� � ������ ! � ����

R r

P s

dq= d

z

E

- 364 -

Física Universitaria: Problemas de Física Campo eléctrico. E01.17

17. Una barra dieléctrica de longitud L está colocada perpendicularmente a una distribución lineal de carga uniforme e infinitamente larga, de densidad lineal de carga � positiva. El extremo más próximo de la barra a la carga lineal dista de ésta D. La barra posee una carga total Q, también positiva y distribuida uniformemente en toda su longitud. Determinar la fuerza que se ejerce sobre la barra.

Comenzamos encontrando la expresión de la intensidad del campo eléctrico a una distancia y de una distribución lineal de carga uniforme e infinitamente larga. Aplicamos el teorema de Gauss a una superficie gaussiana de forma cilíndrica cuyo eje sea la propia distribución lineal de carga: esto es,

0 0lateral lateral

d d d 22S

lE S E S E yl Ey

� ��

� ��� � � � �� � �E S�� � �

Puesto que el campo eléctrico no es uniforme a lo largo de toda la barra dieléctrica, para calcular la fuerza ejercida sobre la barra debemos proceder por integración, descomponiéndola en elementos de longitud dy de carga d dq y�)� , con /Q L�) � :

0 0 0

d dd d ln2 2 2

D L

D

y y D LF E q Fy y D

�� �� ���� �� ��

�) ) ) �� � � ��

o sea

0 0

ln ln2 2

D L Q D LFD L D

�� ��� ��

) � �� �

+Q

L

D

Q

L

Dy

dy

E

l

y�

- 365 -

Física Universitaria: Problemas de Física Campo eléctrico. E01.18

18. Un plano infinito posee una densidad de carga superficial de +4.5 nC/m2 y coincide con el plano yz en el origen; un segundo plano infinito posee una densidad de carga superficial de -4.5 nC/m2 y se localiza en un plano paralelo al plano yz en x = 2 m. Determinar la fuerza eléctrica ejercida sobre una carga puntual de 10 nC situada en: (a) x = 1,8 m y (b) x = 5 m.

a) El campo eléctrico en la zona comprendida entre los dos planos paralelos es uniforme y su intensidad es

9

int 120

4.5 10 508 V/m8.854 10

E ��

�� � �

y la fuerza ejercida sobre la partícula cargada es 9 610 10 5 508 .08 10 NF qE ��� � � �� �

b) El campo eléctrico fuera de la zona comprendida entre los dos planos paralelos es nulo, por poseer estos densidades de cargas iguales y opuestas. Obviamente, la fuerza sobre la partícula cargada también será nula.

B A

E=0

E=508 V/m

- 366 -

Física Universitaria: Problemas de Física Capacidad eléctrica. E02.1

1. Una esfera maciza conductora de radio R, situada en el vacío, está cargada con una carga +Q. a) Determinar

el campo eléctrico y el potencial que existen en un punto P, exterior a la esfera, situado a una distancia r del centro de la misma. b) Ídem en un punto P interior a la esfera. c) Calcular la capacidad del conductor. d) ¿En qué cambiarían los resultados anteriores si la esfera conductora fuese hueca?

a) Aplicaremos el teorema de Gauss a una superficie gaussiana de radio r concéntrica con la esfera sólida:

0

2

2

0

0

d d d 4

14

S S S

Q QE S E S S

QEr

E E r�

� �

� � � � �

# �

� � �E S�� � �

siendo Q la carga eléctrica contenida en la superficie gaussiana, que es la que posee el conductor. La diferencia de potencial se calcula como la circulación del campo:

( ) ( )V r V� !

0 0

20

dd d4

1( 1)4 4

r

r r

r

Q rr

QV rr

Qr

� �� �

! !

!

!

� � �

� �� �# � �� ��

� � �E r E r� �

b) Aplicaremos el teorema de Gauss a una superficie gaussiana de radio r < R concéntrica con la esfera sólida:

0

0 0dS

Q E�

� � # �� E S��

por se nula la carga eléctrica contenida en la superficie gaussiana, ya que la carga del conductor se sitúa en su superficie. La diferencia de potencial se calcula como la circulación del campo:

( ) ( ) dr

RV r V R� � � E r�

0

14

QR��

por lo que el potencial del conductor es el mismo en todos sus puntos, incluida su superficie. c) Por definición, la capacidad de un conductor viene dada como el cociente de la carga que posee y el potencial al que se encuentra (con V� = 0); esto es,

0

0

4

4

Q QC RQVR

��

��

� � �

de modo que, en el caso de una esfera conductora, es directamente proporcional a su radio. d) Los resultados anteriores no se modifican, ya que la carga Q seguirá situándose en la superficie externa del conductor, presentándose la misma distribución de carga tanto si el conductor es hueco como macizo.

+Q

r E

+Q r

E=0

- 367 -

Física Universitaria: Problemas de Física Capacidad eléctrica. E02.2

2. Calcular la energía electrostática asociada a un conductor esférico, de radio R, que posee una carga eléctrica

+Q.

El potencial al que se encuentra la esfera conductora es

0

14

QVR��

La esfera cargada se comporta como un condensador cuya otra armadura se encuentra en el infinito. La energía almacenada por un condensador cargado viene dada por

2

0 0

1 1 14 82

12

QU VR

QR

QQ�� ��

� � �

Otro método En el interior del conductor, el campo eléctrico es nulo, en tanto que en su exterior el campo eléctrico viene dado por la expresión

20

14

QEr��

siendo r > R, la distancia al centro de la esfera. La densidad de energía en el campo eléctrico viene dada por la expresión

2 22

0 0 2 2 4 2 2 40 0

1 1 12 2 16 32

Q Qu Er r

� �� � � �

� � �

que decrece con la cuarta potencia de la distancia, por lo que para determinar la energía asociada a todo el espacio exterior al conductor deberemos proceder por integración. La energía asociada a una capa esférica de radio r y espesor dr, tal como se ilustra en la figura, cuyo volumen es 2d 4 dr r��� , será

2 2 2

22 2 4 2 2 4 2

0 0 0

dd d 4 d32 32 8

Q Q Q rU r rr r r

�� � � � ��

� � ��

y la energía asociada a todo el espacio exterior al conductor será

0

2

0

2 2

20

d 18 8

18R

R

Q r QUr r

QR���� ��

!! � �

� �� � � �� ��

r dr

2 6 5 30 nCq �� � � �

- 368 -

Física Universitaria: Problemas de Física Capacidad eléctrica. E02.3

3. Una esfera conductora, de radio R, que posee una carga eléctrica +Q, está envuelta por una capa dieléctrica

de espesor 3R y permitividad relativa igual a 3. El conjunto se encuentra aislado en el aire, a una distancia muy grande de cualquier otro conductor. a) Dibujar esquemáticamente la distribución de carga eléctrica en el conductor y la polarización en el dieléctrico. b) Expresar el campo eléctrico en el dieléctrico y en el aire en función de la distancia r al centro de la esfera conductora. c) Calcular el potencial al que se encuentra la esfera conductora. d) Calcular la capacidad eléctrica de la esfera conductora.

a) La carga eléctrica +Q se sitúa sobre la superficie de la esfera conductora, tal como se ilustra en la figura. La capa de material dieléctrico se polariza, orientán-dose los dipolos en la dirección radial, dando lugar a la aparición de carga de polarización, de signos opuestos, en las superficies interior y exterior de la capa dieléctrica. b) Campos eléctricos:

3 en el dieléctrico: d 2

1 44

QE R r Rr��

� * *

3 el aire: a 20

1 44

QE r Rr��

� .

c) Calculamos el potencial de la esfera conductora como la circulación del campo eléctrico desde su superficie hasta el infinito (potencial de referencia nulo), de modo que

V V!�4

2 240

d dd d4 4

R R

R R R

Q r Q rr r�� ��

! !

!� � � � �� � � �E r E r� �

integramos 4

4 0 0

r 0 0 0 r

1 1 1 1 14 4 4 4 4 4

3 1 1 3 14 4 4 4 4 4

R R

R

Q Q Q QVr r R R R

Q Q QR R R

�� �� �� ��

�� � �� �� �

!

� � � � � ���� � � �� � � � � ��� ��� � � � � � � � ��� �� � � �� �� ���

y sustituimos r 3� �

00

0

1 3 114 3 4 28

V Q VR

QR ����

� ���� � � ��� ���

siendo V0 el potencial que tendría la esfera conductora en el aire (vacío) sin la envuelta dieléctrica. d) La capacidad de la esfera conductora será

00

0 2/ 8

8Q QC CV Q R

R����

� � � �

siendo C0 la capacidad que tendría la esfera conductora en el aire (vacío) sin la envuelta dieléctrica.

E 4R

conductor dieléctri

aire

+

+ ++ +

V

QR

Ed

Ea

- 369 -

Física Universitaria: Problemas de Física Capacidad eléctrica. E02.4

4. Una barra dieléctrica de longitud L está colocada perpendicularmente a

una distribución lineal de carga uniforme e infinitamente larga, de densidad lineal de carga � positiva. El extremo más próximo de la barra a la carga lineal dista de ésta D. La barra posee una carga total Q, también positiva y distribuida uniformemente en toda su longitud. Determinar la fuerza que se ejerce sobre la barra.

Comenzamos encontrando la expresión de la intensidad del campo eléctrico a una distancia y de una distribución lineal de carga uniforme e infinitamente larga. Aplicamos el teorema de Gauss a una superficie gaussiana de forma cilíndrica cuyo eje sea la propia distribución lineal de carga: esto es,

0 0lateral lateral

d d d 22S

lE S E S E yl Ey

� ��

� ��� � � � �� � �E S�� � �

Puesto que el campo eléctrico no es uniforme a lo largo de toda la barra dieléctrica, para calcular la fuerza ejercida sobre la barra debemos proceder por integración, descomponiéndola en elementos de longitud dy de carga d dq y�)� , con /Q L�) � :

0 0 0

d dd d ln2 2 2

D L

D

y y D LF E q Fy y D

�� �� ���� �� ��

�) ) ) �� � � ��

o sea

0 0

ln ln2 2

D L Q D LFD L D

�� ��� ��

) � �� �

+Q

L

D

Q

L

Dy

dy

E

l

y�

- 370 -

Física Universitaria: Problemas de Física Capacidad eléctrica. E02.5

5. Un condensador plano, de dieléctrico el vacío, tiene una superficie de armaduras S y una separación entre

ellas h (muy pequeña). Si en sus armaduras se depositan unas cargas eléctricas +Q y –Q, hállense: a) La fuerza de atracción que se ejercen entre sí las armaduras. b) El trabajo que hay que realizar para separar las armaduras al doble (2h). c) Variación de la energía eléctrica del condensador al separarse las armaduras.

La intensidad del campo eléctrico entre las armaduras del condensador viene dada por

0 0

QES

�� �

� �

de modo que es independiente de la separación h entre las armaduras, en tanto que ésta sea suficientemente pequeña. La densidad de energía (u) almacenada en el campo eléctrico está dada por la expresión

2 2 22 2

0 02 20 0 0

d 1 1 1 1 1d d d dd 2 2 2 2 2U Q Q Qu E U E S h h

S S S� �

� � �� � � " � � �B

B

c) Variación de la energía eléctrica del condensador al separar las armaduras al doble (,h = h):

2 2

00

1d 1�2 2

dQU h U hSS

Q� �

� " �

b) Trabajo que hay que realizar para separar las armaduras al doble (,h = h): 2 2

0 0

1 1�2 2

Q Q hU W hS S� �

� � 4 �

a) Fuerza de atracción que se ejercen entre sí las armaduras: 2

0

d 1d 2U QFh S�

� � � �

donde el signo negativo indica que la fuerza es atractiva.

+Q -Q S

h

E

- 371 -

Física Universitaria: Problemas de Física Capacidad eléctrica. E02.6

6. Calcular la capacidad de un condensador plano con tres dieléctricos entre

sus armaduras, distribuidos como se indica en la figura, sabiendo que sus permitividades relativas son iguales a 1, 2 y 3, respectivamente, que todos tienen el mismo espesor h/2 y que los dieléctricos 1 y 2 tienen el mismo tamaño.

La capacidad de un condensador plano de superficie S y separación l entre sus armaduras viene dada por la expresión:

r 0S SCl l� � �� �

Los dos primeros dieléctricos se comportan como una asociación de dos condensadores en paralelo, por lo que presentan una capacidad equivalente

� 1 2 0 0 012/ 2 / 22 3/ 2 / 2

S S SC C Ch h h

� � �� � � � �

La asociación en serie de esta capacidad equivalente con la correspondiente al tercer dieléctrico representa una nueva capacidad equivalente dada por

�eq 3 0 0 0120 0

1 1 1 1 13 6 23 3

/ 2

h h hS SC C C S S Sh h

� � �� �� � � � � � �

de modo que la capacidad del condensador plano con tres dieléctricos es

eq 02 SCh

��

S 1

2 3

h/2

- 372 -

Física Universitaria: Problemas de Física Capacidad eléctrica. E02.7

7. Disponemos de dos condensadores, de capacidades C1 y C2, que han sido cargados, por separado, a las

tensiones V1 y V2 respectivas, siendo V1 � V2. Los conectamos en paralelo, uniendo los bornes de igual polaridad. a) Demostrar que la energía electrostática almacenada en este montaje es inferior a la energía total cuando los condensadores estaban separados. b) ¿Por qué ocurre esto?

a) Energía electrostática total almacenada en los condensadores separados:

�2 21 1 2 2

12

U C V C V� �

Capacidad del condensador equivalente al montaje en paralelo: Ceq = C1 + C2

La carga neta almacenada en los condensadores permanece constante al unirlos en paralelo:

�1 2 1 1 2 2 1 1 2 2

1 2 1 2 1 2

Q Q C V C V C V C VVQ Q C C V C C

�� � � � �� ���) )� � � ���

Energía electrostática total almacenada en los condensadores montados en paralelo:

�21 1 2 22

eq1 2

1 12 2

C V C VU C V

C C�

) � ��

Variación de la energía almacenada

� �

2 2 2 2 2 2 2 2 2 21 1 2 2 1 2 1 2 1 1 1 2 2 1 2 1 2 2

1 2 1 2

22 21 2 1 21 2 1 2 1 2

1 2 1 2

2eq 1 2

2121 122 2

1 02

U U U

C V C V C C VV C V C C V C C V C VC C C C

C C C CVV V V V VC C C C

C V V

)4 � � �� �� � � � � �� �� � �� �� �� � ��

� � � � � � �� �

� � � *

de modo que la energía electrostática disminuye1. b) La energía electrostática disminuye debido a que, durante la conexión de los condensadores, pasa carga de un condensador a otro, estableciéndose una corriente eléctrica a través de los hilos de conexión, desprendiéndose calor en ellos. La cantidad de calor desprendida no depende de la resistencia de los hilos de conexión; si éstos presentan una resistencia pequeña, la intensidad de la corriente será grande, e inversamente.

1 Obsérvese que si fuese V1 = V2, sería �U = 0; y que si C1 = C2 y V2 = 0, sería U’=U/2

+

+

+

+

+

Q=CeqV

Q1=C1V1

Q2=C2V2

Q’1=C1V

Q’2=C2V

- 373 -

Física Universitaria: Problemas de Física Capacidad eléctrica. E02.8

8. Dos condensadores de placas paralelas, cada uno con una capacidad de C1 = C2 = 2 0F, están conectados en

paralelo a una batería de 12 V. a) Determinar la carga en cada condensador y la energía total almacenada por los condensadores. b) A continuación, los condensadores se desconectan de la batería y entre las placas del condensador C2 se inserta un dieléctrico de constante k = 2.5. En estas condiciones, determinar la diferencia de potencial entre las placas de cada condensador, la carga depositada sobre cada uno de ellos y la energía total almacenada por ambos.

a) Ambos soportan la misma tensión (12 V), y como tienen la misma capacidad, será Q1 = Q2 = CV = 2 � 12 = 24 0C

total 1 2 48 �CQ Q Q� � � 2 21 1

1 12 21 22 21 1

2 22 2

2 12 144 �J288 �J

2 12 144 �J

U C VU U U

U C V

�� � � � � �� � � ��� � � � � ���

b) La carga total permanece invariable después de desconectar la batería.

1 1 1 11 2 total

2 1 2 2

2 �Fcon

5 �FC C Q C V

Q Q QC kC Q C V

� �) ) ) )� � �� �� � ) )� �� �� �) ) ) )� � �� �� �

total1 2 total

1 2

48 �C( ) 6.86 V7 �F

QC C V Q VC C

) ) ) )# � � " � � �) )�

Ambos condensadores soportan la misma d.d.p.

1 1

1 2

2 2

2 6.86 13.7 �Ccon 48 �C

5 6.86 34.3 �C

Q C VQ Q

Q C V

�� ) ) )� � � ��� ) )� ��� ) ) )� � � ����

2 21 11 12 2

1 22 21 12 22 2

2 6.86 47 �J165 �J

2 6.86 118 �J

U C VU U U

U C V

� ) ) )� � � � � �� ) ) )� � ��� ) ) )� � � � ���

V C1 C2

12 V 2 �F 2 �F

C1’ C2’2 �F 5 �F

- 374 -

Física Universitaria: Problemas de Física Capacidad eléctrica. E02.9

9. En la asociación de condensadores de la figura se aplica una

diferencia de potencial de 50 V entre A y B. Hállese: a) La carga en cada uno de los condensadores. b) La carga en el condensador equivalente a la asociación de todos ellos. c) Energía almacenada en cada uno de los condensadores. d) Energía almacenada en el condensador equivalente.

Los condensadores están asociados en serie:

ss 1 2

1 1 1 1 1 1 1�F2 2

CC C C

� � � � � �

El condensador C3 está en paralelo con Cs:

3 s 3 1 4 �FC C C� � � � �

a) La d.d.p. entre A y B, VAB, se reparte por igual entre los condensadores C1 y C2, por lo que las d.d.p. en estos y sus cargas son:

1 11 2 AB 1 2 1 12 2 50 2 50 �C5 V 2 25V V V q q C V� � � � � � � � �

El condensador C3 soporta la tensión VAB:

3 AB 3 3 3 150 C50 V 3 5 �0V V q C V� � � � � �

b) El condensador equivalente soporta la d.d.p. VAB:

eq eq AB 200 �C4 50q C V� � � �

c) La energía almacenada en un condensador puede calcularse mediante la fórmula 21

2U CV� , de modo que:

2 2 2 21 2 1 1 3 3 3625 �J 3751 1 1 12 25 3 50

2 2 20 �J

2U U C V U C V� � � � � � � � �

d) La energía almacenada en el condensador equivalente sería: 2 2

eq eq eq1 1 4 50 5000 �J2 2

U C V� � � �

A B

C3

C2 C1

20F 20F

30F

- 375 -

Física Universitaria: Problemas de Física Capacidad eléctrica. E02.10

10. En la asociación serie-paralelo de condensadores que se muestra en la

figura, determinar la capacidad y la tensión de ruptura del condensador equivalente entre los bornes A y B.

Los dos condensadores en paralelo equivalen a uno de capacidad Cp:

p 1 2 2 6 8 �FC C C� � � � �

La asociación en serie equivale a un condensador de capacidad Ceq:

p 3eq

p 3

8 12 96 4.8 �F8 12 20

C CC

C C�

� � � �� �

La tensión de ruptura de los dos condensadores en paralelo será igual a la menor de ellas; esto es, 50 V, como se indica en la figura. En la asociación en serie, los dos condensadores se reparten la tensión y tienen la misma carga:

p 3 AB p 3 AB

p p 3 3 p p 3 3 0

V V V V V VC V C V C V C V

� �� � � �� �� � � �� �� � �� �� �

Y resolviendo ese sistema de dos ecuaciones con dos incógnitas:

3p AB AB AB AB

p 3

p3 AB AB AB AB

p 3

12 3 50 V 83 V20 5

8 2 400 V 1000 V20 5

CV V V V VC C

CV V V V V

C C

��� � � � * *�� ������ � � � * *�� ����

La tensión de ruptura del condensador equivalente será la menor de las anteriormente calculadas; esto es,

rup 83 VV �

Otro método Cargas máximas (ruptura) para cada uno de los “dos” condensadores:

s 38 50 400 �C 12 400 4800 �CQ Q� � � � � � Los dos condensadores en serie tendrán la misma carga, de modo que la tensión de ruptura del condensador equivalente se calcula a partir del que admite menor “carga de ruptura”; i.e.,

ss AB

S

400 83 V4.8

QV VC

� � � �

2 �F/100 V

6 �F/50 V

12 �F/400 V

A B

2 �F/100 V

6 �F/50 V

12 �F/400 V

A B

8 �F/50 V

12 �F/400 V

A B

4.8 �F/83 V B A

Condensador equivalente

- 376 -

Física Universitaria: Problemas de Física Capacidad eléctrica. E02.11

11. En la asociación serie-paralelo de condensadores que se muestra en la

figura, determinar la capacidad y la tensión de ruptura del condensador equivalente entre los bornes A y B.

Los dos condensadores en serie equivalen a uno de capacidad Cs:

1 2s

1 2

2 6 12 1.5 �F2 6 8

C CCC C

�� � � �

� � La asociación en paralelo equivale a un condensador de capacidad Ceq:

eq s 3 1.5 3.5 5 �FC C C� � � � � La tensión de ruptura será, obviamente, inferior a la del condensador situado en la rama inferior; i.e.,

AB 400 VV * En la rama superior, los dos condensadores en serie se reparten la tensión y tienen la misma carga:

1 2 AB 1 2 AB

1 1 2 2 1 1 2 2 0V V V V V VC V C V C V C V

� �� � � �� �� � � �� �� � �� �� � Y resolviendo ese sistema de dos ecuaciones con dos incógnitas:

21 AB AB AB AB

1 2

12 AB AB AB AB

1 2

6 3 100 V 133 V8 42 1 50 V 200 V8 4

CV V V V VC C

CV V V V VC C

��� � � � * *�� ����� � � � * *�� ��� La tensión de ruptura del condensador equivalente será la menor de las anteriormente calculadas; esto es,

rup 133 VV �

Otro método Cargas máximas (ruptura) para cada uno de los tres condensadores:

1 2 32 100 200 �C 6 50 300 �C 3.5 400 1400 �CQ Q Q� � � � � � � � � Los dos condensadores en serie tendrán la misma carga, de modo que la tensión de ruptura del condensador equivalente se calcula a partir del que admite menor “carga de ruptura”; i.e.,

1s AB

S

200 133 V1.5

QV VC

� � � �

5 �F/133 V

2 �F/100 V 6 �F/50 V

3.5 �F/400 V B

BA

A

Condensador equivalente

2 �F/100 V 6 �F/50 V

3.5 �F/400 V A B

- 377 -

Física Universitaria: Problemas de Física Capacidad eléctrica. E02.12

12. Determínese la capacidad equivalente entre los puntos A y B del conjunto de

condensadores de la figura, siendo C la capacidad de cada uno de ellos.

Puesto que todos los condensadores son idénticos, el condensador que se encuentra en la rama CD está “puenteando” dos ramas idénticas, por lo que los nodos C y D se encontrarán siempre al mismo potencial y el condensador situado en esa rama no se cargará nunca; por lo que puede suprimirse. Nos queda el sistema ilustrado en la segunda figura, compuesto por tres ramas (en paralelo) entre A y B. Cada una de las dos ramas superiores, constituidas por dos condensadores en serie, presentan una capacidad:

11

1 1 12CC

C C C� � �

y la capacidad equivalente entre A y B, correspondiente a las tres ramas en paralelo, será:

eq 22 2C CC C C� � � �

A B

D

C

A B

D

C

- 378 -

Física Universitaria: Problemas de Física Corriente continua. E03.1

1. En la figura se muestra un esquema de circuito para la distribución de una señal eléctrica (v.g., televisión por cable) a una gran cantidad de abonados. Cada abonado representa una resistencia de carga RA entre la línea de distribución y tierra, en tanto que la resistencia de la línea de distribución entre los puntos de conexión de los diferentes abonados se considera como una resistencia constate RL. Demostrar que la resistencia equivalente vista desde la entrada de la señal es

�2eq L A L L

1 42

R R R R R� � �

Sugerencia: Puesto que hay un gran número de abonados, la resistencia equivalente no variará apreciablemente si un abonado (digamos el primero) cancela su suscripción. En consecuencia, la resistencia equivalente de la sección del circuito situado a la derecha de la primera resistencia de carga es prácticamente igual a Req.

De conformidad con la sugerencia, el circuito se simplifica en la forma que se indica en la segunda figura, con una resistencia Req en paralelo con la resistencia de carga del primer abonado, lo que representa una resistencia Rpar igual a

A eqpar

par A eq A eq

1 1 1 R RR

R R R R R� � �

La resistencia equivalente vista desde la fuente señal es

A eqeq L par L

A eq

A eqeq L

A eq

R RR R R R

R RR R

R RR R

� � � � �

� ��

De modo que operando y despejando Req obtenemos 2

A eq eq L A L eq A eq

2eq L eq L A 0

R R R R R R R R R

R R R R R

� � � �

� � �

y resolviendo esta ecuación de segundo grado en Req, obtenemos

�2

L L L A 2eq L L L A

4 1 42 2

R R R RR R R R R

3 �� � � �

habiendo descartado el signo negativo porque conduciría a una Req negativa.

RL RL RL

RARA RAfuentede señal

RL

ReqRA

fuente de señal

RL

Rparfuente

de señal

- 379 -

Física Universitaria: Problemas de Física Corriente continua. E03.2

2. Disponemos de un generador de 12.0 V de f.e.m. y 1 � de resistencia interna. Necesitamos suministrar una tensión de 3.0 V a una carga externa, por lo que recurrimos a un divisor de tensión, tal como se esquematiza en la figura. a) Determinar el valor de la resistencia R2 para R1 = 9 �. b) Determinar el generador equivalente entre los bornes A y B. c) Calcular la tensión VAB cuando suministramos corriente a una carga externa de R = 7.5 �.

a) Al no existir carga externa, la intensidad que suministra el generador y la d.d.p. entre los bornes AB son:

circ 2 2

12 121 9 10

IR R R

� � �� � �

2AB 2 2 2 2

2

12 303 12 3 3.33 � 0 310 9

RV IR R R RR

� � � � � � ��

b) La f.e.m. equivalente entre los bornes AB coincide con la d.d.p. entre dichos bornes en circuito abierto; por consiguiente:

eq 3.0 V��

Si cortocircuitamos los bornes AB, la intensidad de cortocircuito será:

corto12.0 1.2 A10

I � �

por lo que la resistencia del generador equivalente será:

eqeq

corto

3.01.2

2.5 �rI

� � ��

También puede calcularse, más rápidamente, teniendo en cuenta que entre AB tenemos dos resistencias de 10 � y 3.33 � en paralelo, por lo que

eq10 3.33 2.5 �10 3.33

r �� �

c) Calculamos la intensidad a partir de las características del generador equivalente:

3.0 3.0 0.3 A 300 mA2.5 7.5 10

I � � � ��

y la d.d.p. pedida será

AB 0.3 7. . 5 V5 2 2V IR� � � �

A

B

12.0

V, 1

� R1

R2 R

A

B3.0

V, 2

.5 �

7.5 �

A

B12

.0 V

, 1 �

9 *

3.33 *

- 380 -

Física Universitaria: Problemas de Física Corriente continua. E03.3

3. Dos baterías con fuerza electromotriz � = 5 V y resistencia interna r = 2 *, pueden conectarse a una resistencia R = 6 * en serie o en paralelo. a) Determinar el generador equivalente y la potencia suministrada a la resistencia R en cada caso. b) ¿Qué método de conexión suministra la mayor potencia a R? ¿Y si R fuese igual a 1 *?

Baterías en serie:

Generador equivalente: s

s

5 5 10 V2 2 4

i

ir r� � + � � ����� � + � � � '��

� �

ss

s,tot

s s s

2 2s,R s

10 W

6 W

(m s po

10 1 A4 6

10 161 6

tencia)

IR

P IRP I R

��� � � ��� ����� � � � �� ' ���� � � � �������

á

ss

s,tot

s s s

2 2s,R s

2

10 2 A4 1

1 0 W

4 W

10 2

2 1

IR

R P IP I R

��� � � ��� ����� ' � � � ����� � � � ������

Baterías en paralelo:

Generador equivalente: p

pp

(id nticas)5 V

1 1 1 1 1 12 2i

rr r

� ������� � + � � � � '����

� é

pp

p,tot

p p p

2 2p, p

3.57

5 0.71 A1 6

6 5 0.71

0.71 6

W

3.06 WR

IR

R P I

P I R

��� � � ��� ������ ' � � � ����� � � � �������

pp

p,tot

p p p

2 2p, p

5 2.5 A1 1

5 12.5 W

6.25 W

(m s potencia)

2.51

2.5 1R

IR

P IR

P I R

��� � � ��� ����� � � � �� ' ���� � � � �������

á

R

5 V, 2 � 5 V, 2 �

R5 V, 2 �

5 V, 2 �

- 381 -

Física Universitaria: Problemas de Física Corriente continua. E03.4

4. Una batería tiene una f.e.m. de 12 V y una resistencia interna de 1.00 �. a) ¿Qué resistencia externa deberemos conectar a la batería para obtener una potencia de 20 W? b) ¿Ídem una potencia de 40 W? c) ¿Qué potencia máxima puede transferir la batería a una resistencia externa?

Aplicamos la ecuación del circuito y sustituimos la intensidad de la corriente en la expresión de la potencia disipada en la resistencia de carga:

22

2( )RI P I R Rr R r R

� � �� �� �

Operamos en est última expresión para despejar R:

2 2 2

2 2 2

2 0

2 0R R R

R R R

P r P R P rR R

P R P r R P r

� � � �

� � � �

Resolvemos esta ecuación de segundo grado con respecto a R:

� � �22 2 2 2 2 22 2 4 2 4

2 2R R R R R

R R

P r P r P r P r P rR

P P

� 3 � � � 3 �� �� � � � �

a) Sustituyendo valores:

� @2 212 2 20 1.0 12 12 4 20 1.0 104 96 5.0 �

0.2 �2 20 40R

� � � 3 � � � 3� � �

b) De nuevo sustituimos valores:

�2 212 2 40 1.0 12 12 4 40 1.0 64 482 40 80

R� � � 3 � � � 3

� ��

i

de modo que no existen soluciones reales. Ocurre que la batería no puede proporcionar esa potencia a la resistencia de carga. c) La potencia máxima corresponde a una carga igual a la resistencia interna de la batería, esto es R = r, de modo que

2 2 2

,máx 2

12 36 W( ) 4 4 1.0RP rr r r

� � � �� �� �

�,r

+

a b

I

R

R

PR

rR1 R2

20 W

36 W

- 382 -

Física Universitaria: Problemas de Física Corriente continua. E03.5

5. Calcular y justificar el valor de R para que el puente de la figura esté equilibrado.

El puente está equilibrado cuando no circula corriente por la rama CD (rama puente) lo que significa que la distribución de intensidades es la que se indica en la figura. Para que se presente tal circunstancia deberá ser nula la diferencia de potencial entre los puntos o nudos C y D; esto es, los puntos C y D están al mismo potencial, de modo que

AC AD 1 2

CB DB 1 2

5 36

V V I IV V RI I

� �

� �

y dividiendo m.a.m. estas dos ecuaciones tenemos 5 3 3 5 6

6R

R� � � �

que es la conocida “regla del producto en cruz”, de modo que 5 6

310 �R �

� �

6 *3 *

5 * R

12 V, r = 1 *

A B

C

D

I1 I1

I2 I2

I

- 383 -

Física Universitaria: Problemas de Física Corriente continua. E03.6

6. a) Determinar la intensidad de corriente que recorre cada una de las ramas del circuito que se muestra en la figura. b) Ídem cuando añadimos una resistencia de 1 � colocada entre A y B.

a) Cada una de las dos ramas soporta una tensión de 12 V, de modo que las intensidades que las recorren son:

1 212 2 A

2 4i i� � �

y la intensidad que suministra el generador es:

1 2 4 Ai i i� � �

Otro método La resistencia equivalente de cada una de las dos ramas es 6 � y la resistencia equivalente de las dos ramas en paralelo es:

1 2

1 2

6 6 36 3 �6 6 12

R RRR R

�� � � �

� �

La intensidad total se reparte por igual entre las dos ramas:

1 212 4 � 2 A3

i i i� � � �

b) El puente no está equilibrado, por lo que aplicamos el método de Maxwell para las intensidades de malla:

13

2

3

1

2

3

12 6 2 4 6 2 40 2 7 1 �= 2 7 1 132 �0 4 1 7 4 1 7

12 2 41 12 5767 10 7 1 4.364 A1 7132 132 1320 1 7

6 12 41 12 2162 12 0 1 1.636 A4 7132 132 1324 0 7

1

III

I

I

I

� � � �� �� � � �� � �� � �� � �� � �� � �� � � � � �� � �� � �� � �� � �� � �� � �� � �� � �� � � �� � �

� ��

� � � � ��

�� � �

� � � � � ��

�6 2 12 12 3602 72 7 0 2.727 A4 1132 132 1324 1 0

��

� � � �� �

� �Las intensidades de rama son:

1 2 3 4 5 64.264 A 2.728 A 1.636 A 1.091 A 1.637 A 2.727 Ai i i i i i� � � � � �

en los sentidos indicados en la figura.

- 384 -

Física Universitaria: Problemas de Física Corriente continua. E03.7

7. En el circuito que se esquematiza en la figura, determinar el valor de la resistencia R desconocida y la intensidad que circula por cada rama.

Abordamos el cálculo de las intensidades de malla por el método matricial de Maxwell:

�1 2 31

2 1 2

3 1 3

2 2525 2 1 10 1 1 0 1 00 1 0 2 2 0

I I IIR I I R I

I I I

� � � ��� � � �� �� � �� � �� � � �� � �� � � �� � �� � � � � � �� � � �� � �� � �� � � �� � �� � � �� � �� � ��� � � �� � ���

Sistema de tres ecuaciones con cuatro incógnitas (las tres intensidades y R) a las que añadiremos la indicación del amperímetro para tener una cuarta ecuación:

3 2 5 mAI I� �

Resolvemos el sistema de las tres primeras ecuaciones: � �4 1 1 2 1 3R R R�� � � � � � �

1

2

3

25 1 1 50 11 0 1 01 30 0 2

2 25 11 501 0 01 31 0 2

2 1 25 25 11 1 1 01 31 0 0

RI R

R

IR

RI R

R

� � �� � �

�� � �

��

� �� � � �

��

Y considerando la cuarta ecuación: �

3 2

25 1 50 25 25 305 mA 3 k�1 3 1 3 1 3 10

R RI I RR R R

� �� � � � � � �

� � � De modo que:

1 2 3200 50 10020 mA 5 mA 10 mA10 10 10

I I I� � � � � �

1k�

25 V

R

1k� 1k�

A5mA

1k�

25 V

R

1k� 1k�

A

I2

I3 I1

15 mA 5 mA 5 mA 10 mA 10 mA

- 385 -

Física Universitaria: Problemas de Física Corriente continua. E03.8

8. Las aristas de un tetraedro están formadas por resistencias. Las seis aristas tienen la misma resistencia R. Hállese el valor de la resistencia equivalente entre dos vértices cualesquiera.

Visto desde uno cualquiera de sus vértices (v.g., D), las resistencias de las aristas del tetraedro se pueden representar en el plano del papel, tal cono se indica en el primer esquema. El primer esquema se transforma obviamente en el segundo, en el que apreciamos que la resistencia CD está “puenteando” en un “puente equilibrado” ABCD. En consecuencia, no pasa corriente por la rama CD, de modo que podemos eliminarla (tercer esquema). El tercer esquema nos muestra tres ramas entre A y B, cuyas resistencias se indican (cuarto esquema). Finalmente, en el cuarto esquema tenemos tres resistencias en paralelo, dos de ellas de valor 2R y otra de valor R; por estar en paralelo, será

eq

1 1 1 1 1 1 2 22 2 2R R R R R R

� �� � � � �

eq 2RR# �

B A

R

R

R

R R

2R

2R

R A B

C

A

D

B

B

D

A

C

R

R A B

R¡2 A B

- 386 -

Física Universitaria: Problemas de Física Corriente continua. E03.9

9. Dado el circuito de la figura determinar la intensidad de corriente que pasa por la resistencia de 2 * situada entre A y B y la d.d.p. entre esos puntos.

Aplicamos el método de Maxwell, con las corrientes de malla indicadas en el esquema:

13

2

3

10 7 2 0 7 2 020 2 9 4 2 9 4 242 �0 0 4 6 0 4 6

III

�� � � �� �� � �� � �� � �� � �� � �� � �� � �� � �� � �� � �� � �� � �� � �� � �� � �

Resolvemos:

1

10 2 01 14020 9 4 0.5785 A2420 4 6

I�

� � �

2

7 10 01 7202 20 4 2.975 A2420 4 6

I�

� � �

La intensidad de la corriente que circula por la rama AB, en el sentido de A�B, es la suma de las dos corrientes de malla; esto es,

AB 1 2 0.5785 2.975 3.55 Ai I I� � � � �

La d.d.p. entre A y B será:

AB AB AB 2 3.55 7.10 VV R i� � � �

10V 20V2*

4* 2*

A

B

I1 I2

I3

3* 5*

- 387 -

Física Universitaria: Problemas de Física Corriente continua. E03.10

10. Determinar la resistencia equivalente entre los puntos A y B del circuito representado en la figura.

Suministramos corriente al circuito conectando un generador de f.e.m. entre los nudos A y B y resolvemos el circuito por el método de las mallas de Maxwell:

13

2

3

8 2 60 2 11 6 348 �0 6 6 14

III

�� � � �� �� �� � �� � �� � �� � �� � �� � � �� � �� � �� � �� � �� � �� � �� � �� � �� �� � �

Tan sólo estamos interesados en la intensidad de corriente suministrada por el generador externo, ya que

111

2 61 0 11 60 6 14

I �� �

� �� � �

��

nos permite calcular la resistencia equivalente entre A y B:

AB1 11

348 34811 6 118

6 14

2.95 �RI

��

� � � � ��

3*

2*2* 6*

6*

A

B

3*

2* 2* 6*

6*

A

B I1

I2

I3 �

RAB

A

B

I1 �

- 388 -

Física Universitaria: Problemas de Física Corriente continua. E03.11

11. En el esquema del circuito de corriente continua de la figura, determínese: a) Las intensidades que circulan por cada rama. b) La diferencia de potencial entre los puntos A y B.

Resulta más cómodo volver a dibujar el circuito en la forma que se indica. Lo resolveremos por el método de Maxwell, considerando las corrientes de malla que se muestran en la figura:

13

2

3

6 8 2 56 2 6 1 174 �

6 5 1 8

III

�� � � �� �� �� � �� � �� � �� � �� � �� � � � �� � �� � �� � �� � �� � �� � �� � �� � �� �� � �

Resolvemos para las corrientes de malla:

2

1

3

18 6 5 02 6 1 0 A

1746 2 5 5 6 81 3486 6 1 2 A8 2 61746 1 8 1 3482 6 6 2 A

1745 1 6

I

I

I

� �� � �� ��� ��� � � � ��� � �� �� � �� � ��� � � �� ��� ��� � � � �� ��� � � �� �� � �� �� ��� �� � �� � � � �� �� � �� � ���� � �� ��

y las intensidades de rama son las que se indican en la figura (circuito en serie). b) La diferencia de potencial entre A y B la calculamos por el camino ACB:

AB 2 2 6 2 VV � � � � �

Método rápido (simetrías): Es fácil observar la simetría que presenta el circuito con respecto al eje que se indica en la figura. Como consecuencia de dicha simetría, los puntos D y E, por un lado, y los puntos B y C, por otro, están al mismo potencial, por lo que no circulará intensidad por las ramas DE y BC y pueden suprimirse. Nos queda un circuito serie, como se indica en la figura (circuito resaltado), por el que circula una intensidad

6 6 122 1 2

21 6

AiR

�� � � �

� � ��

2*

2*

5*3*

1*

1*

6V

6V

A

B

2*

2*

5* 3*

1*1*

6V

6V A

B

I2

I1

I3

C

2 A

2 A

2 A

2 A

2*

2*

5*3*

1*

1*

6V

6V

A

BE

D C

- 389 -

Física Universitaria: Problemas de Física Corriente continua. E03.12

12. En el circuito de la figura, determinar: a) Las intensidades que pasan por cada resistencia. b) Fuerza electromotriz y resistencia interna del generador equivalente entre A y B en la segunda figura.

a) Aplicamos el método de las mallas con la notación de Maxwell, siendo I1, I2, y I3 las corrientes de malla:

1

2

3

10 10 4 0 10 4 00 4 9 2 4 9 2 1084 0 2 2 0 2 2

III

� �� � � �� � � �� � � � � � � � , � � � �� � � � � � � � � � � � � � �

1

2

3

10 4 01 1080 9 2 1 A

1084 2 2

10 10 01 04 0 2 0 A

1080 4 2

10 4 101 2164 9 0 2 A

1080 2 4

I

I

I

�� �� � � � �� , � � � �� �� � � � � �� , � � �

�� ��� � � � � �� , � � � �

6 4 2 31 A 2 A 0 Ai i i i# � � � �

designando por ix las intensidades de corrientes de rama que circulan por las resistencia Rx. b) La f.e.m. del generador equivalente es igual a la diferencia de potencial entre los puntos A y B y la resistencia interna de dicho generador es la resistencia equivalente de las resistencias de 6 * y 4 * en paralelo.

Intensidad que circula por el circuito: 10 V 1 A(6 4)

I

� ��

Diferencia de potencial entre A y B: AB ( 1 A) 4 4 VV � � � � �

Resistencia equivalente entre A y B: eq6 4 24 2.46 4 10

r �

� � ��

Por consiguiente: eq eq4 V 2.4r � ��

A

B

10V 4V

6*

4* 2*

3*

6*

10V

4*

10V 4V

6*

4* 2*

3*

1 2 3

1A

1A 2A

2A

A

B6*

10V

4*1 A

- 390 -

Física Universitaria: Problemas de Física Corriente continua. E03.13

13. En el circuito de la figura, determinar la f.e.m. y la resistencia interna del generador equivalente entre A y B.

Abierto: Aplicamos el método de Maxwell para determinar las intensidades de malla I1 e I2, que son también las intensidades que circulan por las resistencias de 12 � y 8 �, respectivamente.

21

2

16 20 5 275 �6 5 15II �

� �� � � �� �� ��� �� ��� ��� �� � �� � �� �� � �

Resolvemos para las corrientes de malla

1

2

1 17016 5 0.764 A6 15 2751 20020 16 0.145 A5 6 275

I

I

� � ��� �� �� � ��� � ���� � ��� � �� ��� � � �� �� �� �� �� ��

Determinamos ahora la “intensidad en cortocircuito”; esto es, con los bornes AB cortocircuitados. Resolvemos el nuevo circuito por el método de las mallas de Maxwell. La f.e.m. equivalente entre AB es igual a la d.d.p. entre AB (en circuito abierto):

eq AB 12 0.764 8 0.145 8.00 VV� � � � � ��

En corto:

c3

1

2

0 20 12 816 12 20 5 1100 �

6 8 5 15

III

�� �� � � �� � �� ��� � �� ��� � �� � )�� � � �� � �� ��� � �� ��� � �� �� �� � �� )� � �� � �

Resolvemos para la intensidad de corto:

c

0 12 81 220016 20 5 2 A� 11006 5 15

I� �� � �� �� �� � � � �� �� �� ���� ��

La resistencia interna del generador equivalente entre A y B es

eeq

q

c

482

�I

r � � ��

Otro método: Para determinar la resistencia equivalente entre AB, observamos que la resistencia de 5 � está “puenteando” las dos ramas en paralelo que forman el circuito. El puente está equilibrado a efectos de resistencias (aunque circula corriente por él, debido a la batería de 10 V), ya que 3�8 = 2�12, por lo que podemos ignorar (suprimir) la resistencia “puente” y el circuito se reduce al que se muestra en la figura. La resistencia equivalente entre AB es:

eq5 20 1005 20 5

4 �2

r �� � �

A B

10V,5*

4V,2* 6V,3*

12* 8*

I1 I2

A B

2* 3*

12* 8*

A B

10V,5*

4V,2* 6V,3*

12* 8*

I’1 I’2

Ic

- 391 -

Física Universitaria: Problemas de Física Corriente continua. E03.14

14. Consideremos el circuito de c.c. que se muestra en la figura. a) Determinar la f.e.m. y la resistencia interna del generador equivalente entre los bornes A y B. b) Calcular la potencia máxima que puede suministrar el circuito de la figura a una resistencia de carga externa conectada entre A y B. Determinar el valor de dicha resistencia de carga.

a) La f.e.m. del generador equivalente entre AB es igual a la diferencia de potencial entre AB sin carga externa. Para determinarla, resolvemos el c.c. de la figura por el método de las mallas de Maxwell.

21

2

16 30 6 30 6 864 �2 6 30 6 30II �

� �� � � �� ��� ��� �� � ��� ��� �� � �� � �� �� � �

1 21 492 41 1 156 1316 6 30 160.569 A 0.181 A2 30 6 2864 72 864 72

I I� �

�� � � � � � � �

AB 0.569 6 0.181 15 3.417 2.708 6.125 VV � � � � � � �

eq 6.125 V�# �

Determinamos ahora la “intensidad en cortocircuito”; esto es, con los bornes AB cortocircuitados. Resolvemos el nuevo circuito por el método de las mallas de Maxwell.

c'1'2

3

0 21 6 1516 6 30 62 15 6 30

21 6 156 30 6 9234 �15 6 30

III

� �� � � �� ��� �� � �� ��� � �� ��� � �� � �� ��� � �� �� �� �� �� ��� � �� � �� � �

�� � � �

� �

c

0 6 151 529216 30 6 0.5731 A9234 92342 6 30

I�

� � � ��

qeq

e

c

6.1250.5731

10.69 �rI

� � ��

b) Un generador suministra la máxima potencia a una carga externa cuando el valor de ésta coincide con el de la resistencia interna del generador. Por consiguiente,

eq

2 2 2 2eq eq e

ext

mq2

ax 2

10.69 �

876.125 0.877 W = 4 4 4 10.

7 mW69

r

RI R R

r R R

R

PR

� �

� ��� �� � � � � �� �� �� � ��

� � �

A B

12 V, 18 * 6 V, 9 *

4 V 6 *

6 * 15 *

2 1

A B

12 V, 18 * 6 V, 9 *

4 V 6 *

6 * 15 *

2 1

Ic

- 392 -

Física Universitaria: Problemas de Física Corriente continua. E03.15

15. El circuito que se muestra en la figura está constituido por cinco generadores de f.e.m. asociados “en puente”. a) Determinar la f.e.m. y la resistencia del generador equivalente entre A y B. b) Calcular la intensidad que suministra cada generador cuando cortocircuitamos A y B.

Abierto: El valor de la f.e.m. equivalente coincide con el de la diferencia de potencial entre A y B en circuito abierto. Resolvemos el circuito que se indica en la figura:

21

2

12 8 4 32 �12 4 6II �

� �� � � �� �� ��� �� ��� ��� �� � �� � �� �� � �

1

2

1 2412 4 0.75 A12 632 321 488 12 1.50 A4 1232 32

I

I

�� � �

�� � � �

� �

eq AB 2 ( 0.75) 1 1.5 ( 6 3) 1.5 1.5 9 9 VV� � � � � � � � � � � � � ��

En corto: Determinaremos la intensidad en cortocircuito entre A y B utilizando las corrientes de malla que se indican en la figura

c3

1

2

9 3 2 112 2 8 4 48 �12 1 4 6

III

�� � � �� �� �� � �� � �� � �� � �� � �� � � �� � �� � �� � �� � �� � �� � �� � �� � �� � �� � �

c

9 2 11 28812 8 4 6 A48 4812 4 6

I� �

� � � �� �

Ahora, podemos determinar fácilmente la resistencia interna del generador equivalente entre A y B, ya que

eq eqc eq

eq c

1.596

�I rr I

� � � �� �

b) Calculamos las otras dos corrientes de malla con A y B en cortocircuito:

1 2

3 9 1 3 2 91 180 1 722 12 4 3.75 A 2 8 12 1.50 A48 48 48 481 12 6 1 4 12

I I� �

� � � � � � � � �� � � � �

A partir de las corrientes de malla (Ic, I1 y I2) se obtienen fácilmente las corrientes de rama, i.e., las corrientes que suministran cada uno de los generadores. Así, el generador de 12 V (puente) suministra 3.75 – 1.50 = 2.25 A (hacia arriba). Las demás corrientes de rama están indicadas en la figura con sus valores y sentidos propios.

12 V, 4 �

6 V, 2 � 3 V, 1 �

3 V, 1 �

A BI1 I2

6 V, 2 �

6 V, 2 �

6 V, 2 � 3 V, 1 �

3 V, 1 �

A B

Ic

I2 I1

3.75 A

2.25 A 4.50 A

1.50 A

2.25 A 12 V, 4 �

- 393 -

Física Universitaria: Problemas de Física Corriente continua. E03.16

16. En el circuito que se esquematiza en la figura, determinar el generador equivalente (f.e.m. y resistencia interna) entre los terminales A y B.

En abierto: La d.d.p. entre A y B coincide con la f.e.m. equivalente entre esos puntos. Abordamos el cálculo de las intensidades de malla por el método matricial:

21

2

12 5 1 � 24 �12 1 5II

� �� � � �� �� ��� �� ��� ��� �� � �� � �� �� � �

1

2

1 4812 1 2 A12 524 241 485 12 2 A1 1224 24

I

I

�� � �

�� � � �

� �

� �AB eq3 2 1 2 6 6 2 6 10 V 10 VV � � � � � � � � � � � ��

En corto: Para determinar la resistencia interna del generador equivalente entre A y B cortocircui-tamos esos bornes y calculamos la intensidad de cortocircuito (Ic) que circula por la rama A y B de resistencia nula. Aplicamos el método matricial de Maxwell para calcular las intensidades de malla:

c2

1

2

6 4 3 112 3 5 1 40 �12 1 1 5

III

�� �� � � �� � �� ��� � �� ��� � �� � )�� � � �� � �� ��� � �� ��� � �� �� �� � �� )� � �� � �

eq eqc c eq

eq c

6 3 11 240 1012 5 1 6 A 1.67 �40 40 612 1 5

I I rr I

� �� � �� �� �� � � � � � � �� �� �� ���� ��

� �

3 �

3 � 6 V, 1�

A B I1 I2

2 A 2A

2 A 2 A 4 A

6 V, 1�

6 V, 1�

3 �

3 �

6 V, 1�

A B

6 V

, 1�

6 V, 1�

3 �

3 � 6 V, 1�

A B I’1 I’2

6 V, 1� 6 V

, 1�

Ic

- 394 -

Física Universitaria: Problemas de Física Corriente continua. E03.17

17. En el circuito que se esquematiza en la figura: a) Calcular la intensidad de la corriente que circula por la resistencia de 4 � y la d.d.p. entre N y M; b) Determinar el generador equivalente (f.e.m. y resistencia interna) entre los terminales A y B.

Abordamos el cálculo de las intensidades de malla por el método matricial:

21

2

6 7 4 33 �6 4 7II �

� �� � � � �� ��� �� ��� ��� �� � �� � �� � �

1

1

1 186 4 0.5455 A6 733 331 187 6 0.5455 A4 633 33

I

I

� � �

� � �

�4 NM1.091 A 4 1.091 4.364 V 4.4 Vi V� � � =

b) En abierto: La d.d.p. entre A y B coincide con la f.e.m. equivalente entre esos puntos:

� � �AB eq2 0.5455 4 1.091 2 0.5455 6.546 V 6.546 V 6.6 VV � � � � � =�

En corto: Para determinar la resistencia interna del generador equivalente entre A y B cortocircuitamos esos bornes y calculamos la intensidad de cortocircuito (Ic) que circula por la rama A y B de resistencia nula. Aplicamos el método matricial de Maxwell para calcular las intensidades de malla:

c3

1

2

6 3 2 16 2 7 4 48 �6 1 4 7

III

�� �� � � �� � �� ��� � �� ��� � �� � )�� � � �� � �� ��� � �� ��� � �� �� �� � �� )� � �� � �

c 1 2

eq eqc eq

eq c

6 2 11 2166 7 4 4.5 A 3.00 A 1.50 A48 486 4 7

6.546 1.46 �4.5

I I I

I rr I

� �) )� � � � � �

� �

� � � �� �

1�

6 V 2 �

2 �

1 �

6 V

A B 4 �

M

N

1�

6 V 2 �

2 �

1 �

6 V

A B 4 �

I1 I2

0.55 A

0.55 A1.10

A

M

N

0.55 A

0.55 A

1�

6 V 2 �

2 �

1 �

6 V

A B 4 �

I’1 I’2

Ic

- 395 -

Física Universitaria: Problemas de Física Corriente continua. E03.18

18. Determinar la f.e.m. y la resistencia interna del generador equivalente al circuito de se representa en la figura, entre los bornes A y B.

Método general: Abierto:

22

3 3

00 10 60 6 8 0

III I

� �� �� � � � �� ��� ��� �� � �� ��� �� � �� � �� �� � � ���

� �AB 0 12 6 18 V 18 VV � � � � � � � =

En corto:

13

2

3

corto 1

2 3

18 3 2 10 2 10 6 66 �0 1 6 8

18 2 11 7920 10 6 12 A66 660 6 8

6 A

III

I I

I I

�� �)� � � �� � ��� �� � �� ��� � �� �� )� � � � � � �� ��� � �� �� �� �� �� �� � � �)� � �� � �

� �)� � � � �

) )� �

eq eqcorto eq

eq corto

18 1.5 �12

I rr I

� � � �� �

Otro método: En virtud del resultado obtenido en el primer apartado del método general, llegamos a la conclusión de que el puente está equilibrado en f.e.m.’s, ya que no circula corriente por la rama de puente, y en resistencias por lo que podemos suprimirlo. Entonces, el circuito se reduce a asociaciones serie y paralelo de generadores de f.e.m. 1. Asociación de generadores en serie. Se suman las f.e.m y se suman las resistencias internas. 2. Asociación de generadores en paralelo. Como todos tienen la misma f.em., polaridad resistencia interna, la f.e.m. equivalente es la de cualquiera de ellos y la resistencia equivalente corresponde a la de una asociación de resistencias en paralelo; esto es,

1 2eq

eq 1 2 1 2

1 1 1 3 1.5 �2 2

r r rrr r r r r

� � � � � ��

12 V

8 V

2 �

10V 2 �

1 �

1 �

6 V

A B 6 � 2 3

1

18 V 1.5 � A B

4 V

18 V 1.5 �A B

12 V

8 V

2 �

10 V 2 �

1 �

1 �

6 V

A B 6 �

4 V

12 V

8 V

2 �

10 V 2 �

1 �

1 �

6 V

A B

18 V 3 �

A B

18 V 3 �

- 396 -

Física Universitaria: Problemas de Física Corriente continua. E03.19

19. En el circuito que se esquematiza en la figura: a) Determinar el generador equivalente (f.e.m. y resistencia interna) entre los terminales A y B b) Calcular la d.d.p. entre CD en circuito abierto y cortocircuitado entre AB

En abierto: La d.d.p. entre A y B coincide con la f.e.m. equivalente entre esos puntos. Abordamos el cálculo de las intensidades de malla por el método matricial:

21

2

12 3 1 8 �12 1 3II �

� �� � � �� �� ��� �� ��� ��� �� � �� � �� �� � �

1

2

1 2412 1 3 A12 38 81 243 12 3 A1 128 8

I

I

�� � �

�� � � �

� �

� � �AB eq1 3 1 3 6 6 V 6 VV � � � � � � ��

� �CD 1 6 6 0 VV � � � � �

En corto: Para determinar la resistencia interna del generador equivalente entre A y B cortocircuitamos esos bornes y calculamos la intensidad de cortocircuito (Ic) que circula por la rama A y B de resistencia nula. Aplicamos el método matricial de Maxwell para calcular las intensidades de malla:

c2

1

2

6 2 1 112 1 3 1 8 �12 1 1 3

III

�� �� � � �� � �� ��� � �� ��� � �� � )�� � � �� � �� ��� � �� ��� � �� �� �� � �� )� � �� � �

eq eqc c eq

eq c

6 1 11 4 612 3 1 6 A 1 �8 8 612 1 3

I I rr I

� �� � � � � � � �

� �

� �

1 2

2 6 1 2 1 61 4 1 01 12 1 6 A 1 3 12 0 A8 8 8 81 12 3 1 1 12

I I� �

) )� � � � � � � � �� � � � �

� �CD 1 6 6 0 VV � � � � �

Otro método: El cálculo de la resistencia equivalente puede abordarse más fácilmente teniendo en cuenta que el “puente de 5 resistencias” (prescindiendo de las f.e.m.’s) está equilibrado:

6 V

1�

6 V 1 �

1 �

1 �

6 V

A B

1 �

C

D

6 V

1�

6 V 1 �

1 �

1 �

6 V

A B

1 �

I1 I2

3 A 3 A

3 A 3 A 6 A

C

D

6 V

1�

6 V 1 �

1 �

1 �

6 V

A B

1 �

I’1 I’2

Ic

D

C

1�

1 �

1 �

1 �

A B 1 �

1�

1 �

A B 2 �

2 �

A B 1�

1 �

1 �

A B

- 397 -

Física Universitaria: Problemas de Física Corriente continua. E03.20

20. En el circuito que se esquematiza en la figura, determinar el generador equivalente (f.e.m. y resistencia interna) entre los terminales A y B.

En abierto: La d.d.p. entre A y B coincide con la f.e.m. equivalente entre esos puntos. Abordamos el cálculo de las intensidades de malla por el método matricial:

21

2

6 7 3 40 �6 3 7II �

� �� � � �� � �� ��� �� ��� ��� �� � �� � ��� � �

1

2

1 246 3 0.60 A6 740 401 247 6 0.60 A3 640 40

I

I

�� �� � � �

�� � �

� � �

� � �eq AB 3 0.60 1 0.60 6 1.8 0.60 6 7.2 V

3 0.60 3 1.20 3 0.60 1.8 3.60 61.8 7.2 V

V� � � � � � � � � � � � �

� � � � � � � � � � � � �

En corto: Para determinar la resistencia interna del generador equivalente entre A y B cortocircuitamos esos bornes y calculamos la intensidad de cortocircuito (Ic) que circula por la rama A y B de resistencia nula. Aplicamos el método matricial de Maxwell para calcular las intensidades de malla:

c3

1

2

6 4 1 36 1 7 3 72 �

6 3 3 7

III

�� �� � � �� � �� ��� � �� ��� � �� � )�� � � � �� � �� ��� � �� ��� � �� �� �� � �� )� �� � �

eqc eq

c

6 1 31 288 7.26 7 3 4 A 1.8 �72 72 46 3 7

I rI

� �� � �� �� �� � � � � � � �� �� �� ��� ��

Otro método para la resistencia equivalente entre AB: Consideramos tan solo las resistencias de todos los elemento (incluidas las resistencias internas de los generadores) y añadimos un generador entre AB, para disponer de intensidad en el circuito. Entonces, resolvemos para calcular la intensidad que suministra el generador:

31

2 3113

1 11 eq1 11

4 1 3 72 �0 1 7 3

40 �0 3 3 772 1.8 �40

III

I RI

��

� �

� � � �� �� � �� � �� � � � �� � �� � � �� � �� � � � � � �� � �� � �� � � �� � � �� � � �� � �� � � �� �� � �

� � � � �

� �

3 �

3 �

6 V, 1�

A B I1 I2

0.6 A 060A

0.6 A 0.60 A 1.20

A

6 V, 1�

3 �

3 �

3 �

6 V, 1�

A B

6 V, 1� 3 �

3 �

3 �

6 V, 1�

A B I’1 I2’

6 V, 1�

3 �

Ic

3 �

3 �

1�

A B

I1

I2

1� 3 �

13

- 398 -

Física Universitaria: Problemas de Física Corriente continua. E03.21

21. Consideremos el circuito que se representa en la figura una vez que los condensadores se han cargado completamente. a) Calcular la carga y la tensión que soporta cada condensador. b) Determinar el valor máximo aplicable de la f.e.m. del generador si las tensiones de ruptura de los condensadores son 100 V, 200 V y 300 V, respectivamente, tal como se indica en la figura.

En régimen estacionario, los condensadores están completamente cargados y no conducen corriente eléctrica.

100 25 A4

IR

� � ��

a) Dada la simetría que presenta el circuito (puente equilibrado), será

BD1

AD DB AB2

050 V

VV V V

� � �

El condensador de 2�F no soporta tensión alguna y, por tanto, no adquiere carga eléctrica. Cada uno de los dos condensadores de 1�F soporta una tensión de 50 V y adquieren una misma carga

1�F 50 V 50 �CQ CV� � � � b) El condensador de 2�F no soporta tensión alguna y, por tanto, no nos preocupa al efecto f.e.m. máxima aplicable. La f.e.m. máxima aplicable será de 200 V, ya que, al repartirse por igual entre los dos condensadores de 1�F, se alcanza la tensión de ruptura del que la tiene más baja (100 V).

100 V

2 �

2 �

2�F/300V

1�F/100V

1�F/200V

100 V

2 �

2 �

2�F/300V

1�F/100V

1�F/200V

B D

C

A

- 399 -

Física Universitaria: Problemas de Física Corriente continua. E03.22

22. Consideremos el circuito representado en la figura. a) Calcular las intensidades de corriente que circulan por cada rama, indicando su sentido en un esquema. b) Asignando el potencial nulo al punto A, determinar los potenciales en los puntos B, C, D y E.

a) Método de la mallas de Maxwell:

1

2

3

0.25 5 4 00 4 7 3 50

0.50 0 3 5

III

�� � � �� ��� � �� � �� � �� � �� � �� � � �� � �� � �� � �� � �� � �� � �� � �� � �� �� � �

1

2

3

0.25 4 01 0.50 7 3 0.010 A 10 mA50 500.50 3 5

5 0.25 01 2.54 0 3 0.130 A 50 mA50 500 0.50 5

5 4 0.251 6.54 7 0 0.050 A 130 mA50 500 3 0.50

I

I

I

�� � � � � �

� �

�� � � � � � � �

��

� � � � � � �� �

Las corrientes de malla (corregidas) y las corrientes de rama se indican en la segunda figura. b) Cálculo de d.d.p. o de potenciales referidos a VA=0:

ab

BA B

CA C

DA D

EA E

( 0.25) 0.25 V 250 mV4 60 240 mV

( 0.50) 0.50 V 500 mV0

V iR

V VV VV VV V

� + � +

� � � � � � ����� � � � ����� � � � � � ���� � ����

����

1 � 2 �

4 � 3 �

A

B DC

E

I1 I3 I2

0.25

V 0.50 V

130 mA

0.50 V

A

B DC

E

0.25

V 10 130

50

10 mA

60 mA 80 mA

1 � 2 �

3 � 4 �

50 mA

- 400 -

Física Universitaria: Problemas de Física Corriente continua. E03.23

23. En el circuito esquematizado en la figura, calcular: a) La fuerza electromotriz y resistencia interna del generador equivalente entre A y B. b) La intensidad que pasaría por una resistencia de 1 * colocada entre A y B.

a) Por cada una de las ramas circula una intensidad de corriente: 12 2 A6

IR

� � ��

La d.d.p. entre A y B coincide con la f.e.m. equivalente entre esos puntos, de modo que

AB 2( 2) 4(2) 4 8 4 VV � � � � � � �

con el borne A positivo. Para determinar la resistencia interna del generador equivalente entre A y B cortocircuitamos esos bornes y calculamos la intensidad de cortocircuito (icorto) que circula por la rama A y B de resistencia nula. Aplicamos el método matricial de Maxwell para calcular las intensidades de malla:

12 6 2 4 1 30 2 6 0 =96 �20 4 0 6

312 2 41 12 6 00 6 0 4.5 A1 0 696 960 0 6

6 12 41 12 2 02 0 0 1.5 A2 4 696 964 0 6

6 2 121 12 2 62 6 0 3.0 A3 4 096 964 0 0

III

I

I

I

� ���� � � �� � ��� � �� � �� � �� � �� � �� �� � �� � �� � �� � �� �� �� �� �� ��� � �� ���

� �� � �

�� �

� � � ��

��

� � � ��

�1.5 A (A B)c 3 2

4 8 2.67 �eq 1.5 3c

i I I

ri

� � �

� � � ��

b) La intensidad pedida es: 4 12 1.09 A8 / 3 1 11

I � � ��

A B 12 V

2*

4 *2 *

4 *

A B

12 V

2*

4 *2 *

4 *

I1 I2

I3

A B 12

V

2*

4 *2 *

4 *

2 A 2 A

4 V, 8/3*

I

A B 1 *

- 401 -

Física Universitaria: Problemas de Física Corriente continua. E03.24

24. Con un hilo homogéneo de sección constante construimos los lados y las diagonales, que se unen en el centro, de un hexágono regular. Sea R la resistencia de cada uno de los tramos de hilo. a) Determinar la resistencia equivalente entre dos vértices contiguos del hexágono. b) Se conectan esos dos vértices contiguos a una batería de que suministra una intensidad de corriente I. Calcular la intensidad de la corriente en cada uno de los conductores.

Todos los puntos situados sobre la línea de simetría (discontinua) están al mismo potencial y, por tanto, podemos fusionarlos en O. Reduciendo sucesivamente en serie y en paralelo, se

obtiene finalmente que la resistencia equivalente entre A y B es eq1120

R R�

Las intensidades se calculan a partir del último esquema, al que asignamos arbitrariamente una corriente de 1 A. Retrocediendo hacia los esquemas anteriores, teniendo en cuenta que en dos ramas en paralelo, de resistencias R1 y R2, la intensidad se reparte de modo que

21

1 2 1 21 1 21 1 1

1 1 2 2 12 22

1 2

RII I I R RR R RI I I IV I R I R RR R I

R R

��� ��� �� � �� � �� � � � � �� �� �� �� ��� ���

A

C

E F

D

B

O

A

C

E F

D

B

O

� 40 A

7

3

4

22

7

4

3

11 11

2

1 1

E F

D

B

OC

A

1

11/3

1/3

1/3

1/3

11 1

1

1

7/40

3/40

4/40

33/40

4/3

D

B

O C

A

1 1/3 1/3

1 1

1

4/3

33/40 7/40

4/40

3/40

4/7 D

B

O C

A

1 1/3 1/3 1

4/7

33/40

1

7/40

7/40

11/7

B

O

A1/31/3

11/77/40

33/40BA 11/20

11/40 O 11/40

A B

1

1

- 402 -

Física Universitaria: Problemas de Física Corriente continua. E03.25

25. Cada uno de los vértices del hexágono está unido a cada uno de los demás vértices por ramas de resistencia eléctrica R. a) Calcular la resistencia eléctrica equivalente entre los nodos A y B. b) Ídem eliminando la rama AB.

a) Al estar cada vértice unido a cada uno de los restantes, no hay diferencia alguna, a efectos de resistencia eléctrica, entre dos vértices consecutivos o entre dos que no lo sean. Así, nos resultará más fácil aplicar simetrías cuando tomamos dos vértices opuestos, A y D en la figura, ya que los puntos o nudos B � F y E � F se encuentran obviamente al mismo potencial, lo que nos permite reducir el circuito del modo que sigue en las figuras. En el paso del segundo esquema al tercero, hemos suprimido la rama A-BF-CE-D porque corresponde a un puente equilibrado, de modo que por la resistencia “puente” , R/4, no circula corriente y podemos suprimirla.

En definitiva, AB 3RR �

b) Si quitamos una de las resistencias de valor R entre A y B quedará

AB 2RR) �

A

D

F BB

E C

A

D

B=F

C=E

A

D

B=F C=E

R/2 R/2

R/2

R

R/2

R/4

A

D

A

D

R R

R

R/3

A

B

A

B

R R

R/2

- 403 -

Física Universitaria: Problemas de Física Corriente continua. E03.26

26. En el circuito de la figura: a) Determinar la resistencia equivalente entre los bornes A y B. b) Calcular la intensidad de la corriente en cada una de las ramas cuando se conecta el circuito al de la batería, tal como se indica en la figura.

a) Reducimos el circuito comenzando por la “cola”, mediante agrupaciones serie y paralelo.

Descubrimos fácilmente in proceso reiterativo que nos lleva finalmente a la reducción deseada entre los bornes A y B. Por consiguiente, la resistencia equivalente entre A y B es eq 4 �R �

b) La intensidad que proporciona la batería es

circuito

6 1 A6

IR

� � ��

En el primer y segundo esquema, podemos observar fácilmente que la intensidad que llega a cada uno de los nudos C, E, G,… se divide a partes iguales entre la rama vertical y la horizontal; esto es, se divide por dos, de modo que las intensidades en las ramas sucesivas son: 1 A, 500 mA, 250 mA, 125 mA, ...

1 * 1 * 1 * 1 *

1 * 1 * 1 * 1 *

1 *

1 *

4 * 4 * 4 * 2 *6 V

A

B

C G

D F H

E

1 *

1 *

2 *

A

B

C

D

4*

A

B

1 *

1 *

6 V 4*

1 * 1 * 1 * 1 *

1 * 1 * 1 * 1 *

1 *

1 *

4 * 4 * 4 * 2 * 6 V

A

B

C G

D F H

E

0.5A

0.25A1 A

0.25A

0.5A

1 * 1 *

1 * 1 *

4 * 2 *

G

H

1 *

1 *

4 * 4 *

G

H

1 *

1 *

2 *

G

H

- 404 -

Física Universitaria: Problemas de Física Corriente continua. E03.27

27. En una línea de transporte de corriente continua, que tiene 400 m de longitud y 0.4 � de resistencia, se ha producido una derivación a tierra por un mal aislamiento. La corriente de entrada en la línea es de 50 A a 125 V y la de salida de 45 A a 106.5 V. Determinar el punto de la línea en el que se ha producido la derivación y la resistencia eléctrica de la misma (i.e., la resistencia de fuga).

Expresamos la d.d.p. entre los puntos extremos de la línea de transporte (A y B), siendo R la resistencia total de la misma:

�AB 1 1 150 45 5 45V R R R R R� � � � �

lo que nos permite calcular la resistencia del tramo de longitud x:

�AB1

125 106.5 45 0.445 0.1�5 5

V RR� � ��

� � �

de modo que será:

400 100 m0.1 0.4x x� �

Expresamos ahora la d.d.p. entre A y C, a lo largo del camino AFGC:

AC 1 f50 5V R R� �

y de aquí calculamos la resistencia de fuga:

AC 1f

50 125 50 0.1 24 �5 5

V RR � � �� � �

Otro planteamiento (método de Maxwell): Consideramos las dos mallas que se indican en la figura:

1 f f

f 1 f

125 50106.5 45

R R RR R R R

� �� � � �� � �� ��� �� �� ��� �� ��� �� � � ��� � �

Operando, obtenemos dos ecuaciones con dos incógnitas (R1,Rf):

� �

1 f f 1 f

1 ff 1 f

50 45 125 50 5 12545 5 106.5 45 124.550 45 106.5

R R R R RR R RR R R R

� �� � �� � ��� � � �� �� � � � � � �� � � � � � �� ��

Sumando m.a.m. tenemos: 1 15 0.5 0.1�R R� �

Y de la primera ecuación: 1f f

125 50 24 �5

RR R�� �

125

V

106.

5 V

x400 m

fuga

tierra

125

V

106.

5 V

x 400 m

Rf

R1 R-R1

50 A 45 A

5 A

A B

DC

F

G

125

V

106.

5 V

x

400 m

Rf

R1 R-R1

50 A 45 A 5 A

- 405 -

Física Universitaria: Problemas de Física Corriente continua. E03.28

28. Una subestación de corriente continua desarrolla una potencia de 400 kW cuando suministra energía a una instalación situada a 100 m de distancia, a través de una línea eléctrica de 3.5 � de resistencia (cada cable conductor) constituida por conductores de cobre (resistividad, � =1.75 ·10-8 �·m. a) Calcular la potencia que se disipa en la línea eléctrica cuando la tensión que suministra la subestación es de 5 kV. ¿Qué porcentaje de pérdida de potencia se produce en la línea eléctrica? b) Ídem si la tensión fuese de 15 kV. c) Calcular la sección de los cables. d) Determinar la densidad de corriente eléctrica en los cables en ambos casos, así como la intensidad del campo eléctrico en los mismos.

a) Intensidad que circula por la línea: 400 kW 80 A5 kV

I � �

Potencia disipada en la línea: 2 2 linea

linea linea 44.8 kW 44.880 7 0.11240

11.2%0

PP I RP

� � � � � � �

b) Intensidad que circula por la línea: 400 kW 26.6 A15 kV

I � ��

Potencia disipada en la línea: 2 2 linea

linea linea 4.98 k 4.9826.6 7 0.012440

W 1.24%0

PP I RP

� � � � � � ��

c) Sección de los cables: 8

7 2 2

2

1.75 10 100 5 10 m 0.5 mm3.5

0.52 24

0.80 mm

l lR SS RD SS D

� �

�� �

��� �

� � � � � �

� � � �

d) Recordamos la definición de densidad de corriente y la expresión de la ley de Ohm (microscópica):

6 61 22 2 2 2

1 2

A A A A160 10 160 53.3 10 53.3m mm m mm

V V2.80 0.93m m

Ij j jS

E E�

� � � � � � �

� � �E j

3.5 *

R3.5 *

I

I

- 406 -

Física Universitaria: Problemas de Física Corriente continua. E03.29

A

�t

10�

�2 �1

Rt=0

0.1(L-x)

0.1(L-x) 0.1x

0.1x

I1 I2

B

10�

29. Una línea de tranvía, de 10 km de longitud, está alimentada por dos generadores de co-rriente continua, de (1100 V, 10 �) y (1000 V, 10 �) respectivamente, conectados cada uno en un extremo de la línea, como se muestra en la figura. La resistencia eléctrica del cable y de las vías son de 0.1 �/km. El tranvía requiere una intensidad de corriente de 100 A para su funcionamiento. Para una posición genérica, x (km), del tranvía, determinar las intensidades y potencias que suministran al tranvía cada uno de los dos generadores y la tensión de alimentación del mismo.

Resolvemos el circuito por el método de las mallas de Maxwell, siendo R11 y R22 las resistencias de las mallas:

11

22

11 22

10 2 0.1 10 0.210 2 0.1( )

10 0.2(10 ) 12 0.222 �

R x xR L x

x xR R

� � � � �

� � � � �

� � � � �� �

1 t 1 t1

2 t 2211 22 111 t 11 1

2 t 22 2 11 1 t 2 t2

2 t11 22 22

010

0 10

IRR R RR I

R I RI

R R R

� �� �� � ���� � �� �� �� �� � � �� � �� � �� � � � �� � �� � �� �� � �� �� � � � � ��� � �� ����

� � � �� �� �

� � � � � �� �

1 t 2 t 1 2 1 2 11 22t 1 2 t t

11 22 11 22 11 22 11 22 11 22

1 1 R RI I IR R R R R R R R R R

� �� � ��� �� � � � � � � � � � �� �� ���

� � � � � � � �� �

Despejamos la f.c.e.m. del tranvía:

1 2t 2

22 1 11 2 11 22 t11 22t

11 22 11 22

11 22

11200 60 422

IR R R R IR R x x

R R R RR R

� �� � � �

� � �� �

� �� �

Calculamos la tensión de alimentación VAB del tranvía: �AB t t0V I� � � � �� �

Calculamos la intensidad de corriente suministrada por cada generador:

1 t1

11

2 t2

22

t 1 2

1300 2022

900 2022

100 A

xIR

xIR

I I I

� � ��� � ������ � �� � ������ � �

� �

� �

Así como las potencias:

10 km, 0.1 �/km x

10 km, 0.1 �/km

1000 V

10 �

1100 V

10 �

x2 4 6 8 10

V

490

495

500

505

510

515

520

Tensión de alimentación del tranvía en función de la distancia x.

- 407 -

Física Universitaria: Problemas de Física Corriente continua. E03.30

� �

� �

21 AB 1

22 AB 2

2t 1 2

1 11200 60 4 1300 20484

1 11200 60 4 900 2048450 11200 60 411

P V I x x x

P V I x x x

P P P x x

��� � � � � ������� � � � � �����

� � � � �

Otro método:

1

2

1 2

10 2 0.1 10 0.210 2 0.1( )

10 0.2(10 ) 12 0.222

R x xR L x

x xR R

� � � � �

� � � � �

� � � �� � '

Resolvemos el circuito por el método de Kirchof:

1 1 2 2 1 2

1 2 t

1 malla 1 nudo (A) 0

R I R IIRI I II

�� � � � �A � A ��� � � �� � � �A ��� ��

� � ��

1 21 2

2 21

1 2 1 2

1 11

1 2 1 2

1 1

100 12 0.2 1001 1300 201 22 22

100 10 0.2 1001 900 201 22 22

R RR R

R xR I xIIR R R RR xR I xI

IR R R R

� �� ��4 � � � �� �� ��� � �� � �� �� � � � ��� � ����� � �� ��� � � � �� � ����

� �

� �

Calculamos la tensión de alimentación del tranvía y su f.c.e.m.: �

� �

AB t t

2

t AB 1 1 1 1 1 1

0

1300 20 11200 60 41100 10 0.222 22

V I

x x xV I R I R x

� � � � �

� � �� � � � � � � � � � �

� �

� � �

� �

� �

1 AB 1

2

2 AB 2

2

2t 1 2

1 11200 60 4 1300 20484

1 11200 60 4 900 20484

50 11200 60 411

P V I

x x x

P V I

x x x

P P P x x

� � ������� � � ������ � ������� � � ����

� � � � �

x2 4 6 8 10

V

490

495

500

505

510

515

520

Tensión de alimentación del tranvía en función de la distancia x.

A

�t

10�

�2 �1

Rt=0

0.1(L-x)

0.1(L-x) 0.1x

0.1x

I1 I2 It

B

10�

- 408 -

Física Universitaria: Problemas de Física Corriente continua. E03.31

30. En el circuito de la figura, sabiendo que el condensador está inicialmente descargado. a) Determinar la corriente inicial que suministra la batería, inmediatamente después de cerrar el interruptor S. b) Calcular la intensidad de la corriente estacionaria a través de la batería después de transcurrir un largo período de tiempo. c) Determinar la carga del condensador en las circunstancias del apartado anterior.

a) En el instante inicial, t = 0, el condensador conduce y el circuito equivalente, en ese instante, es el que se ilustra en la figura. Las dos resistencias en paralelo equivalen a una

resistencia de

p300 600 200300 600

R �

� ��

de modo que

050 50 0.125 A =

200 200 42

001 5 mAI � � �

b) Transcurrido un tiempo suficientemente grande, desaparece el estado transitorio de carga y el condensador corta la corriente en la rama AB en la que se encuentra, por lo que el circuito equivalente es el que se indica en la figura. La intensidad de corriente que suministra la batería es

50 50 0.0625 A =200 600 80

20

6 .5 mAI! � � ��

c) La tensión entre los bornes del condensador es VAB (ya que por la resistencia de 300 * no circula corriente) de modo que

AB 600 0.0625 600 37.5 VV I R!� � � �

y la carga del condensador será

AB 5 37.5 187.5 �CQ CV� � � �

300 *5

5 0F5

200 *5

50 V5600 *5

S

300 *5

5 0F5

200 *5

50 V5600 *5

S

A5

B5

300 *5

5 0F5

200 *5

50 V5600 *5

S

- 409 -

Física Universitaria: Problemas de Física Corriente continua. E03.32

31. Un condensador de 1 0F tiene deteriorado el dieléctrico, dando fugas de corriente, comportándose como si estuviera en paralelo con una resistencia de 95 *. Se pretende cargar el condensador con una fuente de 24 V de f.e.m. y 1 * de resistencia interna. a) Determinar la máxima diferencia de potencial entre las armaduras del condensador que podemos alcanzar. Dibujar el esquema correspondiente al proceso de carga. b) Si se interrumpe el proceso de carga, calcular el tiempo necesario para descargarse en la relación 1/e; i.e., determinar el valor de la constante de tiempo del condensador. Dibujar el esquema correspondiente al proceso de descarga.

Comenzamos determinando el generador de f.e.m. equivalente entre A y B:

f

eq AB f

feq

f

24 0.25 A95 1

0.25 95 23.75 V

95 1 95 0.99 �95 1 96

IR rV IR

R rRR r

� � �� �

� � � � �

�� � � �

� �

a) La máxima diferencia de potencial entre las armaduras del condensador que podemos alcanzar será de 23.75 V. b) La constante de tiempo en los procesos de carga y de descarga de un condensador de capacidad C a través de una resistencia R viene dada por la expresión RC� � . Carga:

6 6eq 0.99 1 10 0.99 10 s 0.99 �sR C� � �� � � � � � �

Descarga: 6 6

f 95 1 10 95 10 s 95 �sR C� � �� � � � � � �

C

Rf

r I �

A B

Req

I

�eq

A B

C

t

i q

0.37I0

I0

Qf

/0( ) t RCi t I e��

�/f( ) 1 t RCq t Q e�� �

0.63Qf Proceso de carga

t

i q

-0.37I0

-I0

Q0

0.37Q0/

0( ) t RCq t Q e��

/0( ) t RCi t I e��

Proceso de descarga

- 410 -

Física Universitaria: Problemas de Física Corriente continua. E03.33

32. En el circuito representado en la figura, el condensador está inicialmente descargado. Pasamos el conmutador S a la posición A y lo mantenemos en ella durante 10 s, al cabo de los cuales lo pasamos a la posición B. a) Determinar la máxima intensidad de corriente que ha suministrado la batería. b) Calcular la carga y la tensión que adquirió el condensador durante el proceso de carga. c) Calcular la intensidad y la potencia máximas que proporciona el condensador a la resistencia de 10 * durante el proceso de descarga del condensador. d) Comparar las intensidades máximas de carga y de descarga, así como la rapidez de cada uno de esos procesos, justificando y argumentado los resultados.

Proceso de carga del condensador: Constante de tiempo RC = 10 k* � 1 mF = 10 s. a) La intensidad máxima se presenta en el instante en que comienza la carga del condensador:

012V ( 0)

10k1.2 mAI t

� � �

b) La carga final del condensador, si tuviera tiempo de cargarse completamente, sería

f 1 m F 12V = 12 mC ( )Q t� � !

pero, como interrumpimos la carga al cabo de 10 s, el condensador tan solo adquiere

/ 1f (1 ) 12 mC (1 7.59 mC) 0.636 12 mC =t RCQ Q e e� �� � � � � � �

y la tensión del condensador en ese instante es 7.59 mC

1 mF7.59 VQV

C� � �

Proceso de descarga del condensador: Constante de tiempo RC = 10 * � 1 mF = 10 ms. c) La intensidad máxima se presenta en el instante en que comienza la descarga del condensador a través de la resistencia de 10 *:

07.59V 0.7 759 A = ( 0)10

59 mAI t

� � �

y la potencia máxima que proporciona el condensador se presenta en ese mismo instante:

0 0 7.59 0.75 W9 5.75P V I� � � �

d) La intensidad y la rapidez de descarga son mucho mayores que la de carga por tener lugar ésta a través de una resistencia mucho mayor (1000:1), lo que repercute en las correspondientes constantes de tiempo (RC).

12V 10 k*

10*

1 mF

A

B S

12V 10 k*

1 mF

A

i

CARGA

DESCARGA

i

I0

q t

Q

<

i I0

q

t

10 * B

i

1 mF +

- 411 -

Física Universitaria: Problemas de Física Corriente continua. E03.34

33. En el circuito esquematizado en la figura, cerramos el interruptor en el instante t = 0, iniciándose el proceso de carga del condensador. Supongamos que sea despreciable la resistencia interna de la batería y sean R = 1 M*, C = 1 mF y � = 100 V. a) Expresar en función del tiempo la intensidad de la corriente y la carga del condensador. b) Al cabo de 10 s abrimos el interruptor. Calcular el valor de la carga que retiene el condensador y la energía almacenada en el mismo.

a) En un instante genérico, la suma de ls tensiones soportadas por la resistencia y por el condensador es igual a la tensión entre los bornes del generador de f.e.m., de modo que:

ad [1]qV iRC

� � ��

Derivando esta expresión con respecto al tiempo, conseguimos que desaparezca la variable q que representa la carga del condensador en el instante t. Luego, integrando

00

00

0

d 1 d d d 10 0 d ln

e 2]e

d

[

it

--t / R t /C

I

i i t i i tR i tt C i RC i RC I R

ii I

C

I ��

� � � � � � �

� � �

donde � = RC es la constante de tiempo del circuito RC. A partir de las expresiones [1] y [2], se sigue fácilmente que

para 0

f

0 0

0

t q i IR

t i q Q C

��� � � � ����� � ! � � ���

Determinamos la expresión de la carga del condensador en función del tiempo a partir de la expresión de la intensidad, i(t); por nueva integración, tenemos

� �

0 00 0

/0 0

/f

d e d e dd

( )e 1 [3]1 ee t

q t-t / -t /

t-t / t

q I q I tt

q I QRC RCR

� �

� � �

� �

�� � � �� ��

� ��

b) Sustituyendo los valores dados en el enunciado, tenemos

f1 M� 1 mF = 1000 s 100 V 1 mF 100 mCRC Q C� � � � � � � ��

de modo que

� � �

-10/1000 -0.110

262410

10 3

100 1 e 100 1 e 0.995 mC

995 101 1

995 �C

44.95 10 J =2

95 �J2 10

q

qUC

��

� � � � � �

�� � � �

R C

a b c d

- 412 -

Física Universitaria: Problemas de Física Corriente continua. E03.35

34. Expresar en función del tiempo la carga acumulada y la d.d.p. en bornes de cada uno de los condensadores C1, C2 y C3 de la figura considerando t = 0 en el instante en que se cierra el interruptor y que los condensadores están inicialmente descargados.

En el proceso de carga de un condensador a través de una resistencia, la carga que adquiere el mismo y la d.d.p. entre bornes son funciones del tiempo y vienen dadas por las expresiones:

( ) (1 e ) ( ) / (1 e )-t / RC -t / RCq t C v t q C� � � � �� �

Determinamos la capacidad del condensador equivalente entre A y B :

1 212 eq

1 2

3 6 2 �F 2 4 6�F3 6

C CC CC C

�� � � � � �

� �

de modo que 12 V 6 �F =1000 � 6

72�F

C0.006 s= 6 ms =

CRC

�� �

� �

La carga del condensador equivalente y la diferencia de potencial entre los puntos A y B vienen dada por

� �eq eq/0.006 /0.006AB1 72 1 �C 1 12 1 V

t tRC RCt tq e e v e e

� �

� �� � � �� �� �� �� �� � � � � � � �� �� �� �� �� �� �� �� �

�� �

Los condensadores C12 (C1 +C2 en serie) y C3 soportan la misma tensión:

� �

� �

/0.006 /0.006

/0.006 /0.006

48 1 �C 12 1 V3 33 312 122 4 24 1 �C 12 1 V12 3 12 12

t t

t t

q e v eq qq q

C C q e v e

� �

� �

� �� �� � � �� �� �� � � �� �� � � �� �� �� �

Los condensadores C1 y C2 en serie adquieren la mima carga:

� �

/0.006

/0.006

/0.006

1 8 1 V1124 1 �C12 1 22 4 1 V22

t

t

t

qv e

Cq q q e

qv e

C

���� � � ������ � � � ���� � � ������

12V

R=1000 *

C1=30 F

C2=60 F C3=40 F

R

B

A

Ceq �

- 413 -

Física Universitaria: Problemas de Física Corriente continua. E03.36

35. En el circuito de la figura, y una vez estabilizadas las corrientes (pasados los efectos transitorios, i.e., transcurridos un tiempo muy grande) se pide calcular: a) La inten-sidad que circula por cada rama. b) La carga del condensador.

El circuito que alimenta al condensador, entre A y B, es equivalente a un generador de corriente continua, determinar: c) La fuerza electromotriz y resistencia interna del generador equivalente. d) La constante de tiempo de carga y descarga del condensador.

a) Cuando desaparecen los efectos transitorios, el condensador estará completamente cargado y no dejará pasar la corriente, por lo que podemos suprimir la rama en la que se encuentra el condensador. La resistencia equivalente externa y la resistencia total de este circuito se determina fácilmente:

serie par tot

2428 16 24 � 12 � 12 12 24 �

2R R R� � � � � � � �

La intensidad que suministra el generador y las intensidades en cada rama son:

ramastot

1 A242

0.54

A2ii i

R� � � � ��

b) Calculamos la d.d.p. entre los bornes del condensador:

AB AC CB 0 0.5 24 12 VV V V� � � � � �

y la carga del condensador:

AB 10 �F 12 V=120 �CQ CV� � �

c) La f.e.m. equivalente entre A y B viene representada por la d.d.p. entre A y B sin carga. Entonces, del apartado anterior, resulta obvio que eq AB 12 VV� ��

La resistencia equivalente entre A y B se calcula fácilmente a partir del esquema adjunto:

par eqpar

1 1 1 1 1 6 � 4 624 24 12

10 �6

R rR

� � � � � # � � �

por lo que el circuito equivalente al dado es el que se muestra en la figura. d) La constante de tiempo del sistema RC (serie) será:

10 � 10 �F 100 �s 0.1 msRC� � � � � �

4*

8*

24*

16*

12* 24V

A

B

100F

8*

24*

16*

12* 24V

0.5 A

0.5 A 1 A

A

BC

8*

24*

16*

12* 24V

A

BC

4*

10 �12 V

10 �F

- 414 -

Física Universitaria: Problemas de Física Corriente continua. E03.37

36. Un galvanómetro tiene una resistencia interna de 10 �. Cuando pasa una corriente de 15 mA a través del galvanómetro, la aguja avanza una división de la escala. a) ¿Qué resistencia debemos poner en serie del galvanómetro para que el conjunto pueda utilizarse como voltímetro, en el que una división represente a 10 V? b) ¿Qué resistencia debemos poner en paralelo al galvanómetro para que el conjunto pueda utilizarse como amperímetro, en el que una división represente 0.1 A?

a) Sea RS la resistencia serie que debemos colocar. La d.d.p. de 10 V aplicada entre los bornes A y B del voltímetro resultante deberá proporcionar una intensidad de corriente de 15 mA a través del galvanómetro. Aplicando la ley de Ohm entre A y B tenemos:

ABAB G G S S G

G

( )I

VV I R R R R� � � �

Con los datos del problema será:

S10 10 667 10

0.015657R � � � � �

b) Sea RSh la resistencia de derivación (shunt) que debemos colocar en paralelo con el galvanómetro a fin de que parte de la intensidad se desvíe por ella. Por otra parte, la d.d.p. entre A y B será la misma a través de la rama del galvanómetro que a través de la rama del shunt, de modo que tenemos

Sh GG Sh

GSh GG G Sh Sh

Sh

I I II I I

IR RR I R II

� � ���� � �� �� � � �� � ��� �� ���

Con los datos del problema será:

Sh

Sh

100 15 85 mA1 1.1 85 085

I

R

� � � ������ � ����

G IG = 15 mA

RS

RG = 10 �

A B 10 V

G

IG = 15 mA

RS

RG = 10 �

A B

ISh = 85 mA

I = 100 mA

RSh

- 415 -

Física Universitaria: Problemas de Física Campo magnético. E04.1

1. Una partícula cargada, de masa m y carga eléctrica q, se mueve con una velocidad v en el vacío. En estas condiciones, la partícula penetra en una zona, de anchura h, en la que existe un campo magnético uniforme B, en dirección perpendicular a dicho campo. a) Determinar el valor mínimo de B para que la partícula no pueda atravesar la zona. b) ¿Qué desviación experimentará la partícula, tras atravesar la zona, si el campo magnético tiene una intensidad que es la mitad de la calculada en el apartado ante-rior?

a) La fuerza que actúa sobre la partícula cargada viene dada por la fórmula de Lorentz: q� �F v B

de modo que, al ser perpendicular a la velocidad, tan solo modifica la dirección de ésta, dando lugar a una trayectoria circular de radio R, tal que

2

nv mvF qvB ma m RR qB

� � � �

Puesto que el radio de la trayectoria es inversamente proporcional a la intensidad del campo magnético, y se requiere que R � h para que la partícula no atraviese la zona, deberá ser:

mínmvB Bq

vh

mqh

- �

b) Si se reduce la intensidad del campo magnético, aumenta el radio de la trayectoria y, si R � h, la partícula atraviesa la zona y sale desviada un cierto ángulo �. Para

mín1'2 2

mvB Bqh

� �

el radio del arco de trayectoria circular será

2

' 2' mv

qh

mv mvR hqB q

� � �

El ángulo de desviación que experimenta la partícula tras atravesar la zona se determina fácilmente a partir de la figura:

sen 0.5'

30º2

h hR h

�� � �� �

3 3 3 3 3 3 3 3 3 3 3 3 3 3 3 3 3 3 3 3 3 3 3 3 3 3 3 3 3 3

v

h B

C q

3 3 3 3 3 3 3 3 3 3 3 3 3 3 3 3 3 3 3 3 3 3 3 3

v

h B

Cq

CRF

v

3 3 3 3 3 3 3 3 3 3 3 3 3 3 3 3 3 3 3 3 3 3 3 3

v

h B’

Cq

R’

v

C’C

- 416 -

Física Universitaria: Problemas de Física Campo magnético. E04.2

2. Un varilla conductora y homogénea está suspendida mediante dos hilos verticales conductores, tal como se muestra en la figura. La varilla posee una masa de 0.040 kg/m y se encuentra en un campo magnético uniforme, perpendicular al plano de la figura y dirigido hacia adentro, de 3.6 T. Calcular la intensidad de la corriente que debe circular por la varilla para que sea nula la tensión mecánica en los hilos que la soportan. ¿Cuál deberá ser el sentido de la corriente?

En las condiciones descritas en el enunciado del problema, la varilla debe estar soportada exclusivamente por la fuerza que ejerce el campo magnético sobre ella (ausencia de tensión mecánica en los hilos). El peso de la varilla deberá ser igual y de sentido opuesto a la fuerza que ejerce el campo magnético uniforme B sobre un conductor rectilíneo de longitud l por el que circula una corriente i: esto es,

�i� �F l B

donde la dirección de l es la del sentido convencional de la corriente en el conductor. Puesto que nos dan la densidad lineal de masa (�), la masa total de la varilla es m = �l, de modo que

0.040 9.8 0.1093

109 mA.6

P mg l glg ilB

F ilBgi

B

��

� � ��� ��� ����

� � � �

y su sentido es el indicado en la figura.

- 417 -

Física Universitaria: Problemas de Física Campo magnético. E04.3

3. Dos conductores rectilíneos indefinidos están contenidos en el plano xy, paralelos al eje x como se indica en la figura (uno en y = - 6 cm y el otro en y = 6 cm). Por cada uno de ellos circula una intensidad de 20 A. Determinar el campo magnético de inducción B en los puntos del eje x cuando: a) Las dos corrientes circulan en el sentido negativo del eje de las x. b) La corriente en el conductor situado en y = - 6 cm circula en el sentido positivo del eje de las x y la del situado en y = 6 cm en sentido contrario.

a) En el mismo sentido El campo magnético resultante en todos los puntos del eje x es nulo, ya que los conductores contribuyen con campos iguales y opuestos en dichos puntos. Esto es así, por ser idénticas las intensidades de corriente y las distancias de los conductores al eje x. b) En sentidos opuestos El campo magnético creado por una corriente rectilínea indefinida a una distancia h del conductor que la transporta viene dada por la expresión,

0

2iIBh

��

como se deduce fácilmente a partir del Teorema de Ampère, y su sentido es el indicado en la figura, para cada uno de los conductores. En todos los puntos del eje x, ambos conductores contribuyen por igual y en la misma dirección al campo magnético en dicho eje, por lo que será;

01 2

Ih

��

� � �B B B k

Sustituyendo los valores dados en el enunciado: 7 70 204 10 1333 10

0.1 3 �T

063I

h��

� �� � � � � � �B k

z

y

x

y = - 6 cm

y = 6 cm

y B1

B2

� �

y

B1 B2

� �

- 418 -

Física Universitaria: Problemas de Física Campo magnético. E04.4

4. Tres conductores rectilíneos largos y paralelos pasan a través de los vértices de un triángulo equilátero de lado 10 cm como se indica en la figura, donde el punto indica que la corriente sale del papel hacia el lector y la cruz que entra en el papel. Si la intensidad de cada corriente es de 10 A, hallar: a) El campo magnético en el conductor superior debido a los otros dos conductores inferiores. b) La fuerza por unidad de longitud sobre el conductor superior.

Utilizando el teorema de Ampère, calculamos el campo magnético B creado por un conductor rectilíneo indefinido:

0d I�� �� B r�

Por ser dB r� , en todo el camino de integración, será Bdr = B dl. Además, el módulo de B es el mismo en todos los puntos de la línea de circulación, por lo que resulta:

002 2

4dB IBl B r I

r�� �

��� � ��

a) Los módulos de los campos magnéticos pedidos son 01 2

24

IB Br

��

� � en las direcciones

que se indican en la figura. Y el campo magnético resultante es

0 01 2

02 2 3cos30º cos30º 2 cos30º 24 4

2 342

I IB B Br

Irr

��

� �� �

� � � �� �� �� � � � �� �� �� �� �� �

y sustituyendo valores

7 50 3.42 3 2 3 1010 74 0.1

10 T0

IBr

��

�� ��� � � �

b) La fuerza que actúa sobre un conductor rectilíneo que transporta una corriente viene dada por I� �F l B , y por ser &l B será

5 410 3.47 10 3.47 10 N/m

F I l BF IBl

� �

� � � � ��

10 cm

10 cm 10 cm

B

dr

I

r

60º 60º 2

30º 30º

B2

B1

F

B

1

- 419 -

Física Universitaria: Problemas de Física Campo magnético. E04.5

5. a) Enuncie la ley de Ampère. ¿Es válida para toda trayectoria que rodea a un conductor? Indique para que y en que casos resulta útil. b) Cuatro conductores rectilíneos, indefinidos y paralelos en el vacío, transportan corrientes de igual magnitud I = 4 A, tal como se indica en la figura. Determinar el campo magnético en el punto P situado en el centro del cuadrado, determinado por los cuatro conductores cuyo lado mide 0.2 m.

Teorema de Ampère: La circulación del campo de inducción magnética (B) a lo largo de una trayectoria cerrada (C) es igual al producto de 00 por la intensidad neta (I) que fluye a través de cualquier superficie que tenga a la trayectoria de circulación (C) como contorno. Esto es,

0C

d = I�� B l��

Este teorema es válido para cualquier trayectoria cerrada en un campo magnético. Se utiliza para determinar el campo magnético de inducción (B), resultando especialmente útil cuando calculamos la circulación de B a lo largo de una línea de campo magnético y es constante el módulo de B (esto es, B), a lo largo de dicha línea. b) Determinamos el campo magnético B creado por un largo conductor rectilíneo, que transporta una corriente I, a una distancia r del mismo. Aplicando el teorema de Ampère, siendo C una trayectoria circular de radio r que coincide con una línea de campo:

0 00

C C C

2d = d d 22 4

I IB l B l B r I Br r

� �� �

� �� � � � �� � �B l�� � �

y aplicando esta expresión a uno cualquiera de los cuatro conductores, teniendo en cuenta que 2 / 2r l� , siendo l el lado del cuadrado, tenemos

70 0 5.7 �T2 2 2 2 2 4104 4 0.22 / 2

I IBll

� �� �

� �� � � �

0 0total

7total

2 2 84 cos 45º 2 2 (2 2)4 4

8 4100.2

16 �T

I IB B Bl l

B

� �� �

� � � �

�# � �

en la dirección indicada en la figura.

P 0.2 m

Btotal

- 420 -

Física Universitaria: Problemas de Física Campo magnético. E04.6

6. Determinar, razonada y detalladamente, la expresión de la fuerza por unidad de longitud con que interactúan dos conductores indefinidos y paralelos que conducen intensidades i1 e i2.

Comenzamos determinando, mediante el teorema de Ampère, el campo magnético a una distancia r de un largo conductor rectilíneo que transporta una intensidad de corriente I. Para ello calculamos la circulación de campo B a lo largo de una trayectoria circular (línea de campo) situada en un plano perpendicular al conductor:

00

2d d d 24

IB l B l B r I Br

�� �

�� � � � �� � �B l�� � �

Así, los campos magnéticos creados por cada una de las corrientes rectilíneas indefinidas en las posiciones ocupadas por el otro conductor, situado a una distancia h, tienen las direcciones indicadas en la figura y sus módulos son:

0 01 21 2

2 24 4

i iB Bh h

� �� �

� �

y las fuerzas de interacción mutua, que vienen dadas por, ( )I� �F l B , tienen las direcciones indicadas y sus módulos

son

0 01 2 12 1 212 1 2 1 2

0 01 2 21 1 221 2 1 2 1

2 24 4

2 24 4

i i F i ii i lB lh l hi i F i ii i lB lh l h

� �� �� �� �

� � � � �

� � � � �

F l B

F l B

de modo que la fuerzas de interacción mutua es atractiva si ambas corriente tienen la misma dirección (como se indica en la figura) o repulsiva si tienen direcciones opuesta, siendo la fuerza por unidad de longitud igual a

0 1 224

i iFl h��

B1

B2

F12

F21

I1

I2

B r

I

- 421 -

Física Universitaria: Problemas de Física Campo magnético. E04.7

7. Por un conductor cilíndrico, con 2 cm de diámetro, circula una corriente continua de 1 A, uniformemente distribuida en la sección del conductor. Determinar el valor del campo magnético en el interior del conductor en función de la distancia r al eje del mismo.

Sección recta del conductor: 2 2 4 20.01 3.14 10 mS R� � �� � � � �

Densidad de corriente: 3 24

1 3.18 10 A/m3.14 10

IjS �� � � �

Teorema de Ampère:

2

2 2d 22

IBri B IR

rR

r� ��� �

� � �� � � � B l�

Sustituyendo valores: 7

32

4 10 12 0.0

2 10 (S.1

I.)B r r��

�� �

� ��

B

r

B B

B

i� I

- 422 -

Física Universitaria: Problemas de Física Campo magnético. E04.8

8. Sobre una espira cuadrada de lado a, por la que circula una intensidad i, actúa un campo magnético B uniforme y constante. Calcular el momento de las fuerzas que actúan sobre la espira, dirección y sentido en los siguientes casos: a) El campo magnético B es paralelo a una diagonal de la espira. b) El campo magnético B es paralelo a uno de los lados de la espira. c) El campo magnético es perpendicular al plano de la espira. d) El campo magnético forma 30º con la normal al plano de la espira.

El momento dipolar magnético (m) de la espira y el momento dinámico (M) que ejerce el campo magnético uniforme (B) sobre la espira vienen dados por las expresiones siguientes:

22 sen sen

I iai M iSB ia B� �

�� � �� � � # � ��� � ���

m S kM S B

M m B

a) 2 2sen 90ºM ia B ia B� �

2 2

2 / 2 2 / 200 2 / 2 2 / 21 0 0

ia B ia B

� � � ��� � � �� �� �� � �� � �� � �� � �� � �� � � � � �� � �� � �� � �� � �� � ���� � �� �� � �� �� �

M m B

b) 2 2sen 90ºM ia B ia B� �

2 20 1 00 0 11 0 0

ia B ia B� � � � � �� � �� � �� � �� � �� � �� � � � �� � �� � �� � �� � �� � �� � �� � �� � �� � �

M m B

c) 2 sen 0 0M ia B� �

2 20 0 00 0 0 01 1 0

ia B ia B� � � � � �� � �� � �� � �� � �� � �� � � � � �� � �� � �� � �� � �� � �� � �� � �� � �� � �

M m B

d) 2 21sen 30º2

M ia B ia B� �

2

22

0 00 sen 30º1 cos30º

sen 30º 10 0

20 0

ia B

ia Bia B

� � � �� �� �� �� �� �� � � � � �� �� �� �� �� �� �� �� �� � � � � �� �� �� �� �� �� �� �� �� �� �� �� �� �� �� �

M m B

a MB

m

a x

y

z

i

a M

B

m

a x

y

z

i

a B

m

a x

y

z

i

a M

B m

a x

y

z

i

- 423 -

Física Universitaria: Problemas de Física Campo magnético. E04.9

9. Una espira cuadrada de lado a está inclinada ángulo ) respecto al plano horizontal, tal como se muestra en la figura, pudiendo girar alrededor de su eje de simetría paralelo al eje x. Si existe un campo magnético uniforme B = B k y la espira está recorrida por una intensidad de corriente I, determinar el peso que habría que suspender de uno de los lados de la espira (¿cuál?) para que no gire.

Sobre cada uno de los cuatro lados de la espira actúa una fuerza que viene dada por la expresión ( )I� �F l B , donde I es la intensidad de corriente, l es la longitud del lado (en la dirección de la corriente) y B es el campo magnético. Sobre los lados AC y BD de la espira actúan unas fuerzas en las direcciones indicadas en la figura, iguales y opuestas, que por ser paralelas al eje de rotación de la espira no producen momento con respecto al mismo.

Sobre los AB y CD de la espira actúan también fuerzas iguales y opuestas, en las direcciones que se indican en la figura, de módulo

F IaB� que constituyen un par de fuerzas, cuyo momento es

2par sen senM F a Ia B� �� �

en la dirección que se indica en la figura. Así, deberemos compensar dicho momento suspendiendo una pesa en el lado CD de la espira, de modo que proporcione un momento con respecto al eje de rotación

igual y opuesto al del par; esto es, 2

par peso 2 tsen c gos2aM M Ia B P P IaB� � ��� �

Otro modo de calcular Mpar El momento dipolar magnético (m) de la espira y el momento dinámico (Mpar) que ejerce el campo magnético externo (B) sobre la espira son

parpar

2par sen sen

II

M ISB Ia B� �

� ��� � ��� � ���# � �

m SM S B

M m B

que es el mismo resultado obtenido anterior.

z

B B

C

DA

x

yI

)

)

z

B B

C

D A

x

y I )

)

F

F

F=IaB

F=IaB

P=mg

a

)

a cos)

a sen) M

Mpeso

BA

C

P=mg

a

)

M

Mpeso

BA

C

m )

- 424 -

Física Universitaria: Problemas de Física Campo magnético. E04.10

10. Con un conductor de longitud 8a se forman dos espiras cuadradas de lado a exponiéndose a la acción de un campo magnético uniforme y constante como se indica en la figura. Si por el conductor se hace circular una corriente de intensidad i, determinar: a) Las fuerzas que se ejercen sobre los lados 1-2 y 3-4. b) El momento resultante sobre las espiras.

a) Aplicando la expresión de la fuerza que actúa sobre un conductor rectilíneo que transporta una corriente,

�1

con 00

i B� �� �� �� �� � � � �� �� ����

F l B B

resulta

Lado 1-2: 12

2 / 2 12 /

02 02 1

2 00 0

iB ai Ba

� � � �� �� � �� �� �� �� ��� � ��� �� �� �� �� ���� � �

� ��� �� �� ����

��� ��

��

F

Lado 3-4: 34

0 10

00

0 11

0iaBiaB

� � � ��� �� �� �� �� �� � � �� �� ��� �� �� �� �� ����

� �� �� �� �� �� �� �� � � F

b) Calculamos los momentos (M) dinámicos a partir de los correspondientes momentos magnéticos (m = iS) de las espiras. Momento sobre la espira superior:

221 1 1

0 10 01

0

001iia B a Bi

� � � ��� �� �� �� �� �� � � � � � �� �� �� �� �� �� ��

� ��� �� ��� �� ���� �� � ���� M m B S B

Momento sobre la espira inferior:

2 222

2

0 10 01 0

01

0ia Bi ia B

� � � ��� �� �� �� �� �� � � � � � �� �� �� �

� ��� �� ��� �� �� ����� �� �� �� ��� � ��

M m B S B

Momento total: 1 2 0� � �M M M

a

a

i

1

23

4

B

a

a

i

1

2 3

4

B

z x

y

- 425 -

Física Universitaria: Problemas de Física Campo magnético. E04.11

11. Una espira con forma de triángulo rectángulo, de catetos b y c, representada, forma un ángulo de 45º respecto al plano xz. Por la espira circula una intensidad de corriente I en el sentido indicado y puede girar libremente entorno al eje z. Si la espira está dentro de un campo magnético uniforme B = B i, determinar: a) El momento magnético de la espira. b) La fuerza magnética sobre cada lado de la espira. c) El momento de las fuerzas magnéticas sobre la espira.

a) El momento magnético de la espira está definido como I�m S , donde I es la intensidad que la recorre y S la superficie de la espira, de modo que

12

cos 45º 1 11 2 2cos 45º 1 12 2 40 0 0

m IS bcI

m bcI bcI

� �

� � � � � �� � �� � �� � �� � �� � �� � � � � �� � �� � �� � �� � �� � �� � �� � �� � �� � � m

b) La fuerza que ejerce el campo magnético sobre un conductor rectilíneo de longitud l que transporta una corriente I viene dada por �I� �F l B , de modo que:

0 1 00 0 11 0 0

1 1 02 2 21 0 02 2 20 0 1

20

b

c

a b c a b c

I Ib B IbB IbB

I Ic B IcB IcB

IbB

� � � � � �� � �� � �� � �� � �� � �� � � � � � � �� � �� � �� � �� � �� � �� � �� � �� � ��� � � � � � � � �� � �� � �� � �� � �� � �� � � � � � �� � �� � �� � �� � �� � �� � �� � �� � ��� � �

� � � � � � � �

F b B j

F c B k

F F F F F F j2

IcB k

c) El momento de las fuerzas magnéticas sobre la espira o momento dinámico es:

1 1 02 21 0 0

4 40 0 1bcIB bcIB

� � � � � �� � �� � �� � �� � �� � �� � � � � �� � �� � �� � �� � �� � �� � �� � �� � �� � � M m B

y

x

z

B

I b

c 45º

y

x

z

B

I b

c 45º

m a

M

45º

- 426 -

Física Universitaria: Problemas de Física Campo magnético. E04.12

eje xdl -x

as

� es

P dB

I

12. Un conductor muy largo que transporta una corriente I se dobla en la forma indicada en la figura. Determinar el campo magnético en el punto P.

Comenzamos estableciendo la expresión del valor del campo magnético B a una distancia a de un conductor rectilíneo (no infinito) que transporta una intensidad de corriente I. Para ello, recurrimos a la ley de Biot-Savart, con la notación que se indica en la figura:

0 02 2

2

send d sen send con4 4 cotg d d

sen

s

aa s sxI B I

ar s x a x

�� � � �� �

� ��

��� � ��� ��� " � ���� � �����

�l eB

de modo que

1 22

2

1

1

20 0 0

2 2

send sen sen d cos4 sen 4 4

a I IB Ia a a

��

��

� � ��� � � � �

� � � �� � � �� �

0

1 2(cos cos )4

IBa

�� �

�# � �

Para los conductores verticales (semi-infinitos) será:

0 0vert (cos 0º cos135º ) 21

44 2IBa

Ia

��� �

� ��� ��� �� ���� � � �

Para el conductor horizontal será:

0 0orz

0h

2 2(cos 45º cos135º )4 4 2 2

24

I I IBa aa

� ��

���

� ��� �� � � � �� �� ����

y el campo magnético en el punto P, habida cuenta de que los tres conductores contribuyen en la misma dirección (perpendicular al plano del dibujo y saliente), será:

0vert h

0orz

2 (1 2)4

22 2 1 24 2

IB B B Iaa

��

��

� �� ���� ��� � � � � �� �� �� ���� ���

I

a a

P

2a

- 427 -

Física Universitaria: Problemas de Física Campo magnético. E04.13

13. Hallar la corriente que debe pasar por una bobina estrecha, de 15 espiras, y 10 cm de radio, para que en su centro se produzca un campo magnético igual al campo magnético terrestre en el ecuador, que vale 70 0T.

Cada elemento infinitesimal de la espira contribuye al campo magnético en el centro de la espira de acuerdo con la expresión

02 (ley de Biot-Savart)

dd4

rIr

��

��

l eB

que, aplicada a una espira de radio R, queda en la forma

02

dd4

lB IR

��

Sumamos los módulos de todas las contribuciones de los elementos dl para obtener el campo magnético en el centro de la espira:

20 0 0

espira 2 20

d d 24 4 2

RI I IB B l RR R R

�� � ��

� �� � � �� �

y para una bobina de N espiras apretadas será:

0

2N IB

R�

de modo que 6

70

2 2 0.1 70 10 0.743 A =4 10 15

743 mARBIN� �

� � �� � �

� �

er

dl

dB B

I

- 428 -

Física Universitaria: Problemas de Física Campo magnético. E04.14

14. Un disco aislante de radio R está cargado eléctricamente con una carga +Q repartida uniformemente por su superficie. Calcúlese el campo magnético B que se origina en su centro al girar alrededor de su eje de simetría con velocidad angular '.

La densidad de carga superficial es 2

QR

��

Consideramos una corona circular r y espesor dr, cuya superficie diferencial es d 2 dS r r�� y que posee una carga eléctrica infinitesimal

2 2

2d d 2 d dQ Qq S r r r rR R

� ��

� � �

Esta carga se mueve con una velocidad v, por lo que crea un campo magnético dado por la ley de Biot-Savart:

02d d

4rq

r��

��

v eB

con ( ) ( )r r r r r rr r r r� � � � � � � � � � � � � � �v � r � e v e � e e e � e �

por lo que el campo elemental creado por la rotación ' de la corona es:

0 0 02 2 2 2

2d d d d4 4 2

Qr Q rq r r rr R r R

� � �� � �

� � �� �B �

Integrando la expresión anterior, dado que todas las coronas circulares contribuyen en la misma dirección, obtenemos

0 0 02 20

d2 2 2

RQ QR QrR R R� � �� � �

� � ��B � � �

Así, el campo magnético en el centro del disco tiene la mima dirección que ' y viene dado por la expresión:

0

2QR

��

�B �

'

r

dr

r

dr

v='� rer

B

- 429 -

Física Universitaria: Problemas de Física Campo magnético. E04.15

x

y

er

dq r

dB

er

dq

r dB

O B

15. Una semicircunferencia, de radio r, de material dieléctrico está uniformemente cargada con una densidad lineal de carga �. Cuando la hacemos girar con velocidad angular ' constante alrededor de su diámetro, calcular el campo magnético B que se producirá en el punto O.

Descomponemos la semicircunferencia en elementos de longitud dl = r d�, con una carga dq = �dl = �r d�. Campo magnético creado por una carga (dq) en movimiento (ley de Biot):

02d d

4q

r��

�� rv eB

con cos cossen

sen0 0

x x ry

y r

� ��

� � � �� � � �� � ��� �� � �� �� � �� � �� �� �� � � �� �� � ��� �� �� � re r

La velocidad del elemento de carga dq genérico es: 0

0 00 0

xy

y

� � � � � �� � �� � �� � �� � �� � �� � � � �� � �� � �� � �� � �� � �� � �� � �� � �� � � v � r

De modo que

20 cos sen sen d0 sen cos sen cos d

0 0 0

yy r

y

� � � � �� � � � � � �

� �� � � � � �� ��� � �� � � �� � � �� � � �� � � �� � � � � � � �� � � �� � � �� � � �� � � � �� � �� � �� � � �� � � ��� � � � rv e

Y sustituyendo en la ley de Biot, obtenemos: 2 2

0 0 02 2

sen d sen dd dd sen cos d sen cos d4 4 40 0

q r rr r

� � � �� � �� �� � � � �� � � �

� � �

� � � �� �� �� �� �� �� �� � � � � �� �� �� �� �� �� �� �� �� � rB v e

Integramos para todo el arco semicircular, entre 0 y �:

2

00

2

00

sen 2sen d2 4 2

sensen cos d 02

��

��

� � �� �

�� � �

�� � �� � �� � ��� � �� � ��� � ��� � �� �� � �� � ��

Y el campo resultante en O es

0 0

0

/ 20

4 80

8

�� �

� �

�� ���

� ��� �� �� �� �� �� ����

�# B

B i

de modo que su dirección es la de �, i.e., la del diámetro.

r

�O

- 430 -

Física Universitaria: Problemas de Física Campo magnético. E04.16

16. Por dos conductores rectilíneos e indefinidos, situados perpendicularmente entre sí, en un plano, circulan intensidades constantes I1 e I2, como se indica en la figura. a) Determinar el vector campo magnético creado por dichas corrientes en un punto genérico P del plano Oxy. b) ¿En qué punto del plano Oxy el campo es nulo?

a) Comenzamos determinando, mediante el teorema de Ampère, el campo magnético a una distancia r de un largo conductor rectilíneo que transporta una intensidad de corriente I. Para ello calculamos la circulación de campo B a lo largo de una trayectoria circular (línea de campo) situada en un

plano perpendicular al conductor:

00d d d 2

2B l B Br II

rl B � �

��

� � � � � � � � �B l� � �

El campo magnético en P(x,y) se obtiene como la superposición de los campos magnéticos creados por cada uno de los conductores en dicho punto; esto es,

0 01 21 2

0 1 2

2 2 2I II I

y x y x��

� �� �

� � � � �� ��� � �� �� ���

B B B k k k

b) La condición de que el campo magnético sea nulo nos lleva a escribir

0 1 2 1 2 1

2

0 02

I I I I IB y xy x y x I

��

� ���� � � � � � �� �� ���

que es la ecuación de una recta que pasa por el origen de coordenadas y cuya pendiente es I1/I2.

I2

I1 O x

P(x,y)y

I

r B

- 431 -

Física Universitaria: Problemas de Física Campo magnético. E04.17

17. Determinar el flujo magnético a través de una espira cuadrada de lado a, situada a una distancia b de un hilo conductor, contenido en su mismo plano, que transporta una corriente constante I.

Comenzamos determinando, mediante el teorema de Ampère, el campo magnético a una distancia r de un largo conductor rectilíneo que transporta una intensidad de

corriente I. Para ello calculamos la circulación de campo B a lo largo de una trayectoria circular (línea de campo) situada en un plano perpendicular al conductor:

00

2d d d 24

IB l B l B r I Br

�� �

�� � � � � �� � �B l� � �

Así pues, el campo magnético es perpendicular al plano de la espira, su sentido es hacia adentro y su magnitud decrece con la distancia al hilo rectilíneo conductor. Calculamos el flujo a través de una franja estrecha, de espesor dr, de superficie dS = adr, situada a una distancia r del hilo.

0 02 dd = d d d 24 4

I rB S a r Iar r

� ��

� �� � � �B S

e integramos sobre toda la superficie de la espira,

0 0 2 ld24

n4

b a

b

b aIab

rIar

��

��

� ��

I

a

a

b

I

r

a

b

a

dr

I

r B

- 432 -

Física Universitaria: Problemas de Física Inducción magnética. E05.1

1. En una zona del espacio existe un campo magnético uniforme B = -Bk. Una varilla delgada, conductora, de longitud L está situada paralelamente al eje Oy. Determinar la fuerza electromotriz inducida en la varilla cuando ésta se mueve con velocidad constante v e indicar el extremo de la varilla que estaría a mayor potencial en los siguientes casos: a) v = vi, b) v = vj, c) v = vk.

La f.e.m. inducida en un conductor rectilíneo, de longitud l, que se mueve con velocidad v en un campo magnético uniforme B viene dada por el producto mixto de esas tres magnitudes; i.e.,

�� �l v B� �

de donde se siguen fácilmente los resultados.

a) �0 0

0 00 0

lv lvB

B� � � � �

�l v B� � ,

o sea, dirigida desde C hacia A (en la dirección de L), por lo que el extremo A estará a mayor potencial que el C.

b) �0 00 0 00 0

lv

B� � � �

�l v B� �

por lo que todos los puntos de la varilla se encuentran al mismo potencial.

a) �0 00 0 00 0

lvB

� � � ��

l v B� �

por lo que todos los puntos de la varilla se encuentran al mismo potencial.

+ + + + +

+ + + + +

+ + + + +

+ + + + +

A

C

y

x

l

- 433 -

Física Universitaria: Problemas de Física Inducción magnética. E05.2

2. Por un conductor rectilíneo e indefinido circula una intensidad I. Un segundo conductor de longitud l, está situado perpendicularmente al primer conductor y se desplaza en la misma dirección y sentido que la intensidad con una velocidad v. Determinar la diferencia de potencial inducida que se origina entre los extremos del segundo conductor.

Comenzamos determinando, mediante el teorema de Ampère, el campo magnético a una distancia r de un largo conductor rectilíneo que transporta una intensidad de corriente I. Para ello calculamos la circulación de campo B a lo largo de una trayectoria circular (línea de campo) situada en un plano perpendicular al conductor:

00d d d 2

2IB l B l B r I Br

�� �

�� � � � �� � �B l�� � �

Como el conductor MN se mueve en el campo magnético creado por el otro conductor, se induce en el una f.e.m.. Como el campo magnético B no es constante en todo el conductor MN, consideramos un elemento de longitud dr, en el que la f.e.m. inducida será

d d ( ) dv B r� � �l v B� �

en la dirección del producto vectorial de v�B; esto es, de N hacia M, como se ilustra en la figura. La f.e.m. inducida en todo el conductor se obtiene por integración:

00 dd2

ln2

a l

a

Iv rv B r Br

Iv a la

��

��

� ��

� �� ��

La diferencia de potencial entre los extremos del conductor en movimiento coincide con el valor de la f.e.m. anteriormente calculada, estando el extremo M a mayor potencial que el N.

� � � � �

� � � � �

� � � � �

� � � � �

� � � � �

� � � � �

I

r

l a

v

dr

B

NM

Br

I

- 434 -

Física Universitaria: Problemas de Física Inducción magnética. E05.3

3. Una varilla conductora, de longitud L, está girando con velocidad angular constante (') alrededor de un eje fijo perpendicular a ella y que pasa por su extremo. En el mismo plano de rotación de la varilla, un conductor rectilíneo indefinido transporta una corriente eléctrica constante (I) y pasa por el extremo de la varilla. Determinar la f.e.m. inducida en la varilla en función del tiempo.

De conformidad con la notación que se indica en la figura, el campo magnético creado por una corriente rectilínea indefinida en un elemento del conductor viene dado por

0 0

2 2 cosI IBx r t

� �� � �

� �

La velocidad de dicho elemento es v = �r, en la dirección que se indica en la figura. Como consecuencia de su movimiento, en el elemento del conductor se induce una f.e.m. dada por la expresión

0

0

d d ( ) d d2 cos

d d2 cos

Iv B r r rr t

I r

��� �

� �� �

� � � � � � �

# � �

r v B�

e integrando a lo largo de toda la varilla, para tener en cuenta las contribuciones de todos los elementos, obtenemos la f.e.m. inducida en la varilla:

00

0d

c2 2 oscosL

r I LI� �� ��

� ��

� � � ���

en la dirección que se indica en la figura.

I

'

L

I 't

x

B v

dr

r x

d�

D

- 435 -

Física Universitaria: Problemas de Física Inducción magnética. E05.4

4. Una varilla conductora, de longitud L, está girando con velocidad angular constante (�) alrededor de un eje fijo perpendicular a ella y que pasa por su centro. En el mismo plano de rotación de la varilla, un conductor rectilíneo indefinido transporta una corriente eléctrica constante (I). Determinar la f.e.m. inducida en la varilla en el instante que se indica en la figura.

El campo magnético creado por un conductor rectilíneo indefinido, a una distancia h del

mismo viene dado por la expresión 0 24

IBh

��

� y, en la

figura adjunta, es perpendicular al plano del dibujo, entrante a la derecha del hilo conductor. Por consiguiente, en un elemento infinitesimal de la varilla, de longitud dx, situado a una distancia x de su centro, la magnitud del

campo magnético es 0 24

IBb x

��

��

. Puesto que dicho

elemento se está moviendo con una velocidad v = �x, generará una f.e.m. inducida elemental dada por

0 02 dd d ( ) d d4 4

II x xx vB x xb x b x

� � ��� �

� � � � � � � � �� �

l v B�

en la dirección indicada en la figura. Integrando la expresión anterior, para sumar las contribuciones de todos los elementos de la varilla, tenemos

/2 /20 0 0 0

/2 /2

d d 1 d ln4 4 4 4

L L a L

L L a

I I I Ix x h b b a Lh h L bb x h h a

� � � � � � � �� � � �

� � �

� �

� � � �� �� �� �� � � � � � � � � �� �� �� �� �� � �� � ��

habiendo utilizado el siguiente cambio da variable d dh b x h x� � � . El resultado anterior lo rescribimos en la forma:

0 1

2( ) ln

4I a La L L

a� ��

� �� ��� � � �� ��� �

Otro método: Ley de Faraday

0

0 0

/20 0

/2

1

2

d ( d ) d d d d d d d2

d d dd dd 2 d 2

d ... ( ) ln2 4

L

L

IS r r r r B S r rb r

II r rr rt b r t b rI Ir r a La L L

b r a

�� � � �

�� � �� �� �

� � � �� �

� � � ��

� � � � � �� �

� �� ��� � � � � � �� ��� ��

a L

�I

a L �

I

v B

b h

y

x

dr r

dSd��

- 436 -

Física Universitaria: Problemas de Física Inducción magnética. E05.5

5. La espira rectangular de base b = 0.2 m y altura h = 0.5 m de la figura se traslada con velocidad constante v = 3 j (m/s) en una región del espacio donde existe un campo magnético no homogéneo dado por B = y i (T). Determinar la fuerza electromotriz inducida en la espira, indicando en un esquema el sentido de la corriente que se origina.

La f.e.m. inducida sobre un conductor rectilíneo, de longitud l, que se mueve se mueve con una velocidad v en un campo magnético B, viene dada por la expresión:

( )� � �l v B� y su sentido es el del producto vectorial v×B. En el caso en que los tres vectores sean ortogonales entre sí, la expresión anterior se reduce a

l v B�� Las f.e.m. inducidas sobre los lados superior e inferior de la espira son nulos; ya que el producto mixto es nulo por ser l �v. Las f.e.m. inducidas sobre los lados laterales de la espira tienen el sentido que se indica en la figura, siendo sus magnitudes:

1 1 2 2 ( )hv B hv y hv B hv y b� � � � �� �

lo que da como resultado una f.e.m. neta en la espira en el sentido horario, tal como se ilustra en la figura, siendo su magnitud:

2 1 ( )hv y b hv y h Svbv� � � � � � �� � � donde S es la superficie de la espira. La espira estará recorrida por una corriente eléctrica inducida en el sentido horario, cuya intensidad será

SvIR R

� ��

siendo R la resistencia de la espira.

h v

b

yx

z

v

�1 �2I

- 437 -

Física Universitaria: Problemas de Física Inducción magnética. E05.6

6. Una bobina rectangular de 80 vueltas, 20 cm de anchura y 30 cm de longitud está situada en un campo magnético B = 0.8 T perpendicular al plano de la bobina y dirigido hacia dentro de la página. Como se indica en la figura, tan sólo la mitad de la bobina se encuentra en la región del campo magnético. La resistencia de la bobina es de 30 *. Determinar la magnitud y dirección de la corriente inducida al desplazarse la bobina con una velocidad de 2 m/s en los siguientes casos: a) hacia la derecha, b) hacia arriba y c) hacia abajo.

Ley de la inducción de Faraday: ddt

N �� ��

a) En este caso el flujo a través de la bobina permanece constante, por lo que será: 0 0I� ��

b) El flujo ( )2hBS Blx Bl vt� � � � � varía (aumenta) con el tiempo,

de modo que d

80 0.8 0.20 2 25.6 Vdt

N NBlv�

� � � � � � ��

(ley de Lenz)0.85325. A630

IR

� � � ��

c) El flujo ( )2hBS Blx Bl vt�� � � � varía (disminuye) con el tiempo,

de modo que d

80 0.8 0.20 2 25.6 Vdt

N NBlv�

� � � � � � ��

(ley de Lenz)0.85325. A630

IR

� � � ��

Otro método: A partir de la expresión de la f.e.m. inducida sobre un conductor rectilíneo en movimiento en un campo magnético, ( )� � �l v B� (producto mixto), se siguen fácilmente los siguientes resultados: a) 0�� , por ser l v� (el producto mixto es nulo) b)

80 0.20 2 0.8 25.6 V

0.853 A25.630

NlvB

IR

� � � � � �

� � �

��

c)

80 0.20 2 0.8 25.6 V

25. 0.853 A630

NlvB

IR

� � � � � � � � ��

� �� �

��

+ + + + + + + + + + + + + + + + + + + + + + + + + + + + + +

30cm

20cm

80 vueltas

x

+ + + + + + + + + + + + + + + + + + + + + + + + + + + + + +

l

x

h

v

I

l Bv

+I

-v

-I

+ + + + + + + + + + + + + + + + + + + + + + + + + + + + + +

l

x

h

v

I

- 438 -

Física Universitaria: Problemas de Física Inducción magnética. E05.7

7. Un conductor rectilíneo indefinido y una espira cuadrada están situados en un mismo plano. La espira, de lado a, tiene el lado más cercano al conductor paralelo al mismo y a una distancia igual a su lado. Determínese la fuerza electromotriz inducida en la espira cuando por el conductor rectilíneo circula una corriente

seni I t�� .

El teorema de Ampère nos permite determinar la intensidad del campo magnético a una distancia r de un largo conductor rectilíneo que transporta una intensidad de corriente i. Para ello, calculamos la circulación de campo B a lo largo de una trayectoria circular (línea de campo) situada en un plano perpendicular al conductor:

00d d d 2

2iB l B l B r i Br

�� �

�� � � � �� � �B l�� � �

El campo magnético es perpendicular al plano de la espira, su sentido es hacia adentro y su magnitud decrece con la distancia al hilo rectilíneo conductor. Calculamos el flujo a través de una franja estrecha, de espesor dr, de superficie dS = adr, situada a una distancia r del hilo.

0 0 dd = d d d2 2

i rB S a r iar r

� ��

� �� � � �B S

e integramos sobre toda la superficie de la espira, 2

0 0 0d ln 2 ln 2 sen2 2 2

a

a

ria ia Ia tr

� � �� �

� � �� ���� � � �� ��� �

De acuerdo con la ley de Faraday, la f.e.m. inducida en la espira es

0 0d dln 2 s ln 2end

c2 d

os2

Ia tt

tt

I a��

���

��

�� �� � ���� �� ��� �� � � � ��� ���

a

i

a

B r

i

i r

a

a

a

dr

� � � � � � � � � � � � � � � � � � � � � � � � � � � � � � � � � � � � � � � � � � � � � � � � � � � � � � � �

- 439 -

Física Universitaria: Problemas de Física Inducción magnética. E05.8

8. Una espira cuadrada de lado a, tiene una resistencia R, y penetra perpendicularmente en una franja donde existe un campo magnético B, con una velocidad v que forma un ángulo de 45º con el límite de la franja. Determínese la fuerza que hay que realizar cuando penetre en dicha franja, y la fuerza al salir de la misma. Indíquese el sentido de la intensidad inducida y la fuerza en ambos casos.

Fuerza electromotriz inducida por un conductor rectilíneo de longitud l que se mueve con una velocidad v en un campo magnético B:

ind ( )� � �l v B�

Fuerza que ejerce un campo magnético B sobre un conductor rectilíneo, de longitud l, que transporta una corriente I:

( )I� �F l B

En ambos casos, tan solo la componente “vertical” de la velocidad, i.e. 2 / 2v , contribuye a la corriente inducida.

Entrando

ind2 2

2 2a vB avB� �� (de izquierda a derecha)

indind

22

avBIR R

� ��

(sentido antihorario)

2 2

ind2

2a vBF I aB

R� � (sentido hacia arriba)

apF F� � (hacia abajo, se opone a entrar)

Saliendo

ind2 2

2 2a vB avB� �� (de izquierda a derecha)

indind

22

avBIR R

� ��

(sentido horario)

2 2

ind2

2a vBF I aB

R� � (sentido hacia arriba)

apF F� � (hacia abajo, se opone a salir)

Nota: Las fuerzas sobre los lados “laterales” son iguales y opuestas, por lo que tienen una resultante nula.

a v

� � � � � �� � � � � �

� � � � � � � � � � � � � � � � � � � � � � � � � � � � � � � � � � � � � � � � � � � � � � � � � � � � � � � 2

2v

a

F �

I

Fap

I

� � � � � � � � � � � � � � � � � � � � � � � � � � � � � � � � � � � � � � � � � � � � � � � � � � � � � � �

a

F

I

Fap

I

- 440 -

Física Universitaria: Problemas de Física Inducción magnética. E05.9

9. En una región del espacio existe un campo magnético giratorio cuya expresión en función del tiempo viene dada por

0 0cos senB t B t� �� �B j k

Una espira cuadrada de lado a gira alrededor del eje Ox con igual velocidad angular '. Determinar la f.e.m. inducida si: a) el sentido de giro de la espira coincide con el del campo magnético, b) el sentido es opuesto.

Expresamos vectorialmente la superficie contorneada por la espira; esto es cos( ) sen ( ) cos senS S t S t S t S t� � � �� 3 � 3 � 3j k j k

donde el doble signo de la velocidad corresponde a las dos posibilidades previstas en el enunciado del problema. El flujo a través de la espira es

02 20 0

00

0 0 ctecos cos (cos sen )

cos 2sen sen

B SB t S t B S t t

B S tB t S t� � � � �

�� �

� � � �� � �� � ��� �� � �� �� � � � 3 �� � �� �� �� � �� �� � �� � �� �3� � B S �

Y la f.e.m. inducida viene dada por 2 2

ind 0 0d d= (cos sen ) ( 2cos sen 2sen cos )d d

B S t t B S t t t tt t�

� � � � � � �� � � 3 � � � 3�

ind 00

02 (cos sen sen cos )

2 sen2B S t t t t

B S t� � � � �

� �

���# � � ����� �

a) El sentido de giro de la espira coincide con el del campo magnético:

ind 02 (cos sen sen s 0co )B S t t t t� � � � �� � ��

lo que resulta obvio, ya que al girar la espira con la misma velocidad angular y en el mismo sentido con que lo hace el vector campo magnético, el flujo a través de ella permanece constante. b) El sentido de giro de la espira es opuesto al del campo magnético:

20

ind 0 02 (cos sen sen cos

2 sen(

) 2 (2sen cos

2 )

)B S t t t ta

B S tB t

t� � � � �

� �

�� � � �

lo que también resulta obvio, ya que el resultado anterior se puede escribir en la forma 2 2

ind 0 02 sen(2 ) senB a t B a t� � � �� , ya que al girar la espira con la misma velocidad angular, pero en sentido opuesto al de rotación del vector campo magnético, equivale a una rotación de la espira con una velocidad angular * = 2' en un campo magnético estacionario.

a y

x

z

- 441 -

Física Universitaria: Problemas de Física Inducción magnética. E05.10

10. La espira de forma cuadrada, de lado a y resistencia eléctrica R, que se muestra en la figura, gira alrededor del eje x con velocidad angular constante �. En el instante t = 0, su posición es la representada en la figura. Supongamos que existe un campo magnético no uniforme de dirección constante y paralela a la del eje z, tal que su módulo venga dado por B = Ky, donde K es una constante positiva. a) Determinar la intensidad que circula por la espira en un instante genérico t; i.e., i(t). b) Representar el sentido de la intensidad cuando la espira se encuentra en cada uno de los cuadrantes que recorre en su movimiento.

a) La f.e.m. inducida sobre un conductor rectilíneo, de longitud l, que se mueve con una velocidad v en un campo magnético uniforme B viene dada por

( )� �l v B� � Puesto que la f.e.m. tiene el sentido definido por el producto vectorial ( )�v B , tan solo se induce f.e.m. a lo largo del lado de la espira opuesto al eje de rotación. Además, el campo magnético, aunque no es uniforme, presenta un valor constante a lo largo de ese lado (conductor), por lo que podemos aplicar la expresión anterior; esto es,

� � �3 31sen cos sen sen cos sen 22

avB t a a Ka t t Ka t t Ka t� � � � � � � � �� � � ��

Otro método Como la espira se encuentra en movimiento (rotación), el flujo que la atraviesa varía en el transcurso del tiempo; esta variación del flujo produce una f.e.m. inducida en la espira. En un instante genérico t, la posición de la espira estará determinada por el ángulo �t. Para calcular el flujo que la atraviesa en ese instante, observamos que el campo magnético no tiene la misma intensidad en todos los puntos de la superficie de la espira. En consecuencia, procedemos por integración, descomponiendo la superficie de la espira en bandas estrechas de espesor infinitesimal d�, tal como se ilustra en la figura, y superficie dS = ad�, en las que el campo magnético tiene un valor constante B = Ky = K�cos�t. Así, obtenemos

� � 2 3 2

0

1d d cos cos d cos cos d cos2

aB S t K t a t Ka t Ka t� � � � � � � � � �� � � � �� � � �B S�

Aplicamos la ley de Faraday para determinar la f.e.m. inducida en la bobina en un instante genérico,

�3 3ind

d 1 12cos sen sen 2d 2 2

Ka t t Ka tt�

� � � � �� � � �E

La intensidad de la corriente que circula por la espira en un instante genérico es

�3

indind

1 sen 22

Kai tR R

��� �

b) De acuerdo con la regla de Lenz y con la del producto ( )�v B , el sentido de la corriente inducida es la indicada para cada uno de los cuadrantes.

- 442 -

Física Universitaria: Problemas de Física Inducción magnética. E05.11

11. Una bobina rectangular, de 80 vueltas y de dimensiones 20 cm � 30 cm, está situada en un campo magnético B = 0.8 T dirigido hacia dentro de la página. Como indica la figura, sólo la mitad de la bobina se encuentra en la región del campo magnético. La resistencia eléctrica de la bobina es de 30 *. Determinar la magnitud y dirección de la corriente inducida al desplazarse la bobina con una velocidad de 2 m/s (a) hacia la derecha, (b) hacia arriba y (c) hacia abajo.

La f.e.m. inducida en un conductor rectilíneo en movimiento en un campo magnético externo viene dada por el producto mixto ( )� �l v B� � . En los tres supuestos considerados, tan solo contribuyen a la f.e.m. inducida neta los conductores situados en el lado superior de la bobina, en los que

00 00

a

B

� � � �� �� �� �� �� �� �� �� �� �� �� �� �� �� ��� � l B

a) En este caso será 00

v� ��� �� �� � �� �� ���� v , por lo que el producto mixto será

nulo ( l v� ). Así, 0 0I� ��

b) En este caso será 0

0v

� ��� �� �� � �� �� ���� v , por lo que

0 00 0 0 00 0 0 0

a a vB NavBN v N NavB IRB

� � � � � � � � � ��� � � � �� � � � � �� � � � �� � � � �� � � � �� � � � � �� � � � �� � � � �� � � � �� � � � �� � � � �� � � � �� � � � �� � � � ��� � � � � � � �

25.680 0.20 2 0.8 25.6 V 0.85 A30

NavB I �� � � � � � � � � � � ��

en el sentido antihorario.

c) En este caso será 0

0v

� ��� �� �� �� �� �� ���� v , por lo que

0 00 0 0 00 0 0 0

a a vB NavBN v N NavB IRB

� � � � � � � � � �� � � � �� � � � �� � � � �� � � � �� � � � �� � � � � �� � � � �� � � � �� � � � �� � � � �� � � � �� � � � �� � � � �� � � � ��� � � � � � � �

25.680 0.20 2 0.8 25.6 .85 A30

0VNavB I� � � � � � � ��

en el sentido horario.

� � � � � � � � � � � � � � � �

a =20cm

b =30cm

� � � � � � � � � � � � � � � �

a =20cm

l

xy +I

- 443 -

Física Universitaria: Problemas de Física Inducción magnética. E05.12

12. Una espira conductora circular elástica se expansiona a una velocidad constante, de modo que su radio viene dado por R = R0 + vt. La espira se encuentra en una región de campo magnético constante perpendicular a la misma. Determinar la fuerza electromotriz generada en la espira. Despreciar los efectos posibles de autoinducción.

La fuerza electromotriz inducida en la espira viene dada por la expresión: ddt�

� �� (ley de inducción de Faraday)

con

�20B S B R vt� �� � �

de modo que

� � �2 20 0 0

d d 2d d

B R vt B R vt Bv R vtt t� � �� � � �� � � � � � � � �� � � �� �

Esto es,

�02 Bv R vt�� � �� �

en el sentido antihorario (ley de Lenz).

�555�555�555�555�555�555�555�555�555�555�555�555�555�555�555�555�555�555�555�555�555�555�555�555�555�555�555�555�555�555�555�555�555�555�555�555�555�555�555

R

B

- 444 -

Física Universitaria: Problemas de Física Inducción magnética. E05.13

13. Un anillo delgado y conductor, de radio r, se encuentra en un campo magnético uniforme de dirección perpendicular al plano del anillo y que varía con el tiempo según la ley B = kt, donde k es una constante positiva. Determinar la intensidad del campo eléctrico en el anillo (su módulo y su sentido). Explicar la naturaleza de dicho campo eléctrico.

Calculamos el flujo magnético a través del anillo 2BS r kt� �� �

Puesto que el flujo varía con el tiempo, se induce en el anillo una f.e.m. que viene dada por la ley de Faraday:

2dd

r kt�

�� � � ��

y tiene el sentido que se indica en la figura (antihorario). La f.e.m. se define como la circulación del campo eléctrico (no-electrostático) a lo largo de todo el anillo. Calculamos dicha circulación a lo largo de una línea de campo:

ne ne ne ned d d 2E l E l rE�� � � �� � �E l� �� � �

de modo que 2

ne 2 2 2r k krE

r r�

� �� � � � ��

y tiene el mismo sentido que la f.e.m.

Ene

� � � � � � � � � � � � � � � � � � � � � � � � � � � � � � � � � � � � � � � � � � � � � � � � � � � � � � � � � � � � � � � � � � � � � � � � � � � � � � � � � � � � � � � � � � � � � � � � � � � � � � � � � � � � � � � � � � � � � � � � � � � � � � � � � � � � � � � � � � � � � � � � � � � � � � � � � �

B

fem

- 445 -

Física Universitaria: Problemas de Física Inducción magnética. E05.14

14. Una varilla metálica gira con velocidad angular constante alrededor de un eje per-pendicular a ella y que pasa por uno de sus extremos, deslizando sobre un anillo conductor de radio l, como se esquematiza en la figura. El eje de la varilla está conectado al borne positivo de un generador de f.e.m., cuyo borne negativo está conectado al anillo. Si existe un campo magnético uniforme perpendicular al plano del anillo, determinar la velocidad angular que adquirirá la varilla y el sentido de ésta.

El generador produce una corriente eléctrica, en el sentido indicado en la figura, que tomaremos como sentido positivo. La varilla se encuentra en un campo magnético externo (B) y está recorrida por la intensidad de corriente I. En consecuencia, la varilla está sometida a una fuerza �I� �F l B , uniformemente distribuida que da lugar a un momento resultante que la hace girar en el sentido horario con velocidad angular creciente (movimiento acelerado de rotación). Como consecuencia del movimiento de la varilla conductora en un campo magnético externo (B), se induce en ella una cierta f.e.m.. El campo no-electrostático en la varilla, dado por,

ne � �E v B , está dirigido a lo largo de la varilla, hacia el extremo fijo de la misma (sentido negativo). Cada elemento de la varilla en rotación tiene una velocidad distinta, esto es, v = r', siendo r la distancia del elemento al extremo fijo de la varilla. Calculamos la f.e.m. inducida mediante integración:

� 2ind necond cond cond 0

1d d d d2

lvB r B r r Bl� �� � � � � � � � �� � � �E l v B l� � �

La varilla quedará libre de la acción de fuerzas y momentos cuando alcance una cierta velocidad angular tal que la f.e.m. inducida sea igual (y opuesta) a la del generador, ya que entonces la intensidad de la corriente será nula. Esto es,

ind ind0IR� � � �� � � �

de modo que

ind ind0IR� � � �� � � �

l B

. . . . . .

. . . . . .

. . . . . .

. . . . . .

. . . . . .

l

B

. . . . . .

. . . . . .

. . . . . .

. . . . . .

. . . . . .

� v F

�ind

�ind

I I

I I

- 446 -

Física Universitaria: Problemas de Física Inducción magnética. E05.15

B débil

B

l

S N

B

15. Una bobina muy larga tiene 1000 espiras/m de longitud y está recorrida por la una intensidad i = 3 cos 100 t (en unidades del S.I.) En su interior y sobre su mismo eje, colocamos una pequeña bobina de 1 cm de radio y 50 espiras independientes de las de la bobina larga. a) Calcular la intensidad del campo magnético en el interior de la bobina larga. b) Determinar el valor del flujo magnético a través de la bobina pequeña. c) Calcular la f.e.m. inducida en la bobina pequeña.

a) Determinamos el campo magnético B en el interior de la bobina o solenoide largo a partir del Teorema de Ampère, calculando la circulación de B a lo largo de una línea de campo:

0d d dB l B l Bl Ni�� � � �� � �B r�� � �

de donde

00 0

7

3

cos

4 10 1000 3cos100 0.0012 cos100

3.77 10 cos100 (S.I.)

NiB ni nI tl

t t

B t

�� � �

� ��

� � � �

� � � � �

# � �

que puede considerarse uniforme en el interior de la bobina.

b) Flujo ligado a través de la bobina pequeña:

� �23 550 3.77 10 cos100 0.01 5.92 10 cos100 (S.I.)NBS t t� �� �� � � � � �

c) Determinamos la f.e.m. a partir de la Ley de la Inducción de Faraday:

�5 3d d 5.92 10 cos100 5.92 10 sen100 (S.I.)d d

t tt t� � �� � � � � � ��

de modo que máx 5.92 mV�� .

- 447 -

Física Universitaria: Problemas de Física Inducción magnética. E05.16

16. Determinar el coeficiente de autoinducción de un solenoide tórico constituido por N espiras. El radio medio del toro es R y su sección cuadrada de lado a.

Determinamos el campo magnético B en el interior del solenoide a partir del Teorema de Ampère, calculando la circulación de B a lo largo de la línea de campo circular de radio R:

0d d d 2B l B l B R NI� �� � � �� � �B r�� � �

de donde

0 0

2NI NIBl R� �

�� �

El flujo ligado en el solenoide será 2 2

0 0

2N S N SN NSB I Il R

� ��

�� � �

El coeficiente de autoinducción se define como el flujo ligado por unidad de intensidad de corriente:

2 20 0( )

2N S N SNL

I l R� ��

�� � �

o bien, con S = a2, tenemos 2

02 2

02( )

2N a NL

RN

la

I�� �

�� � �

Cálculo más detallado. La intensidad del campo magnético B no es constante en toda la sección del solenoide toroidal, sino que decrece con la distancia r al eje de simetría de rotación del mismo, ya que

0 0

2NI NIBl r� �

�� �

por lo que el cálculo correcto del flujo a través de una espira se debe hacer mediante integración; i.e.,

2 20 0 0 0

2 2

2d 2d d ln ln2 2 2 2 2

2

a aR R

a aS R R

aRNI NIa NIa NIar R aa r ar r R aR

� � � ��

� � � �

� �

� �

� �� � � � �

��� � �B S�

de modo que el flujo ligado y la autoinducción vienen dados por

022

02 ( )ln2 2

2ln2 2

N Ia R a NN LR a

N aI

R aR a

� �� �

���

� ��

���

R

espira

dr

a

a

R

Eje del toroide

dS

r

- 448 -

Física Universitaria: Problemas de Física Inducción magnética. E05.17

17. En el esquema de la figura, determinar: a) La intensidad que pasa por cada rama en el instante inicial (t = 0), cuando se conecta la batería. b) La intensidad que pasa por cada rama cuando alcanza el estado estacionario (t = $).

a) Inicialmente el condensador está descargado por lo que AB 0qVC

� � .

Al ser AB 0V � por la resistencia de 4 * no pasará corriente.

En la rama de la bobina será ABd 0d

iV Lt

� � , luego en ese instante i no

varía; como antes era nula, ahora seguirá siéndolo. En definitiva, la corriente circula tal como se indica en la figura, a través de la resistencia de 12 * y del condensador (que “deja pasar”), siendo su intensidad

12 1 A12

IR

� � ��

siendo nula en los otros elementos. b) Una vez haya transcurrido un tiempo suficientemente largo (t � $), se alcanza el estado estacionario y todas esas magnitudes permanecerán constantes.

En la rama de la bobina será ABd d0 0d d

i iV Lt t

� � � y la bobina “deja

pasar” la corriente. Por ser AB 0V � , la intensidad por la rama de la resistencia de 4 * será nula. Por la misma razón será nula la intensidad en la rama del condensador, ya que habrán pasado todos los efectos transitorios de carga del condensador. En definitiva, la corriente circula tal como se indica en la figura, a través de la resistencia de 12 * y de la bobina (que “deja pasar”), siendo su intensidad

12 1 A12

IR

� � ��

siendo nula en los otros elementos.

12* 12V

4*

100H

100F

A B

12* 12V

4*

100H

100F

12* 12V

4*

100H

100F

A B

- 449 -

Física Universitaria: Problemas de Física Inducción magnética. E05.18

t

i

If

�/f( ) 1 t RCi t I e�� �0.63If

18. Consideremos un circuito RL en serie conectado a un generador de f.e.m. en corriente continua. En el instante t = 0 se cierra el interruptor S y se inicia la corriente de cierre en el circuito. a) Expresar la intensidad de la corriente de cierre en función del tiempo. b) Definir la constante de tiempo de circuito RL y explicar su significado. c) Explicar el comportamiento del circuito RL a partir de los elementos resistencia y autoinducción.

a) En el instante inicial el interruptor (S) está abierto y la autoinducción está “descargada”. Al conectar el generador (t = 0), comienza a circular la corriente. Calculamos la d.d.p. entre a y b a través de los elementos pasivos y a través de generador (de resistencia interna nula):

abddR L

iV v v Ri Lt

� � � � � �

Separamos las variables i y t: d d dd

i i tL iRt iR L

� � ��

��

E integramos

�0 0

d d/

i ti R tR i L

��� ��

utilizando el cambio de variable: �/

d du R i

u i

� � ����� � ���

� �

�0

/d ln ln / ln/

i R iu Ru R i tu R L

�� �� � � � � � � � �� ��

��

� � � �

/ln 1 1

/ / /

R Rt tL L

R i R i it e eR L R R

� ��� � � � � �

�� � �

� �/

1f f

f

0 01

1 0.63

/

tt i

i eR t i I e I

L Rt i I

R

��

����� � �� �� �� �� �� �# � � � �� �� �� �� ��� �� � ! � �����

b) Constante de tiempo ( � ): tiempo que deberá transcurrir en el proceso de cierre hasta que la intensidad de la corriente alcanza el 63% de su valor final. c) Al aumentar la intensidad de la corriente, se induce una f.e.m. en la L tal que se opone al aumento de la intensidad, por lo que retrasa el aumento de está; la L se comporta como un elemento de inercia que almacena energía en el campo magnético que se va creando en la autoinducción a medida que pasa la corriente.

R L

i

�ind

S

i

a bc

R L

� S

- 450 -

Física Universitaria: Problemas de Física Inducción magnética. E05.19

19. Un circuito serie, alimentado con tensión alterna de 125 V y 50 Hz de frecuencia, está formado por una resistencia de 8 *, una autoinducción 12 * de reactancia y un condensador de 6 * de reactancia. a) ¿Qué intensidad circulará por el circuito? b) Si la corriente alterna varía su frecuencia a 25 Hz, sin variar su tensión, ¿qué intensidad circulará por el circuito? c) Determinar elemento en paralelo necesario para corregir completamente el factor de potencia, en cada caso.

a) Tensión alterna de 125 V y 50 Hz de frecuencia 2 100 rad/s� �� �� �

36.9º8 12 -6 = 8+ 6 = 10 � � j j j� (inductivo)

0º36.9º

36.9º

125= 12.5 10 7.5 A

10 �� � � � j�� �

b) Tensión alterna de 125 V y 25 Hz de frecuencia 2 50 rad/s� �� �) � � "

1 62

2 12

LL

L

CC

C

X L XX LX C XX C

� �

� �

� �

� �

� ) ) )�� )� � � # ��� )��� )� )� � � # ��� ) ) )��

-36.9º8 6 -12 8 - 6 10 ) � � � �j j j� (capacitativo)

0º36.9º

36.9º

12512.5 10 7.5 A

10 �

)) � � � � �

)j�� �

c1) Como el circuito es inductivo, hay que colocar un condensador en paralelo.

react senC

VI I CVX

� �� � �

6sen 7.5 191 10 F =100 1

191�F25

ICV�

� ��� � � �

c2) Como el circuito es capacitativo, hay que colocar una autoinducción en paralelo.

react senL

V VI IX L

��

� � �

3125 106 10 H =sen 50 7.

10 mH5

6VLI� � �

�� � � ��

125 V 50 Hz

12 * 8j * -6j *

125 V 25 Hz

12 * 6j * -12j *

- 451 -

Física Universitaria: Problemas de Física Corriente alterna. E06.1

1. Para determinar la resistencia Rb y la autoinducción Lb de una bobina se la coloca en serie con una resis-

tencia pura y calibrada de 1 k� y se miden las caídas de tensión en esta resistencia, en la bobina y en el circuito serie completo, obteniéndose los siguientes resultados a 50 Hz: 180 V, 50 V y 220 V, respectiva-mente. a) Dibujar el diagrama fasorial de tensiones. b) Determinar la resistencia Rb y la autoinducción Lb de la bobina.

a) La intensidad de la corriente (I) es la misma en la resistencia y en la bobina. En la resistencia, la intensidad y la tensión están en fase. En la bobina (impedancia inductiva), la intensidad está retrasada respecto de la tensión; o lo que lo mismo, la tensión está adelantada respecto de la intensidad. En consecuencia, el diagrama fasorial de tensiones es el que mostramos en la figura. b) Puesto que R b� �� � � (suma fasorial), será

2 2 2 2 2 22 2 2 R b

R b R bR b

220 180 502 cos cos 0.752 2 180 50

V V VV V V V VV V

� �� � � �

� � � � � �� �

de modo que la tensión en la bobina está adelantada 41.4º�� respecto de la intensidad. La intensidad de la corriente es

180 180 mA1000

RVIR

� � �

La impedancia de la bobina es

41.4ºbb 41.4º

50278 208 184 �

i 0.180� � � � � j�� �

de modo que su resistencia y autoinducción valen:

b

bb

208 �

184 585 mH2 50

R

XL� �

� � ��

I

Vb=50V

VR=180V

V=220 V +

R Z I

VR Vb

V

bobina

- 452 -

Física Universitaria: Problemas de Física Corriente alterna. E06.2

2. En una red de 220 V, 50 Hz, se desea instalar una lámpara incandescente especificada para consumir una

potencia de 60 W a una tensión máxima de 120 V. Para conectar esta lámpara a la red antes indicada se pretende instalar un condensador en serie con la misma. a) ¿Qué capacidad deberá tener dicho condensador? b) Si se variara la frecuencia a 60 Hz, ¿qué potencia consumiría la lámpara antes y después de colocar el mismo condensador?

a) Determinamos la intensidad de trabajo de la lámpara y su resistencia eléctrica a partir de los valores nominales de su potencia y tensión de trabajo:

nomnom

nom

nom

nom

60 0.5 A120

120 2400.5

PP VI IV

VVI RR I

� � � � �

� � � � � *

Si conectásemos la lámpara directamente a la red de 220 V, la potencia que consumiría sería

2 2220 202 W240

VPR

� � �

muy superior a la potencia nominal (60 W) y la lámpara se fundiría. Para evitarlo, colocamos una reactancia en serie con la lámpara para aumentar la impedancia de la rama y disminuir la intensidad de la corriente a través de ella. Determinamos el valor de la reactancia apropiada para limitar el paso de intensidad por la lámpara:

nomnom

2 2 2 2 2 2 2

220 440 �0.5

440 240 369 �

V VI ZZ I

Z R X X Z R

� � � �

� � � � � � �

Conocida la reactancia, determinamos la capacidad del condensador:

61 1 1 8.63 10 F 8.63 �F100 369

X CC X� � �

�� � � � � ��

b) Si variamos la frecuencia, manteniendo el mismo condensador, variará la reactancia y la intensidad que circula por la resistencia.

2 2 2 2

2 2

1 100 369 308 � 240 308 369 �120

220 0.60 A 0.60 240 86 W369

X Z R XC

VI P I RZ

�� �

�) ) )� � � � � � � � )

) ) )� � � � � � �)

lo que resulta excesivo por superar en un 44% a la potencia nominal de la lámpara (60 W) por lo que ésta se fundirá a corto plazo.

220 V, 50 Hz

I = 0.5 A

X

R = 240 *

120 V, 50 Hz

I = 0.5 A R = 240 *

- 453 -

Física Universitaria: Problemas de Física Corriente alterna. E06.3

3. Con corriente alterna monofásica de 220 V y 50 Hz se alimenta una

carga. Al conectarle en paralelo un condensador de 891 �F se corrige el factor de potencia desde 0.4 a 0.8. a) Determinar la intensidad que consume dicha carga. b) Calcular la capacidad del condensador que debiera sustituir al anterior para corregir totalmente el factor de potencia.

Cálculos previos:

3 -1

2 50 100 rad/s0.891 mF 891�F 100 0.891 10 0.28 �

cos 0.4 66.4º cos 0.8 36.9ºC C� � �

� �� � � �

� � �

� � � � � �� � � �

a) Deducción de la expresión para corrección parcial del factor de potencia a partir del diagrama fasorial:

act corr act

act

(tg tg ) (tg tg )

0.28 220(tg tg ) tg 66.4º tg 36.9

40

C

VI I I CVX

CVI

� � � � �

�� �

) )� � � � �

�# � � �

)� �

66.4º

36.9

act

acº

t

100 A

5

40 100 Acos cos 66.4º

40 50 Acos cos36.9º

0 A

II

II

)�

� � �

)) � � �

)

b) para corregir totalmente el factor de potencia deberá ser +” = 0, de modo que:

act corr act

3act

tg tg

tg 40 tg 66.4º 1.32 10120 220

1.32 mF

C

VI I I CVX

ICV

� � �

�� �

� � �

�# � � � � �

En estas condiciones, será 0º40 A))�� .

220 V50 Hz IC CZ

IC = Icorr

Iact

I

I’

+ +’

- 454 -

Física Universitaria: Problemas de Física Corriente alterna. E06.4

4. A una red de c.a. de 125 V y 5 Hz se conectan las cargas que se indican en la

figura:

C

1

2

(capacidad pura)

(inductivo)

(inductivo)

= 2.5 �; 28 �; f.p. 0.8 28 �; f.p. 0.6

XZZ

� �

� �

a) Calcular la intensidad que circula por cada carga. b) Calcular la potencia consumida por las cargas. c) Determinar la capacidad del condensador que hay que colocar en paralelo con la carga total para corregir totalmente el factor de potencia.

Cálculos de impedancias:

� �

C 90º

1 37º

2 53º

90º37º 53º1 245º 45ºpar

1 245º

C par 37º

2.5 2.5 �

28 22.4 16.8 �28 16.8 22.4 �

78428 28 10 2 10 10 14.14 2 �39.2 39.2 39.4 2

2.5 10 10 10 7.5 12.5 �

+

�� � �

� � � �

� � � � � �

� � � � �

jjj

jj

j j j

���

� �� � �

� � � a) Determinación de intensidades:

37º

2

37º

C C 37º 90º 127º

par par 37º 45º 8º

8ºpar1

1 37º

8ºpar

422 53º

1258 6 A

12.5

10 2.5 25 V

10 1

10 A

5.05

4.14 141.4 V

141.428

141.

A

5.04

85

2A

� �

� � � � �

� � � �

� � � �

� � �

� � �

j�� �� ��� � ��� ��� �

b) Factor de potencia y potencia consumida: (inductivo)37º f.p. cos37º

cos 125 10 0.8 1000 W0.8

1 kWP VI+

+� � � �� � � � � �

c) Corrección total del factor de potencia con condensador:

react sen1/

sen 153 �10 sen 37º100

F125

VI I CVC

ICV

� ��

�� �

� � �

�� � �

- 455 -

Física Universitaria: Problemas de Física Corriente alterna. E06.5

5. Un circuito de corriente

alterna de 220 V está compuesto de seis elemen-tos (resistencias, capa-cidades y autoinducciones puras) colocados como se indica en la figura, en la que se indican los valores de sus resistencias o de sus reactancias capacitativas o inductivas. Determínense: a) La diferencia de potencial entre los bornes de cada uno de los elementos (con sus defases). b) Las intensidades que circulan por cada uno de los seis elementos (con sus defases). c) La potencia consumida por cada uno de los elementos.

3 impedancias en serie: s 10 10 10 10 �� � � �j j�

3 impedancias en paralelo: pp

1 1 1 1 1 1�1 10 10 1

� � � � ��j j

��

Impedancia total (A-E): AE 11���

Intensidad total y en los elementos en serie: 0AE0

AE 0

22020 A

11� � ��� �

a) Tensiones parciales: b) Intensidades parciales:

AB 0 0 0

BC 0 90º 90º

CD 0 90º 90º

DE 0 0 0

20 10 200 V

20 10 200 V

20 10 200 V

20 1 20 V

� �

� �

� � �

� � �

� � �

� � �

����

0DE1 90º

1 90º

0DE2 90º

2 90º

0DE3 0

3 0

202 A

10

202 A

10

2020 A

1

��

��

� � �

� � �

� � �

�� ��� ��� �

c) Tan sólo se consume potencia en los elementos resistivos:

2 21

2 22

20 10 4000 W4 400 W

20 1 400 W

P I RP

P I R

��� � � � � ���� � � � ��

R=10* XC=10* XL=10*

~ 220 V R=1 *

XL=10*

XC=10*

I

I1

I2

I3

A B C D E

- 456 -

Física Universitaria: Problemas de Física Corriente alterna. E06.6

I

V

retrasada

adelantadaC

V

I

Ireact =I sen+

Iact=I cos++

Icond

6. En el circuito de la figura, hállense: a) La intensidad total y en cada uno de los elementos. b) La diferencia de tensión en bornes de cada elemento. c) La potencia total y la consumida por cada elemento. d) Características del elemento que tendríamos que conectar en paralelo para corregir totalmente el factor de potencia.

Cálculo de impedancias: 2 100 rad/s� �� �� �

� �

� �

390º

90º6

45º p 45ºp

37º (inductivo)

100 22.28 10 7 � 7 7 �

1 1 1 8 � 8 8 �100 398 10 0.125

11 1 1 0.125 0.125 0.177 5.66 4 4 �8 8

7 4 4 4 3 5 �

L L

C C

X L

XC

� �

� �

��

� � � � � � �

� � � � � � �� �

� � � � � � � ��

� � � � � �

j

j

j jj

j j j

���

a) y b) Cálculo de intensidades y de tensiones:

�0º37º 82ºp

36.87º 82º

37º 90º 53º82ºp

8ºp p 37º 45º 82º90º

22044 35.2 26.4 A 2495 31 A

844 7 308 V 249

31 A44 5.66 249 V 8

R

L L

CC

R� �

��

� � ��

��� � � � � � �� �� � � � �� �� �� �� �� � �� �� �� � � � �� �� � � � �� �������

j�� �� �� �� ��� �� �

c) Cálculo de potencias:

(inductivo) (obvio)

(obvio)

7 744 Wf.p. cos cos37º 0.8

cos90º 0cos 220 44 cos37º 7 744 W

cos90º 0

R R R

L L L

C C C

P V IP V I

P VIP V I

��

� � ���� � � �� �� � � �� �� �� � � � ��� �� � ���

Tan solo se consume potencia en el elemento resistivo. d) Corrección total del factor de potencia: El factor de potencia inductivo se corrige con un condensador en paralelo con la carga.

react

cond

react

atrasada: sen

adelantada: 1/

sensenC

I IV VI CVX C

I II CV CV V

��

�� �

� �

� � �

� � �

Sustituyendo valores: 626.4 382 10 F 382 �F

100 220C

��� � � �

8 * 220 V

50 Hz

22.28 mH 398 �F

F

- 457 -

Física Universitaria: Problemas de Física Corriente alterna. E06.7

V

I

I sen +

Iact=I cos+

+’

Icond

+

I’

I’sen +’

Iact tg + Iact tg +’

7. En el circuito de la figura, determínense: a) La intensidad total e intensidades que circulan por cada rama. b) Diferencias de tensión en bornes de cada elemento. c) Potencia disipada en el circuito y factor de potencia. d) Diagrama fasorial de tensiones e intensidades. e) Capacidad del condensador a colocar en paralelo con el circuito para corregir el factor de potencia a 0.95.

Datos: 1 20.12 2.58 � H mFR L L C� �

� � � �

Cálculo de impedancias:

1 2 1 2 90º

3

90º

rad2 2 50 100s

0.12100 12 12 12

1 10 4 4 4100 2.5

L L L L

C C

X X L

XC

� �� � �

� � �

� � �

� � � �

� � � � � � � � �

�� � � � � �

j

j

� �

90º 90º 0A Bpar 90º 37º

A B 90º

(inductivo)12 4 48

6 8 6 12j 8 6 1012 4 8

��� � � � � � � � � �

� �j j

j j� �� �� �

a) Cálculo de intensidades y tensiones:

2 2

1

1

37º 0 37º0º37º

37º 90º 53º37º

127ºpar217º

90ºpar par 37º 90º 127º

127ºpar37º

90º

22 8 176 V22022 A = (17.6 - 13.2 ) A

22 12 264 V10

13211 A

1222 6 132 V

13233 A

4

R R

L L

LL

CC

� �

��

��

� � ��

��

� � ���� � � � � � � ����

� � ��� � � � � �

� � �

j� ���� � ���

�� �� �� �� �

���

c) Potencia y factor de potencia: (inductivo)cos 220 22 cos37º 3872 W f.p. cos cos37º 0.8P VI = =� �� � � � � �

Puesto que la instalación resulta inductiva, deberemos colocar un condensador en paralelo que compense la corriente reactiva retrasada de la instalación. e) Corrección parcial:

� �

act act cond

act

act

tg tg

tg tg1/

tg tg

17.6 tg36.87º tg18.19º100 22

107 �F0

I I IVI CV

CI

CV

C

� �

� � ��

� �

)� �

)� � �

)��

� �� �

R

L2

L1 C220 V50 Hz

- 458 -

Física Universitaria: Problemas de Física Corriente alterna. E06.8

8. a) En el circuito de C.A. que se representa en la figura, calcular la

diferencia de potencial (módulo y fase) entre los puntos A y B ¿Cuánto marca el voltímetro? b) Dibujar el diagrama fasorial de intensidades. c) Explicar si el circuito es capacitativo o inductivo. Calcular la impedancia del circuito.

Calculamos la frecuencia angular y las reactancias de los elementos reactivos:

3

90º

90º3

2� 100� rad/s

100 12.73 10 4 � 4 4 �1 1 3 3 �3 �

100 1.06 10

LL

CC

X L

XC

� �

� �

� �

��

� ��� � � � � � �� � � ��� �� � �� � � � �� � �� ��� � ���

jj

��

a) Las impedancias de cada una de las ramas son:

1 37º

2 53º

4 3 5 �

3 4 5 ��

� � � ����� � � ���

jj

��

Las intensidades en cada rama son:

0º1 37º

1 37º

0º2 53º

2 53º

22044 A 35.2 26.4 A

5

22044 A 26.4 35.2 A

5

��

��

��� � � � � �������� � � � � ������

j

j

�� ��� �

�1 1 8º61.6 8.8 62.2 A�� � � � �j� � �

La d.d.p. entre los puntos A y B viene dada por

AB 1 1 2 2 1 L 2 C� � � � �� � � � � � � � �

esto es, � � �

� � � � �AB 106º

AB 106º

35.2 26.4 4 26.4 35.2 3 61.6 211.2 220 V

35.2 26.4 3 26.4 35.2 4 61.6 211.2 220 V�

� � � � � � � � �

� � � � � � � � �

j j j

j j j j j

��

b) En la figura mostramos el diagrama fasorial de intensidades. c) Sin necesidad de calcular, vemos que el circuito es inductivo, ya que la intensidad está retrasada con respecto a la tensión aplicada. En efecto, si calculamos la impedancia de las dos ramas en paralelo, tenemos:

� �37º 53º 16º 16º1 1

8º8º1 1

5 5 25 253.54 �

4 3 3 4 7 1 50� � � �

��

� � � � �� � � � �j j j� �� � �

O bien

0º8º

2203.54 �

62.2 ��

� � ��� �

220 V50 Hz

4 * 3 *

BA

12.73 mH 1.06 mF

V

I1 I2

I

A

3 *

B

4j *

4 *

-3j *

220 V50 Hz

37º

53º

I1

I2

I

V

- 459 -

Física Universitaria: Problemas de Física Corriente alterna. E06.9

9. a) En el circuito de C.A. que se representa en la figura, calcular la

diferencia de potencial (módulo y fase) entre los puntos A y B ¿Cuánto marca el voltímetro? b) Dibujar los diagramas fasoriales de impedancias e intensidades. c) Con base en los diagramas fasoriales (sin calcular), explicar si el circuito es capacitativo o inductivo.

Calculamos la frecuencia angular y las reactancias de los elementos reactivos:

3

90º

90º6

2� 100� rad/s

100 9.55 10 3 � 3 3 �1 1 4 4 �4 �

100 796 10

LL

CC

X L

XC

� �

� �

� �

��

� ��� � � � � � �� � � ��� �� � �� � � � �� � �� ��� � ���

jj

��

a) Las impedancias de cada una de las ramas son:

1 37º

2 53º

4 3 5 �

3 4 5 ��

� � � ����� � � ���

j

j

��

Las intensidades en cada rama son:

0º1 37º

1 37º

0º2 53º

2 53º

22044 A 35.2 26.4 A

5

22044 A 26.4 35.2 A

5

��

��� � � � � ��������� � � � � �����

j

j

�� ��� �

La d.d.p. entre los puntos A y B viene dada por

AB 1 1 2 C 1 L 2 2� � � � �� � � � � � � � �

esto es,

AB 37º 0º 53º 90º 37º 37º

AB 37º 90º 53º 0º 53º 53º

44 4 44 4 176 176 0

44 3 44 3 132 132 0� � � �

� � � � � � �

� � � � � �

��

de modo que los punto A y B están al mismo potencial y el voltímetro marca cero. b) En la figura mostramos los diagramas fasoriales de impedancias e intensidades. c) Sin necesidad de calcular, vemos que el circuito es capacitativo, ya que la intensidad está adelantada con respecto a la tensión aplicada. En efecto, si calculamos la intensidad total y la impedancia de las dos ramas en paralelo, tenemos:

� � �1 1 8º35.2 26.4 26.4 35.2 61.6 8.8 62.2 A�� � � � � � � � �j j j� � �

� �37º 53º 16º 16º1 1

8º8º1 1

0º8º

5 5 25 253.54 �

4 3 3 4 7 1 50220

62.2 A3.54

� � � ��

� � � � �� � � � �

� � �

j j j� �� � ��� �

220 V 50 Hz

4 *

3 *

BA

9.55 mH

796 �F

V

I1 I2

I

A

3 *

B

4 *

3 *

4 *

220 V50 Hz

Z2

Z1 37º

53º

I1

I2

I

Z V8º

37º

53º

- 460 -

Física Universitaria: Problemas de Física Corriente alterna. E06.10

10. En el circuito de c.a. representado en la figura, determinar el elemento (bobina o condensador) que hay que

colocar entre A y B para corregir completamente el factor de potencia. Datos: R = 4 *, C = 1.061 mF, L = 9.549 mH.

Calculamos las reactancias de la bobina y del condensador: 3

3

100 9.549 10 3�1 1 3 �

100 1.061 10

L

C

X L

XC

� �

� �

� � � � �

� � �� �

Las impedancias de las ramas en paralelo son:

1 37º

2 37º

4 3 5 �

4 3 5 �L

C

R XR X �

� � � � �

� � � � �

j jj j

��

La impedancia de estas dos ramas en conjunto es

� �37º 37º 0º1 2

12 0º1 2 0º

5 5 253.13 �

4 3 4 3 8�� � � �

� � � �j j� �� � �

La impedancia de la carga completa es

0º 0º120º

12

3.13 41.75 �

3.13 4R

R

� � �� �� �� � �

Por consiguiente, la carga es puramente resistiva, su factor de potencia es cos 0º = 1, y no necesita corrección alguna.

L

R R

C R

A

B

220 V50 Hz

- 461 -

Física Universitaria: Problemas de Física Corriente alterna. E06.11

11. a) En el circuito de C.A. que se representa en la figura, calcular la

diferencia de potencial (módulo y fase) entre los puntos A y B ¿Cuánto marca el voltímetro? b) Dibujar los diagramas fasoriales de impedancias e intensidades. c) Explicar si el circuito es capacitativo o inductivo.

Calculamos la frecuencia angular y las reactancias de los elementos reactivos:

3

90º

90º6

2� 100� rad/s

100 9.55 10 3 � 3 3 �1 1 4 4 �4 �

100 796 10

LL

CC

X L

XC

� �

� �

� �

��

� ��� � � � � � �� � � ��� �� � �� � � � �� � �� ��� � ���

jj

��

a) Las impedancias de cada una de las ramas son:

1 37º

2 53º

4 3 5 �

3 4 5 ��

� � � ����� � � ���

j

j

��

Las intensidades en cada rama son:

0º1 37º

1 37º

0º2 53º

2 53º

22044 A 35.2 26.4 A

5

22044 A 26.4 35.2 A

5

��

��� � � � � ��������� � � � � �����

j

j

�� ��� �

�1 1 8º61.6 8.8 62.2 A�� � � � �j� � �

La d.d.p. entre los puntos A y B viene dada por

AB 1 1 2 2 1 L 2 C� � � � �� � � � � � � � �

esto es, � � �

� � � �AB 106º

AB 106º

35.2 26.4 4 26.4 35.2 3 61.6 211.2 220 V

35.2 26.4 3 26.4 35.2 4 61.6 211.2 220 V

� � � � � � � � �

� � � � � � � � �

j j j

j j j j j

��

de modo que los punto A y B están al mismo potencial y el voltímetro marca cero. b) En la figura mostramos los diagramas fasoriales de impedancias e intensidades. c) Sin necesidad de calcular, vemos que el circuito es capacitativo, ya que la intensidad está adelantada con respecto a la tensión aplicada. En efecto, si calculamos la impedancia de las dos ramas en paralelo, tenemos:

� �37º 53º 16º 16º1 1

8º8º1 1

5 5 25 253.54 �

4 3 3 4 7 1 50� � � �

��

� � � � �� � � � �j j j� �� � �

o bien

0º8º

2203.54 �

62.2 �� � ��� �

220 V 50 Hz

4 * 3 *

BA

9.55 mH 796 �F

V

I1 I2

I

A

3 *

B

3j *

4 *

-4j *

220 V50 Hz

Z2

Z1 37º

53º

I1

I2

I

Z V

37º

53º

- 462 -

Física Universitaria: Problemas de Física Corriente alterna. E06.12

I1 I2

I

A

40j *

B

-40j � 30 *

250 V50 Hz

30 *

VR

VC

VAB 53º

37º

37º

200

150

70

V

12. Al circuito de la figura se le aplica una tensión alterna de 250 V a 50 Hz. a) Determinar las intensidades que circulan por cada rama y la intensidad total. Dibujar el diagrama fasorial de intensidades. b) Calcular la diferencia de potencial entre los puntos A y B y su desfase con respecto a la tensión de alimentación.

a) Calculamos la frecuencia angular y las reactancias de los elementos reactivos:

3

90º

6

90º

2 100 rad/s100 127.3 10 40 �

40 40 �

1 10 40 �100 79.6

40 40 �

L

L

C

C

X L

XC

� �� �

� �

� �

� �

� � � � �

� � �

� � ��

� � �

j

j

Las impedancias de cada una de las ramas y las intensidades que las recorren son:

0º1 53º

1 53º 1 53º

2 53º 0º2 53º

2 53º

2505 3 4 A

30 40 50 � 50

30 40 50 � 2505 3 4 A

50

�� �

��

��� � � � � ���� � � �� �� � � �� �� � �� �� � � � � � �����

jj

jj

��� �� �� �

Impedancia total:

53 531 2T 0

1 2

50 50 2500 41.67 �60 60

�� � � ��� �� � �

Intensidad total: � �T 1 2 0º

0T 0

T 0

3 4 3 4 6 A

2506 A

41.67

� � � � � � � ������� � � �����

j j� � ��� �

b) La d.d.p. entre los puntos A y B viene dada por AB 1 1 2 C� �� � � � � , de modo que operando…

En forma módulo-argumento:

� � �AB 53º 0º 53º 90º 53º 37º

180º

5 30 5 40 150 200

90 120 160 120 70 0 70 V� � � � �� � � � �

� � � � � � � �j j j

En forma binómica: � � � � � �

AB

180º

3 4 30 3 4 40

90 160 120 120 70 0 70 V

� � � � � �

� � � � � � � � �

j j j

j j

Esto es, 70 V en contrafase con la tensión aplicada.

A

30*

B

79.6 �F

127.3 mH

30*

250 V 50 Hz

53º

I1

I2

I V53º

- 463 -

Física Universitaria: Problemas de Física Corriente alterna. E06.13

13. En el circuito de la figura, existe una tensión de 220 V a 50 Hz entre los

puntos A y B. Hállese la diferencia de tensión alterna entre los puntos C y D, y su desfase con respecto a la tensión de alimentación.

Calculamos la frecuencia angular y las reactancias de los elementos reactivos:

90º

390º

2 100 rad/s1 1000 1000 5 � 5 5 �

100 2 20050100 10 5 � 5 5 �

C C

L L

XC

X L

� �� ��

� �

� ��

� �

� � � � � � �

� � � � � �

j

j

Las impedancias de cada una de las ramas y las intensidades que las recorren son:

0º1 45º

45º 45º11

0º45º22 45º

45º2

22031.11 A

5 5 50 � 502205 5 50 � 31.11 A50

� �

��

��� � � ��� �� � � � ��� � � �� �� � �� ��� � � ������

j

j

�� ���� � �

La d.d.p. entre los puntos C y D viene dada por

CD CA AD AC AD 1 1 2 L� � � � � � � �� � � � � � � � �

de modo que

CD 45º 0º 45º 90º 45º 45º31.11 5 31.11 5 156 156 0�� � � � � � ��

resultando que los puntos C y D están al mismo potencial.

2 mF�

50 mH�

5*

5* C

D

B A 220V 50Hz

5*

5* C

D

BA 220V 50Hz

-5j *

+5j *

- 464 -

Física Universitaria: Problemas de Física Corriente alterna. E06.14

14. En el circuito de corriente alterna de la figura se desea conocer: a) La

intensidad total y por rama. b) La diferencia de potencial en bornes de cada elemento. c) Diagrama fasorial de dichas intensidades y diferencia de potencial.

Datos: 1 0.034�, F, H.

300R C L

� �� � �

Cálculo de las reactancias y de las impedancias de rama:

1 37º90º

2 37º90º

3 0º

0.03 4 3 j 5100 3 3 3 j4 3 j 5

1 300 3 3 j3 4 4100

L L

CC

X L

XC

� ��

�� �

��

� �� � � � � '� � � � � ' ��� � � � ' ��� ��� � � � '� � �� � �� � � '� � ��� � � '� � � � '� ����

�����

a) Impedancia total e intensidad total: 1

37º 37º 0º 0ºT 1 1 1

0ºT 0º T

T 0º0º (en fase con la tensi n)

1 1 1 1 0.2 0.2 0.25 0.2cos37º 0.2cos37º 0.25 0.57

124.4220

1.751.75

A

�� �� � � � � � � � � � '

# � ' � � �

� � � ��� � � ó

b) Intensidades de rama:

37º

37º

0º1

1 37º

0º2

2 37º

0º3

00

º

44 A

44 A

2205

2205

522

45

0A

��

��� � � ��������� � � �������� � � ������

�� ��� ��� �

c) Tensiones que soportan cada uno de los elementos:

@

1 1 1 37º 0º 37º

1 37º 90º 53º

2 2 2 37º 0º 37º

2 37º 90º 53º

3 0º

44 4 176 V

44 3 132 V

44 4 176 V

44 3 132 V

220 V

R R

L L

R R

C C

R

� �

� �

� �

�� � � ��������� � � ��������� � � ��������� � � ������� ��������

� � �� � �� � �� � ��

I1

R R

L C

220V 50Hz

R

I2 I3

I1

I2

I3

IT

VL

VR1

V

VC

VR2 V

VR3 = V

- 465 -

Física Universitaria: Problemas de Física Corriente alterna. E06.15

15. En una línea de corriente alterna de 220 V y 50 Hz se conectan en paralelo las tres cargas siguientes: Z1 = 30 * con cos +1 = 0.8 inductivo Z2 = 20 * con cos +2 = 0.85 inductivo Z3 = 40 * con cos +3 = 0.9 inductivo a) Calcular la intensidad que circula por cada una de las cargas, y la intensidad total. b) Determinar la potencia total consumida y el factor de potencia del conjunto. c) Evaluar la capacidad del condensador que hay que colocar para corregir totalmente el factor de potencia.

Determinamos las impedancias de cada carga: � � �

1 1 1 36.9º

2 2 2 31.8º

3 3 3 25.8º

cos 0.8 36.9º 30 24.0 18.0

cos 0.85 31.8º 20 17.0 10.5

cos 0.9 25.8º 40 36.0 17.4

� �

� �

� �

� � � � �

� � � � �

� � � � �

j

j

j

���

a) Las intensidades en cada una de las carga son:

0º11

1 36.9º

0º22

2 31.8º

36.9º

31.8

33

3 25.8º

º

25.8º

7.3

1

2205.9 4.4 A

30

2209.4 5.8 A

20

2205.0 2.4 A

40

1.0

5.5

� � � � �

� � � � �

� � � � �

j

j

j

�� ��� ��� �

y la intensidad total es � 32 1.1 3 820.3 1 22.6 3.9 A�� � � � � �j� � � �

La impedancia total es:

0º31.8º

31.8º

2209.2

23.9

� � ��� �

b) El factor de potencia y la potencia consumida son f.p. cos31.8º

cos 220 23.90.85

4.47 k0.85 4467 WWP VI �� �

� � � � � �

c) Deberemos colocar un condensador en paralelo que compense la corriente reactiva retrasada de la instalación:

reactreact cond 1/

IVI I CC V� �

� � �

de modo que 412.6 1.82 10

10182

0�

220FC

��� � � �

I1 I2 I3V

1 2 3

I

Iact Ireact

I

+

- 466 -

Física Universitaria: Problemas de Física Corriente alterna. E06.16

16. En el circuito de la figura, la tensión entre bornes de la bobina es

el doble que la tensión que soporta el condensador C1. a) Cal-cular la d.d.p. entre los bornes del generador y la intensidad que éste proporciona al circuito. b) Determinar los valores de L, C1 y C2. c) Evaluar la impedancia total del circuito y su factor de potencia.

rama 3: AB 3 3 0º 0º 0º12 8 96 V� � �� � �

rama 2:

0ºAB2 90º 2 2

2 90º 2 2

96 1 1 15.33 597 �F18 100 5.33

Z CC Z

� � ��

� � � � � � ��

�� �

rama 1:

AB AB AB1 1 1 90º

1 1 1

1

1

1 96 10.67 10.671 9V V VI Z LZ C IL

C

L I

� ��

� � � � � � � ��

12I

21 1 1

3 11 1 1

2 6

2 2

2 1 1 110.67 298 �F100 10.67

2 68 mH100 298 10

L LC C C

Z CC C C

L

�� � �

� � � �

� �

� �

� � � � � ��

� ��

Asociación de impedancias en paralelo: 1

37º1 2 3

1 1 1 1 1 1 1 1 0.125 0.094 0.15310.67 5.33 8 10.67 5.33 8

�� � � � � � � � � � � � ��

j j jj j� � � �

de modo que 37º6.4 ��� y cos cos37º 0.8�� � (capacitativo)

Intensidad suministrada por el generador:

0º37º

37º

(adelantada)96

15 A 6.4 �

� � ��� �

A

50Hz L

8*

12 A 18 A 9 A

C1C2

A A

1 2 3A

B

- 467 -

Física Universitaria: Problemas de Física Corriente alterna. E06.17

I1 I2

I

A

11*

B

-18j � 11*

220 V 50 Hz

40j �

17. Al circuito de la figura se le aplica una tensión alterna de 220 V a 50 Hz. a) Determinar las intensidades que circulan por cada rama y la intensidad total. Dibujar el diagrama fasorial de intensidades. b) Calcular la diferencia de potencial entre los puntos A y B y su desfase con respecto a la tensión de alimentación.

a) Calculamos la frecuencia angular y las reactancias de los elementos reactivos:

3

90º

6

90º

2 100 rad/s100 127.3 10 40 �

40 40 �

1 10 18 �100 176.8

18 18 �

L

L

C

C

X L

XC

� �� �

� �

� �

� �

� � � � �

� � �

� � ��

� � �

j

j

Las impedancias de cada una de las ramas y las intensidades que las recorren son:

0º1 0º

1 0º1

2 90º 0º2 90º

2 90º

22010 10 A

2211 11 22 �40 18 22 =22 � 220

10 10 A22 �

��� � � � ���� � � �� �� �� �� �� � �� �� � � � � � �����

j j jj

�� ��� �� �

Impedancia total: �

0º 90º 0º 90º 90º 90º1 2T 45º

45º1 2

22 22 22 22 22 2215.6 �

22 22 22 1 1 1 1 2� � � � � �

� � � �j j j� �� � �

Intensidad total: T 1 2 45º

0T 45º

T 45º

10 10 14 A220

14 A15.6

� � � � � ������� � � �����

j� � ��� �

b) Calculamos la d.d.p. entre los puntos A y B por el… Camino superior:

� � � � �AB 1 1 2 10 11 10 40 110 400 290 VL� � � � � � � � � � �j j� � � � �

Camino inferior: � � � � �AB 1 2 2 10 11 10 18 110 180 290 VC� � � � � � � � � �j j� � � � �

Esto es, 290 V en fase con la tensión aplicada.

A

11*

B

176.8 �F

127.3 mH

11*

220 V 50 Hz

I2=10A

I=14A

V 45º I1=10A 110 V 400 V

290 V

110 V 180 V

290 V

- 468 -

Física Universitaria: Problemas de Física Corriente alterna. E06.18

18. En el circuito de la figura, hállense: a) La intensidad total y en cada

uno de los elementos. b) La diferencia de tensión en bornes de cada elemento. c) Potencia y factor de potencia. d) Capacidad del condensador que hay que colocar en paralelo para corregir totalmente el factor de potencia.

Cálculos preliminares: 3

6

2

par 90º

36.9º

100 6.37 10 2 � 2 �1 1 4 � 4 �

100 796 10(2 )( 4 ) 8 4 4 �

(2 ) ( 4 ) 28 4 2 8 6 10 �

L L

C C

X L

XC

� �

� �

�� � � � � � ������ � � � � ��� � ����� � �� � � � ��� � � ���� � � � � � ����

j

j

j j j jj j jj j j

a) Determinación de intensidades:

:

0

36.9º

par par 36.9º 90º 53.1º

53.1º

90º

53.1

36.9º

36.

º

90º

143.1º

220(17.6 13.2 ) A

10

22 4 88 V

882

22 A

882

44 A

44 A

LL

L

CC

C �

�� � � � �

� � � �

� � �

� � �

j�� �� ��

�� ��� �

b) Tensión que soporta cada elemento:

36.9º

53.1º

53

36.9º 0º

36.9º 90º .1º

176 V

88 V

22 8

22 2 44 V

R

R

L L

�� � � �

� � � �

� ���� ��

c) Factor de potencia y potencia (inductivo)3 0.8

3876.9º cos cos36.9º

cos 220 22 0.8 2 WP VI� �

�� � �

� � � � �

d) Corrección total del factor de potencia con condensador:

react

19

sen1/

sen 22 sen 36.9 1�Fº100 220

VI I CVC

ICV

� ��

�� �

� � �

�� � �

8*

220 V 50 Hz

6.37mH

6.37mH 796�F

IC

IL

I

37º

Vpar

VL

37º

V

VR

191�F 220 V50 Hz

- 469 -

Física Universitaria: Problemas de Física Corriente alterna. E06.19

19. En el esquema de la figura determínense: a) La intensidad que

circula por cada rama. b) La diferencia de potencial entre A y B. c) La potencia y el factor de potencia. d) Valor de la capacidad que hay que colocar en paralelo para corregir el factor de potencia.

a) Calculamos las impedancias de cada elemento y de las asociaciones en paralelo y total:

82º 82º

82º 82º

1 700 7 � 1 7 50 7.07 �1000.07 1 7 50 7.07 �100 7 �

C C

LL

XC

X L

�� �

� ��

� �

� �

��� � � � ��� � � � ���� �� �� � � � � �� �� � � ������

j

j

82º 82ºp 0º 0º

50 50 50 25 40(1 7 ) (1 7 ) 2

C L

C L

� �� � � � �� � � �j j� �� �� �

0º0º p p 0º 0º 0º

(en fase)220

5.5 A 5.5 25 137.5 V40

� � � � � � ��� � ���

0º 0º82º 82º

82º 82º

137.5 137.519.45 A 19.45 A

50 50C LC L

�� �

� � � � � �� �� �� �

b) Cálculo de la d.d.p. entre A y B:

AB 90º1 1 (2.75 19.25 ) (2.75 19.25 ) 38.5 38.5 VC L C L� � � � � � � � � � � �j j j� � � � �

c) Factor de potencia y potencia: 0 f.p. cos 1 cos 220 5.5 1210 WP VI� � �� � � � � � �

d) No hay nada que corregir

0,07 H�

1 F700�

15*

220 V 50 Hz

1* 1*

BA

- 470 -

Física Universitaria: Problemas de Física Corriente alterna. E06.20

20. En el circuito de corriente alterna de la figura se desea conocer: a) La

intensidad total y por rama. b) La diferencia de potencial en bornes de cada elemento. b) Diagrama vectorial de dichas intensidades y diferencia de potencial.

Datos: 4� 1061�F 9.55 mHR C L� � �

1 0

32 90º

3 0

4 90º6

5 0

4 �

9.55 10 100 3 3 �

4 �

1 1 3 3 �1061 10 100

4 �

R

LR

C

� �

� �

��

� �

� � � � � �

� �

� � � � �� �

j j j

jj j

���

a) Calculamos las intensidades a partir de la ley de Ohm para C.A.:

0 01 37º

1 2 37º

0 02 37º

3 4 37º

03 0

5 0

1 2 3 0 (en fase con la tension)

220 22044 A 35.2 26.4 A

4 3 5

220 22044 A 35.2 26.4 A

4 3 5

22055 A 55 A

4

125.4 A

� � � � � �� �

� � � � � �� �

� � � �

� � � �

jj

jj

�� � ��� � �

�� �� � � � b) Calculamos las tensiones:

1 1 1 37º 0 37º

2 1 2 37º 90º 53º

3 2 3 37º 0 37º

4 2 4 37º 90º 53º

5 0

44 4 176 V

44 3 132 V

44 4 176 V

44 3 132 V

220 V

� �

� �

� � � �

� � � �

� � � �

� � � �

� �

� � �� � �� � �� � �� �

c) Diagrama fasorial:

L

R R

C R 220 V

50 Hz

D

AB

C

IT

I1 I2

I3

Z2

A

B

C D I1

I2 I3

Z1 Z3

Z4

Z5

- 471 -

Física Universitaria: Problemas de Física Corriente alterna. E06.21

0.4H

5*

220 V50 Hz

0.07H

�1*

1*

BA

1F

700� IBIA

I 21. En el circuito representado en la figura, determinar: a) Intensidad en cada rama. b) Diferencia potencial entre A y B. c) Potencia y factor de potencia. d) Dibujar el diagrama fasorial.

Cálculo de impedancias:

B BB

rad2 2 50 10s

1 700 7 � 7 �100

0.07100 7 � 7 j �

0.4100 40 � 40 j �

C C

L L

L L

XC

X L

X L

� �� � �

�� �

� ��

� ��

� � � �

� � � � �

� � � � � �

� � � � �

j�

82º 82ºA b

0ºA Bpar 0º 53º

A B 0º

(inductivo)

1 7 = 50 1 7 = 5050

25 � 30 40 = 502

� �� � � �

� � � � ��

j j

j

� �� �� �� �

a) Cálculo de intensidades:

0º53º

53º

par par 53º 0º 53º

53ºparA 29º

82ºAA

53ºparB 135º

82ºB

(retrasada)220

4.4 A = (2.64 - 3.52 ) A50

4.4 25 110 V

11015.56 (13.64 7.48 ) A

50110

15.56 ( 11.00 11.00 ) A50

� �

��

�� � ��

�� � � � ���

� � � � �� � �� � � � � � ���

j

j

j

�� �� ���� �

� ��� �

B�

b) Diferencia potencial entre A y B:

BAB A A B 29º 0º 135º 90º 29º 45º

127º

V I I X 15.56 1 15.56 7 15.56 108.89

(13.64 7.48 ) (77.00-77.00 ) ( 63.36 84.48 ) 105.6LR � �� � � � � � �

� � � � �j j j c) Potencia y factor de potencia:

(inductivo)cos 220 4.4 cos53º 580.80 W f.p. cos cos53º 0.6P VI = =+ +� � � � � �

0º 53º 53º

580.80 W220 4.4 968 (580.80 774.40 ) W

774.40 VAPQ� �

��� � � � � � ��

j� ��

�A

�B ����

+

��

P

+

Q

- 472 -

Física Universitaria: Problemas de Física Corriente alterna. E06.22

22. En el circuito que se muestra en la figura, el alternador suministra una

tensión alterna de 50 Hz y en la resistencia de 4 � se disipa una potencia de 16 W. a) Calcular la intensidad en cada rama y la tensión entre bornes del generador, así como la intensidad de la corriente suministrada por éste. b) Determinar el factor de potencia de toda la carga. c) Evaluar la potencia consumida y la potencia reactiva de la carga y de cada uno de los elementos.

a) Calculamos las impedancias de cada rama y la de la carga total.

79º1 79º

68º2 68º

1 5 26 5.10 �

4 10 116 10.77 �

� �

� �

� � � �

� � � �

j

j

��

� � �79 68º 11º 11º1 2

56º1 2 45º

capacitativo5.10 10.77 54.92 54.92

7.77 �1 5 4 10 5 5 7.07

� � ��� � � � �

� � � � �j j j� �� � �

Calculamos la intensidad en la segunda rama a partir de la potencia disipada en la resistencia:

2 / 16 / 4 2 AP I R I P R� � � � La tensión entre A y B (suministrada por el generador) será:

2 2 68º 68º 0º2 10.77 21.54 V� �� � �� � �

Las intensidades en cada rama y la suministrada por el generador son:

0º1 79º

1 79º

2 68º

0º56º

56º

21.544.22 = 0.81 4.14 A

5.10

2 0.75 1.85 A

21.542.77 1.56 2.29 A

7.77

��

� � � �

� � �

� � � � �

j

j

j

�� ���� �

b) La impedancia de la carga total es capacitativa, con un argumento (ángulo)

(capacitativo)56º f.p. cos 0.57� �� � � c) Cálculo de las potencias: En la resistencia de la rama 1 se consume:

2 21 4.22 1 18 WP I R� � � �

Las potencias reactivas en el condensador y en la bobina son: 2

cond22

bob

4.22 5 89 W

2.00 10 40 W

PP I X

P

�� � � ��� �� � � ���

La potencia total:

� �0º 56º 56º

capacitativa

34 W21.54 2.77 60 34 49 W

49 W PQ

� ���� � � � � �� ���j� ��

5 �

1 � 4 �

10 �

5 �

1 �

I1 I2

I A

B

4 �

10 �

I1

I2

I

Z1

Z2

Z

56º

-56º

- 473 -

Física Universitaria: Problemas de Física Corriente alterna. E06.23

23. Cuando se conecta un circuito RLC serie a una línea de 120 V

eficaces y 60 Hz de frecuencia, circula una corriente eficaz de 11 A, adelantada 45º respecto de la tensión. a) Hallar la potencia suministrada al circuito y la resistencia óhmica del mismo. b) Si la autoinducción es L = 0.05 H, hallar la capacidad C presente en el circuito. c) ¿Qué capacidad o autoinducción deberán añadirse para que el factor de potencia sea 1?

2 2 60 120 rad/s� �� � �� � � �

045º

45º

12010.9 � =(7.71-7.71j) �

11 �� � ��� �

de modo que el circuito RCL es capacitativo. a) La resistencia óhmica es la parte real de la impedancia:

(capacitativo)7.71 7.71� =� -45ºR X �� � � La potencia suministrada al circuito es cos 120 11 cos 9335 W4 .4P V I �� � � � � � � b) Calculamos la capacidad pedida:

120 0.05 18.85 �18.85 ( 7.71) 26.

99.9 �F

56 �1 1 1

120 26.56

L

L C C L

CC

X LX X X X X X

X CC X

� �

� � �

� � � �

� � � � � � � �

� � � ��

c) Para corregir completamente el factor de potencia necesitamos colocar una autoinducción en paralelo con la carga total:

reactcorr

corr

sen

120 0.041 Hsen 120 11 sen 45º

41 mH

VI IL

VLI

��

� � �

� �

� � � �� �

L

R

C120 V 60 Hz

B

A

Lcorr

I

Iact

Icorr Ireact

45º

- 474 -

Física Universitaria: Problemas de Física Corriente alterna. E06.24

I

V

retrasada

adelantadaC

V

I

Ireact =I sen+

Iact=I cos++

Icond

24. A una red de corriente alterna de 220 V/50 Hz se conectan (en paralelo) un motor que consume 3 kW con un factor de potencia 0.8 inductivo y 50 lámparas fluorescentes de 40 W (cada una) y factor de potencia 0.4 inductivo. a) Determinar la intensidad total que consume esta instalación. b) Calcular el factor de potencia del consumo. c) Dibujar el diagrama fasorial de tensiones e intensidades. d) Encontrar la capacidad del condensador que hay que colocar en paralelo para corregir totalmente el factor de potencia.

Con los datos del enunciado, completamos la tabla que se adjunta, sirviéndonos de las expresiones que se indican en los encabezamientos de las columnas de la Tabla:. Generador: 220 V / 50 Hz; intensidades en amperios (A)

Elementos P (W) cos � �º actI P V� react act tgI I �� act cosI I ��

1 motor 3000 0.8 ind 36.9º 13.6 10.2 17.0

50 tubos �40 2000 0.4 ind 66.4º 9.1 20.8 22.7

Totales 5000 0.59 ind 53.9º 22.7 31.1 38.5

ab) Calculamos la intensidad que consume la instalación, el desfase y el factor de potencia del consumo:

2 2 2 2act react

react

act

(inductivo)

22.7 31.131.1= arctg arctg22.7

cos cos 53.9º

38.5 A

53.9º

0.59

I I III

� � � � �

� �

� �

c) En la figura adjunta presentamos los diagramas fasoriales de las intensidades y de la tensión (la misma en todos los elementos en paralelo). d) Puesto que la instalación es inductiva, deberemos colocar un elemento capacitativo (condensador) en paralelo con la carga para corregir el factor de potencia:

react

react 4

1/31.1

10 250 �

0 2 0F

C

V VI CVX C

ICV

��

� �

� � �

� � ��

1 37º17I �

2 66º22.7I �54º38.5I �

V

- 475 -

Física Universitaria: Problemas de Física Corriente alterna. E06.25

I

V

retrasada

adelantadaC

V

I

Ireact =I sen+

Iact=I cos++

Icond

25. Una instalación eléctrica (220 V, 50 Hz) se compone de los siguientes consumos: 50 lámparas fluorescentes de 40 W cada una, y factor de potencia 0.40 (inductivo) 1 motor de 2 kW de consumo y factor de potencia 0.80 (inductivo) 2 calefactores eléctricos (resistivos) de 0.5 kW cada uno a) Calcular las intensidades que producen cada uno de esos tres grupos de consumo. b) Calcular el factor de potencia del conjunto. c) Determinar el elemento (indicando condensador o bobina, y su valor) que hay que añadir en paralelo para corregir totalmente el factor de potencia.

Utilizando las relaciones que se indican, completamos la tabla:

act act act

react act react act

react

act

cos

cossen tg tg

arctg

PP VI VI I IV II I I I I

II

�� � �

��� � � ��� ����� � � ���

Grupo Elementos P (W) f.p.=cos + + Iact (A) Ireact (A) I (A)

1 50 tubos 40 W 2000 0.40 (i) 66.42º 9.09 20.83 22.73

2 1 motor 2000 0.80 (i) 36.87º 9.09 6.82 11.36

3 2 calef 500 W 1000 1.00 0.00º 4.55 0.00 4.55

Totales 5000 0.64 (i) 50.58 22.73 27.65 35.79

a) Intensidades en cada uno de los tres grupos son: Lámparas: 1 66.4º22.73 A��� (retrasada)

Motor: 2 36.9º11.36 A��� (retrasada)

Calefactores: 3 0º4.55 A�� (en fase)

b) El factor de potencia del conjunto vale 0.64 (inductivo) c) Puesto que la instalación resulta inductiva, deberemos colocar un condensador en paralelo

que compense la corriente reactiva retrasada de la instalación:

reactreact cond react 1 /

IVI I I CC V� �

� � �

de modo que 427.65 4 10

100 220400 �FC

��� � � �

Iact

Ireact I

+

I1 I2 I3V

1 2 3

I

- 476 -

Física Universitaria: Problemas de Física Corriente alterna. E06.26

I

V

retrasada

adelantadaC

V

I

Ireact =I sen+

Iact=I cos++

Icond

26. A una red alterna de 220 V / 50 Hz se conectan (en paralelo) un motor que consume 1.5 kW con factor de potencia inductivo 0.8 y 50 lámparas fluorescentes de 40 W (cada una) y factor de potencia 0.4 también inductivo. a) Calcular la intensidad total que consume esta instalación. b) Hallar el factor de potencia del conjunto. c) Determinar a capacidad del condensador que hay que colocar en paralelo para corregir totalmente el factor de potencia.

Utilizando las relaciones que se indican, completamos la tabla:

act act act

react act react act

react

act

cos

cossen tg tg

arctg

PP VI VI I IV II I I I I

II

�� � �

��� � � ��� ����� � � ���

Grupo Elementos P (W) f.p.=cos + + Iact (A) Ireact (A) I (A)

1 1 motor 1500 0.80 (i) 36.9º 6.82 5.11 8.52

2 50 tubos 40 W 2000 0.40 (i) 66.4º 9.09 20.83 22.73

Totales 3500 0.52 (i) 58.5º 15.91 25.94 30.43

a) Intensidad total que consume la instalación: 58.5º30.43 A���

b) El factor de potencia del conjunto vale 0.52 (inductivo) c) El factor de potencia inductivo se corrige con un condensador en paralelo con la carga.

react

cond

reactreact

atrasada: sen

adelantada: 1/

senC

I IV VI CVX C

I II CV CV V

��

��

� �

� � �

� � �

Sustituyendo valores: 425.94 3.75 10 375 �F

100 220C

��� � � �

Iact

Ireact I

+

I1 V

1 I2 2

I

- 477 -

Física Universitaria: Problemas de Física Corriente alterna. E06.27

V

I

I sen +

Iact=I cos+

+’

Icond

+

I’

I’sen+’

Iact tg + Iact tg +’

27. En una estación de bombeo (220 V – 50 Hz) tenemos instalados 3 motores y un grupo de resistencias con las siguientes características:

Motor 1: 2 CV, cos 2 = 0.85 Motor 2: 5 CV, cos 2 = 0.7 Motor 3: 3 CV, cos 2 = 0.6 Resistencias: 3 CV = 2208 W a) Calcular las intensidades absorbidas por cada motor o grupo. b) ¿Qué condensador habrá que poner para llevar la instalación completa a un cos + = 0.95? Datos: 1 CV = 736 W.

Utilizando las relaciones que se indican, completamos la tabla:

act

act act

reactreact act react act

act

coscos

sen tg tg arctg

IIPP VI VI IV

II I I I II

��

� � � �

���� �� �� �� � �� �� � � �� �� �� �� � � �� �� ���

Elementos P (W) f.p.=cos + + Iact (A) Ireact (A) I (A)

Motor1 1472 0.85 (i) 31.79º 6.69 4.15 7.87

Motor 2 3680 0.70 (i) 45.57º 16.73 17.07 23.90

Motor 3 2208 0.60 (i) 53.13 10.04 13.39 16.73

Resistencias 2208 1.00 0.00º 10.04 0.00 10.40

9568 0.78 (i) 38.51º 43.49 34.61 55.76

a) Intensidades en cada uno de los cuatro grupos son: Motor 1: 1 31.79º7.87 A��� (retrasada)

Motor 2: 2 45.57º23.90 A��� (retrasada)

Motor 3: 3 53.13º16.73 A��� (retrasada)

Resistencias: 4 0º10.04 A�� (en fase)

b) El factor de potencia del conjunto vale 0.78 (inductivo) Puesto que la instalación resulta inductiva, deberemos colocar un condensador en paralelo que compense la corriente reactiva retrasada de la instalación: Corrección parcial:

� � �

act act cond act

act

tg tg tg tg1/

tg tg 43.49 tg 38.51º tg18.19º100 2

22

94 0

�F

VI I I I CVC

IC

V

� � � � ��

� �

� �

) )� � � � �

)� � �� � �

Iact

Ireact I

+

- 478 -

Física Universitaria: Problemas de Física Corriente alterna. E06.28

I

V retrasada

adelantadaC

V

I

Ireact =I sen+

Iact=I cos++

Icond

28. Se realiza la iluminación de una sala de trabajo con 200 lámparas fluorescentes de 220 V/40 W y un factor de potencia de 0.4 (inductivo). a) Determinar la intensidad total que las alimenta. b) La capacidad de condensador que deberá colocarse en paralelo a la entrada de la instalación para corregir totalmente el factor de potencia. c) La capacidad de los condensadores que deberían colocarse en paralelo con cada una de las lámparas fluorescentes para corregir totalmente el factor de potencia. Nota: Esta es una alternativa a la solución del apartado anterior.

Cada tubo consume 40 W / 220 V con cos�=0.4 inductivo (� = 66.4º).

cos40 0.455 A

cos 220 0.4

P VIPI

V

� � ��

0

66.4º

220 V

0.455 A 0.182 0.417 A�

� ����� � � ��� j

��

a) Intensidad total: total 66.4º 66.4º200 0.455 90.9 A 36.4 83.3 An � �� � � � � � j� �

Corrección total del factor de potencia: El factor de potencia inductivo se corrige con un condensador en paralelo con la carga.

react

cond

react

atrasada: sen

adelantada: 1/

sensenC

I IV VI CVX C

I II CV CV V

��

�� �

� �

� � �

� � �

b) Con condensador único para toda la instalación: 3

total83.3 1.2 10 F 1.2 mF

100 220C

��� � � �

c) Con un condensador para cada uno de los tubos: 3

tubo0.417 6 10 6F 6 F

100 220C �

��� � � �

Obsérvese que estos 200 condensadores en paralelo equivalen a

eq tubo200 200 6 �F 1200 �F 1.2 mFC C� � � � �

220 V 50 Hz

200 tubos 40 W

- 479 -

Física Universitaria: Problemas de Física Corriente alterna. E06.29

V

I

I sen +

Iact=I cos+

+’

Icond

+

I’

I’sen+’

Iact tg + Iact tg +’

29. Un generador de corriente alterna (200 V) suministra una intensidad de 100 A a una instalación que tiene un factor de potencia inductivo de 0.6. Cuando ponemos un condensador en paralelo con dicha instalación, el factor de potencia pasa a valer 0.9, también inductivo. a) Hallar la intensidad que pasa por el mencionado condensador. b) ¿Qué intensidad suministra el generador después de la corrección del factor de potencia?

Puesto que el factor de potencia es inductivo, la intensidad de corriente está atrasada con respecto al voltaje un ángulo:

arccos0.6 53.13º�� �

La intensidad será: 53.13º0100 60 80 A�� � � j�

La impedancia total de la instalación será:

053.13º

53.13º

2002 �

100 �

� � � �� �� �� �

Cuando ponemos el condensador en paralelo, el retraso de la intensidad se reduce hasta un valor:

arccos 0.9 25.84º�) � � a) Por el condensador circula una corriente adelantada 90º respecto de la tensión, cuyo valor se determina fácilmente a partir del diagrama de intensidades:

� �

cond act act acttg tg tg tg

60 tg53.13º tg 25.84º 51 A

I I I I� � � �) )� � � � �

� � �

cond 90º51 A���

b) Después de colocar el condensador, el generador suministra una corriente I’ que se determina a partir del diagrama de intensidades:

� �act act -25.84ºtg 60 60 tg 25.84º 60 2 1 A9 9AI I �) )� � � � � � �j j j�

200V

~100 A

Z

200V

~ C

I’ Icond Icarga

Z

- 480 -